Compilation Sa GI

You might also like

Download as pdf or txt
Download as pdf or txt
You are on page 1of 360

MANAGEMENT OF CLIENTS WITH INGESTIVE DISORDERS

Nurse Berlinda is assigned to a 41-year-old client who has a diagnosis of chronic pancreatitis. The nurse reviews the laboratory
result, anticipating a laboratory report that indicates a serum amylase level of:

o A. 45 units/L

o B. 100 units/L

o C. 300 units/L

o D. 500 units/L

2. Question
A male client who is recovering from surgery has been advanced from a clear liquid diet to a full liquid diet. The client is looking
forward to the diet change because he has been “bored” with the clear liquid diet. The nurse would offer which full liquid item to
the client?

 A. Tea

 B. Gelatin

 C. Custard

 D. Popsicle

2. 3. Question
Nurse Juvy is caring for a client with cirrhosis of the liver. To minimize the effects of the disorder, the nurse teaches the client
about foods that are high in thiamine. The nurse determines that the client has the best understanding of the dietary measures to
follow if the client states an intention to increase the intake of:

 A. Pork

 B. Milk

 C. Chicken

 D. Broccoli

3. 4. Question
Nurse Oliver checks for residual before administering a bolus tube feeding to a client with a nasogastric tube and obtains a
residual amount of 150 mL. What is the appropriate action for the nurse to take?

 A. Hold the feeding

 B. Reinstill the amount and continue with administering the feeding

 C. Elevate the client’s head at least 45 degrees and administer the feeding

 D. Discard the residual amount and proceed with administering the feeding

4. 5. Question
A nurse is inserting a nasogastric tube in an adult male client. During the procedure, the client begins to cough and has difficulty
breathing. Which of the following is the appropriate nursing action?

 A. Quickly insert the tube


 B. Notify the physician immediately

 C. Remove the tube and reinsert when the respiratory distress subsides

 D. Pull back on the tube and wait until the respiratory distress subsides

6. Question
Nurse Ryan is assessing for correct placement of a nasogastric tube. The nurse aspirates the stomach contents and checks the
contents for pH. The nurse verifies correct tube placement if which pH value is noted?

 A. 3.5

 B. 7.0

 C. 7.35

 D. 7.5

5. 7. Question
A nurse is preparing to remove a nasogastric tube from a female client. The nurse should instruct the client to do which of the
following just before the nurse removes the tube?

 A. Exhale

 B. Inhale and exhale quickly

 C. Take and hold a deep breath

 D. Perform a Valsalva maneuver

6. 8. Question
Nurse Joy is preparing to administer medication through a nasogastric tube that is connected to suction. To administer the
medication, the nurse would:

 A. Position the client supine to assist in medication absorption

 B. Aspirate the nasogastric tube after medication administration to maintain patency

 C. Clamp the nasogastric tube for 30 minutes following administration of the medication

 D. Change the suction setting to low intermittent suction for 30 minutes after medication administration

7. 9. Question
A nurse is preparing to care for a female client with esophageal varices who just had a Sengstaken-Blakemore tube inserted. The
nurse gathers supplies, knowing that which of the following items must be kept at the bedside at all times?

 A. An obturator

 B. Kelly clamp

 C. An irrigation set

 D. A pair of scissors

8. 10. Question
Dr. Smith has determined that the client with hepatitis has contracted the infection from contaminated food. The nurse
understands that this client is most likely experiencing what type of hepatitis?
 A. Hepatitis A

 B. Hepatitis B

 C. Hepatitis C

 D. Hepatitis D

9. 11. Question
A client is suspected of having hepatitis. Which diagnostic test result will assist in confirming this diagnosis?

 A. Elevated hemoglobin level

 B. Elevated serum bilirubin level

 C. Elevated blood urea nitrogen level

 D. Decreased erythrocyte sedimentation rate

10. 12. Question


The nurse is reviewing the physician’s orders written for a male client admitted to the hospital with acute pancreatitis. Which
physician order should the nurse question if noted on the client’s chart?

 A. NPO status

 B. Nasogastric tube inserted

 C. Morphine sulfate for pain

 D. An anticholinergic medication

11. 13. Question


A female client being seen in a physician’s office has just been scheduled for a barium swallow the next day. The nurse writes
down which instruction for the client to follow before the test?

 A. Fast for 8 hours before the test

 B. Eat a regular supper and breakfast

 C. Continue to take all oral medications as scheduled

 D. Monitor own bowel movement pattern for constipation

12. 14. Question


The nurse is performing an abdominal assessment and inspects the skin of the abdomen. The nurse performs which assessment
technique next?

 A. Palpates the abdomen for size

 B. Palpates the liver at the right rib margin

 C. Listens to bowel sounds in all four quadrants

 D. Percusses the right lower abdominal quadrant

13. 15. Question


Polyethylene glycol-electrolyte solution (GoLYTELY) is prescribed for the female client scheduled for a colonoscopy. The client
begins to experience diarrhea following the administration of the solution. What action by the nurse is appropriate?
 A. Start an IV infusion

 B. Administer an enema

 C. Cancel the diagnostic test

 D. Explain that diarrhea is expected

14. 16. Question


The nurse is caring for a male client with a diagnosis of chronic gastritis. The nurse monitors the client knowing that this client is
at risk for which vitamin deficiency?

 A. Vitamin A

 B. Vitamin B12

 C. Vitamin C

 D. Vitamin E

17. Question
The nurse is reviewing the medication record of a female client with acute gastritis. Which medication, if noted on the client’s
record, would the nurse question?

 A. Digoxin (Lanoxin)

 B. Furosemide (Lasix)

 C. Indomethacin (Indocin)

 D. Propranolol hydrochloride (Inderal)

15. 18. Question


The nurse is assessing a male client 24 hours following a cholecystectomy. The nurse noted that the T-tube has drained 750 mL of
green-brown drainage since the surgery. Which nursing intervention is appropriate?

 A. Clamp the T-tube

 B. Irrigate the T-tube

 C. Notify the physician

 D. Document the findings

16. 19. Question


The nurse is monitoring a female client with a diagnosis of peptic ulcer. Which assessment findings would most likely indicate
perforation of the ulcer?

 A. Bradycardia

 B. Numbness in the legs

 C. Nausea and vomiting

 D. A rigid, board-like abdomen

17. 20. Question


A male client with a peptic ulcer is scheduled for a vagotomy and the client asks the nurse about the purpose of this procedure.
Which response by the nurse best describes the purpose of a vagotomy?
 A. Halts stress reactions

 B. Heals the gastric mucosa

 C. Reduces the stimulus to acid secretions

 D. Decreases food absorption in the stomach

18. 21. Question


The nurse is caring for a female client following a Billroth II procedure. Which postoperative order should the nurse question and
verify?

 A. Leg exercises

 B. Early ambulation

 C. Irrigating the nasogastric tube

 D. Coughing and deep-breathing exercises

19. 22. Question


The nurse is providing discharge instructions to a male client following gastrectomy and instructs the client to take which
measure to assist in preventing dumping syndrome?

 A. Ambulate following a meal

 B. Eat high carbohydrate foods

 C. Limit the fluid taken with meal

 D. Sit in a high-Fowler’s position during meals

20. 23. Question


The nurse is monitoring a female client for the early signs and symptoms of dumping syndrome. Which of the following indicate
this occurrence?

 A. Sweating and pallor

 B. Bradycardia and indigestion

 C. Double vision and chest pain

 D. Abdominal cramping and pain

21. 24. Question


The nurse is preparing a discharge teaching plan for the male client who had umbilical hernia repair. What should the nurse
include in the plan?

 A. Irrigating the drain

 B. Avoiding coughing

 C. Maintaining bed rest

 D. Restricting pain medication

22. 25. Question


The nurse is instructing the male client who has an inguinal hernia repair how to reduce postoperative swelling following the
procedure. What should the nurse tell the client?
 A. Limit oral fluid

 B. Elevate the scrotum

 C. Apply heat to the abdomen

 D. Remain in a low-fiber diet

23. 26. Question


The nurse is caring for a hospitalized female client with a diagnosis of ulcerative colitis. Which finding, if noted on assessment of
the client, would the nurse report to the physician?

 A. Hypotension

 B. Bloody diarrhea

 C. Rebound tenderness

 D. A hemoglobin level of 12 mg/dL

24. 27. Question


The nurse is caring for a male client postoperatively following the creation of a colostomy. Which nursing diagnosis should the
nurse include in the plan of care?

 A. Sexual dysfunction

 B. Body image, disturbed

 C. Fear related to poor prognosis

 D. Nutrition: more than body requirements, imbalanced

25. 28. Question


The nurse is reviewing the record of a female client with Crohn’s disease. Which stool characteristics should the nurse expect to
note documented in the client’s record?

 A. Diarrhea

 B. Chronic constipation

 C. Constipation alternating with diarrhea

 D. Stools constantly oozing from the rectum

26. 29. Question


The nurse is performing colostomy irrigation on a male client. During the irrigation, the client begins to complain of abdominal
cramps. What is the appropriate nursing action?

 A. Notify the physician

 B. Stop the irrigation temporarily

 C. Increase the height of the irrigation

 D. Medicate for pain and resume the irrigation

27. 30. Question


The nurse is teaching a female client how to perform a colostomy irrigation. To enhance the effectiveness of the irrigation and
fecal returns, what measure should the nurse instruct the client to do?
 A. Increase fluid intake

 B. Place heat on the abdomen

 C. Perform the irrigation in the evening

 D. Reduce the amount of irrigation solution

28. 31. Question


For a client in hepatic coma, which outcome would be the most appropriate?

 A. The client is oriented to time, place, and person.

 B. The client exhibits no ecchymotic areas.

 C. The client increases oral intake to 2,000 calories/day.

 D. The client exhibits increased serum albumin level.

29. 32. Question


Jordin is a client with jaundice who is experiencing pruritus. Which nursing intervention would be included in the care plan for the
client?

 A. Administering vitamin K subcutaneously

 B. Applying pressure when giving I.M. injections

 C. Decreasing the client's dietary protein intake

 D. Keeping the client's fingernails short and smooth

30. 33. Question


Marie, a 51-year-old woman, is diagnosed with cholecystitis. Which diet, when selected by the client, indicates that the nurse’s
teaching has been successful?

 A. 4-6 small meals of low-carbohydrate foods daily

 B. High-fat, high-carbohydrate meals

 C. Low-fat, high-carbohydrate meals

 D. High-fat, low protein meals

31. 34. Question


The hospital administrator had undergone percutaneous transhepatic cholangiography. Which assessment finding indicates
complication after the operation?

 A. Fever and chills

 B. Hypertension

 C. Bradycardia

 D. Nausea and diarrhea

35. Question
When planning home care for a client with hepatitis A, which preventive measure should be emphasized to protect the client’s
family?
 A. Keeping the client in complete isolation

 B. Using good sanitation with dishes and shared bathrooms

 C. Avoiding contact with blood-soiled clothing or dressing

 D. Forbidding the sharing of needles or syringes

36. Question
For Jayvin who is taking antacids, which instruction would be included in the teaching plan?

 A. "Take the antacids with 8 oz of water."

 B. "Avoid taking other medications within 2 hours of this one."

 C. "Continue taking antacids even when pain subsides."

 D. "Weigh yourself daily when taking this medication."

37. Question
Which clinical manifestation would the nurse expect a client diagnosed with acute cholecystitis to exhibit?

 A. Jaundice, dark urine, and steatorrhea

 B. Acute right lower quadrant (RLQ) pain, diarrhea, and dehydration

 C. Ecchymosis petechiae, and coffee-ground emesis

 D. Nausea, vomiting, and anorexia

38. Question
Pierre, who is diagnosed with acute pancreatitis, is under the care of Nurse Bryan. Which intervention should the nurse include in
the care plan for the client?

 A. Administration of vasopressin and insertion of a balloon tamponade

 B. Preparation for a paracentesis and administration of diuretics

 C. Maintenance of nothing-by-mouth status and insertion of nasogastric (NG) tube with low intermittent suction

 D. Dietary plan of a low-fat diet and increased fluid intake to 2,000 ml/day

39. Question
When teaching a client about pancreatic function, the nurse understands that pancreatic lipase performs which function?

 A. Transports fatty acids into the brush border

 B. Breaks down fat into fatty acids and glycerol

 C. Triggers cholecystokinin to contract the gallbladder

 D. Breaks down protein into dipeptides and amino acids

40. Question
A 52-year-old man was referred to the clinic due to increased abdominal girth. He is diagnosed with ascites by the presence of a
fluid thrill and shifting dullness on percussion. After administering diuretic therapy, which nursing action would be most effective
in ensuring safe care?
 A. Measuring serum potassium for hyperkalemia

 B. Assessing the client for hypervolemia

 C. Measuring the client's weight weekly

 D. Documenting precise intake and output

41. Question
Which assessment finding indicates that lactulose is effective in decreasing the ammonia level in the client with hepatic
encephalopathy?

 A. Passage of two or three soft stools daily

 B. Evidence of watery diarrhea

 C. Daily deterioration in the client's handwriting

 D. Appearance of frothy, foul-smelling stools

42. Question
Nurse Farrah is providing care for Kristoff who has jaundice. Which statement indicates that the nurse understands the rationale
for instituting skin care measures for the client?

 A. "Jaundice is associated with pressure ulcer formation."

 B. "Jaundice impairs urea production, which produces pruritus."

 C. "Jaundice produces pruritus due to impaired bile acid excretion."

 D. "Jaundice leads to decreased tissue perfusion and subsequent breakdown."

43. Question
Which rationale supports explaining the placement of an esophageal tamponade tube in a client who is hemorrhaging?

 A. Allowing the client to help insert the tube

 B. Beginning teaching for home care

 C. Maintaining the client's level of anxiety and alertness

 D. Obtaining cooperation and reducing fear

44. Question
For Rico who has chronic pancreatitis, which nursing intervention would be most helpful?

 A. Allowing liberalized fluid intake

 B. Counseling to stop alcohol consumption

 C. Encouraging daily exercise

 D. Modifying dietary protein

45. Question
Mr. Hasakusa is in end-stage liver failure. Which interventions should the nurse implement when addressing hepatic
encephalopathy? Select all that apply.
 A. Assessing the client's neurologic status every 2 hours

 B. Monitoring the client's hemoglobin and hematocrit levels

 C. Evaluating the client's serum ammonia level

 D. Monitoring the client's handwriting daily

 E. Preparing to insert an esophageal tamponade tube

 F. Making sure the client's fingernails are short

46. Question
For a client with hepatic cirrhosis who has altered clotting mechanisms, which intervention would be most important?

 A. Allowing complete independence of mobility

 B. Applying pressure to injection sites

 C. Administering antibiotics as prescribed

 D. Increasing nutritional intake

47. Question
A client with advanced cirrhosis has been diagnosed with hepatic encephalopathy. The nurse expects to assess for:

 A. Malaise

 B. Stomatitis

 C. Hand tremors

 D. Weight loss

48. Question
A client diagnosed with chronic cirrhosis who has ascites and pitting peripheral edema also has hepatic encephalopathy. Which of
the following nursing interventions are appropriate to prevent skin breakdown? Select all that apply.

 A. Range of motion every 4 hours

 B. Turn and reposition every 2 hours

 C. Abdominal and foot massages every 2 hours

 D. Alternating air pressure mattress

 E. Sit in chair for 30 minutes each shift

49. Question
Which of the following will the nurse include in the care plan for a client hospitalized with viral hepatitis?

 A. Increase fluid intake to 3000 ml per day

 B. Adequate bed rest

 C. Bland diet

 D. Administer antibiotics as ordered


50. Question
Spironolactone (Aldactone) is prescribed for a client with chronic cirrhosis and ascites. The nurse should monitor the client for
which of the following medication-related side effects?

 A. Jaundice

 B. Hyperkalemia

 C. Tachycardia

 D. Constipation

1. 1. Question
During preparation for bowel surgery, a male client receives an antibiotic to reduce intestinal bacteria. Antibiotic therapy may
interfere with the synthesis of which vitamin and may lead to hypoprothrombinemia?

o A. Vitamin A

o B. Vitamin D

o C. Vitamin E

o D. Vitamin K

2. 2. Question
When evaluating a male client for complications of acute pancreatitis, the nurse would observe for:

 A. Increased intracranial pressure

 B. Decreased urine output

 C. Bradycardia

 D. Hypertension

3. 3. Question
A male client with a recent history of rectal bleeding is being prepared for a colonoscopy. How should the nurse position the
client for this test initially?

 A. Lying on the right side with legs straight

 B. Lying on the left side with knees bent

 C. Prone with the torso elevated

 D. Bent over with hands touching the floor

4. 4. Question
A male client with extreme weakness, pallor, weak peripheral pulses, and disorientation is admitted to the emergency
department. His wife reports that he has been “spitting up blood.” A Mallory-Weiss tear is suspected, and the nurse begins taking
the client’s history from the client’s wife. The question by the nurse that demonstrates her understanding of Mallory-Weiss
tearing is:

 A. “Tell me about your husband’s alcohol usage.”

 B. “Is your husband being treated for tuberculosis?”

 C. “Has your husband recently fallen or injured his chest?”

 D. “Describe spices and condiments your husband uses on food.”

5. 5. Question
Which of the following nursing interventions should the nurse perform for a female client receiving enteral feedings through a
gastrostomy tube?

 A. Change the tube feeding solutions and tubing at least every 24 hours.

 B. Maintain the head of the bed at a 15-degree elevation continuously.

 C. Check the gastrostomy tube for position every 2 days.

 D. Maintain the client on bed rest during the feedings.

6. 6. Question
A male client is recovering from a small bowel resection. To relieve pain, the physician prescribes meperidine (Demerol), 75 mg
I.M. every 4 hours. How soon after administration should meperidine onset of action occur?

 A. 5 to 10 minutes

 B. 15 to 30 minutes

 C. 30 to 60 minutes

 D. 2 to 4 hours

7. 7. Question
The nurse is caring for a male client with cirrhosis. Which assessment findings indicate that the client has deficient vitamin K
absorption caused by this hepatic disease?

 A. Dyspnea and fatigue

 B. Ascites and orthopnea

 C. Purpura and petechiae

 D. Gynecomastia and testicular atrophy

8. 8. Question
Which condition is most likely to have a nursing diagnosis of fluid volume deficit?

 A. Appendicitis

 B. Pancreatitis

 C. Cholecystitis
 D. Gastric ulcer

9. 9. Question
While a female client is being prepared for discharge, the nasogastric (NG) feeding tube becomes clogged. To remedy this
problem and teach the client’s family how to deal with it at home, what should the nurse do?

 A. Irrigate the tube with warm water.

 B. Advance the tube into the intestine.

 C. Apply intermittent suction to the tube.

 D. Withdraw the obstruction with a 30-ml syringe.

10. 10. Question


A male client with pancreatitis complains of pain. The nurse expects the physician to prescribe meperidine (Demerol) instead of
morphine to relieve pain because:

 A. Meperidine provides a better, more prolonged analgesic effect.

 B. Morphine may cause spasms of Oddi’s sphincter.

 C. Meperidine is less addictive than morphine.

 D. Morphine may cause hepatic dysfunction.

11. 11. Question


Mandy, an adolescent girl is admitted to an acute care facility with severe malnutrition. After a thorough examination, the
physician diagnoses anorexia nervosa. When developing the plan of care for this client, the nurse is most likely to include which
nursing diagnosis?

 A. Hopelessness

 B. Powerlessness

 C. Chronic low self-esteem

 D. Deficient knowledge

12. 12. Question


Which diagnostic test would be used first to evaluate a client with upper GI bleeding?

 A. Endoscopy

 B. Upper GI series

 C. Hemoglobin (Hb) levels and hematocrit (HCT)

 D. Arteriography

13. 13. Question


A female client who has just been diagnosed with hepatitis A asks, “How could I have gotten this disease?” What is the
nurse’s best response?

 A. “You may have eaten contaminated restaurant food.”

 B. “You could have gotten it by using I.V. drugs.”


 C. “You must have received an infected blood transfusion.”

 D. “You probably got it by engaging in unprotected sex.”

14. 14. Question


When preparing a male client, age 51, for surgery to treat appendicitis, the nurse formulates a nursing diagnosis of Risk for
infection related to inflammation, perforation, and surgery. What is the rationale for choosing this nursing diagnosis?

 A. Obstruction of the appendix may increase venous drainage and cause the appendix to rupture.

 B. Obstruction of the appendix reduces arterial flow, leading to ischemia, inflammation, and rupture of the
appendix.

 C. The appendix may develop gangrene and rupture, especially in a middle-aged client.

 D. Infection of the appendix diminishes necrotic arterial blood flow and increases venous drainage.

15. 15. Question


A female client with hepatitis C develops liver failure and GI hemorrhage. The blood products that would most likely bring about
hemostasis in the client are:

 A. Whole blood and albumin.

 B. Platelets and packed red blood cells.

 C. Fresh frozen plasma and whole blood.

 D. Cryoprecipitate and fresh frozen plasma.

16. Question
To prevent gastroesophageal reflux in a male client with a hiatal hernia, the nurse should provide which of the following
discharge instructions?

 A. “Lie down after meals to promote digestion.”

 B. “Avoid coffee and alcoholic beverages.”

 C. “Take antacids with meals.”

 D. “Limit fluid intake with meals.”

17. Question
The nurse caring for a client with small bowel obstruction would plan to implement which nursing intervention first?

 A. Administering pain medication

 B. Obtaining a blood sample for laboratory studies

 C. Preparing to insert a nasogastric (NG) tube

 D. Administering I.V. fluids

16. 18. Question


A female client with dysphagia is being prepared for discharge. Which outcome indicates that the client is ready for discharge?

 A. The client doesn’t exhibit rectal tenesmus.

 B. The client is free from esophagitis and achalasia.


 C. The client reports diminished duodenal inflammation.

 D. The client has normal gastric structures.

17. 19. Question


A male client undergoes total gastrectomy. Several hours after surgery, the nurse notes that the client’s nasogastric (NG) tube has
stopped draining. How should the nurse respond?

 A. Notify the physician.

 B. Reposition the tube.

 C. Irrigate the tube.

 D. Increase the suction level.

18. 20. Question


What laboratory finding is the primary diagnostic indicator for pancreatitis?

 A. Elevated blood urea nitrogen (BUN)

 B. Elevated serum lipase

 C. Elevated aspartate aminotransferase (AST)

 D. Increased lactate dehydrogenase (LD)

19. 21. Question


A male client with cholelithiasis has a gallstone lodged in the common bile duct. When assessing this client, the nurse expects to
note:

 A. Yellow sclera

 B. Light amber urine

 C. Circumoral pallor

 D. Black, tarry stools

20. 22. Question


Nurse Hannah is teaching a group of middle-aged men about peptic ulcers. When discussing risk factors for peptic ulcers, the
nurse should mention:

 A. A sedentary lifestyle and smoking.

 B. A history of hemorrhoids and smoking.

 C. Alcohol abuse and a history of acute renal failure.

 D. Alcohol abuse and smoking.

21. 23. Question


While palpating a female client’s right upper quadrant (RUQ), the nurse would expect to find which of the following structures?

 A. Sigmoid colon

 B. Appendix
 C. Spleen

 D. Liver

22. 24. Question


A male client has undergone a colon resection. While turning him, wound dehiscence with evisceration occurs. The
nurse’s first response is to:

 A. Call the physician.

 B. Place saline-soaked sterile dressings on the wound.

 C. Take blood pressure and pulse.

 D. Pull the dehiscence closed.

23. 25. Question


The nurse is monitoring a female client receiving paregoric to treat diarrhea for drug interactions. Which drugs can produce
additive constipation when given with an opium preparation?

 A. Antiarrhythmic drugs

 B. Anticholinergic drugs

 C. Anticoagulant drugs

 D. Antihypertensive drugs

24. 26. Question


A male client is recovering from an ileostomy that was performed to treat inflammatory bowel disease. During discharge
teaching, the nurse should stress the importance of:

 A. Increasing fluid intake to prevent dehydration.

 B. Wearing an appliance pouch only at bedtime.

 C. Consuming a low-protein, high-fiber diet.

 D. Taking only enteric-coated medications.

25. 27. Question


The nurse is caring for a female client with active upper GI bleeding. What is the appropriate diet for this client during the first 24
hours after admission?

 A. Regular diet

 B. Skim milk

 C. Nothing by mouth

 D. Clear liquids

28. Question
A male client has just been diagnosed with hepatitis A. On assessment, the nurse expects to note:

 A. Severe abdominal pain radiating to the shoulder.

 B. Anorexia, nausea, and vomiting.


 C. Eructation and constipation.

 D. Abdominal ascites.

26. 29. Question


A female client with viral hepatitis A is being treated in an acute care facility. Because the client requires enteric precautions, the
nurse should:

 A. Place the client in a private room.

 B. Wear a mask when handling the client’s bedpan.

 C. Wash the hands after touching the client.

 D. Wear a gown when providing personal care for the client.

27. 30. Question


Which of the following factors can cause hepatitis A?

 A. Contact with infected blood.

 B. Blood transfusions with infected blood.

 C. Eating contaminated shellfish.

 D. Sexual contact with an infected person.

28. 31. Question


The correct sequence for abdominal assessment is:

 A. Inspection, percussion, palpation, auscultation.

 B. Inspection, auscultation, palpation, percussion.

 C. Inspection, palpation, auscultation, percussion.

 D. Inspection, percussion, auscultation, palpation.

29. 32. Question


Peritonitis can occur as a complication of:

 A. Septicemia

 B. Multiple organ failure

 C. Hypovolemic shock

 D. Peptic ulcer disease

30. 33. Question


A patient has become very depressed postoperatively after receiving a colostomy for GI cancer. He does not participate in his
colostomy care or looks at the stoma. An appropriate nursing diagnosis for this situation is:

 A. Ineffective Individual Coping

 B. Knowledge Deficit

 C. Impaired Adjustment
 D. Anxiety

31. 34. Question


Patients with esophageal varices would reveal the following assessment:

 A. Increased blood pressure

 B. Increased heart rate

 C. Decreased respiratory rate

 D. Increased urinary output

32. 35. Question


The nurse would anticipate using which medication if sclerotherapy has not been used?

 A. Neomycin

 B. Propranolol

 C. Vasopressin

 D. Cimetidine

33. 36. Question


The nurse must be alert for complications with Sengstaken-Blakemore intubation including:

 A. Pulmonary obstruction

 B. Pericardiectomy syndrome

 C. Pulmonary embolization

 D. Cor pulmonale

34. 37. Question


Peptic ulcer disease may be caused by which of the following?

 A. Helicobacter pylori

 B. Clostridium difficile

 C. Candida albicans

 D. Staphylococcus aureus

35. 38. Question


Pain control with peptic ulcer disease includes all of the following except:

 A. Promoting physical and emotional rest.

 B. Identifying stressful situations.

 C. Eating meals when desired.

 D. Administering medications that decrease gastric acidity.


36. 39. Question
Nitrosocarcinogen production can be inhibited with the intake of:

 A. Vitamin C

 B. Vitamin E

 C. Carbohydrates

 D. Fiber

37. 40. Question


The nurse can expect a 60-year old patient with ischemic bowel to report a history of:

 A. Diabetes mellitus

 B. Asthma

 C. Addison's Disease

 D. Cancer of the bowel

38. 41. Question


During the initial assessment of a patient post-endoscopy, the nurse notes absent bowel sounds, tachycardia, and abdominal
distention. The nurse would anticipate:

 A. Ischemic bowel

 B. Peritonitis

 C. Hypovolemic shock

 D. Perforated bowel

39. 42. Question


Which of the following tests can be useful as a diagnostic and therapeutic tool in the biliary system?

 A. Ultrasonography

 B. MRI

 C. Endoscopic retrograde cholangiopancreatography (ERCP)

 D. Computed tomography scan (CT scan)

40. 43. Question


To inhibit pancreatic secretions, which pharmacologic agent would you anticipate administering to a patient with acute
pancreatitis?

 A. Nitroglycerin

 B. Somatostatin

 C. Pancrelipase

 D. Pepcid
41. 44. Question
Your patient’s ABG reveals an acidic pH, an acidic CO2, and a normal bicarbonate level. Which of the following indicates this acid-
base disturbance?

 A. Respiratory acidosis

 B. Respiratory alkalosis

 C. Metabolic acidosis

 D. Metabolic alkalosis

42. 45. Question


A clinical manifestation of acute pancreatitis is epigastric pain. Your nursing intervention to facilitate relief of pain would place the
patient in a:

 A. Knee-chest position

 B. Semi-Fowler's position

 C. Recumbent position

 D. Low-Fowler's position

43. 46. Question


What assessment finding of a patient with acute pancreatitis would indicate a bluish discoloration around the umbilicus?

 A. Grey-Turner's sign

 B. Homan's sign

 C. Rovsing's sign

 D. Cullen's sign

44. 47. Question


A patient with severe cirrhosis of the liver develops hepatorenal syndrome. Which of the following nursing assessment data
would support this?

 A. Oliguria and azotemia

 B. Metabolic alkalosis

 C. Decreased urinary concentration

 D. Weight gain of less than 1 lb per week

45. 48. Question


Which phase of hepatitis would the nurse incur strict precautionary measures at?

 A. Icteric

 B. Non-icteric

 C. Post-icteric

 D. Pre-icteric

46. 49. Question


You are caring for Rona, a 35-year-old female in a hepatic coma. Which evaluation criteria would be the most appropriate?

 A. The patient demonstrates an increase in the level of consciousness.

 B. The patient exhibits improved skin integrity.

 C. The patient experiences no evident signs of bleeding.

 D. The patient verbalizes decreased episodes of pain.

47. 50. Question


What is the primary nursing diagnosis for a 4th to 10th-day postoperative liver transplant patient?

 A. Excess Fluid Volume

 B. Risk for Rejection

 C. Impaired Skin Integrity

 D. Decreased Cardiac Output

1. 1. Question
A patient with chronic alcohol abuse is admitted with liver failure. You closely monitor the patient’s blood pressure because of
which change that is associated with liver failure?

o A. Hypoalbuminemia

o B. Increased capillary permeability

o C. Abnormal peripheral vasodilation

o D. Excess renin release from the kidneys

2. 2. Question
You’re assessing the stoma of a patient with a healthy, well-healed colostomy. You expect the stoma to appear:

 A. Pale, pink and moist

 B. Red and moist

 C. Dark or purple-colored

 D. Dry and black

3. 3. Question
You’re caring for a patient with a sigmoid colostomy. The stool from this colostomy is:

 A. Formed
 B. Semisolid

 C. Semiliquid

 D. Watery

4. 4. Question
You’re advising a 21 y.o. with a colostomy who reports problems with flatus. What food should you recommend?

 A. Peas

 B. Cabbage

 C. Broccoli

 D. Yogurt

5. 5. Question
You have to teach ostomy self-care to a patient with a colostomy. You tell the patient to measure and cut the wafer:

 A. To the exact size of the stoma.

 B. About 1/16” larger than the stoma.

 C. About 1/8” larger than the stoma.

6. 6. Question
You’re performing an abdominal assessment on Brent who is 52 y.o. In which order do you proceed?

 A. Observation, percussion, palpation, auscultation

 B. Observation, auscultation, percussion, palpation

 C. Percussion, palpation, auscultation, observation

 D. Palpation, percussion, observation, auscultation

7. 7. Question
You’re doing preoperative teaching with Gertrude who has ulcerative colitis who needs surgery to create an ileoanal reservoir.
Which information do you include?

 A. A reservoir is created that exits through the abdominal wall.

 B. A second surgery is required 12 months after the first surgery.

 C. A permanent ileostomy is created.

 D. The surgery occurs in two stages.

8. 8. Question
You’re caring for Carin who has just had ileostomy surgery. During the first 24 hours post-op, how much drainage can you expect
from the ileostomy?

 A. 100 ml

 B. 500 ml

 C. 1500 ml
 D. 5000 ml

9. 9. Question
You’re preparing a teaching plan for a 27 y.o. named Jeff who underwent surgery to close a temporary ileostomy. Which
nutritional guidelines do you include in this plan?

 A. There is no need to change eating habits.

 B. Eat six small meals a day.

 C. Eat the largest meal in the evening.

 D. Restrict fluid intake.

10. 10. Question


Arthur has a family history of colon cancer and is scheduled to have a sigmoidoscopy. He is crying as he tells you, “I know that I
have colon cancer, too.” Which response is most therapeutic?

 A. “I know just how you feel.”

 B. “You seem upset.”

 C. “Oh, don’t worry about it, everything will be just fine.”

 D. “Why do you think you have cancer?”

11. 11. Question


You’re caring for Beth who underwent a Billroth II procedure (surgical removal of the pylorus and duodenum) for treatment of a
peptic ulcer. Which findings suggest that the patient is developing dumping syndrome, a complication associated with this
procedure?

 A. Flushed, dry skin.

 B. Headache and bradycardia.

 C. Dizziness and sweating.

 D. Dyspnea and chest pain.

12. 12. Question


You’re developing the plan of care for a patient experiencing dumping syndrome after a Billroth II procedure. Which dietary
instructions do you include?

 A. Omit fluids with meals.

 B. Increase carbohydrate intake.

 C. Decrease protein intake.

 D. Decrease fat intake.

13. 13. Question


You’re caring for Lewis, a 67 y.o. patient with liver cirrhosis who developed ascites and requires paracentesis. Relief of which
symptom indicates that the paracentesis was effective?

 A. Pruritus

 B. Dyspnea
 C. Jaundice

 D. Peripheral Neuropathy

14. 14. Question


You’re caring for Jane, a 57 y.o. patient with liver cirrhosis who developed ascites and requires paracentesis. Before her
paracentesis, you instruct her to:

 A. Empty her bladder.

 B. Lie supine in bed.

 C. Remain NPO for 4 hours.

 D. Clean her bowels with an enema.

15. 15. Question


After abdominal surgery, your patient has a severe coughing episode that causes wound evisceration. In addition to calling the
doctor, which intervention is most appropriate?

 A. Irrigate the wound & organs with Betadine.

 B. Cover the wound with a saline-soaked sterile dressing.

 C. Apply a dry sterile dressing & binder.

 D. Push the organs back & cover with moist sterile dressings.

16. 16. Question


You’re caring for Betty with liver cirrhosis. Which of the following assessment findings leads you to suspect hepatic
encephalopathy in her?

 A. Asterixis

 B. Chvostek’s sign

 C. Trousseau’s sign

 D. Hepatojugular reflux

17. 17. Question


You are developing a care plan for Sally, a 67 y.o. patient with hepatic encephalopathy. Which of the following do you include?

 A. Administering a lactulose enema as ordered.

 B. Encouraging a protein-rich diet.

 C. Administering sedatives, as necessary.

 D. Encouraging ambulation at least four times a day.

18. 18. Question


You have a patient with achalasia (incomplete muscle relaxation of the GI tract, especially sphincter muscles). Which medications
do you anticipate to administer?

 A. isosorbide dinitrate (Isordil)

 B. digoxin (Lanoxin)
 C. captopril (Capoten)

 D. propranolol (Inderal)

19. 19. Question


The student nurse is preparing a teaching care plan to help improve nutrition in a patient with achalasia. You include which of the
following:

 A. Swallow foods while leaning forward.

 B. Omit fluids at mealtimes.

 C. Eat meals sitting upright.

 D. Avoid soft and semi soft foods.

20. 20. Question


Britney, a 20 y.o. student is admitted with acute pancreatitis. Which laboratory findings do you expect to be abnormal for this
patient?

 A. Serum creatinine and BUN

 B. Alanine aminotransferase (ALT) and aspartate aminotransferase (AST)

 C. Serum amylase and lipase

 D. Cardiac enzymes

21. 21. Question


A patient with Crohn’s disease is admitted after 4 days of diarrhea. Which of the following urine-specific gravity values do you
expect to find in this patient?

 A. 1.005

 B. 1.011

 C. 1.020

 D. 1.030

22. 22. Question


Your goal is to minimize David’s risk of complications after a herniorrhaphy. You instruct the patient to:

 A. Avoid the use of pain medication.

 B. Cough and deep breath Q2H.

 C. Splint the incision if he can’t avoid sneezing or coughing.

 D. Apply heat to scrotal swelling.

23. 23. Question


Janice is waiting for discharge instructions after her herniorrhaphy. Which of the following instructions do you include?

 A. Eat a low-fiber diet.

 B. Resume heavy lifting in 2 weeks.


 C. Lose weight, if obese.

 D. Resume sexual activity once discomfort is gone.

24. 24. Question


Develop a teaching care plan for Angie who is about to undergo a liver biopsy. Which of the following points do you include?

 A. “You’ll need to lie on your stomach during the test.”

 B. “You’ll need to lie on your right side after the test.”

 C. “During the biopsy, you’ll be asked to exhale deeply and hold it.”

 D. “The biopsy is performed under general anesthesia.”

25. 25. Question


Stephen is a 62 y.o. patient that has had a liver biopsy. Which of the following groups of signs alert you to a possible
pneumothorax?

 A. Dyspnea and reduced or absent breath sound over the right lung.

 B. Tachycardia, hypotension, and cool, clammy skin.

 C. Fever, rebound tenderness, and abdominal rigidity.

 D. Redness, warmth, and drainage at the biopsy site.

26. 26. Question


Michael, 42 y.o. The man is admitted to the med-surg floor with a diagnosis of acute pancreatitis. His BP is 136/76, pulse 96,
Resps 22, and temp 101. His past history includes hyperlipidemia and alcohol abuse. The doctor prescribes an NG tube. Before
inserting the tube, you explain the purpose to the patient. Which of the following is the most accurate explanation?

 A. “It empties the stomach of fluids and gas.”

 B. “It prevents spasms at the sphincter of Oddi.”

 C. “It prevents air from forming in the small intestine and large intestine.”

 D. “It removes bile from the gallbladder.”

27. 27. Question


Jason, a 22 y.o. accident victim, requires an NG tube for feeding. What should you immediately do after inserting an NG tube for
liquid enteral feedings?

 A. Aspirate for gastric secretions with a syringe.

 B. Begin feeding slowly to prevent cramping.

 C. Get an X-ray of the tip of the tube within 24 hours.

 D. Clamp off the tube until the feedings begin.

28. Question
Stephanie, a 28 y.o. accident victim, requires TPN. The rationale for TPN is to provide:

 A. Necessary fluids and electrolytes to the body.

 B. Complete nutrition by the I.V. route.


 C. Tube feedings for nutritional supplementation.

 D. Dietary supplementation with liquid protein given between meals.

28. 29. Question


Type A chronic gastritis can be distinguished from type B by its ability to:

 A. Cause atrophy of the parietal cells.

 B. Affect only the antrum of the stomach.

 C. Thin the lining of the stomach walls.

 D. Decrease gastric secretions.

29. 30. Question


Matt is a 49 y.o. with a hiatal hernia that you are about to counsel. Health care counseling for Matt should include which of the
following instructions?

 A. Restrict intake of high-carbohydrate foods.

 B. Increase fluid intake with meals.

 C. Increase fat intake.

 D. Eat three regular meals a day.

30. 31. Question


Jerod is experiencing an acute episode of ulcerative colitis. Which is the priority for this patient?

 A. Replace lost fluid and sodium.

 B. Monitor for increased serum glucose level from steroid therapy.

 C. Restrict the dietary intake of foods high in potassium.

 D. Note any change in the color and consistency of stools.

32. Question
A 29 y.o. patient has an acute episode of ulcerative colitis. What diagnostic test confirms this diagnosis?

 A. Barium Swallow

 B. Stool examination

 C. Gastric analysis

 D. Sigmoidoscopy

31. 33. Question


Eleanor, a 62 y.o. woman with diverticulosis is your patient. Which interventions would you expect to include in her care?

 A. Low-fiber diet and fluid restrictions.

 B. Total parenteral nutrition and bed rest.

 C. High-fiber diet and administration of psyllium.


 D. Administration of analgesics and antacids.

32. 34. Question


Regina is a 46 y.o. woman with ulcerative colitis. You expect her stools to look like:

 A. Watery and frothy

 B. Bloody and mucous

 C. Firm and well-formed

 D. Alternating constipation and diarrhea

33. 35. Question


Donald is a 61 y.o. man with diverticulitis. Diverticulitis is characterized by:

 A. Periodic rectal hemorrhage.

 B. Hypertension and tachycardia.

 C. Vomiting and elevated temperature.

 D. Crampy and lower left quadrant pain and low-grade fever.

34. 36. Question


Brenda, a 36 y.o. patient is on your floor with acute pancreatitis. Treatment for her includes:

 A. Continuous peritoneal lavage.

 B. Regular diet with increased fat.

 C. Nutritional support with TPN.

 D. Insertion of a T tube to drain the pancreas.

35. 37. Question


Glenda has cholelithiasis (gallstones). You expect her to complain of:

 A. Pain in the right upper quadrant, radiating to the shoulder.

 B. Pain in the right lower quadrant, with rebound tenderness.

 C. Pain in the left upper quadrant, with shortness of breath.

 D. Pain in the left lower quadrant, with mild cramping.

36. 38. Question


After an abdominal resection for colon cancer, Madeline returns to her room with a Jackson-Pratt drain in place. The purpose of
the drain is to:

 A. Irrigate the incision with a saline solution.

 B. Prevent bacterial infection of the incision.

 C. Measure the amount of fluid lost after surgery.

 D. Prevent accumulation of drainage in the wound.


37. 39. Question
Anthony, a 60 y.o. patient, has just undergone a bowel resection with a colostomy. During the first 24 hours, which of the
following observations about the stoma should you report to the doctor?

 A. Pink color

 B. Light edema

 C. Small amount of oozing

 D. Trickles of bright red blood

38. 40. Question


You’re teaching Anthony how to use his new colostomy. How much skin should remain exposed between the stoma and the ring
of the appliance?

 1/16”

 1/4”

 1/2”

 1”

39. 41. Question


Claire, a 33 y.o. is on your floor with a possible bowel obstruction. Which intervention is a priority for her?

 A. Obtain daily weights.

 B. Measure abdominal girth.

 C. Keep strict intake and output.

 D. Encourage her to increase fluids.

40. 42. Question


Your patient has a GI tract that is functioning, but has the inability to swallow foods. Which is the preferred method of feeding for
your patient?

 A. TPN

 B. PPN

 C. NG feeding

 D. Oral liquid supplements

41. 43. Question


Your patient is complaining of abdominal pain during assessment. What is your priority?

 A. Auscultate to determine changes in bowel sounds.

 B. Observe the contour of the abdomen.

 C. Palpate the abdomen for a mass.

 D. Percuss the abdomen to determine if fluid is present.

42. 44. Question


Before bowel surgery, Lee is to administer enemas until clear. During administration, he complains of intestinal cramps. What do
you do next?

 A. Discontinue the procedure.

 B. Lower the height of the enema container.

 C. Complete the procedure as quickly as possible.

 D. Continue administration of the enema as ordered without making any adjustments.

43. 45. Question


Leigh Ann is receiving pancrelipase (Viokase) for chronic pancreatitis. Which observation best indicates the treatment is effective?

 A. There is no skin breakdown.

 B. Her appetite improves.

 C. She lost more than 10 lbs.

 D. Stools are less fatty and decrease in frequency.

44. 46. Question


Ralph has a history of alcohol abuse and has acute pancreatitis. Which lab value is most likely to be elevated?

 A. Calcium

 B. Glucose

 C. Magnesium

 D. Potassium

45. 47. Question


Anna is 45 y.o. and has a bleeding ulcer. Despite multiple blood transfusions, her HGB is 7.5g/dl and HCT is 27%. Her doctor
determines that surgical intervention is necessary and she undergoes partial gastrectomy. Postoperative nursing care includes:

 A. Giving pain medication Q6H.

 B. Flushing the NG tube with sterile water.

 C. Positioning her in high Fowler’s position.

 D. Keeping her NPO until the return of peristalsis.

46. 48. Question


Sitty, a 66 y.o. patient underwent a colostomy for a ruptured diverticulum. She did well during the surgery and returned to your
med-surg floor in stable condition. You assess her colostomy 2 days after surgery. Which finding do you report to the doctor?

 A. Blanched stoma

 B. Edematous stoma

 C. Reddish-pink stoma

 D. Brownish-black stoma
47. 49. Question
Sharon has cirrhosis of the liver and develops ascites. What intervention is necessary to decrease the excessive accumulation of
serous fluid in her peritoneal cavity?

 A. Restrict fluids.

 B. Encourage ambulation.

 C. Increase sodium in the diet.

 D. Give antacids as prescribed.

48. 50. Question


Katrina is diagnosed with lactose intolerance. To avoid complications with lack of calcium in the diet, which food should be
included in the diet?

 A. Fruit

 B. Whole grains

 C. Milk and cheese products

 D. Dark green, leafy vegetables

1. 1. Question
Which of the following complications is thought to be the most common cause of appendicitis?

o A. A fecalith

o B. Bowel kinking

o C. Internal bowel occlusion

o D. Abdominal bowel swelling

2. 2. Question
Which of the following terms best describes the pain associated with appendicitis?

 A. Aching

 B. Fleeting

 C. Intermittent

 D. Steady

3. 3. Question
Which of the following nursing interventions should be implemented to manage a client with appendicitis?

 A. Assessing pain.

 B. Encouraging oral intake of clear fluids.


 C. Providing discharge teaching.

 D. Assessing for symptoms of peritonitis.

4. 4. Question
Which of the following definitions best describes gastritis?

 A. Erosion of the gastric mucosa.

 B. Inflammation of a diverticulum.

 C. Inflammation of the gastric mucosa.

 D. Reflux of stomach acid into the esophagus.

5. 5. Question
Which of the following substances is most likely to cause gastritis?

 A. Milk

 B. Bicarbonate of soda, or baking soda

 C. Enteric-coated aspirin

 D. Nonsteroidal anti-inflammatory drugs

6. 6. Question
Which of the following definitions best describes diverticulosis?

 A. An inflamed outpouching of the intestine.

 B. A noninflamed outpouching of the intestine.

 C. The partial impairment of the forward flow of intestinal contents.

 D. An abnormal protrusion of an organ through the structure that usually holds it.

7. 7. Question
Which of the following types of diets is implicated in the development of diverticulosis?

 A. Low-fiber diet

 B. High-fiber diet

 C. High-protein diet

 D. Low-carbohydrate diet

8. 8. Question
Which of the following mechanisms can facilitate the development of diverticulosis into diverticulitis?

 A. Treating constipation with chronic laxative use, leading to dependence on laxatives.

 B. Chronic constipation causing an obstruction, reducing forward flow of intestinal contents.

 C. Herniation of the intestinal mucosa, rupturing the wall of the intestine.

 D. Undigested food blocking the diverticulum, predisposing the area to bacterial invasion.
9. 9. Question
Which of the following symptoms indicated diverticulosis?

 A. No symptoms exist.

 B. Change in bowel habits.

 C. Anorexia with low-grade fever.

 D. Episodic, dull, or steady midabdominal pain.

10. 10. Question


Which of the following tests should be administered to a client suspected of having diverticulosis?

 A. Abdominal ultrasound

 B. Barium enema

 C. Barium swallow

 D. Gastroscopy

11. 11. Question


Medical management of the client with diverticulitis should include which of the following treatments?

 A. Reduced fluid intake.

 B. Increased fiber in the diet.

 C. Administration of antibiotics.

 D. Exercises to increase intra-abdominal pressure.

12. 12. Question


Crohn’s disease can be described as a chronic relapsing disease. Which of the following areas in the GI system may be involved
with this disease?

 A. The entire length of the large colon.

 B. Only the sigmoid area.

 C. The entire large colon through the layers of mucosa and submucosa.

 D. The small intestine and colon; affecting the entire thickness of the bowel.

13. Question
Which area of the alimentary canal is the most common location for Crohn’s disease?

 A. Ascending colon

 B. Descending colon

 C. Sigmoid colon

 D. Terminal ileum

13. 14. Question


Which of the following factors is believed to be linked to Crohn’s disease?

 A. Constipation

 B. Diet

 C. Hereditary

 D. Lack of exercise

14. 15. Question


Which of the following factors is believed to cause ulcerative colitis?

 A. Acidic diet

 B. Altered immunity

 C. Chronic constipation

 D. Emotional stress

15. 16. Question


Fistulas are most common with which of the following bowel disorders?

 A. Crohn’s disease

 B. Diverticulitis

 C. Diverticulosis

 D. Ulcerative colitis

16. 17. Question


Which of the following areas is the most common site of fistulas in clients with Crohn’s disease?

 A. Anorectal

 B. Ileum

 C. Rectovaginal

 D. Transverse colon

17. 18. Question


Which of the following associated disorders may a client with ulcerative colitis exhibit?

 A. Gallstones

 B. Hydronephrosis

 C. Nephrolithiasis

 D. Toxic megacolon

18. 19. Question


Which of the following associated disorders may the client with Crohn’s disease exhibit?

 A. Ankylosing spondylitis
 B. Colon cancer

 C. Malabsorption

 D. Lactase deficiency

19. 20. Question


Which of the following symptoms may be exhibited by a client with Crohn’s disease?

 A. Bloody diarrhea

 B. Narrow stools

 C. N/V

 D. Steatorrhea

20. 21. Question


Which of the following symptoms is associated with ulcerative colitis?

 A. Dumping syndrome

 B. Rectal bleeding

 C. Soft stools

 D. Fistulas

21. 22. Question


If a client had irritable bowel syndrome, which of the following diagnostic tests would determine if the diagnosis is Crohn’s
disease or ulcerative colitis?

 A. Abdominal computed tomography (CT) scan

 B. Abdominal x-ray

 C. Barium swallow

 D. Colonoscopy with biopsy

22. 23. Question


Which of the following interventions should be included in the medical management of Crohn’s disease?

 A. Increasing oral intake of fiber.

 B. Administering laxatives.

 C. Using long-term steroid therapy.

 D. Increasing physical activity.

23. 24. Question


In a client with Crohn’s disease, which of the following symptoms should not be a direct result of antibiotic therapy?

 A. Decrease in bleeding.

 B. Decrease in temperature.
 C. Decrease in body weight.

 D. Decrease in the number of stools.

24. 25. Question


Surgical management of ulcerative colitis may be performed to treat which of the following complications?

 A. Gastritis

 B. Bowel herniation

 C. Bowel outpouching

 D. Bowel perforation

25. 26. Question


Which of the following medications is most effective for treating the pain associated with irritable bowel disease?

 A. Acetaminophen

 B. Opiates

 C. Steroids

 D. Stool softeners

26. 27. Question


During the first few days of recovery from ostomy surgery for ulcerative colitis, which of the following aspects should be
the first priority of client care?

 A. Body image

 B. Ostomy care

 C. Sexual concerns

 D. Skin care

27. 28. Question


Colon cancer is most closely associated with which of the following conditions?

 A. Appendicitis

 B. Hemorrhoids

 C. Hiatal hernia

 D. Ulcerative colitis

29. Question
Which of the following diets is most commonly associated with colon cancer?

 A. Low-fiber, high fat

 B. Low-fat, high-fiber
 C. Low-protein, high-carbohydrate

 D. Low carbohydrate, high protein

28. 30. Question


Which of the following diagnostic tests should be performed annually over age 50 to screen for colon cancer?

 A. Abdominal CT scan

 B. Abdominal x-ray

 C. Colonoscopy

 D. Fecal occult blood test

29. 31. Question


Radiation therapy is used to treat colon cancer before surgery for which of the following reasons?

 A. Reducing the size of the tumor.

 B. Eliminating the malignant cells.

 C. Curing cancer.

 D. Helping the bowel heal after surgery.

30. 32. Question


Which of the following symptoms is a client with colon cancer most likely to exhibit?

 A. A change in appetite.

 B. A change in bowel habits.

 C. An increase in body weight.

 D. An increase in body temperature.

31. 33. Question


A client has just had surgery for colon cancer. Which of the following disorders might the client develop?

 A. Peritonitis

 B. Diverticulosis

 C. Partial bowel obstruction

 D. Complete bowel obstruction

32. 34. Question


A client with gastric cancer may exhibit which of the following symptoms?

 A. Abdominal cramping

 B. Constant hunger

 C. Feeling of fullness

 D. Weight gain
33. 35. Question
Which of the following diagnostic tests may be performed to determine if a client has gastric cancer?

 A. Barium enema

 B. Colonoscopy

 C. Gastroscopy

 D. Serum chemistry levels

34. 36. Question


A client with gastric cancer can expect to have surgery for resection. Which of the following should be the nursing management
priority for the preoperative client with gastric cancer?

 A. Discharge planning

 B. Correction of nutritional deficits

 C. Prevention of DVT

 D. Instruction regarding radiation treatment

35. 37. Question


Care for the postoperative client after gastric resection should focus on which of the following problems?

 A. Body image

 B. Nutritional needs

 C. Skincare

 D. Spiritual needs

36. 38. Question


Which of the following complications of gastric resection should the nurse teach the client to watch for?

 A. Constipation

 B. Dumping syndrome

 C. Gastric spasm

 D. Intestinal spasms

37. 39. Question


A client with rectal cancer may exhibit which of the following symptoms?

 A. Abdominal fullness

 B. Gastric fullness

 C. Rectal bleeding

 D. Right upper quadrant pain

38. 40. Question


A client with which of the following conditions may be likely to develop rectal cancer?

 A. Adenomatous polyps

 B. Diverticulitis

 C. Hemorrhoids

 D. Peptic ulcer disease

39. 41. Question


Which of the following treatments is used for rectal cancer but not for colon cancer?

 A. Chemotherapy

 B. Colonoscopy

 C. Radiation

 D. Surgical resection

40. 42. Question


Which of the following conditions is most likely to directly cause peritonitis?

 A. Cholelithiasis

 B. Gastritis

 C. Perforated ulcer

 D. Incarcerated hernia

43. Question
Which of the following symptoms would a client in the early stages of peritonitis exhibit?

 A. Abdominal distention

 B. Abdominal pain and rigidity

 C. Hyperactive bowel sounds

 D. Right upper quadrant pain

41. 44. Question

Which of the following laboratory results would be expected in a client with peritonitis?

 A. Partial thromboplastin time above 100 seconds

 B. Hemoglobin level below 10 mg/dL

 C. Potassium level above 5.5 mEq/L

 D. White blood cell count above 15,000

42. 45. Question


Which of the following therapies is not included in the medical management of a client with peritonitis?
 A. Broad-spectrum antibiotics

 B. Electrolyte replacement

 C. I.V. fluids

 D. Regular diet

43. 46. Question


Which of the following aspects is the priority focus of nursing management for a client with peritonitis?

 A. Fluid and electrolyte balance

 B. Gastric irrigation

 C. Pain management

 D. Psychosocial issues

44. 47. Question


A client with irritable bowel syndrome is being prepared for discharge. Which of the following meal plans should the nurse give
the client?

 A. Low fiber, low-fat

 B. High fiber, low-fat

 C. Low fiber, high-fat

 D. High-fiber, high-fat

45. 48. Question


A client presents to the emergency room, reporting that he has been vomiting every 30 to 40 minutes for the past 8 hours.
Frequent vomiting puts him at risk for which of the following?

 A. Metabolic acidosis with hyperkalemia

 B. Metabolic acidosis with hypokalemia

 C. Metabolic alkalosis with hyperkalemia

 D. Metabolic alkalosis with hypokalemia

46. 49. Question


Five days after undergoing surgery, a client develops a small bowel obstruction. A Miller-Abbott tube is inserted for bowel
decompression. Which nursing diagnosis takes priority?

 A. Imbalanced nutrition: Less than body requirements

 B. Acute pain

 C. Deficient fluid volume

 D. Excess fluid volume

47. 50. Question


When teaching an elderly client how to prevent constipation, which of the following instructions should the nurse include?
 A. “Drink 6 glasses of fluid each day.”

 B. “Avoid grain products and nuts.”

 C. “Add at least 4 grams of bran to your cereal each morning.”

 D. “Be sure to get regular exercise.”

1. 1. Question
In a client with diarrhea, which outcome indicates that fluid resuscitation is successful?

o A. The client passes formed stools at regular intervals.

o B. The client reports a decrease in stool frequency and liquidity.

o C. The client exhibits firm skin turgor.

o D. The client no longer experiences perianal burning.

2. 2. Question
When teaching a community group about measures to prevent colon cancer, which instruction should the nurse include?

 A. “Limit fat intake to 20% to 25% of your total daily calories.”

 B. “Include 15 to 20 grams of fiber into your daily diet.”

 C. “Get an annual rectal examination after age 35.”

 D. “Undergo sigmoidoscopy annually after age 50.”

3. 3. Question
A 30-year old client experiences weight loss, abdominal distention, crampy abdominal pain, and intermittent diarrhea after the
birth of her 2nd child. Diagnostic tests reveal gluten-induced enteropathy. Which foods must she eliminate from her diet
permanently?

 A. Milk and dairy products

 B. Protein-containing foods

 C. Cereal grains (except rice and corn)

 D. Carbohydrates

4. 4. Question
After a right hemicolectomy for treatment of colon cancer, a 57-year old client is reluctant to turn while on bed rest. Which action
by the nurse would be appropriate?

 A. Asking a coworker to help turn the client.

 B. Explaining to the client why turning is important.


 C. Allowing the client to turn when he’s ready to do so.

 D. Telling the client that the physician’s order states he must turn every 2 hours.

5. 5. Question
A client has a percutaneous endoscopic gastrostomy tube inserted for tube feedings. Before starting a continuous feeding, the
nurse should place the client in which position?

 A. Semi-Fowlers

 B. Supine

 C. Reverse Trendelenburg

 D. High Fowler’s

6. 6. Question
An enema is prescribed for a client with suspected appendicitis. Which of the following actions should the nurse take?

 A. Prepare 750 ml of irrigating solution warmed to 100° F.

 B. Question the physician about the order.

 C. Provide privacy and explain the procedure to the client.

 D. Assist the client to the left lateral Sims position.

7. 7. Question
The client being seen in a physician’s office has just been scheduled for a barium swallow the next day. The nurse writes down
which of the following instructions for the client to follow before the test?

 A. Fast for 8 hours before the test.

 B. Eat a regular supper and breakfast.

 C. Continue to take all oral medications as scheduled.

 D. Monitor own bowel movement pattern for constipation.

8. 8. Question
The nurse is monitoring a client for the early signs of dumping syndrome. Which symptom indicates this occurrence?

 A. Abdominal cramping and pain

 B. Bradycardia and indigestion

 C. Sweating and pallor

 D. Double vision and chest pain

9. 9. Question
The nurse is preparing a discharge teaching plan for the client who had an umbilical hernia repair. Which of the following would
the nurse include in the plan?

 A. Restricting pain medication.

 B. Maintaining bedrest.
 C. Avoiding coughing.

 D. Irrigating the drain.

10. 10. Question


The nurse is caring for a hospitalized client with a diagnosis of ulcerative colitis. Which finding, if noted on assessment of the
client, would the nurse report to the physician?A. Bloody diarrhea

 A. Bloody diarrhea

 B. Hypotension

 C. A hemoglobin of 12 mg/dL

 D. Rebound tenderness

11. 11. Question


The nurse is reviewing the record of a client with Crohn’s disease. Which of the following stool characteristics would the nurse
expect to note documented on the client’s record?

 A. Chronic constipation

 B. Diarrhea

 C. Constipation alternating with diarrhea.

 D. Stool constantly oozes from the rectum.

12. 12. Question


The nurse is performing colostomy irrigation on a client. During the irrigation, a client begins to complain of abdominal cramps.
Which of the following is the most appropriate nursing action?

 A. Notify the physician.

 B. Increase the height of the irrigation.

 C. Stop the irrigation temporarily.

 D. Medicate with dilaudid and resume the irrigation.

13. 13. Question


The nurse is teaching the client how to perform a colostomy irrigation. To enhance the effectiveness of the irrigation and fecal
returns, what measure should the nurse instruct the client to do?

 A. Increase fluid intake.

 B. Reduce the amount of irrigation solution.

 C. Perform the irrigation in the evening.

 D. Place heat on the abdomen.

14. 14. Question


The nurse is reviewing the physician’s orders written for a client admitted with acute pancreatitis. Which physician order would
the nurse question if noted on the client’s chart?

 A. NPO status
 B. Insert a nasogastric tube

 C. An anticholinergic medication

 D. Morphine for pain

15. 15. Question


The nurse is doing an admission assessment on a client with a history of duodenal ulcer. To determine whether the problem is
currently active, the nurse would assess the client for which of the following most frequent symptom(s) of duodenal ulcer?

 A. Pain that is relieved by food intake.

 B. Pain that radiates down the right arm.

 C. N/V

 D. Weight loss

16. 16. Question


The nurse instructs the ileostomy client to do which of the following as a part of essential care of the stoma?

 A. Cleanse the peristomal skin meticulously.

 B. Take in high-fiber foods such as nuts.

 C. Massage the area below the stoma.

 D. Limit fluid intake to prevent diarrhea.

17. 17. Question


The client who has undergone the creation of a colostomy has a nursing diagnosis of Disturbed body image. The nurse would
evaluate that the client is making the most significant progress toward identified goals if the client:

 A. Watch the nurse empty the colostomy bag.

 B. Look at the ostomy site.

 C. Read the ostomy product literature.

 D. Practice cutting the ostomy appliance.

18. 18. Question


The nurse is assessing for stoma prolapse in a client with a colostomy. The nurse would observe which of the following if stoma
prolapse occurred?

 A. Sunken and hidden stoma

 B. Dark- and bluish-colored stoma

 C. Narrowed and flattened stoma

 D. Protruding stoma

19. 19. Question


The client with a new colostomy is concerned about the odor from the stool in the ostomy drainage bag. The nurse teaches the
client to include which of the following foods in the diet to reduce odor?

 A. Yogurt
 B. Broccoli

 C. Cucumbers

 D. Eggs

20. 20. Question


The nurse has given instructions to the client with an ileostomy about foods to eat to thicken the stool. The nurse determines that
the client needs further instructions if the client starts to eat which of the following foods to make the stools less watery?

 A. Pasta

 B. Boiled rice

 C. Bran

 D. Low-fat cheese

21. 21. Question


The client has just had surgery to create an ileostomy. The nurse assesses the client in the immediate post-op period for which of
the following most frequent complications of this type of surgery?

 A. Intestinal obstruction

 B. Fluid and electrolyte imbalance

 C. Malabsorption of fat

 D. Folate deficiency

22. 22. Question


The nurse is doing pre-op teaching with the client who is about to undergo the creation of a Kock pouch. The nurse interprets
that the client has the best understanding of the nature of the surgery if the client makes which of the following statements?

 A. “I will need to drain the pouch regularly with a catheter.”

 B. “I will need to wear a drainage bag for the rest of my life.”

 C. “The drainage from this type of ostomy will be formed.”

 D. “I will be able to pass stool from my rectum eventually.”

23. 23. Question


The client with a colostomy has an order for irrigation of the colostomy. The nurse used which solution for irrigation?A. Distilled
water

 A. Distilled water

 B. Tap water

 C. Sterile water

 D. Lactated Ringer’s

24. 24. Question


A nurse is monitoring a client admitted to the hospital with a diagnosis of appendicitis. The client is scheduled for surgery in 2
hours. The client begins to complain of increased abdominal pain and begins to vomit. On assessment, the nurse notes that the
abdomen is distended and the bowel sounds are diminished. Which of the following is the most appropriate nursing
intervention?
 A. Administer dilaudid.

 B. Notify the physician.

 C. Call and ask the operating room team to perform the surgery as soon as possible.

 D. Reposition the client and apply a heating pad in a warm setting to the client’s abdomen.

25. 25. Question


The client has been admitted with a diagnosis of acute pancreatitis. The nurse would assess this client for pain that is:

 A. Severe and unrelenting, located in the epigastric area and radiating to the back.

 B. Severe and unrelenting, located in the left lower quadrant and radiating to the groin.

 C. Burning and aching, located in the epigastric area and radiating to the umbilicus.

 D. Burning and aching, located in the left lower quadrant and radiating to the hip.

26. 26. Question


The client with Crohn’s disease has a nursing diagnosis of acute pain. The nurse would teach the client to avoid which of the
following in managing this problem?

 A. Lying supine with the legs straight.

 B. Massaging the abdomen.

 C. Using antispasmodic medication.

 D. Using relaxation techniques.

27. 27. Question


A client with ulcerative colitis has an order to begin salicylate medication to reduce inflammation. The nurse instructs the client to
take the medication:

 A. 30 minutes before meals

 B. On an empty stomach

 C. After meals

 D. On arising

28. 28. Question


During the assessment of a client’s mouth, the nurse notes the absence of saliva. The client is also complaining of pain near the
area of the ear. The client has been NPO for several days because of the insertion of a NG tube. Based on these findings, the
nurse suspects that the client is developing which of the following mouth conditions?

 A. Stomatitis

 B. Oral candidiasis

 C. Parotitis

 D. Gingivitis

29. 29. Question


The nurse evaluates the client’s stoma during the initial post-op period. Which of the following observations should be
reported immediately to the physician?
 A. The stoma is slightly edematous.

 B. The stoma is dark red to purple.

 C. The stoma oozes a small amount of blood.

 D. The stoma does not expel stool.

30. 30. Question


When planning care for a client with ulcerative colitis who is experiencing symptoms, which client care activities can the nurse
appropriately delegate to an unlicensed assistant? Select all that apply.

 A. Assessing the client’s bowel sounds.

 B. Providing skincare following bowel movements.

 C. Evaluating the client’s response to antidiarrheal medications.

 D. Maintaining intake and output records.

 E. Obtaining the client’s weight.

31. 31. Question


Which goal of the client’s care should take priority during the first days of hospitalization for an exacerbation of ulcerative colitis?

 A. Promoting self-care and independence.

 B. Managing diarrhea.

 C. Maintaining adequate nutrition.

 D. Promoting rest and comfort.

32. 32. Question


A client’s ulcerative colitis symptoms have been present for longer than 1 week. The nurse recognizes that the client should be
assessed carefully for signs of which of the following complications?

 A. Heart failure

 B. DVT

 C. Hypokalemia

 D. Hypocalcemia

33. 33. Question


A client who has ulcerative colitis has persistent diarrhea. He is thin and has lost 12 pounds since the exacerbation of his
ulcerative colitis. The nurse should anticipate that the physician will order which of the following treatment approaches to help
the client meet his nutritional needs?

 A. Initiate continuous enteral feedings.

 B. Encourage a high protein, high-calorie diet.

 C. Implement total parenteral nutrition.

 D. Provide six small meals a day.

34. 34. Question


Digoxin preparations and absorbents should not be given simultaneously. As a nurse, you are aware that if these agents are given
simultaneously, which of the following will occur?

 A. Increased absorption of digoxin.

 B. Decreased absorption of digoxin.

 C. Increased absorption of the absorbent.

 D. Decreased absorption of the absorbent.

35. 35. Question


When used with hyperacidic disorders of the stomach, antacids are given to elevate the gastric pH to:

 A. 2.0

 B. 4.0

 C. 6.0

 D. >8.0

36. 36. Question


One of your patients is receiving digitalis orally and is also to receive an antacid at the same time. Your most appropriate action,
based on the pharmacokinetics of antacids, is to:

 A. Delay the digitalis for 1 to 2 hours until the antacid is absorbed.

 B. Give the antacid at least 2 to 4 hours before administering the digitalis.

 C. Administer both medications as ordered and documented in the nurse’s notes.

 D. Contact the physician regarding the drug interaction and request a change in the time of dosing of the drugs.

37. 37. Question


The nurse would teach patients that antacids are effective in the treatment of hyperacidity because they:

 A. Neutralize gastric acid.

 B. Decrease stomach motility.

 C. Decrease gastric pH.

 D. Decrease duodenal pH.

38. 38. Question


The nurse would monitor for which of the following adverse reactions to aluminum-containing antacids such as aluminum
hydroxide (Amphojel)?

 A. Diarrhea

 B. Constipation

 C. GI upset

 D. Fluid retention

39. 39. Question


The nurse would question an order for which type of antacid in patients with chronic renal failure?
 A. Aluminum-containing antacids

 B. Calcium-containing antacids

 C. Magnesium-containing antacids

 D. All of the above.

40. 40. Question


The nurse would monitor a patient using sodium bicarbonate to treat gastric hyperacidity for signs and symptoms of:A.
Metabolic alkalosis

 A. Metabolic alkalosis

 B. Metabolic acidosis

 C. Hyperkalemia

 D. Hypercalcemia

41. 41. Question


Which of the following nursing diagnoses is appropriate for a patient receiving famotidine (Pepcid)?

 A. Increased risk for infection due to immunosuppression.

 B. Potential risk for bleeding related to thrombocytopenia.

 C. Alteration in urinary elimination related to retention.

 D. Alteration in tissue perfusion related to hypertension.

42. 42. Question


Histamine2-receptor antagonists:

 A. Compete with histamine for binding sites on the parietal cells.

 B. Irreversibly bind to H+/K+ATPase.

 C. Cause a decrease in stomach pH.

 D. Decrease signs and symptoms of allergies related to histamine release.

43. 43. Question


Proton pump inhibitors cause:

 A. Gastric ulcer formation

 B. GERD

 C. Achlorhydria

 D. Diverticulosis

44. 44. Question


A patient unable to tolerate oral medications may be prescribed which of the following proton pump inhibitors to be
administered intravenously?

 A. lansoprazole (Prevacid)
 B. omeprazole (Prilosec)

 C. pantoprazole (Protonix)

 D. esomeprazole (Nexium)

45. 45. Question


When administering sucralfate (Carafate) to a patient with a nasogastric tube, it is important to:

 A. Crush the tablet into a fine powder before mixing with water.

 B. Administer with a bolus tube feeding.

 C. Allow the tablet to dissolve in water before administering.

 D. Administer with an antacid for maximum benefit.

46. Question
Sucralfate (Carafate) achieves a therapeutic effect by:

 A. Neutralizing gastric acid.

 B. Enhancing gastric absorption.

 C. Forming a protective barrier around gastric mucosa.

 D. Inhibiting gastric acid secretion.

46. 47. Question


To avoid fecal impaction, psyllium (Metamucil) should be administered with at least how many ounces of fluid?

 A. 4

 B. 6

 C. 8

 D. 10

47. 48. Question


Bismuth subsalicylate (Pepto-Bismol), as an absorbent, has which of the following mechanisms of action?

 A. Decreased GI motility.

 B. Decreased gastric secretions.

 C. Increased fluid absorption.

 D. Binding to diarrhea-causing bacteria for excretion.

48. 49. Question


Side effects of loperamide (Imodium) include all of the following except?

 A. Diarrhea

 B. Epigastric pain

 C. Dry mouth
 D. Anorexia

49. 50. Question


The mechanism of action of diphenoxylate (Lotomil) is:

 A. An increase in intestinal excretion of water.

 B. An increase in intestinal motility.

 C. A decrease in peristalsis in the intestinal wall.

 D. A decrease in the reabsorption of water in the bowel.

1. 1. Question
The alimentary canal is a continuous, coiled, hollow muscular tube that winds through the ventral cavity and is
open at both ends. Its solid organs include all of the following except:

o A. Increased intrathoracic pressure.

o B. Weakness of the esophageal muscle.

o C. Increased esophageal muscle pressure.

o D. Weakness of the diaphragmatic muscle.

2. 2. Question
Risk factors for the development of Hiatal hernias are those that lead to increased abdominal pressure. Which of the following
complications can cause increased abdominal pressure?

 A. Obesity

 B. Volvulus

 C. Constipation

 D. Intestinal obstruction

3. 3. Question
Which of the following symptoms is common with a hiatal hernia?

 A. Left arm pain

 B. Lower back pain

 C. Esophageal reflux

 D. Abdominal cramping

4. 4. Question
Which of the following tests can be performed to diagnose a hiatal hernia?

 A. Colonoscopy

 B. Lower GI series
 C. Barium swallow

 D. Abdominal x-rays

5. 5. Question
Which of the following measures should the nurse focus on for the client with esophageal varices?

 A. Recognizing hemorrhage.

 B. Controlling blood pressure.

 C. Encouraging nutritional intake.

 D. Teaching the client about varices.

6. 6. Question
Which of the following tests can be used to diagnose ulcers?

 A. Abdominal x-ray

 B. Barium swallow

 C. Computed tomography (CT) scan

 D. Esophagogastroduodenoscopy (EGD)

7. 7. Question
Which of the following best describes the method of action of medications, such as ranitidine (Zantac), which are used in the
treatment of peptic ulcer disease?

 A. Neutralize acid.

 B. Reduce acid secretions.

 C. Stimulate gastrin release.

 D. Protect the mucosal barrier.

8. 8. Question
The hospitalized client with GERD is complaining of chest discomfort that feels like heartburn following a meal. After
administering an ordered antacid, the nurse encourages the client to lie in which of the following positions?

 A. Supine with the head of the bed flat.

 B. On the stomach with the head flat.

 C. On the left side with the head of the bed elevated 30 degrees.

 D. On the right side with the head of the bed elevated 30 degrees.

9. 9. Question
The nurse is caring for a client following a Billroth II procedure. On review of the post-operative orders, which of the following, if
prescribed, would the nurse question and verify?

 A. Irrigating the nasogastric tube.

 B. Coughing and deep breathing exercises.


 C. Leg exercises

 D. Early ambulation

10. 10. Question


The nurse is providing discharge instructions to a client following gastrectomy. Which measure will the nurse instruct the client to
follow to assist in preventing dumping syndrome?

 A. Eat high-carbohydrate foods.

 B. Limit the fluids taken with meals.

 C. Ambulate following a meal.

 D. Sit in a high-Fowler's position during meals.

11. 11. Question


The nurse instructs the nursing assistant on how to provide oral hygiene for a client who cannot perform this task for himself.
Which of the following techniques should the nurse tell the assistant to incorporate into the client’s daily care?

 A. Assess the oral cavity each time mouth care is given and record observations.

 B. Use a soft toothbrush to brush the client’s teeth after each meal.

 C. Swab the client’s tongue, gums, and lips with a soft foam applicator every 2 hours.

 D. Rinse the client’s mouth with mouthwash several times a day.

12. 12. Question


A client with suspected gastric cancer undergoes an endoscopy of the stomach. Which of the following assessments made after
the procedure would indicate the development of a potential complication?

 A. The client complains of a sore throat.

 B. The client displays signs of sedation.

 C. The client experiences a sudden increase in temperature.

 D. The client demonstrates a lack of appetite.

13. 13. Question


A client has been diagnosed with adenocarcinoma of the stomach and is scheduled to undergo a subtotal gastrectomy (Billroth II
procedure). During preoperative teaching, the nurse is reinforcing information about the procedure. Which of the following
explanations is most accurate?

 A. The procedure will result in enlargement of the pyloric sphincter.

 B. The procedure will result in anastomosis of the gastric stump to the jejunum.

 C. The procedure will result in removal of the duodenum.

 D. The procedure will result in repositioning of the vagus nerve.

14. 14. Question


After a subtotal gastrectomy, the nurse should anticipate that nasogastric tube drainage will be what color for about 12 to 24
hours after surgery?

 A. Dark brown
 B. Bile green

 C. Bright red

 D. Cloudy white

15. 15. Question


After a subtotal gastrectomy, care of the client’s nasogastric tube and drainage system should include which of the following
nursing interventions?

 A. Irrigate the tube with 30 ml of sterile water every hour if needed.

 B. Reposition the tube if it is not draining well.

 C. Monitor the client for N/V, and abdominal distention.

 D. Turn the machine to high suction if the drainage is sluggish on low suction.

16. 16. Question


Which of the following would be an expected nutritional outcome for a client who has undergone a subtotal gastrectomy for
cancer?

 A. Regain weight loss within 1 month after surgery.

 B. Resume normal dietary intake of three meals per day.

 C. Control nausea and vomiting through regular use of antiemetics.

 D. Achieve optimal nutritional status through oral or parenteral feedings.

17. 17. Question


The client with GERD complains of a chronic cough. The nurse understands that in a client with GERD this symptom may be
indicative of which of the following conditions?

 A. Development of laryngeal cancer.

 B. Irritation of the esophagus.

 C. Esophageal scar tissue formation.

 D. Aspiration of gastric contents.

18. Question
Which of the following dietary measures would be useful in preventing esophageal reflux?

 A. Eating small, frequent meals.

 B. Increasing fluid intake.

 C. Avoiding air swallowing with meals.

 D. Adding a bedtime snack to the dietary plan.

19. Question
A client is admitted to the hospital after vomiting bright red blood and is diagnosed with a bleeding duodenal ulcer. The client
develops a sudden, sharp pain in the mid-epigastric area along with a rigid, board-like abdomen. These clinical
manifestations most likely indicate which of the following?
 A. An intestinal obstruction has developed.

 B. Additional ulcers have developed.

 C. The esophagus has become inflamed.

 D. The ulcer has perforated.

18. 20. Question


When obtaining a nursing history on a client with a suspected gastric ulcer, which signs and symptoms would the nurse expect to
see? Select all that apply.

 A. Epigastric pain at night

 B. Relief of epigastric pain after eating

 C. Vomiting

 D. Weight loss
19.
20. 21. Question
The nurse is caring for a client who has had a gastroscopy. Which of the following symptoms may indicate that the client is
developing a complication related to the procedure? Select all that apply.

 A. The client complains of a sore throat.

 B. The client has a temperature of 100*F.

 C. The client appears drowsy following the procedure.

 D. The client complains of epigastric pain.

 E. The client experiences hematemesis.

21. 22. Question


A client with peptic ulcer disease tells the nurse that he has black stools, which he has not reported to his physician. Based on this
information, which nursing diagnosis would be appropriate for this client?

 A. Ineffective coping related to fear of diagnosis of chronic illness.

 B. Deficient knowledge related to unfamiliarity with significant signs and symptoms.

 C. Constipation related to decreased gastric motility.

 D. Imbalanced nutrition: Less than body requirements due to gastric bleeding.

22. 23. Question


A client with a peptic ulcer reports epigastric pain that frequently awakens her at night, a feeling of fullness in the abdomen, and
a feeling of anxiety about her health. Based on this information, which nursing diagnosis would be most appropriate?

 A. Imbalanced Nutrition: Less than Body Requirements related to anorexia.

 B. Disturbed Sleep Pattern related to epigastric pain.

 C. Ineffective Coping related to exacerbation of duodenal ulcer.

 D. Activity Intolerance related to abdominal pain.

23. 24. Question


While caring for a client with peptic ulcer disease, the client reports that he has been nauseated most of the day and is now
feeling lightheaded and dizzy. Based upon these findings, which nursing actions would be most appropriate for the nurse to
take? Select all that apply.

 A. Administering an antacid hourly until nausea subsides.

 B. Monitoring the client’s vital signs.

 C. Notifying the physician of the client’s symptoms.

 D. Initiating oxygen therapy.

 E. Reassessing the client in an hour.

24. 25. Question


A client is to take one daily dose of ranitidine (Zantac) at home to treat her peptic ulcer. The nurse knows that the client
understands proper drug administration of ranitidine when she says that she will take the drug at which of the following times?

 A. Before meals

 B. With meals

 C. At bedtime

 D. When pain occurs

25. 26. Question


A client has been taking aluminum hydroxide 30 mL six times per day at home to treat his peptic ulcer. He tells the nurse that he
has been unable to have a bowel movement for 3 days. Based on this information, the nurse would determine which of the
following is the most likely cause of the client’s constipation?

 A. The client has not been including enough fiber in his diet.

 B. The client needs to increase his daily exercise.

 C. The client is experiencing a side effect of the aluminum hydroxide.

 D. The client has developed a gastrointestinal obstruction.

26. 27. Question


A client is taking an antacid for treatment of a peptic ulcer. Which of the following statements best indicates that the client
understands how to correctly take the antacid?

 A. “I should take my antacid before I take my other medications.”

 B. “I need to decrease my intake of fluids so that I don’t dilute the effects of my antacid.”

 C. “My antacid will be most effective if I take it whenever I experience stomach pains.”

 D. “It is best for me to take my antacid 1 to 3 hours after meals.”

27. 28. Question


The nurse is caring for a client with chronic gastritis. The nurse monitors the client, knowing that this client is at risk for which of
the following vitamin deficiencies?

 A. Vitamin A

 B. Vitamin B12
 C. Vitamin C

 D. Vitamin E

28. 29. Question


The nurse is reviewing the medication record of a client with acute gastritis. Which medication, if noted on the client’s record,
would the nurse question?

 A. digoxin (Lanoxin)

 B. indomethacin (Indocin)

 C. furosemide (Lasix)

 D. propranolol hydrochloride (Inderal)

30. Question
The nurse is assessing a client 24 hours following a cholecystectomy. The nurse notes that the T-tube has drained 750ml of
green-brown drainage. Which nursing intervention is most appropriate?

 A. Notify the physician.

 B. Document the findings.

 C. Irrigate the T-tube.

 D. Clamp the T-tube.

29. 31. Question


The nurse provides medication instructions to a client with peptic ulcer disease. Which statement, if made by the client, indicates
the best understanding of the medication therapy?

 A. “The cimetidine (Tagamet) will cause me to produce less stomach acid.”

 B. “Sucralfate (Carafate) will change the fluid in my stomach.”

 C. “Antacids will coat my stomach.”

 D. “Omeprazole (Prilosec) will coat the ulcer and help it heal.”

30. 32. Question


The client with peptic ulcer disease is scheduled for a pyloroplasty. The client asks the nurse about the procedure. The nurse
plans to respond knowing that a pyloroplasty involves:

 A. Cutting the vagus nerve.

 B. Removing the distal portion of the stomach.

 C. Removal of the ulcer and a large portion of the cells that produce hydrochloric acid.

 D. An incision and resuturing of the pylorus to relax the muscle and enlarge the opening from the stomach to the
duodenum.

31. 33. Question


A client with a peptic ulcer is scheduled for a vagotomy. The client asks the nurse about the purpose of this procedure. The nurse
tells the client that the procedure:

 A. Decreases food absorption in the stomach.


 B. Heals the gastric mucosa.

 C. Halts stress reactions.

 D. Reduces the stimulus to acid secretions.

32. 34. Question


The nurse would assess the client experiencing an acute episode of cholecystitis for pain that is located in the right

 A. Upper quadrant and radiates to the left scapula and shoulder.

 B. Upper quadrant and radiates to the right scapula and shoulder.

 C. Lower quadrant and radiates to the umbilicus.

 D. Lower quadrant and radiates to the back.

33. 35. Question


Which of the following tasks should be included in the immediate postoperative management of a client who has undergone
gastric resection?

 A. Monitoring gastric pH to detect complications.

 B. Assessing for bowel sounds.

 C. Providing nutritional support.

 D. Monitoring for symptoms of hemorrhage.

34. 36. Question


If a gastric acid perforates, which of the following actions should not be included in the immediate management of the client?

 A. Blood replacement

 B. Antacid administration

 C. Nasogastric tube suction

 D. Fluid and electrolyte replacement

35. 37. Question


Mucosal barrier fortifiers are used in peptic ulcer disease management for which of the following indications?

 A. To inhibit mucus production.

 B. To neutralize acid production.

 C. To stimulate mucus production.

 D. To stimulate hydrogen ion diffusion back into the mucosa.

36. 38. Question


When counseling a client in ways to prevent cholecystitis, which of the following guidelines is most important?

 A. Eat a low-protein diet.

 B. Eat a low-fat, low-cholesterol diet.


 C. Limit exercise to 10 minutes/day.

 D. Keep weight proportionate to height.

37. 39. Question


Which of the following symptoms best describes Murphy’s sign?

 A. Periumbilical ecchymosis exists.

 B. On deep palpation and release, pain elicited.

 C. On deep inspiration, pain is elicited and breathing stops.

 D. Abdominal muscles are tightened in anticipation of palpation.

38. 40. Question


Which of the following tests is most commonly used to diagnose cholecystitis?

 A. Abdominal CT scan

 B. Abdominal ultrasound

 C. Barium swallow

 D. Endoscopy

39. 41. Question


Which of the following factors should be the main focus of nursing management for a client hospitalized for cholecystitis?

 A. Administration of antibiotics.

 B. Assessment for complications.

 C. Preparation for lithotripsy.

 D. Preparation for surgery.

40. 42. Question


A client being treated for chronic cholecystitis should be given which of the following instructions?

 A. Increase rest.

 B. Avoid antacids.

 C. Increase protein in diet.

 D. Use anticholinergics as prescribed.

41. 43. Question


The client with a duodenal ulcer may exhibit which of the following findings during an assessment?

 A. Hematemesis

 B. Malnourishment

 C. Melena

 D. Pain with eating


42. 44. Question
The pain of a duodenal ulcer can be distinguished from that of a gastric ulcer by which of the following characteristics?

 A. Early satiety

 B. Pain on eating

 C. Dull upper epigastric pain

 D. Pain on an empty stomach

43. 45. Question


The client has orders for a nasogastric (NG) tube insertion. During the procedure, instructions that will assist in the insertion
would be:

 A. Instruct the client to tilt his head back for insertion in the nostril, then flex his neck for the final insertion.

 B. After insertion into the nostril, instruct the client to extend his neck.

 C. Introduce the tube with the client’s head tilted back, then instruct him to keep his head upright for final insertion.

 D. Instruct the client to hold his chin down, then back for insertion of the tube.

44. 46. Question


The most important pathophysiological factor contributing to the formation of esophageal varices is:

 A. Decreased prothrombin formation.

 B. Decreased albumin formation by the liver.

 C. Portal hypertension.

 D. Increased central venous pressure.

45. 47. Question


The client being treated for esophageal varices has a Sengstaken-Blakemore tube inserted to control the bleeding.
The most important assessment is for the nurse to:

 A. Check that the hemostat is on the bedside.

 B. Monitor IV fluids for the shift.

 C. Regularly assess respiratory status.

 D. Check that the balloon is deflated on a regular basis.

46. 48. Question


A female client complains of gnawing epigastric pain for a few hours after meals. At times, when the pain is severe, vomiting
occurs. Specific tests are indicated to rule out:

 A. Cancer of the stomach

 B. Peptic ulcer disease

 C. Chronic gastritis

 D. Pylorospasm
47. 49. Question
When a client has peptic ulcer disease, the nurse would expect a priority intervention to be:

 A. Assisting in inserting a Miller-Abbott tube.

 B. Assisting in inserting an arterial pressure line.

 C. Inserting a nasogastric tube.

 D. Inserting an I.V.

48. 50. Question


A 40-year-old male client has been hospitalized with peptic ulcer disease. He is being treated with a histamine receptor
antagonist (cimetidine), antacids, and diet. The nurse doing discharge planning will teach him that the action of cimetidine is to:

 A. Reduce gastric acid output.

 B. Protect the ulcer surface.

 C. Inhibit the production of hydrochloric acid (HCl).

 D. Inhibit vagus nerve stimulation.

1. 1. Question
Which of the following laboratory values would be the most important to monitor for a patient with pancreatic cancer?

o A. Serum glucose

o B. Radioimmunoassay (RIA)

o C. Creatine phosphokinase (CPK)

o D. Carcinoembryonic antigen (CEA)

2. 2. Question
You observe changes in mentation, irritability, restlessness, and decreased concentration in a patient with cancer of the liver.
Hepatic encephalopathy is suspected and the patient is ordered neomycin enemas. Which of the following information in the
patient’s history would be a contraindication of this order?

 A. Left nephrectomy

 B. Glaucoma in both eyes

 C. Myocardial infarction

 D. Peripheral neuropathy

3. 3. Question
A nursing intervention for a patient with hepatitis B would include which of the following types of isolation.

 A. Universal precautions

 B. Blood transfusions

 C. Enteric isolation
 D. Strict isolation

4. 4. Question
A patient is admitted with lacerated liver as a result of blunt abdominal trauma. Which of the following nursing interventions
would not be appropriate for this patient?

 A. Monitor for respiratory distress.

 B. Monitor for coagulation studies.

 C. Administer pain medications as ordered.

 D. Administer normal saline, crystalloids as ordered.

5. 5. Question
A male client is recovering from a small bowel resection. To relieve pain, the physician prescribes meperidine (Demerol), 75 mg
I.M. every 4 hours. How soon after administration should meperidine onset of action occur?

 A. 5 to 10 minutes

 B. 15 to 30 minutes

 C. 30 to 60 minutes

 D. 2 to 4 hours

6. Question
Nathaniel has severe pruritus due to having hepatitis B. What is the best intervention for his comfort?

 A. Give tepid baths.

 B. Avoid lotions and creams.

 C. Use hot water to increase vasodilation.

 D. Use cold water to decrease the itching.

6. 7. Question
Rob is a 46 y.o. admitted to the hospital with a suspected diagnosis of Hepatitis B. He’s jaundiced and reports weakness. Which
intervention will you include in his care?

 A. Regular exercise.

 B. A low-protein diet.

 C. Allow the patient to select his meals.

 D. Rest period after small, frequent meals.

7. 8. Question
You’re discharging Nathaniel with hepatitis B. Which statement suggests understanding by the patient?

 A. “Now I can never get hepatitis again.”

 B. “I can safely give blood after 3 months.”

 C. “I’ll never have a problem with my liver again, even if I drink alcohol.”
 D. “My family knows that if I get tired and start vomiting, I may be getting sick again.”

8. 9. Question
Gail is scheduled for a cholecystectomy. After completion of preoperative teaching, Gail states,” If I lie still and avoid turning after
the operation, I’ll avoid pain. Do you think this is a good idea?” What is the best response?

 A. “You’ll need to turn from side to side every 2 hours.”

 B. “It’s always a good idea to rest quietly after surgery.”

 C. “The doctor will probably order you to lie flat for 24 hours.”

 D. “Why don’t you decide about activity after you return from the recovery room?”

9. 10. Question
You’re caring for a 28 y.o. woman with hepatitis B. She’s concerned about the duration of her recovery. Which response isn’t
appropriate?

 A. Encourage her to not worry about the future.

 B. Encourage her to express her feelings about the illness.

 C. Discuss the effects of hepatitis B on future health problems.

 D. Provide avenues for financial counseling if she expresses the need.


10. 11. Question
Elmer is scheduled for a proctoscopy and has an I.V. The doctor wrote an order for 5mg of I.V. diazepam(Valium). Which order is
correct regarding diazepam?

 A. Give diazepam in the I.V. port closest to the vein.

 B. Mix diazepam with 50 ml of dextrose 5% in water and give it over 15 minutes.

 C. Give diazepam rapidly I.V. to prevent the bloodstream from diluting the drug mixture.

 D. Question the order because I.V. administration of diazepam is contraindicated.

11. 12. Question


Annabelle is being discharged with a colostomy, and you’re teaching her about colostomy care. Which statement correctly
describes a healthy stoma?

 A. “At first, the stoma may bleed slightly when touched.”

 B. “The stoma should appear dark and have a bluish hue.”

 C. “A burning sensation under the stoma faceplate is normal.”

 D. “The stoma should remain swollen away from the abdomen.”

12. 13. Question


A patient who underwent abdominal surgery now has a gaping incision due to delayed wound healing. Which method is correct
when you irrigate a gaping abdominal incision with sterile normal saline solution, using a piston syringe?

 A. Rapidly instill a stream of irrigating solution into the wound.

 B. Apply a wet-to-dry dressing to the wound after the irrigation.

 C. Moisten the area around the wound with normal saline solution after the irrigation.
 D. Irrigate continuously until the solution becomes clear or all of the solution is used.

13. 14. Question


Hepatic encephalopathy develops when the blood level of which substance increases?

 A. Ammonia

 B. Amylase

 C. Calcium

 D. Potassium

14. 15. Question


Your patient recently had abdominal surgery and tells you that he feels a popping sensation in his incision during a coughing
spell, followed by severe pain. You anticipate an evisceration. Which supplies should you take to his room?

 A. A suture kit.

 B. Sterile water and a suture kit.

 C. Sterile water and sterile dressings.

 D. Sterile saline solution and sterile dressings.

15. 16. Question


Findings during an endoscopic exam include a cobblestone appearance of the colon in your patient. The findings are
characteristic of which disorder?

 A. Ulcer

 B. Crohn’s disease

 C. Chronic gastritis

 D. Ulcerative colitis

16. 17. Question


What information is correct about stomach cancer?

 A. Stomach pain is often a late symptom.

 B. Surgery is often a successful treatment.

 C. Chemotherapy and radiation are often successful treatments.

 D. The patient can survive for an extended time with TPN.

17. 18. Question


Dark, tarry stools indicate bleeding in which location of the GI tract?

 A. Upper colon

 B. Lower colon

 C. Upper GI tract

 D. Small intestine
18. 19. Question
A patient has an acute upper GI hemorrhage. Your interventions include:

 A. Treating hypovolemia.

 B. Treating hypervolemia.

 C. Controlling the bleeding source.

 D. Treating shock and diagnosing the bleeding source.

19. 20. Question


You promote hemodynamic stability in a patient with upper GI bleeding by:

 A. Encouraging oral fluid intake.

 B. Monitoring central venous pressure.

 C. Monitoring laboratory test results and vital signs.

 D. Giving blood, electrolyte, and fluid replacement.

20. 21. Question


You’re preparing a patient with a malignant tumor for colorectal surgery and subsequent colostomy. The patient tells you he’s
anxious. What should your initial step be in working with this patient?

 A. Determine what the patient already knows about colostomies.

 B. Show the patient some pictures of colostomies.

 C. Arrange for someone who has a colostomy to visit the patient.

 D. Provide the patient with written material about colostomy care.

21. 22. Question


Your patient, Christopher, has a diagnosis of ulcerative colitis and has severe abdominal pain aggravated by movement, rebound
tenderness, fever, nausea, and decreased urine output. This may indicate which complication?

 A. Fistula

 B. Bowel perforation

 C. Bowel obstruction

 D. Abscess

22. 23. Question


A patient has a severe exacerbation of ulcerative colitis. Long-term medications will probably include:

 A. Antacids

 B. Antibiotics

 C. Corticosteroids

 D. Histamine2-receptor blockers

23. 24. Question


The student nurse is teaching the family of a patient with liver failure. You instruct them to limit which foods in the patient’s diet?

 A. Meats and beans

 B. Butter and gravies

 C. Potatoes and pasta

 D. Cakes and pastries

24. 25. Question


An intubated patient is receiving continuous enteral feedings through a Salem pump tube at a rate of 60ml/hr. Gastric residuals
have been 30-40ml when monitored Q4H. You check the gastric residual and aspirate 220ml. What is your first response to this
finding?

 A. Notify the doctor immediately.

 B. Stop the feeding, and clamp the NG tube.

 C. Discard the 220ml and clamp the NG tube.

 D. Give a prescribed GI stimulant such as metoclopramide (Reglan).

25. 26. Question


Your patient with peritonitis is NPO and complaining of thirst. What is your priority?

 A. Increase the I.V. infusion rate.

 B. Use diversion activities.

 C. Provide frequent mouth care.

 D. Give ice chips every 15 minutes.

26. 27. Question


Kevin has a history of peptic ulcer disease and vomits coffee-ground emesis. What does this indicate?

 A. He has fresh, active upper GI bleeding.

 B. He needs immediate saline gastric lavage.

 C. His gastric bleeding occurred 2 hours earlier.

 D. He needs a transfusion of packed RBCs.

27. 28. Question


A 53 y.o. patient has undergone a partial gastrectomy for adenocarcinoma of the stomach. An NG tube is in place and is
connected to low continuous suction. During the immediate postoperative period, you expect the gastric secretions to be which
color?

 A. Brown

 B. Clear

 C. Red

 D. Yellow

28. 29. Question


Your patient has a retractable gastric peptic ulcer and has had a gastric vagotomy. Which factor increases as a result of
vagotomy?

 A. Peristalsis

 B. Gastric acidity

 C. Gastric motility

 D. Gastric pH

29. 30. Question


Christina is receiving an enteral feeding that requires a concentration of 80 ml of supplement mixed with 20 ml of water. How
much water do you mix with an 8 oz (240ml) can of feeding?

 A. 60 ml

 B. 70 ml

 C. 80 ml

 D. 90 ml

30. 31. Question


Which stoma would you expect a malodorous, enzyme-rich, caustic liquid output that is yellow, green, or brown?

 A. Ileostomy

 B. Ascending colostomy

 C. Transverse colostomy

 D. Descending colostomy

31. 32. Question


George has a T tube in place after gallbladder surgery. Before discharge, what information or instructions should be given
regarding the T tube drainage?

 A. “If there is any drainage, notify the surgeon immediately.”

 B. “The drainage will decrease daily until the bile duct heals.”

 C. “First, the drainage is dark green; then it becomes dark yellow.”

 D. “If the drainage stops, milk the tube toward the puncture wound.”

32. 33. Question


Your patient Maria takes NSAIDS for her degenerative joint disease, and has developed peptic ulcer disease. Which drug is useful
in preventing NSAID-induced peptic ulcer disease?

 A. calcium carbonate (Tums)

 B. famotidine (Pepcid)

 C. misoprostol (Cytotec)

 D. sucralfate (Carafate)

33. 34. Question


The student nurse is participating in a colorectal cancer screening program. Which patient has the fewest risk factors for colon
cancer?

 A. Janice, a 45 y.o. with a 25-year history of ulcerative colitis.

 B. George, a 50 y.o. whose father died of colon cancer.

 C. Herman, a 60 y.o. who follows a low-fat, high-fiber diet.

 D. Sissy, a 72 y.o. with a history of breast cancer.

35. Question
Your patient, post-op drainage of a pelvic abscess secondary to diverticulitis, begins to cough violently after drinking water. His
wound has ruptured and a small segment of the bowel is protruding. What’s your priority?

 A. Ask the patient what happened, call the doctor, and cover the area with a water-soaked bed sheet.

 B. Obtain vital signs, call the doctor, and obtain emergency orders.

 C. Have a CAN hold the wound together while you obtain vital signs, call the doctor and flex the patient’s knees.

 D. Have the doctor called while you remain with the patient, flex the patient’s knees, and cover the wound with
sterile towels soaked in sterile saline solution.

PRACTICE TEST QUESTIONS


Downloaded from FILIPINO NURSES CENTRAL
{www.filipinonursescentral.wordpress.com}

Situation 1: Children have a special fascination with the workings of the digestive system. To fully understand
the digestive processes, Nurse Lavigña must be knowledgeable of the anatomy and physiology of the
gastrointestinal system.

1. The alimentary canal is a continuous, coiled, hollow muscular tube that winds through the ventral cavity and is open at both ends.
Its solid organs include all of the following except:
a. liver
b. gall bladder
c. stomach
d. pancreas

2. Pharynx is lined with mucous membranes and mucous secreting glands to ease the passage of food. The larygngopharynx serves as
passageway for:
a. air only
b. air and water
c. food, fluids and air
d. air and food

3. Once food has been placed in the mouth, both mechanical and chemical digestions begin. The six activities of the
digestive process are:
a. ingestion, mastication, digestion, deglutition, absorption, egestion
b. ingestion, mastication, deglutition, digestion, absorption, egestion
c. deglutition, ingestion, mastication, egestion, absorption, defecation
d. ingestion, digestion, mastication, deglutition, absorption, defecation
4. Most digestive activity occurs in the pyloric region of the stomach. What hormone stimulates the chief cells to produce
pepsinogen?
a. Gastrin

b
c. Pe
HCp
l sin
d. Insulin

5. What pancreatic enzyme aids in the digestion of carbohydrates?


a. Lipase
b. Trypsin
c. Amylase
d. Chymotrypsin

Situation 2: Nurse Dorina is going to perform an abdominal examination to Mr. Lim who was admitted due
to on and off pain since yesterday.

6. How will you position Mr. Lim prior to procedure?


a. supine with knees flexed
b. prone
c. lying on back
d. sim’s

7. To identify any localized bulging, distention and peristaltic waves, Nurse Dorina must perform which of the following?
a. Auscultation
b. Inspection
c. Palpation
d. Percussion

8. In order to identify areas of tenderness and swelling, Nurse Dorina must do:
a. deep palpation
b. light palpation
c. percussion
d. palpation

9. Mr. Lim verbalized pain on the right iliac region. Nurse Dorina knows that the organ affected would be the:
a. liver
b. sigmoid colon
c. appendix
d. duodenum

10. Mr. Lim felt pain upon release of Nurse Dorina’s hand. This can be referred as:
a. referred pain
b. rebound tenderness
c. direct tenderness
d. indirect tenderness
Situation 3: Mrs. Cruz was admitted in the Medical Floor due to pyrosis, dyspepsia
and difficulty of swallowing.

11. Based from the symptoms presented, Nurse Yoshi might suspect:
a. Esophagitis
b. Hiatal hernia
c. GERD
d. Gastric Ulcer

12. What diagnostic test would confirm the type of problem Mrs. Cruz have?
a. barium enema
b. barium swallow
c. colonoscopy
d. lower GI series

another diagnostic test. Options a and d are the same. Option c is incorrect.
13. Mrs. Cruz complained of pain and difficulty in swallowing. This term is referred as:
a. Odynophagia
b. Dysphagia
c. Pyrosis
d. Dyspepsia

14. To avoid acid reflux, Nurse Yoshi should advice Mrs. Cruz to avoid which type of diet?
a. cola, coffee and tea
b. high fat, carbonated and caffeinated beverages
c. beer and green tea
d. lechon paksiw and bicol express

15. Mrs. Cruz’ body mass index (BMI) is 25. You can categorized her as:
a. normal
b. overweight
c. underweight
d. obese

Situation 4: Nurse Gloria is the staff nurse assigned at the Emergency Department. During her shift, a
patient was rushed – in the ED complaining of severe heartburn, vomiting and pain that radiates to the
flank. The doctor suspects gastric ulcer.

16. What other symptoms will validate the diagnosis of gastric ulcer?
a. right epigastric pain
b. pain occurs when stomach is empty
c. pain occurs immediately after meal
d. pain not relieved by vomiting

17. What diagnostic test would yield good visualization of the ulcer crater?
a. Endoscopy
b. Gastroscopy
c. Barium Swallow
d. Histology

18. Peptic ulcer disease particularly gastric ulcer is thought to be cause by which of the following microorgamisms?
a. E. coli
b. H. pylori
c. S. aureus
d. K. pnuemoniae
19. She is for occult blood test, what specimen will you collect?
a. Blood
b. Urine
c. Stool
d. Gastric Juice

20. Preparation of the client for occult blood examination is:


a. Fluid intake limited only to 1 liter/day
b. NPO for 12 hours prior to obtaining of specimen
c. Increase fluid intake
d. Meatless diet for 48 hours prior to obtaining of specimen

21. The client with IBS asks Nurse June what causes the disease. Which of the following responses by Nurse June would be
most appropriate?
a. “This is an inflammation of the bowel caused by eating too much roughage”
b. “IBS is caused by a stressful lifestyle”
c. “The cause of this condition is unknown”
d. “There is thinning of the intestinal mucosa caused by ingestion of gluten”

22. Which of the following alimentary canal is the most common location for Chron’s disease?
a. Descending colon
b. Jejunum
c. Sigmoid Colon
d. Terminal Ileum
23. Which of the following factors is believed to be linked to Chron’s disease?
a. Diet

b. Const pation
c. Heredity
d. Lack of exercise

Answer: C. Heredity

24. How about ulcerative colitis, which of the following factors is believed to cause it?
a. Acidic diet

b
c. A
Clt
hreore
nd
ic icmm
onsu
tin
pity
ation
d. Emotional stress

Answer: B. Altered immunity

25. Mr. Jung, had ulcerative colitis for 5 years and was admitted to the hospital. Which of the following factors was most likely of
greatest significance in causing an exacerbation of the disease?
a. A demanding and stressful job
b. Changing to a modified vegetarian diet
c. Beginning a weight training program
d. Walking 2 miles everyday

Situation 6: A patient was admitted in the Medical Floor at St. Luke’s Hospital. He was asymptomatic. The
doctor suspects diverticulosis.

26. Which of the following definitions best describes diverticulosis?


a. An inflamed outpouching of the intestine
b. A non – inflamed outpouching of the intestine
c. The partial impairment of the forward flow of instestinal contents
d. An abnormal protrusions of an oxygen through the structure that usually holds it

27. Which of the following types of diet is implicated in the development of


diverticulosis?
a. Low – fiber diet
b. High – fiber diet
c. High – protein diet
d. Low – carbohydrate diet

28. To improve Mr. Trinidad’s condition, your best nursing intervention and teaching is:
a. Reduce fluid intake
b. Increase fiber in the diet
c. Administering of antibiotics
d. Exercise to increase intraabdominal pressure

29. Upon review of Mr. Trinidad’s chart, Nurse Drew noticed that he weighs 121 lbs and his height is 5 ft, 4 in. After computing for
his Body Mass Index (BMI), you
can categorize him as:
a. obese
b. normal
c. obese
d. underweight

Situation 7: Manny, 6 years old was admitted at Cardinal Santos Hospital due to increasing frequency of bowel
movements, abdominal cramps and distension.

30. Diarrhea is said to be the leading cause of morbidity in the Philippines. Nurse Harry knows that diarrhea is present if:
a. passage of stool is more than 3 bowel movements per week
b. passage of stool is less than 3 bowel movements per day
c. passage of stool is more than 3 bowel movements per day
d. passage of stool is less than 3 bowel movements per week

31. Diarrhea is believed to be caused by all of the following except


a. increase intestinal secretions
b. altered immunity
c. decrease mucosal absorption
d. altered motility

32. What life threatening condition may result in persistent diarrhea?


a. hypokalemia
b. dehydration
c. cardiac dysrhytmias
d. leukocytosis

33. Voluminous, watery stools can deplete fluids and electrolytes. The acid base imbalance that can occur is:
a. metabolic alkalosis
b. metabolic acidosis
c. respiratory acidosis
d. respiratory alkalosis

34. What is the immediate home care management for diarrhea?


a. Milk
b. Imodium
c. Water
d. Oresol

Situation 8: Mr. Sean is admitted to the hospital with a bowel obstruction. He complained of colicky pain and

inability to pass stool.


35. Which of these findings by Nurse Leonard, would indicate that the obstruction is in the early stages?
a. high pitched tinkling or rumbling bowel sounds
b. hypoactive bowel sounds
c. no bowel sounds auscultated
d. normal bowel sounds heard in all four quadrants

36. Nasogastric tube was inserted to Mr. Sean. The NGT’s primary purpose is:
a. nutrition
b. decompression of bowel
c. passage for medication
d. aspiration of gastric contents

37. Mr. Sean has undergone surgery. Post – operatively, which of the following findings is normal?
a. absent bowel sounds
b. bleeding
c. hemorrhage
d. bowel movement

Bowel movement occurs only after flatus and bowel sounds are noted.

38. Client education should be given in order to prevent constipation. Nurse Leonard’s health teaching should include
which of the following?
a. use of natural laxatives
b. fluid intake of 6 glasses per day
c. use of OTC laxatives
d. complete bed rest

39. Four hours post – operatively, Mr. Sean complains of guarding and rigidity of the abdomen. Nurse Leonard’s initial intervention
is:
a. assess for signs of peritonitis
b. call the physician
c. administer pain medication
d. ignore the client

Situation 9: Mr. Gerald Liu, 19 y/o, is being admitted to a hospital unit complaining of severe pain in the
lower abdomen. Admission vital signs reveal an oral temperature of 101.2 0F.

40. Which of the following would confirm a diagnosis of appendicitis?


a. The pain is localized at a position halfway between the umbilicus and the right iliac crest.
b. Mr. Liu describes the pain as occurring 2 hours after eating
c. The pain subsides after eating
d. The pain is in the left lower quadrant

41. Which of the following complications is thought to be the most common cause of appendicitis?
a. A fecalith
b. Internal bowel occlusion
c. Bowel kinking
d. Abdominal wall swelling Answer: A. A

The doctor ordered for a complete blood

count. After the test, Nurse Ray received

the result from the laboratory. Which

laboratory values will confirm the diagnosis

of appendicitis?
a. RBC 5.5 x 106/mm3
b. Hct 44 %
c. WBC 13, 000/mm3

d. Hgb 15 g/dL

42. Signs and symptoms include pain in the RLQ of the abdomen that may be localize at McBurney’s point. To relieve pain, Mr.
Liu should assume which position?
a. Prone
b. Supine, stretched out
c. Sitting
d. Lying with legs drawn upl

43. After a few minutes, the pain suddenly stops without any intervention. Nurse Ray might suspect that:
a. the appendix is still distended
b. the appendix may have ruptured
c. an increased in intrathoracic pressure will occur
d. signs and symptoms of peritonitis occur
Situation 10: Nurse Nico is caring to a 38-year-old female, G3P3 client who has been diagnosed with
hemorrhoids.

46. Which of the following factors would most likely be a primary cause of her
hemorrhoids?
a. Her age
b. Three vaginal delivery pregnancies
c. Her job as a school teacher
d. Varicosities in the legs

47. Client education should include minimizing client discomfort due to hemorrhoids. Nursing management should
include:
a. Suggest to eat low roughage diet
b. Advise to wear silk undergarments
c. Avoid straining during defecation
d. Use of sitz bath for 30 minutes

48. The doctor orders for Witch Hazel 5 %. Nurse Nico knows that the action of this astringent is:
a. temporarily relieves pain, burning, and itching by numbing the nerve endings
b. causes coagulation (clumping) of proteins in the cells of the perianal skin or the lining of the anal canal
c. inhibits the growth of bacteria and other organisms
d. causes the outer layers of skin or other tissues to disintegrate

49. Which position would be ideal for the client in the early postoperative period after hemorrhoidectomy?
a. High Fowler’s
b. Supine
c. Side – lying
d. Trendelenburg’s

50. Nurse Nico instructs her client who has had a hemorrhoidectomy not to used sitz bath until at
least 12 hours postoperatively to avoid which of the following complications?
a. Hemorrhage
b. Rectal Spasm
c. Urinary retention
d. Constipation
1. Nurse Berlinda is assigned to a 41-year-old client who has a diagnosis of chronic pancreatitis. The nurse
reviews the laboratory result, anticipating a laboratory report that indicates a serum amylase level of:

A. 45 units/L
B. 100 units/L
C. 300 units/L
D. 500 units/L

2. A male client who is recovering from surgery has been advanced from a clear liquid diet to a full liquid
diet. The client is looking forward to the diet change because he has been “bored” with the clear liquid diet.
The nurse would offer which full liquid item to the client?

A. Tea
B. Gelatin
C. Custard
D. Popsicle

3. Nurse Juvy is caring for a client with cirrhosis of the liver. To minimize the effects of the disorder, the
nurse teaches the client about foods that are high in thiamine. The nurse determines that the client has the
best understanding of the dietary measures to follow if the client states an intension to increase the intake of:

A. Pork
B. Milk
C. Chicken
D. Broccoli

4. Nurse Oliver checks for residual before administering a bolus tube feeding to a client with a nasogastric
tube and obtains a residual amount of 150 mL. What is appropriate action for the nurse to take?

A. Hold the feeding


B. Reinstill the amount and continue with administering the feeding
C. Elevate the client’s head at least 45 degrees and administer the feeding
D. Discard the residual amount and proceed with administering the feeding

5. A nurse is inserting a nasogastric tube in an adult male client. During the procedure, the client begins to
cough and has difficulty breathing. Which of the following is the appropriate nursing action?

A. Quickly insert the tube


B. Notify the physician immediately
C. Remove the tube and reinsert when the respiratory distress subsides
D. Pull back on the tube and wait until the respiratory distress subsides

6. Nurse Ryan is assessing for correct placement of a nosogartric tube. The nurse aspirates the stomach
contents and check the contents for pH. The nurse verifies correct tube placement if which pH value is
noted?

A. 3.5
B. 7.0
C. 7.35
D. 7.5

7. A nurse is preparing to remove a nasogartric tube from a female client. The nurse should instruct the
client to do which of the following just before the nurse removes the tube?

A. Exhale
B. Inhale and exhale quickly
C. Take and hold a deep breath
D. Perform a Valsalva maneuver

8. Nurse Joy is preparing to administer medication through a nasogastric tube that is connected to suction.
To administer the medication, the nurse would:

A. Position the client supine to assist in medication absorption


B. Aspirate the nasogastric tube after medication administration to maintain patency
C. Clamp the nasogastric tube for 30 minutes following administration of the medication
D. Change the suction setting to low intermittent suction for 30 minutes after medication administration
9. A nurse is preparing to care for a female client with esophageal varices who has just has a Sengstaken-
Blakemore tube inserted. The nurse gathers supplies, knowing that which of the following items must be
kept at the bedside at all times?

A. An obturator
B. Kelly clamp
C. An irrigation set
D. A pair of scissors

10. Dr. Smith has determined that the client with hepatitis has contracted the infection form contaminated
food. The nurse understands that this client is most likely experiencing what type of hepatitis?

A. Hepatitis A
B. Hepatitis B
C. Hepatitis C
D. Hepatitis D

11. A client is suspected of having hepatitis. Which diagnostic test result will assist in confirming this
diagnosis?

A. Elevated hemoglobin level


B. Elevated serum bilirubin level
C. Elevated blood urea nitrogen level
D. Decreased erythrocycle sedimentation rate

12. The nurse is reviewing the physician’s orders written for a male client admitted to the hospital with
acute pancreatitis. Which physician order should the nurse question if noted on the client’s chart?

A. NPO status
B. Nasogastric tube inserted
C. Morphine sulfate for pain
D. An anticholinergic medication

13. A female client being seen in a physician’s office has just been scheduled for a barium swallow the next
day. The nurse writes down which instruction for the client to follow before the test?

A. Fast for 8 hours before the test


B. Eat a regular supper and breakfast
C. Continue to take all oral medications as scheduled
D. Monitor own bowel movement pattern for constipation

14. The nurse is performing an abdominal assessment and inspects the skin of the abdomen. The nurse
performs which assessment technique next?

A. Palpates the abdomen for size


B. Palpates the liver at the right rib margin
C. Listens to bowel sounds in all for quadrants
D. Percusses the right lower abdominal quadrant

15. Polyethylene glycol-electrlyte solution (GoLYTELY) is prescribed for the female client scheduled for a
colonoscopy. The client begins to experience diarrhea following administration of the solution. What action
by the nurse is appropriate?

A. Start an IV infusion
B. Administer an enema
C. Cancel the diagnostic test
D. Explain that diarrhea is expected

16. The nurse is caring for a male client with a diagnosis of chronic gastritis. The nurse monitors the client
knowing that this client is at risk for which vitamin deficiency?

A. Vitamin A
B. Vitamin B12
C. Vitamin C
D. Vitamin E
17. The nurse is reviewing the medication record of a female client with acute gastritis. Which medication,
if noted on the client’s record, would the nurse question?

A. Digoxin (Lanoxin)
B. Furosemide (Lasix)
C. Indomethacin (Indocin)
D. Propranolol hydrochloride (Inderal)

18. The nurse is assessing a male client 24 hours following a cholecystectomy. The nurse noted that the T
tube has drained 750 mL of green-brown drainage since the surgery. Which nursing intervention is
appropriate?

A. Clamp the T tube


B. Irrigate the T tube
C. Notify the physician
D. Document the findings

19. The nurse is monitoring a female client with a diagnosis of peptic ulcer. Which assessment findings
would most likely indicate perforation of the ulcer?

A. Bradycardia
B. Numbness in the legs
C. Nausea and vomiting
D. A rigid, board-like abdomen

20. A male client with a peptic ulcer is scheduled for a vagotomy and the client asks the nurse about the
purpose of this procedure. Which response by the nurse best describes the purpose of a vagotomy?

A. Halts stress reactions


B. Heals the gastric mucosa
C. Reduces the stimulus to acid secretions
D. Decreases food absorption in the stomach

21. The nurse is caring for a female client following a Billroth II procedure. Which postoperative order
should the nurse question and verify?

A. Leg exercises
B. Early ambulation
C. Irrigating the nasogastric tube
D. Coughing and deep-breathing exercises

22. The nurse is providing discharge instructions to a male client following gastrectomy and instructs the
client to take which measure to assist in preventing dumping syndrome?

A. Ambulate following a meal


B. Eat high carbohydrate foods
C. Limit the fluid taken with meal
D. Sit in a high-Fowler’s position during meals

23. The nurse is monitoring a female client for the early signs and symptoms of dumping syndrome. Which
of the following indicate this occurrence?

A. Sweating and pallor


B. Bradycardia and indigestion
C. Double vision and chest pain
D. Abdominal cramping and pain

24. The nurse is preparing a discharge teaching plan for the male client who had umbilical hernia repair.
What should the nurse include in the plan?

A. Irrigating the drain


B. Avoiding coughing
C. Maintaining bed rest
D. Restricting pain medication
25. The nurse is instructing the male client who has an inguinal hernia repair how to reduce postoperative
swelling following the procedure. What should the nurse tell the client?

A. Limit oral fluid


B. Elevate the scrotum
C. Apply heat to the abdomen
D. Remain in a low-fiber diet

26. The nurse is caring for a hospitalized female client with a diagnosis of ulcerative colitis. Which finding,
if noted on assessment of the client, would the nurse report to the physician?

A. Hypotension
B. Bloody diarrhea
C. Rebound tenderness
D. A hemoglobin level of 12 mg/dL

27. The nurse is caring for a male client postoperatively following creation of a colostomy. Which nursing
diagnosis should the nurse include in the plan of care?

A. Sexual dysfunction
B. Body image, disturbed
C. Fear related to poor prognosis
D. Nutrition: more than body requirements, imbalanced

28. The nurse is reviewing the record of a female client with Crohn’s disease. Which stool characteristics
should the nurse expect to note documented in the client’s record?

A. Diarrhea
B. Chronic constipation
C. Constipation alternating with diarrhea
D. Stools constantly oozing form the rectum

29. The nurse is performing a colostomy irrigation on a male client. During the irrigation, the client begins
to complain of abdominal cramps. What is the appropriate nursing action?

A. Notify the physician


B. Stop the irrigation temporarily
C. Increase the height of the irrigation
D. Medicate for pain and resume the irrigation

30. The nurse is teaching a female client how to perform a colostomy irrigation. To enhance the
effectiveness of the irrigation and fecal returns, what measure should the nurse instruct the client to do?

A. Increase fluid intake


B. Place heat on the abdomen
C. Perform the irrigation in the evening
D. Reduce the amount of irrigation solution

31. A patient with chronic alcohol abuse is admitted with liver failure. You closely monitor the patient’s
blood pressure because of which change that is associated with the liver failure?

A. Hypoalbuminemia
B. Increased capillary permeability
C. Abnormal peripheral vasodilation
D. Excess rennin release from the kidneys

32. You’re assessing the stoma of a patient with a healthy, well-healed colostomy. You expect the stoma to
appear:

A. Pale, pink and moist


B. Red and moist
C. Dark or purple colored
D. Dry and black
33. You’re caring for a patient with a sigmoid colostomy. The stool from this colostomy is:

A. Formed
B. Semisolid
C. Semiliquid
D. Watery

34. You’re advising a 21 y.o. with a colostomy who reports problems with flatus. What food should you
recommend?

A. Peas
B. Cabbage
C. Broccoli
D. Yogurt

35. You have to teach ostomy self care to a patient with a colostomy. You tell the patient to measure and
cut the wafer:

A. To the exact size of the stoma.


B. About 1/16” larger than the stoma.
C. About 1/8” larger than the stoma.
D. About 1/4″ larger than the stoma.

36. You’re performing an abdominal assessment on Brent who is 52 y.o. In which order do you proceed?

A. Observation, percussion, palpation, auscultation


B. Observation, auscultation, percussion, palpation
C. Percussion, palpation, auscultation, observation
D. Palpation, percussion, observation, auscultation

37. You’re doing preoperative teaching with Gertrude who has ulcerative colitis who needs surgery to
create an ileoanal reservoir. Which information do you include?

A. A reservoir is created that exits through the abdominal wall.


B. A second surgery is required 12 months after the first surgery.
C. A permanent ileostomy is created.
D. The surgery occurs in two stages.

38. You’re caring for Carin who has just had ileostomy surgery. During the first 24 hours post-op, how
much drainage can you expect from the ileostomy?

A. 100 ml
B. 500 ml
C. 1500 ml
D. 5000 ml

39. You’re preparing a teaching plan for a 27 y.o. named Jeff who underwent surgery to close a temporary
ileostomy. Which nutritional guideline do you include in this plan?

A. There is no need to change eating habits.


B. Eat six small meals a day.
C. Eat the largest meal in the evening.
D. Restrict fluid intake.

40. Arthur has a family history of colon cancer and is scheduled to have a sigmoidoscopy. He is crying as
he tells you, “I know that I have colon cancer, too.” Which response is most therapeutic?

A. “I know just how you feel.”


B. “You seem upset.”
C. “Oh, don’t worry about it, everything will be just fine.”
D. “Why do you think you have cancer?”

41. You’re caring for Beth who underwent a Billroth II procedure (surgical removal of the pylorus and
duodenum) for treatment of a peptic ulcer. Which findings suggest that the patient is developing dumping
syndrome, a complication associated with this procedure?
A. Flushed, dry skin.
B. Headache and bradycardia.
C. Dizziness and sweating.
D. Dyspnea and chest pain.

42. You’re developing the plan of care for a patient experiencing dumping syndrome after a Billroth II
procedure. Which dietary instructions do you include?

A. Omit fluids with meals.


B. Increase carbohydrate intake.
C. Decrease protein intake.
D. Decrease fat intake.

43. You’re caring for Lewis, a 67 y.o. patient with liver cirrhosis who develops ascites and requires
paracentesis. Relief of which symptom indicated that the paracentesis was effective?

A. Pruritus
B. Dyspnea
C. Jaundice
D. Peripheral Neuropathy

44. You’re caring for Jane, a 57 y.o. patient with liver cirrhosis who develops ascites and requires
paracentesis. Before her paracentesis, you instruct her to:

A. Empty her bladder.


B. Lie supine in bed.
C. Remain NPO for 4 hours.
D. Clean her bowels with an enema.

45. After abdominal surgery, your patient has a severe coughing episode that causes wound evisceration. In
addition to calling the doctor, which intervention is most appropriate?

A. Irrigate the wound & organs with Betadine.


B. Cover the wound with a saline soaked sterile dressing.
C. Apply a dry sterile dressing & binder.
D. Push the organs back & cover with moist sterile dressings.

46. You’re caring for Betty with liver cirrhosis. Which of the following assessment findings leads you to
suspect hepatic encephalopathy in her?

A. Asterixis
B. Chvostek’s sign
C. Trousseau’s sign
D. Hepatojugular reflex

47. You are developing a careplan on Sally, a 67 y.o. patient with hepatic encephalopathy. Which of the
following do you include?

A. Administering a lactulose enema as ordered.


B. Encouraging a protein-rich diet.
C. Administering sedatives, as necessary.
D. Encouraging ambulation at least four times a day.

48. You have a patient with achalasia (incomplete muscle relaxtion of the GI tract, especially sphincter
muscles). Which medications do you anticipate to administer?

A. Isosorbide dinitrate (Isordil)


B. Digoxin (Lanoxin)
C. Captopril (Capoten)
D. Propanolol (Inderal)

49. The student nurse is preparing a teaching care plan to help improve nutrition in a patient with achalasia.
You include which of the following:

A. Swallow foods while leaning forward.


B. Omit fluids at mealtimes.
C. Eat meals sitting upright.
D. Avoid soft and semisoft foods.

50. Britney, a 20 y.o. student is admitted with acute pancreatitis. Which laboratory findings do you expect
to be abnormal for this patient?

A. Serum creatinine and BUN


B. Alanine aminotransferase (ALT) and aspartate aminotransferase (AST)
C. Serum amylase and lipase
D. Cardiac enzymes

51. A patient with Crohn’s disease is admitted after 4 days of diarrhea. Which of the following urine
specific gravity values do you expect to find in this patient?

A. 1.005
B. 1.011
C. 1.020
D. 1.030

52. Your goal is to minimize David’s risk of complications after a heriorrhaphy. You instruct the patient to:

A. Avoid the use of pain medication.


B. Cough and deep breathe Q2H.
C. Splint the incision if he can’t avoid sneezing or coughing.
D. Apply heat to scrotal swelling.

53. Janice is waiting for discharge instructions after her herniorrhaphy. Which of the following instructions
do you include?

A. Eat a low-fiber diet.


B. Resume heavy lifting in 2 weeks.
C. Lose weight, if obese.
D. Resume sexual activity once discomfort is gone.

54. Develop a teaching care plan for Angie who is about to undergo a liver biopsy. Which of the following
points do you include?

A. “You’ll need to lie on your stomach during the test.”


B. “You’ll need to lie on your right side after the test.”
C. “During the biopsy you’ll be asked to exhale deeply and hold it.”
D. “The biopsy is performed under general anesthesia.”

55. Stephen is a 62 y.o. patient that has had a liver biopsy. Which of the following groups of signs alert you
to a possible pneumothorax?

A. Dyspnea and reduced or absent breath sounds over the right lung
B. Tachycardia, hypotension, and cool, clammy skin
C. Fever, rebound tenderness, and abdominal rigidity
D. Redness, warmth, and drainage at the biopsy site

56. Michael, a 42 y.o. man is admitted to the med-surg floor with a diagnosis of acute pancreatitis. His BP
is 136/76, pulse 96, Resps 22 and temp 101. His past history includes hyperlipidemia and alcohol abuse. The
doctor prescribes an NG tube. Before inserting the tube, you explain the purpose to patient. Which of the
following is a most accurate explanation?

A. “It empties the stomach of fluids and gas.”


B. “It prevents spasms at the sphincter of Oddi.”
C. “It prevents air from forming in the small intestine and large intestine.”
D. “It removes bile from the gallbladder.”

57. Jason, a 22 y.o. accident victim, requires an NG tube for feeding. What should you immediately do after
inserting an NG tube for liquid enteral feedings?

A. Aspirate for gastric secretions with a syringe.


B. Begin feeding slowly to prevent cramping.
C. Get an X-ray of the tip of the tube within 24 hours.
D. Clamp off the tube until the feedings begin.

58. Stephanie, a 28 y.o. accident victim, requires TPN. The rationale for TPN is to provide:

A. Necessary fluids and electrolytes to the body.


B. Complete nutrition by the I.V. route.
C. Tube feedings for nutritional supplementation.
D. Dietary supplementation with liquid protein given between meals.

59. Type A chronic gastritis can be distinquished from type B by its ability to:

A. Cause atrophy of the parietal cells.


B. Affect only the antrum of the stomach.
C. Thin the lining of the stomach walls.
D. Decrease gastric secretions.

60. Matt is a 49 y.o. with a hiatal hernia that you are about to counsel. Health care counseling for Matt
should include which of the following instructions?

A. Restrict intake of high-carbohydrate foods.


B. Increase fluid intake with meals.
C. Increase fat intake.
D. Eat three regular meals a day.

61. Jerod is experiencing an acute episode of ulcerative colitis. Which is priority for this patient?

A. Replace lost fluid and sodium.


B. Monitor for increased serum glucose level from steroid therapy.
C. Restrict the dietary intake of foods high in potassium.
D. Note any change in the color and consistency of stools.

62. A 29 y.o. patient has an acute episode of ulcerative colitis. What diagnostic test confirms this diagnosis?

A. Barium Swallow.
B. Stool examination.
C. Gastric analysis.
D. Sigmoidoscopy.

63. Eleanor, a 62 y.o. woman with diverticulosis is your patient. Which interventions would you expect to
include in her care?

A. Low-fiber diet and fluid restrictions.


B. Total parenteral nutrition and bed rest.
C. High-fiber diet and administration of psyllium.
D. Administration of analgesics and antacids
.
64. Regina is a 46 y.o. woman with ulcerative colitis. You expect her stools to look like:

A. Watery and frothy.


B. Bloody and mucoid.
C. Firm and well-formed.
D. Alternating constipation and diarrhea.

65. Donald is a 61 y.o. man with diverticulitis. Diverticulitis is characterized by:

A. Periodic rectal hemorrhage.


B. Hypertension and tachycardia.
C. Vomiting and elevated temperature.
D. Crampy and lower left quadrant pain and low-grade fever.

66. Brenda, a 36 y.o. patient is on your floor with acute pancreatitis. Treatment for her includes:

A. Continuous peritoneal lavage.


B. Regular diet with increased fat.
C. Nutritional support with TPN.
D. Insertion of a T tube to drain the pancreas.

67. Glenda has cholelithiasis (gallstones). You expect her to complain of:

A. Pain in the right upper quadrant, radiating to the shoulder.


B. Pain in the right lower quadrant, with rebound tenderness.
C. Pain in the left upper quadrant, with shortness of breath.
D. Pain in the left lower quadrant, with mild cramping

68. After an abdominal resection for colon cancer, Madeline returns to her room with a Jackson-Pratt drain
in place. The purpose of the drain is to:

A. Irrigate the incision with a saline solution.


B. Prevent bacterial infection of the incision.
C. Measure the amount of fluid lost after surgery.
D. Prevent accumulation of drainage in the wound.

69. Anthony, a 60 y.o. patient, has just undergone a bowel resection with a colostomy. During the first 24
hours, which of the following observations about the stoma should you report to the doctor?

A. Pink color.
B. Light edema.
C. Small amount of oozing.
D. Trickles of bright red blood.

70. Your teaching Anthony how to use his new colostomy. How much skin should remain exposed between
the stoma and the ring of the appliance?

A. 1/16”
B. 1/4″
C. 1/2”
D. 1”

71. Claire, a 33 y.o. is on your floor with a possible bowel obstruction. Which intervention is priority for
her?

A. Obtain daily weights.


B. Measure abdominal girth.
C. Keep strict intake and output.
D. Encourage her to increase fluids.

72. Your patient has a GI tract that is functioning, but has the inability to swallow foods. Which is the
preferred method of feeding for your patient?

A. TPN
B. PPN
C. NG feeding
D. Oral liquid supplements

73. You’re patient is complaining of abdominal pain during assessment. What is your priority?

A. Auscultate to determine changes in bowel sounds.


B. Observe the contour of the abdomen.
C. Palpate the abdomen for a mass.
D. Percuss the abdomen to determine if fluid is present.

74. Before bowel surgery, Lee is to administer enemas until clear. During administration, he complains of
intestinal cramps. What do you do next?

A. Discontinue the procedure.


B. Lower the height of the enema container.
C. Complete the procedure as quickly as possible.
D. Continue administration of the enema as ordered without making any adjustments.
75. Leigh Ann is receiving pancrelipase (Viokase) for chronic pancreatitis. Which observation best
indicates the treatment is effective?

A. There is no skin breakdown.


B. Her appetite improves.
C. She loses more than 10 lbs.
D. Stools are less fatty and decreased in frequency.

76. Ralph has a history of alcohol abuse and has acute pancreatitis. Which lab value is most likely to be
elevated?

A. Calcium
B. Glucose
C. Magnesium
D. Potassium

77. Anna is 45 y.o. and has a bleeding ulcer. Despite multiple blood transfusions, her HGB is 7.5g/dl and
HCT is 27%. Her doctor determines that surgical intervention is necessary and she undergoes partial
gastrectomy. Postoperative nursing care includes:

A. Giving pain medication Q6H.


B. Flushing the NG tube with sterile water.
C. Positioning her in high Fowler’s position.
D. Keeping her NPO until the return of peristalsis.

78. Sitty, a 66 y.o. patient underwent a colostomy for ruptured diverticulum. She did well during the
surgery and returned to your med-surg floor in stable condition. You assess her colostomy 2 days after
surgery. Which finding do you report to the doctor?

A. Blanched stoma
B. Edematous stoma
C. Reddish-pink stoma
D. Brownish-black stoma

79. Sharon has cirrhosis of the liver and develops ascites. What intervention is necessary to decrease the
excessive accumulation of serous fluid in her peritoneal cavity?

A. Restrict fluids
B. Encourage ambulation
C. Increase sodium in the diet
D. Give antacids as prescribed

80. Katrina is diagnosed with lactose intolerance. To avoid complications with lack of calcium in the diet,
which food should be included in the diet?

A. Fruit
B. Whole grains
C. Milk and cheese products
D. Dark green, leafy vegetables

81. Nathaniel has severe pruritus due to having hepatitis B. What is the best intervention for his comfort?

A. Give tepid baths.


B. Avoid lotions and creams.
C. Use hot water to increase vasodilation.
D. Use cold water to decrease the itching.

82. Rob is a 46 y.o. admitted to the hospital with a suspected diagnosis of Hepatitis B. He’s jaundiced and
reports weakness. Which intervention will you include in his care?

A. Regular exercise.
B. A low-protein diet.
C. Allow patient to select his meals.
D. Rest period after small, frequent meals.
83. You’re discharging Nathaniel with hepatitis B. Which statement suggests understanding by the patient?

A. “Now I can never get hepatitis again.”


B. “I can safely give blood after 3 months.”
C. “I’ll never have a problem with my liver again, even if I drink alcohol.”
D. “My family knows that if I get tired and start vomiting, I may be getting sick again.”

84. Gail is scheduled for a cholecystectomy. After completion of preoperative teaching, Gail states,”If I lie
still and avoid turning after the operation, I’ll avoid pain. Do you think this is a good idea?” What is the best
response?

A. “You’ll need to turn from side to side every 2 hours.”


B. “It’s always a good idea to rest quietly after surgery.”
C. “The doctor will probably order you to lie flat for 24 hours.”
D. “Why don’t you decide about activity after you return from the recovery room?”

85. You’re caring for a 28 y.o. woman with hepatitis B. She’s concerned about the duration of her recovery.
Which response isn’t appropriate?

A. Encourage her to not worry about the future.


B. Encourage her to express her feelings about the illness.
C. Discuss the effects of hepatitis B on future health problems.
D. Provide avenues for financial counseling if she expresses the need.

86. Elmer is scheduled for a proctoscopy and has an I.V. The doctor wrote an order for 5mg of I.V.
diazepam(Valium). Which order is correct regarding diazepam?

A. Give diazepam in the I.V. port closest to the vein.


B. Mix diazepam with 50 ml of dextrose 5% in water and give over 15 minutes.
C. Give diazepam rapidly I.V. to prevent the bloodstream from diluting the drug mixture.
D. Question the order because I.V. administration of diazepam is contraindicated.

87. Annebell is being discharged with a colostomy, and you’re teaching her about colostomy care. Which
statement correctly describes a healthy stoma?

A. “At first, the stoma may bleed slightly when touched.”


B. “The stoma should appear dark and have a bluish hue.”
C. “A burning sensation under the stoma faceplate is normal.”
D. “The stoma should remain swollen away from the abdomen.”

88. A patient who underwent abdominal surgery now has a gaping incision due to delayed wound healing.
Which method is correct when you irrigate a gaping abdominal incision with sterile normal saline solution,
using a piston syringe?

A. Rapidly instill a stream of irrigating solution into the wound.


B. Apply a wet-to-dry dressing to the wound after the irrigation.
C. Moisten the area around the wound with normal saline solution after the irrigation.
D. Irrigate continuously until the solution becomes clear or all of the solution is used.

89. Hepatic encephalopathy develops when the blood level of which substance increases?

A. Ammonia
B. Amylase
C. Calcium
D. Potassium

90. Your patient recently had abdominal surgery and tells you that he feels a popping sensation in his
incision during a coughing spell, followed by severe pain. You anticipate an evisceration. Which supplies
should you take to his room?

A. A suture kit.
B. Sterile water and a suture kit.
C. Sterile water and sterile dressings.
D. Sterile saline solution and sterile dressings.
91. Findings during an endoscopic exam include a cobblestone appearance of the colon in your patient. The
findings are characteristic of which disorder?

A. Ulcer
B. Crohn’s disease
C. Chronic gastritis
D. Ulcerative colitis

92. What information is correct about stomach cancer?

A. Stomach pain is often a late symptom.


B. Surgery is often a successful treatment.
C. Chemotherapy and radiation are often successful treatments.
D. The patient can survive for an extended time with TPN.

93. Dark, tarry stools indicate bleeding in which location of the GI tract?

A. Upper colon.
B. Lower colon.
C. Upper GI tract.
D. Small intestine.

94. A patient has an acute upper GI hemorrhage. Your interventions include:

A. Treating hypovolemia.
B. Treating hypervolemia.
C. Controlling the bleeding source.
D. Treating shock and diagnosing the bleeding source.

95. You promote hemodynamic stability in a patient with upper GI bleeding by:

A. Encouraging oral fluid intake.


B. Monitoring central venous pressure.
C. Monitoring laboratory test results and vital signs.
D. Giving blood, electrolyte and fluid replacement.

96. You’re preparing a patient with a malignant tumor for colorectal surgery and subsequent colostomy.
The patient tells you he’s anxious. What should your initial step be in working with this patient?

A. Determine what the patient already knows about colostomies.


B. Show the patient some pictures of colostomies.
C. Arrange for someone who has a colostomy to visit the patient.
D. Provide the patient with written material about colostomy care.

97. Your patient, Christopher, has a diagnosis of ulcerative colitis and has severe abdominal pain
aggravated by movement, rebound tenderness, fever, nausea, and decreased urine output. This may indicate
which complication?

A. Fistula.
B. Bowel perforation.
C. Bowel obstruction.
D. Abscess.

98. A patient has a severe exacerbation of ulcerative colitis. Long-term medications will probably include:

A. Antacids.
B. Antibiotics.
C. Corticosteroids.
D. Histamine2-receptor blockers.

99. The student nurse is teaching the family of a patient with liver failure. You instruct them to limit which
foods in the patient’s diet?

A. Meats and beans.


B. Butter and gravies.
C. Potatoes and pastas.
D. Cakes and pastries.

100. An intubated patient is receiving continuous enteral feedings through a Salem sump tube at a rate of
60ml/hr. Gastric residuals have been 30-40ml when monitored Q4H. You check the gastric residual and
aspirate 220ml. What is your first response to this finding?

A. Notify the doctor immediately.


B. Stop the feeding, and clamp the NG tube.
C. Discard the 220ml, and clamp the NG tube.
D. Give a prescribed GI stimulant such as metoclopramide (Reglan).

Part 1

1. Nurse Berlinda is assigned to a 41-year-old client who has a diagnosis of


chronic pancreatitis. The nurse reviews the laboratory result, anticipating a
laboratory report that indicates a serum amylase level of:

a. 45 units/L
b. 100 units/L
c. 300 units/L
d. 500 units/L

2. A male client who is recovering from surgery has been advanced from a clear
liquid diet to a full liquid diet. The client is looking forward to the diet change
because he has been “bored” with the clear liquid diet. The nurse would offer
which full liquid item to the client?

a. Tea
b. Gelatin
c. Custard
d. Popsicle
3. Nurse Juvy is caring for a client with cirrhosis of the liver. To minimize the
effects of the disorder, the nurse teaches the client about foods that are high in
thiamine. The nurse determines that the client has the best understanding of the
dietary measures to follow if the client states an intension to increase the intake
of:

a. Pork
b. Milk
c. Chicken
d. Broccoli

4. Nurse Oliver checks for residual before administering a bolus tube feeding to a
client with a nasogastric tube and obtains a residual amount of 150 mL. What is
appropriate action for the nurse to take?

a. Hold the feeding


b. Reinstill the amount and continue with administering the feeding
c. Elevate the client’s head at least 45 degrees and administer the feeding
d. Discard the residual amount and proceed with administering the feeding

5. A nurse is inserting a nasogastric


tube in an adult male client. During the procedure, the client begins to cough and
has difficulty breathing. Which of the following is the appropriate nursing action?

a. Quickly insert the tube


b. Notify the physician immediately
c. Remove the tube and reinsert when the respiratory distress subsides
d. Pull back on the tube and wait until the respiratory distress subsides

6. Nurse Ryan is assessing for correct placement of a nosogartric tube. The nurse
aspirates the stomach contents and check the contents for pH. The nurse verifies
correct tube placement if which pH value is noted?

a. 3.5
b. 7.0
c. 7.35
d. 7.5
7. A nurse is preparing to remove a nasogartric tube from a female client. The
nurse should instruct the client to do which of the following just before the nurse
removes the tube?

a. Exhale
b. Inhale and exhale quickly
c. Take and hold a deep breath
d. Perform a Valsalva maneuver

8. Nurse Joy is preparing to administer medication through a nasogastric


tube that is connected to suction. To administer the medication, the nurse
would:

a. Position the client supine to assist in medication absorption


b. Aspirate the nasogastric tube after medication administration to maintain patency
c. Clamp the nasogastric tube for 30 minutes following
administration of the medication
d. Change the suction setting to low intermittent suction for 30 minutes after
medication administration

9. A nurse is preparing to care for a female client with esophageal varices who has
just has a Sengstaken-Blakemore tube inserted. The nurse gathers supplies,
knowing that which of the following items must be kept at the bedside at all
times?

a. An obturator
b. Kelly clamp
c. An irrigation set
d. A pair of scissors

10. Dr. Smith has determined that the client with hepatitis has contracted the
infection form contaminated food. The nurse understands that this client is most
likely experiencing what type of hepatitis?

a. Hepatitis A
b. Hepatitis B
c. Hepatitis C
d. Hepatitis D

11. A client is suspected of having hepatitis. Which diagnostic test result will
assist in confirming this diagnosis?

a. Elevated hemoglobin level


b. Elevated serum bilirubin level
c. Elevated blood urea nitrogen level
d. Decreased erythrocycle sedimentation rate

12. The nurse is reviewing the physician’s orders written for a male client admitted
to the hospital with acute pancreatitis. Which physician order should the nurse
question if noted on the client’s chart?

a. NPO status
b. Nasogastric tube inserted
c. Morphine sulfate for pain
d. An anticholinergic medication

13. A female client being seen in a physician’s office has just been scheduled for a
barium swallow the next day. The nurse writes down which instruction for the client
to follow before the test?
a. Fast for 8 hours before the test
b. Eat a regular supper and breakfast
c. Continue to take all oral medications as scheduled
d. Monitor own bowel movement pattern for constipation

14. The nurse is performing an abdominal assessment and inspects the


skin of the abdomen. The nurse performs which assessment technique
next?

a. Palpates the abdomen for size


b. Palpates the liver at the right rib margin
c. Listens to bowel sounds in all for quadrants
d. Percusses the right lower abdominal quadrant

15. Polyethylene glycol-electrlyte solution (GoLYTELY) is prescribed for the


female client scheduled for a colonoscopy. The client begins to experience
diarrhea following administration of the solution. What action by the nurse is
appropriate?

a. Start an IV infusion
b. Administer an enema
c. Cancel the diagnostic test
d. Explain that diarrhea is expected

16. The nurse is caring for a male client with a diagnosis of chronic gastritis.
The nurse monitors the client knowing that this client is at risk for which
vitamin deficiency?

a. Vitamin A
b. Vitamin B12
c. Vitamin C
d. Vitamin E

17. The nurse is reviewing the medication record of a female client with acute
gastritis. Which medication, if noted on the client’s record, would the nurse
question?

a. Digoxin (Lanoxin)
b. Furosemide (Lasix)
c. Indomethacin (Indocin)
d. Propranolol hydrochloride (Inderal)

18. The nurse is assessing a male client 24 hours following a cholecystectomy. The
nurse noted that the T tube has drained 750 mL of green-brown drainage since the
surgery. Which nursing intervention is appropriate?

a. Clamp the T tube


b. Irrigate the T tube
c. Notify the physician
d. Document the findings

19. The nurse is monitoring a female client with a diagnosis of peptic


ulcer. Which assessment findings would most likely indicate perforation of
the ulcer?

a. Bradycardia
b. Numbness in the legs
c. Nausea and vomiting
d. A rigid, board-like abdomen

20. A male client with a peptic ulcer is scheduled for a vagotomy and the client
asks the nurse about the purpose of this procedure. Which response by the nurse
best describes the purpose of a vagotomy?
a. Halts stress reactions
b. Heals the gastric mucosa
c. Reduces the stimulus to acid secretions
d. Decreases food absorption in the stomach

21. The nurse is caring for a female client following a Billroth II


procedure. Which postoperative order should the nurse question and
verify?

a. Leg exercises
b. Early ambulation
c. Irrigating the nasogastric tube
d. Coughing and deep-breathing exercises

22. The nurse is providing discharge instructions to a male client following


gastrectomy and instructs the client to take which measure to assist in preventing
dumping syndrome?

a. Ambulate following a meal


b. Eat high carbohydrate foods
c. Limit the fluid taken with meal
d. Sit in a high-Fowler’s position during meals

23. The nurse is monitoring a female client for the early signs and symptoms of
dumping syndrome. Which of the following indicate this occurrence?

a. Sweating and pallor


b. Bradycardia and indigestion
c. Double vision and chest pain
d. Abdominal cramping and pain

24. The nurse is preparing a discharge teaching plan for the male client who had
umbilical hernia repair. What should the nurse include in the plan?

a. Irrigating the drain


b. Avoiding coughing
c. Maintaining bed rest
d. Restricting pain medication

25. The nurse is instructing the male client who has an inguinal hernia repair how
to reduce postoperative swelling following the procedure. What should the nurse tell
the client?

a. Limit oral fluid


b. Elevate the scrotum
c. Apply heat to the abdomen
d. Remain in a low-fiber diet

26. The nurse is caring for a hospitalized female client with a diagnosis of ulcerative
colitis. Which finding, if noted on assessment of the client, would the nurse report
to the physician?

a. Hypotension
b. Bloody diarrhea
c. Rebound tenderness
d. A hemoglobin level of 12 mg/dL

27. The nurse is caring for a male client postoperatively following creation of a
colostomy. Which nursing diagnosis should the nurse include in the plan of care?

a. Sexual dysfunction
b. Body image, disturbed
c. Fear related to poor prognosis
d. Nutrition: more than body requirements, imbalanced

28. The nurse is reviewing the record of a female client with Crohn’s disease.
Which stool characteristics should the nurse expect to note documented in the
client’s record?

a. Diarrhea
b. Chronic constipation
c. Constipation alternating with diarrhea
d. Stools constantly oozing form the rectum

29. The nurse is performing a colostomy irrigation on a male client. During the
irrigation, the client begins to complain of abdominal cramps. What is the
appropriate nursing action?

a. Notify the physician


b. Stop the irrigation temporarily
c. Increase the height of the irrigation
d. Medicate for pain and resume the irrigation

30. The nurse is teaching a female client how to perform a colostomy irrigation. To
enhance the effectiveness of the irrigation and fecal returns, what measure should
the nurse instruct the client to do?

a. Increase fluid intake


b. Place heat on the abdomen
c. Perform the irrigation in the evening
d. Reduce the amount of irrigation solution

Part 2

1. During preparation for bowel surgery, a male client receives an antibiotic to


reduce intestinal bacteria. Antibiotic therapy may interfere with synthesis of which
vitamin and may lead to hypoprothrombinemia?

a. vitamin A
b. vitamin D
c. vitamin E
d. vitamin K

2. When evaluating a male client for complications of acute pancreatitis, the


nurse would observe for:

a. increased intracranial pressure.


b. decreased urine output.
c. bradycardia.
d. hypertension.

3. A male client with a recent history of rectal bleeding is being prepared for a
colonoscopy.How should the nurse position the client for this test initially?

a. Lying on the right side with legs straight


b. Lying on the left side with knees bent
c. Prone with the torso elevated
d. Bent over with hands touching the floor

4. A male client with extreme weakness, pallor, weak peripheral pulses, and
disorientation is admitted to the emergency department. His wife reports that he
has been “spitting up blood.” A Mallory-Weiss tear is suspected, and the nurse
begins taking a client history from the client’s wife. The question by the nurse that
demonstrates her understanding of Mallory- Weiss tearing is:

a. “Tell me about your husband’s alcohol usage.”


b. “Is your husband being treated for tuberculosis?”
c. “Has your husband recently fallen or injured his chest?”
d. “Describe spices and condiments your husband uses on food.”

5. Which of the following nursing interventions should the nurse perform for a
female client receiving enteral feedings through a gastrostomy tube?

a. Change the tube feeding solutions and tubing at least every 24 hours.
b. Maintain the head of the bed at a 15-degree elevation continuously.
c. Check the gastrostomy tube for position every 2 days.
d. Maintain the client on bed rest during the feedings.

6. A male client is recovering from a small-bowel resection. To relieve pain, the


physician prescribes meperidine (Demerol), 75 mg I.M. every 4 hours. How soon
after administration should meperidine’s onset of action occur?

a. 5 to 10 minutes
b. 15 to 30 minutes
c. 30 to 60 minutes
d. 2 to 4 hours

7. The nurse is caring for a male client with cirrhosis. Which assessment findings
indicate that the client has deficient vitamin K absorption caused by this hepatic
disease?

a. Dyspnea and fatigue


b. Ascites and orthopnea
c. Purpura and petechiae
d. Gynecomastia and testicular atrophy

8. Which condition is most likely to have a nursing diagnosis of fluid volume deficit?

a. Appendicitis
b. Pancreatitis
c. Cholecystitis
d. Gastric ulcer

9. While a female client is being prepared for discharge, the nasogastric (NG)
feeding tube becomes clogged. To remedy this problem and teach the client’s
family how to deal with it at home, what should the nurse do?

a. Irrigate the tube with cola.


b. Advance the tube into the intestine.
c. Apply intermittent suction to the tube.
d. Withdraw the obstruction with a 30-ml syringe.

10. A male client with pancreatitis complains of pain. The nurse expects the
physician to prescribe meperidine (Demerol) instead of morphine to relieve pain
because:
a. meperidine provides a better, more prolonged analgesic effect.
b. morphine may cause spasms of Oddi’s sphincter.
c. meperidine is less addictive than morphine.
d. morphine may cause hepatic dysfunction.

11. Mandy, an adolescent girl is admitted to an acute care facility with severe
malnutrition. After a thorough examination, the physician diagnoses anorexia
nervosa. When developing the plan of care for this client, the nurse is most likely
to include which nursing diagnosis?

a. Hopelessness
b. Powerlessness
c. Chronic low self esteem
d. Deficient knowledge

12. Which diagnostic test would be used first to evaluate a client with upper GI
bleeding?

a. Endoscopy
b. Upper GI series
c. Hemoglobin (Hb) levels and hematocrit (HCT)
d. Arteriography

13. A female client who has just been diagnosed with hepatitis A asks, “How
could I have gotten this disease?” What is the nurse’s best response?

a. “You may have eaten contaminated restaurant food.”


b. “You could have gotten it by using I.V. drugs.”
c. “You must have received an infected blood transfusion.”
d. “You probably got it by engaging in unprotected sex.”

14. When preparing a male client, age 51, for surgery to treat appendicitis, the
nurse formulates a nursing diagnosis of Risk for infection related to inflammation,
perforation, and surgery. What is the rationale for choosing this nursing diagnosis?

a. Obstruction of the appendix may increase venous drainage and cause the
appendix to rupture.
b. Obstruction of the appendix reduces arterial flow, leading to
ischemia, inflammation, and rupture of the appendix.
c. The appendix may develop gangrene and rupture, especially in a middle-aged
client.
d. Infection of the appendix diminishes necrotic arterial blood flow and increases
venous drainage.

15. A female client with hepatitis C develops liver failure and GI hemorrhage.
The blood products that would most likely bring about hemostasis in the client
are:

a. whole blood and albumin.


b. platelets and packed red blood cells.
c. fresh frozen plasma and whole blood.
d. cryoprecipitate and fresh frozen plasma.

16. To prevent gastroesophageal reflux in a male client with hiatal hernia, the
nurse should provide which discharge instruction?

a. “Lie down after meals to promote digestion.”


b. “Avoid coffee and alcoholic beverages.”
c. “Take antacids with meals.”
d. “Limit fluid intake with meals.”
17. The nurse caring for a client with small-bowel obstruction would plan to
implement which nursing intervention first?

a. Administering pain medication


b. Obtaining a blood sample for laboratory studies
c. Preparing to insert a nasogastric (NG) tube
d. Administering I.V. fluids

18. A female client with dysphagia is being prepared for discharge. Which outcome
indicates that the client is ready for discharge?

a. The client doesn’t exhibit rectal tenesmus.


b. The client is free from esophagitis and achalasia.
c. The client reports diminished duodenal inflammation.
d. The client has normal gastric structures.

19. A male client undergoes total gastrectomy. Several hours after surgery, the
nurse notes that the client’s nasogastric (NG) tube has stopped draining. How
should the nurse respond?

a. Notify the physician


b. Reposition the tube
c. Irrigate the tube
d. Increase the suction level

20. What laboratory finding is the primary diagnostic indicator for pancreatitis?

a. Elevated blood urea nitrogen (BUN)


b. Elevated serum lipase
c. Elevated aspartate aminotransferase (AST)
d. Increased lactate dehydrogenase (LD)

21. A male client with cholelithiasis has a gallstone lodged in the common bile
duct. When assessing this client, the nurse expects to note:

a. yellow sclerae.
b. light amber urine.
c. circumoral pallor.
d. black, tarry stools.

22. Nurse Hannah is teaching a group of middle-aged men about peptic


ulcers. Whendiscussing risk factors for peptic ulcers, the nurse should
mention:

a. a sedentary lifestyle and smoking.


b. a history of hemorrhoids and smoking.
c. alcohol abuse and a history of acute renal failure.
d. alcohol abuse and smoking.

23. While palpating a female client’s right upper quadrant (RUQ), the nurse would
expect to find which of the following structures?

a. Sigmoid colon
b. Appendix
c. Spleen
d. Liver
24. A male client has undergone a colon resection. While turning him, wound
dehiscence with evisceration occurs. The nurse’s first response is to:

a. call the physician.


b. place saline-soaked sterile dressings on the wound.
c. take a blood pressure and pulse.
d. pull the dehiscence closed.

25. The nurse is monitoring a female client receiving paregoric to treat diarrhea
for drug interactions. Which drugs can produce additive constipation when given
with an opium preparation?

a. Antiarrhythmic drugs
b. Anticholinergic drugs
c. Anticoagulant drugs
d. Antihypertensive drugs

26. A male client is recovering from an ileostomy that was performed to treat
inflammatory bowel disease. During discharge teaching, the nurse should stress the
importance of:

a. increasing fluid intake to prevent dehydration.


b. wearing an appliance pouch only at bedtime.
c. consuming a low-protein, high-fiber diet.
d. taking only enteric-coated medications.

27. The nurse is caring for a female client with active upper GI bleeding.
What is the appropriate diet for this client during the first 24 hours after
admission?

a. Regular diet
b. Skim milk
c. Nothing by mouth
d. Clear liquids

28. A male client has just been diagnosed with hepatitis A. On assessment,
the nurse expects to note:

a. severe abdominal pain radiating to the shoulder.


b. anorexia, nausea, and vomiting.
c. eructation and constipation.
d. abdominal ascites.

29. A female client with viral hepatitis A is being treated in an acute care facility.
Because the client requires enteric precautions, the nurse should:

a. place the client in a private room.


b. wear a mask when handling the client’s bedpan.
c. wash the hands after touching the client.
d. wear a gown when providing personal care for the client.

30. Which of the following factors can cause hepatitis A?

a. Contact with infected blood


b. Blood transfusions with infected blood
c. Eating contaminated shellfish
d. Sexual contact with an infected person

Part 3
1. The client had been diagnosed to have a cholelithiasis. He had undergone laparoscopic
cholecystectomy. Which of the following does the nurse recognize as normal signs andsymptoms
afterthe surgery?

a) abdominal pain and bloating


b) diminished lung sounds
c) bile-stained vomitus
d) hyperactive bowel sounds

2. The client has been diagnosed to have cancer of the colon. She is for colostomy. The client says, "The
doctor told me that there are complications of colostomy." The best initial action by the nurse is

a) discuss complications of colostomy to the patient


b) provide pre-operation teachings
c) ask what are the complications of colostomy
d) ask the client to sign consent form

3. A nurse is caring for a client with colostomy created 3 days earlier. The client is beginning to pass
malodorous flatus from stoma. The nurse interprets that:

a) this is normal, expected event


b) this indicates inadequate preoperative bowel preparation
c) the client is experiencing early signs of impaired circulation
d) the client should not have the nasogastric tube movement

4. A client who has gastrostomy tube for feeding refuses to participate in the plan of care, will not
make eye contact and does not speak to family or visitors. A nurse assesses that this client is using
which typeof coping mechanism?

a) self-control
b) distancing
c) problem-solving
d) accepting responsibility

5. A nurse is preparing a diet plan for a post-gastrectomy client to prevent dumping syndrome. Which
ofthe following would not be a component of this teaching plan?

a) lie down after eating


b) drink liquids with meals
c) eat small meals, six times daily
d) avoid concentrated sweets

Situation: Mr. Greg, a 49 year old CEO is diagnosed as having ulcer disease.

6. Mr. Greg's ulcer perforates into the peritoneal cavity. To relieve the pain caused by perforation,
Mr.Greg is most likely to:
a) lie on his left side
b) turn into his stomach
c) rigidly maintain the supine position
d) draw his knees up to his abdomen

7. Mr. Greg is placed on the bland diet and receives medications to decrease gastric acidity.
Whichmedication reduces hydrochloride acid secretion?

a) cimetidine (tagamet)
b) sucralfate (carafets)
c) aluminum hydroxide (amphogel)
d) aspirin

8. Mr. Greg is scheduled for an upper GI series. Which intervention should the nurse perform
afterprocedure?

a) testing stool for occult blood


b) give the patient a laxative
c) assessing for the gag reflex
d) administer double dose of antacids to prevent excessive HCL production

9. What diet should the nurse recommend for a child with celiac disease?

a) wheat and oats


b) rice and corn
c) cookies and ice cream
d) pasta and noodles

10. Which of the following foods should not be included in the diet of the client with diverticulitis?

a) rice and steamed chicken


b) tomato and cucumber
c) pasta and orange slices
d) roasted turkey and spaghetti

11. A client had undergone gastric resection. Which of the following is not to be included in the
nursingcare plan for the client to prevent dumping syndrome?

a) small, frequent feeding


b) high protein, low carbohydrate diet
c) lying down after meals
d) taking fluids with meals

12. When is the best time to administer sucralfate?

a) one hour before meals


b) 30 minutes after meals
c) with meals
d) 2 hours after meals

13. Which of the following should the nurse include when giving health teachings in a client
withgastroesophageal reflux?

a) lie down after meals


b) sleep with the head of bed elevated
c) eat high carbohydrate diet
d) eat low protein diet
14. Which of the following should the nurse advise to a client who had undergone partial
gastrectomy?

a) drink fluid with meals


b) lie down after meals
c) increase fats in the diet
d) assume upright position during and after meals

15. Which of the following statements when made by the mother of a child with celiac disease
indicatesthat she understands the diet of her child?

a) my child can eat rice


b) my child can eat oats
c) my child can eat biscuits
d) my child can eat pasta

16. Which of the following assessment findings should concern the nurse most, when assessing
clientwho had undergone colonoscopy?

a) abdominal distention
b) 300 ml of bile-stained vomitus
c) complaints of anal pain
d) complaints of drowsiness and fatigue

17. A client has hepatic cirrhosis and gastric bleeding. Which of the following tasks may be delegated
tothe nursing assistant?

a) assist the client in taking a bath


b) hourly intake and output monitoring
c) assist the client to sit before changing the bed linen
d) assist the client in ambulation

18. A client diagnosed with gastric ulcer is for discharge. Which of the following should be included by
thenurse in the health teachings regarding diet?

a) you must eat bland diet


b) you can eat most foods as long as they don't bother your stomach
c) you should refrain from eating fruits and vegetables
d) you should eat low fiber diet

19. The client who was diagnosed to have gastric cancer had undergone gastrectomy. Which of
thefollowing statements when made by the client indicates that he understands the health
teachings

a) I'll take vitamin K for life


b) I'll take vitamin B12 for life
c) I'll take vitamin C for life
d) I'll take vitamin B6 for life

20. The client had been diagnosed to have liver cirrhosis and esophageal varices. Which of the
followingshould the nurse include when giving health teachings? Select all that apply

a) avoid spicy foods


b) avid straining at stool
c) increase fluid intake
d) open mouth if coughing or sneezing could not be avoided
e) avoid bending or stooping
f) take acetaminophen instead of aspirin for pain
g) avoid heavy lifting
21. Which of the following findings indicates effectiveness of Viokase?

a) abdominal pain relieved


b) steatorrhea has decreased
c) vomiting has stopped
d) jaundice has diminished

22. The client had undergone ileostomy. He has nasogastric tube connected to intermittent suction,
withIV fluid and foley catheter. Which of the following physician's instructions requires intervention
by thew nurse?
a) remove the NGT on the third day postop
b) remove foley catheter after 24 hours
c) irrigate ileostomy at bed time
d) clear liquid diet once peristalsis returns

23. Which of the following manifestations characterize pancreatitis?

a) right upper quadrant pain


b) bile-stained vomitus
c) epigastric pain that is not relieved by vomiting
d) elevated serum calcium

24. The client is diagnosed with acute pancreatitis. Which of the following signs and symptoms will
theclient manifest?

a) right upper quadrant (RUQ) pain


b) bluish discoloration at the periumbilical area
c) left lower quadrant (LLQ) pain
d) pain at the epigastric region

25. The client is diagnosed to have acute pancreatitis. Which laboratory findings signify the diagnosis?

a) elevated SGOT, SGPT


b) elevated BUN, serum creatinine
c) elevated FBS, ESR
d) elevated serum amylase, lipase

26. The client had gastrectomy 2 days ago. To prevent dumping syndrome, which of the following is not
acomponent of the interventions?

a) eating small frequent meals


b) lying down after meals for 30 minutes
c) drinking fluids during meals
d) avoiding concentrated sugar

27. After liver biopsy, what is the most appropriate next action of the RN?

a) take vital signs


b) place the client in right sided-lying position
c) place the client in semi-fowler's position
d) place the client in left sided-lying position

28. A fluid challenge is begun with a postop gastric surgery client. Which assessment will give the
bestindication of the client's response to this treatment?

a) CVP (central venous pressure) reading and hourly urine output


b) blood pressure and apical rate
c) lung sounds and arterial blood gases
d) electrolytes, BUN levels

29. Which of the following facts best explains why the duodenum is not removed during a
subtotalgastrectomy?

a) the head of the pancreas is adherent to the duodenal wall


b) the common bile duct empties into the duodenal villi
c) the wall of the jejunum contains no intestinal villi
d) the jejunum receives its blood supply through the duodenum

30. Which of the following expected outcomes should the nurse inform the client after
laparoscopiccholecystectomy?

a) redness and swelling on the operative site


b) serosanguinous drainage on the dressing
c) shoulder pain for 24 hours
d) nausea and vomiting for 24 hours

31. Which of the following interventions does the nurse expect to give to a client after
gastrointestinalseries?

a) analgesic
b) laxative
c) antiemetic
d) sedative

32. The client has been diagnosed to have acute pancreatitis. Which of the following is not a
componentof nursing care for the client?

a) administer morphine sulfate for pain


b) administer calcium supplement as ordered
c) administer digestive enzymes with each meal and snack
d) administer IV therapy as ordered

33. The client has been diagnosed to have VRE (Vancomycin-resistant enterocolitis).Which
of thefollowing is appropriate nursing action when caring for the client?

a) wear mask when entering the client's room


b) wear gloves when caring for the client
c) wear mask and gloves when performing procedures to the client
d) wear gown and mask when caring for the client

34. A patient was diagnosed to have Laennec's cirrhosis. Which of the following symptoms should
beassessed first?

a) inability to write
b) jaundice
c) increased BUN
d) ascites

35. A nurse assists a physician in performing a liver biopsy. After the procedure, which of the
followingpositions should the nurse place the patient?
a) prone position
b) supine position
c) right side-lying position with a pillow under the puncture site
d) left side-lying position with a pillow under the puncture site

36. The client had been diagnosed to have acute pancreatitis. Which of the following signs
andsymptoms most likely are experienced by the client? Select all that apply

a) pain in the left upper quadrant of the abdomen


b) bile-stained vomitus
c) elevated serum amylase
d) hypercalcemia
e) steatorrhea
f) hypoglycemia
g) weight loss

37. The child has been diagnosed to have Hirschprung's disease (Aganglionic megacolon) by
rectalbiopsy. Which of the following findings most likely is experienced by the child?

a) ribbon-like stool
b) currant jelly-like stool
c) olive-sized mass in the abdomen
d) sausage-like mass in the abdomen

38. The nurse assists the physician during paracentesis. In which position does the nurse place the
client?

a) lying position
b) sitting position
c) prone position
d) side-lying position

39. The client had been diagnosed to have complete intestinal obstruction. Which of the
followingassessment findings will the nurse expect?

a) medium-pitched gurgling sounds


b) high-pitched tinkling bowel sounds
c) absence of bowel sounds
d) increased bowel sounds

40. Which of the following physician's orders for a client with acute pancreatitis should be questioned
bythe nurse?

a) zantac 300mg 1 tablet BID


b) morphine sulfate 5mg/IV every 6 hours
c) bland, low fat diet
d) meperidine hydrochloride 50mg/IV every 4 hours

41. The client with liver cirrhosis has developed esophageal varices. Which of the following should
thenurse advise the client to avoid? Select all that apply

a) coughing
b) straining at stool
c) yawning
d) bending and stooping
e) swallowing
f) heavy lifting
g) spicy foods

42. The client had undergone Billroth II surgery. Which of the following health teachings should be
takento the client before discharge? Select all that apply

a) take dry, high protein foods


b) limit concentrated carbohydrates
c) lie down in left side-lying position after meals
d) take fluids with meals
e) take small, frequent meals
f) start meal with hot foods and beverages

43. The nurse is taking care of a patient with a positive clostridium difficile culture result. The
chargenurse will intervene if she observes the nurse to be

a) wearing gloves in handling secretions


b) washing hands before and after entering the room and giving care to the patient
c) instructing the patient to wash hands with antimicrobial soap
d) wearing gown, gloves, mask, and cap while giving care to the patient

44. After gastrojejunostomy, which of the following instructions should be included in the health
teachingsof the patient?

a) limit carbohydrates in your diet


b) increase fluid intake
c) avoid lying down after eating
d) limit proteins in your diet

45. Arrange in sequence the following actions when cleaning an abdominal incision.

a) prepare supplies
b) apply sterile gloves
c) do hand washing apply clean gloves
d) remove soiled dressings
e) clean from the top to the bottom of the abdominal incision
g) apply sterile dressings

- C, D, A, B, E, F

46. A client has Sengstaken-Blakemore tube. The nurse, during change-of-shift report should remind
thenext shift nurse to:

a) keep scissors at bedside


b) avoid instilling fluid into the aspiration port
c) keep tracheostomy tray at bedside
d) deflate the balloon for 15 to 30 minutes every 2 hours

47. The nurse is developing the plan of care for a client receiving continuous ambulatory
peritonealdialysis (CAPD). Which is the priority complication of CAPD to be addressed in the plan
of care?

a) bleeding
b) pain
c) outflow problems
d) infection

Situation: Bobby, a 13 year old is being seen in the emergency room for possible appendicitis.

48. An important nursing action to perform when preparing Bobby for an appendectomy is to:

a) administer saline enemas to cleanse the bowels


b) apply heat to reduce pain
c) measure abdominal girth
d) continuously monitor pain

49. Which of the following would indicate that Bobby's appendix has ruptured?
a) diaphoresis
b) anorexia
c) pain at Mc Burney's point
d) relief from pain

50. A nurse is making a home health visit and finds the client experiencing right lower quadrant
abdominalpain, which has decreased in intensity over the last day. The client also has a rigid abdomen
and a temperature of 103.6 F. The nurse should intervene by:
a) administering Tylenol (acetaminophen) for the elevated temperature
b) advising the client to increase oral fluids
c) asking the client when she last had a bowel movement
d) notifying the physician

51.A nurse is working in the emergency room and receives a client with suspected botulism.
Which action is a priority for the nurse to initiate?

a) administer vaccine for botulism


b) initiate isolation
c) induce vomiting
d) administer antibiotics

52. The nurse is assigned to a 40-year old client who has a diagnosis of chronic pancreatitis. The nurse
reviews the laboratory result, anticipating a laboratory report that indicates a serum amylase level of:

a) 45 units/L
b) 100 units/L
c) 300 units/L
d0 500 units/L

53. An adult client was diagnosed with acute pancreatitis 9 days ago. The nurse interprets that the
client is recovering from this episode if the serum lipase level decreases to which of the following
values, whichis just below the upper limit of normal?

a) 20 unit/L
b) 80 unit/L
c) 135 unit/L
d) 350 unit/L

54. A client who is recently has been started on enteral feedings begins to complain of abdominal
cramping, followed by the passage of two liquid stools. A nurse notes that the client has abdominal
distention as well. The nurse reviews the nutritional content on the label of the can of feeding to see
if ithas which of the following ingredients?

a) lactose
b) sucrose
c) fructose
d) maltose

55. A nurse is caring for a client with cirrhosis of the liver. To minimize the effects of the disorder, the
nurse teaches the client about foods that are high in thiamine. The nurse determines that the client
hasthe best understanding of the dietary measures to follow if the client states an intention to
increase theintake of:

a) pork
b) milk
c) chicken
d) broccoli
56. A nurse is monitoring a postoperative client after abdominal surgery for signs ofcomplications.
Thenurse assesses the client for the presence of Homan's sign and determines that his sign is
positive if which of the following is noted?

a) incisional pain
b) absent bowel sounds
c) pain with dorsiflexion of the foot
d) crackles on auscultation of the lungs
57. A nurse is assessing for correct placement of a nasogastric tube. The nurse aspirates the
stomach contents and checks the contents for pH. The nurse verifies correct tube placement if
which pH value isnoted?

a) 3.5
b) 7.0
c) 7.35
d) 7.5

58. An adult client with a history of gastrointestinal bleeding has a platelet count of 300,000
cells/mm3.Which action by the nurse is most appropriate after seeing the laboratory results?

a) report the abnormally low count


b) report the abnormally high count
c) place the client on bleeding precautions
d) place the normal report in the client's medical record

59. An adult client with cirrhosis has been following a diet with optimal amounts of protein because
neitheran excess or a deficiency of protein has been helpful. The nurseevaluates the client's status as
being most satisfactory if the total protein level in which of the following values?

a) 0.4 g/dL
b) 3.7 g/dL
c) 6.4 g/dL
d) 9.8 g/dL

60. A client who is recovering from surgery has been advanced from a clear liquid diet to a full
liquid diet. The client is looking forward to the diet change because he has been "bored"
with the clear liquid diet. The nurse would offer which full liquid item to the client?

a) tea
b) gelatin
c) custard
d) popsicle
A male client with a history of cirrhosis and alcoholism is admitted with severe
dyspnea resulted to ascites. The nurse should be aware that the ascites is most
likely the result of increased…
a. Pressure in the portal
vein b. Production of
serum albumin c.
Secretion of bile salts

d. Interstitial osmotic pressure

A client has Gastroesophageal Reflux Disease (GERD). The nurse should teach
the client that after every meals, the client should…
a. Rest in sitting position
b. Take a short walk
c. Drink plenty of water
d. Lie down at least 30 minutes

After gastroscopy, an adaptation that indicates major complication


would be: a. Nausea and vomiting
b. Abdominal distention
c. Increased GI
motility d. Difficulty
in swallowing

A client who has undergone a cholecystectomy asks the nurse whether there are
any dietary restrictions that must be followed. Nurse Hilary would recognize that
the dietary teaching was well understood when the client tells a family member
that:
a. “Most people need to eat a high protein diet for 12 months after
surgery” b. “I should not eat those foods that upset me before the
surgery”
c. “I should avoid fatty foods as long as I live”

d. “Most people can tolerate regular diet after this type of surgery”

Nurse Rachel teaches a client who has been recently diagnosed with hepatitis A
about untoward signs and symptoms related to Hepatitis that may develop. The one
that should be reported immediately to the physician is:
a.
Restlessness
b. Yellow
urine c.
Nausea
d. Clay- colored stools

1. On assessment of a patient with a colostomy, you note the stoma is located on the
right area of the abdomen. Due to its location, this is known as what type of colostomy?
 A. Descending Colostomy
 B. Transverse
 C. Ileostomy
 D. Ascending Colostomy
The answer is D. This is known as an ascending colostomy.

2. Thinking back to the patient in Question 1, what type of stool would you expect the
stoma to be excreting?

 A. Liquid stool
 B. Lose to partly formed stool
 C. Similar to normal stool
 D. Semi-solid stool
The answer is A. Stool from an ascending colostomy will be liquid. Stool from a Transverse
Colostomy: lose to partly formed stool, Descending/Sigmoid: similar to normal solid consistency.
An ileostomy will always excrete liquid stool.

3. True or False: An ileostomy is a surgical opening created to bring the large intestine to
the surface of the abdomen.

 True
 False
The answer is FALSE. An ileostomy is a surgical opening created to bring the SMALL (not
large) intestine to the surface of the abdomen.

4. Describe, in order, how food travels from the stomach to the rectum:

 A. It exits the stomach into: the cecum to the jejunum to the ileum, then into the
duodenum, descending colon, transverse colon, ascending colon, sigmoid colon, and

rectum.
 B. It exits the stomach into: the duodenum to the ileum to the jejunum, then into the
cecum, ascending colon, sigmoid colon, descending colon, transverse colon, and rectum.
 C. It exits the stomach into: the ileum to the jejunum to the duodenum, then into the
cecum, sigmoid colon, transverse colon, descending colon, ascending colon, and rectum.
 D. It exits the stomach into: the duodenum to the jejunum to the ileum, then into the
cecum, ascending colon, transverse colon, descending colon, sigmoid colon, and
rectum.
The answer is D.

5. A patient has a double-barrel colostomy of the transverse colon. You note on


assessment two stomas, a proximal and distal stoma. What type of stool do you expect
to drain from the proximal and distal stomas?

 A. Proximal: lose to partly formed stool; Distal: mucous


 B. Proximal: liquid stool; Distal: mucous
 C. Proximal: mucous; Distal: lose to partly formed stool
 D. Proximal & Distal: lose to partly formed stool
The answer is A. The proximal will drain stool while the distal will NOT. The distal will drain
mucous. Since it is a double-barrel colostomy of the transverse colon, you can expect the stool
to be lose to partly formed.

6. A patient is 8 hours post-opt from an colostomy placement. Which finding requires


immediate nursing action?

 A. The stoma is swollen and large.


 B. The stoma is black.
 C. The stoma is not draining any stool.
 D. The patient states the site is tender.
The answer is B. An assessment finding of a stoma being black is not a normal finding but
represents compromised circulation to the stoma. It requires immediate physician notification.
The stoma should look red and be shiny/moist. It is normal for a stoma to be swollen and large
after surgery (this will subside after a few months), and it is normal for the site to be tender due
to the surgery (this will subside as well) and for the stoma to not be draining any stool yet. It can
take approximately 2 day before stool drains from a colostomy.

7. A patient, who had a colostomy placed yesterday, calls on the call light to say their
surgical dressing "fell off". You will re-apply what type of dressing over the stoma?

 A. Wet to dressing
 B. No dressing is needed. You will keep it open to air.
 C. Petroleum gauze dressing
 D. Telfa gauze
The answer is C. A petroleum gauze dressing will be kept in place (or a sterile dry dressing)
until a pouch system is in place.

8. You're providing teaching to a patient with an ileostomy on how to change their pouch
drainage system. Which statement is INCORRECT about how to change a pouching
system for an ostomy?

 A. Empty the pouch when it is 1/3 to 1/2 full.


 B. Change the pouching system every 3-5 days.
 C. When measuring the stoma for skin barrier placement, be sure the opening of
the skin barrier is a 2/3 inch larger than the stoma.
 D. Keep the skin around the stoma clean and dry at all times.
The answer is C. This statement is INCORRECT. When measuring the stoma for skin barrier
placement, be sure the opening of the skin barrier is a 1/8 inch larger than the stoma.....not 2/3
inch.

9. You receive a doctor's order for a patient to take Aspirin EC by mouth daily. The
patient has the following medication history: diabetes type 2, peripheral vascular
disease, and a permanent ileostomy. What is your next nursing action?

 A. Administer the medication as ordered.


 B. Crush the medication and mix it in applesauce.
 C. Hold the medication and notify the doctor the patient has an ileostomy.
 D. Crush the medication and mix it in pudding.
The answer is C. Aspirin EC is an enteric-coated form of Aspirin. A patient with an ileostomy
should not take enteric-coated or sustained-released medications. Enteric-coating medications
don't dissolve until reaching a specific part of the small intestine, and sustained-released
medications release slowly over a period of time. Remember a patient with an ileostomy does
not have the ability to fully utilize the function of the small intestine and this medication will not
be able to perform properly. The nurse should hold the medication and notify the doctor for
further orders.

10. You're providing diet teaching to a patient with an ileostomy. Which foods should the
patient consume in very small amounts or completely avoid?
 A. Peanut butter, bananas, rice
 B. Corn, popcorn, nut and seeds

 C. Grape juice, bread, and pasta


 D. Vinegar, soft drinks, and cured meats
The answer is B. The foods in option B are difficult to digest and could block the stoma. The
patient should either consume these foods in VERY small amounts or avoid them all together.

11. A patient, who has a colostomy, asks what type of foods they should avoid to
decrease odorous gas. You would tell the patient to avoid:

 A. Onions, alcoholic beverages, eggs, and cabbage


 B. Beef, fried foods, lettuce, and rice
 C. Apple, pears, nuts, and wheat
 D. Potatoes, peas, carrots, and chicken
The answer is A. The patient should avoid foods like: onions, alcoholic beverages, eggs, and
cabbage etc. to decrease odorous gas.

12. A patient is 2 days post-opt from an ileostomy placement. Which finding requires
immediate nursing action?

 A. The stoma is excreting liquid stool.

 B. The patient's potassium level is 2.0


 C. The stoma is bright red and moist.
 D. The patient reports mucoid drainage from the anus.
The answer is B. A patient with an ileostomy is at risk for electrolyte imbalances. The nurse
should monitor the patient (especially after ileostomy surgery) for these imbalances. A normal
potassium is 3.5 to 5.1. Therefore, a level of 2.0 requires immediate nursing action. The nurse
should contact the physician for further orders. All the other options are normal findings found
with an ileostomy.

13. Which type of colostomy can allow a patient to have bowel continence?

 A. Descending or Sigmoid Colostomy


 B. Ascending or Transverse Colostomy
 C. Transverse or Descending Colostomy
 D. Ascending or Descending Colostomy
The answer is A. Patients with a colostomy in locations most distal in the GI track have the
highest chance of bowel continence (hence, learn to control their bowel movements).

14. True or False: Patients with an ileostomy are at greater risk for dehydration and an
electrolyte imbalance.

 True
 False
The answer is True.

1. True or False: A patient with Crohn's Disease can experience inflammation in the large
intestine that affects mainly the mucosa (inner layer) of the bowel.

 True
 False
The answer is FALSE: A patient with Crohn's Disease can experience inflammation throughout
the GI Tract (mainly in the terminal ileum and beginning of the colon) from the mouth to anus
(not just the large intestine) and it affects the WHOLE bowel lining (not just the mucosa layer).

2. A patient with Crohn's Disease is MOST likely to have the disease is what part of the GI
tract?

 A. Rectum
 B. Duodenum of the small intestine
 C. Terminal Ileum
 D. Descending colon
The answer is C. Crohn's disease is MOST likely to affect the terminal ileum. However, it may
affect any area of the GI tract.
3. You're providing teaching to a patient who has been newly diagnosed with Crohn's
Disease. Which statement by the patient's spouse requires re-education?

 A. "Crohn's Disease can be scattered throughout the GI tract in patches with some areas

appearing healthy while others are diseased."


 B. "There is no cure for Crohn's Disease."
 C. "Strictures are a common complication with Crohn's Disease."
 D. "Crohn's Disease can cause the haustra of the large intestine to lose its form."
The answer is D. All the statements are true except option D. ULCERATIVE COLITIS can cause
the haustra of the large intestine to lose its form. This is not common with Crohn's Disease.

4. A physician is explaining to a patient that the patient has a type of Crohn's Disease
that is found in both the ileum and colon. As the nurse, you know this type of Crohn's
Disease is called?

 A. Gastroduodenal Crohn's Disease


 B. Granulomatous Colitis
 C. Ileitis
 D. Ileocolitis
The answer is D. This patient has ileocolitis which affects parts of the colon and ileum.
Gastroduodenal Crohns affects the stomach and duodenum which is the first part of the small
intestine. Granulomatous colitis affects only the colon. Ileitis affects the ileum.

5. Select ALL of the following that are complications associated with Crohn's Disease:

 A. Cobble-stone appearance of GI lining


 B. Lead-pipe sign
 C. Toxic megacolon

 D. Fistula

 E. Abscess

 F. Anal Fissure
The answers are A, D, E, and F. These are all complications found with Crohn's Disease. Lead-
pipe sign and toxic megacolon are complications associated with ulcerative colitis.

6. Your patient with Crohn's Disease is admitted with an opening that has formed
between the bowel and bladder. As the nurse, you know this is what type of complication
associated with this disease?

 A. Enterovesical Fistula
 B. Rectovaginal Stricture

 C. Enteroenteric Fistula
 D. Perianal Fissure
The answer is A. These scenario describes a fistula which is an abnormal passage that forms
deep in the wall of the intestine to form an opening between intestine to intestine, intestine to
organ, or intestine to skin’s surface. This specific patient is experiencing an enterovesical fistula
which is an abnormal passage between the bowel and bladder.

7. A patient experiencing a flare-up with Crohn's Disease is ordered complete bowel rest
by the physician. You are administering TPN (total parental nutrition) per physician
order. When developing the patient's nursing plan of care, which nursing diagnosis is
MOST important to include in the care plan?

 A. Risk for allergy response

 B. Risk for unstable blood glucose level


 C. Risk for imbalance nutrition: more than body requirements
 D. Risk for imbalanced nutrition: less than body requirements
The answer is D. Patients with Crohn's Disease are at risk for undernourishment due to how the
disease process effects the body (small intestine is inflamed which is the area of the gut that
absorbs most of the nutrients from food). With severe cases of Crohn's Disease, the physician
may order complete bowel rest (where the patient will be nothing by mouth (NPO) and nothing
will enter the GI system....so the patient is at even more risk for imbalance nutrition. Physicians
may prescribe an IV solution (total parental nutrition) which will be given through a central line
that contains nutrients (so it enters the bloodstream...bypassing the gut). However, there is still
a risk for imbalanced nutrition (less than body requirements) for the patient. The nurse must
monitor the patient's nutrition status very closely such as daily weights, hydration status,
electrolytes, skin, etc.

8. A patient is receiving treatment for Crohn's Disease. Which food found on the patient's
food tray should the patient avoid?
 A. Fresh Salad
 B. White rice
 C. Baked chicken
 D. Cooked skinless apples
The answer is A. Patients who are experiencing flare-ups of Crohn's Disease should avoid high
fiber foods, foods that are hard to digest, spicy foods, dairy products etc. Therefore, the patient
should avoid a fresh salad. This contain vegetables which are high in fiber and hard to digest.
The gut needs to rest. It is best for the patient to consume low fiber and high protein foods.
White rice and fruits/vegetables that are cooked/skinless are low in fiber. Baked chicken is a
good source of protein for the patient.

9. A physician has prescribed a patient with a severe case of Crohn's Disease to take a
drug that works by suppressing the immune system. This medication achieves this by
blocking a protein that plays a role the inflammatory process. Which drug does this
describe?

 A. Azathioprine
 B. Sulfasalazine
 C. Infliximab
 D. Prednisone
The answer is C. Infliximab (Remicade) is a TNF-blocker (biologic drug) which blocks tumor
necrosis factor which plays a role in the inflammatory response system. Azathioprine is an
immunosupressor which suppresses the immune system but does NOT block TNF.
Sulfasalazine is an 5-Aminosalicylate which is an anti-inflammatory medication. Prednisone is a
corticosteroid which decreases inflammation.

10. A patient with Crohn's Disease is taking corticosteroids. The patient is complaining of
extreme thirst, polyuria, and blurred vision. What is your next nursing action?

 A. Check the patient's blood glucose


 B. Give the patient a food containing sugar (ex: orange juice)
 C. Administer oxygen via nasal cannula
 D. D. Assess bowel sounds
The answer is A. A side effect of corticosteroids is hyperglycemia. Extreme thirst, polyuria, and
blurred vision are classic signs and symptoms of hyperglycemia. Therefore, the nurse should
check the patient's blood glucose to confirm the hyperglycemia.
1. A patient had a cholecystectomy and has a t-tube in place. You're helping the nursing
student understand how to care for the t-tube. The nursing student asks you where the t-
tube is located in the body. Your response is the:

 A. Cystic duct
 B. Hepatic duct
 C. Bile duct
 D. Pancreatic duct
The answer is C. The t-tube is located in the bile duct. It will serve as a drain to help remove bile
from the liver until the common bile duct is healed.

2. The nurse helps the patient with a t-tube get up from the bed and sit in the bedside
chair. Where will the nurse make it priority to position the tubing and drainage bag of the
t-tube?

 A. Slightly elevated above the t-tube insertion site


 B. At heart level
 C. Midline with the t-tube insertion site
 D. At or below the waist
The answer is D. The t-tube drainage bag and tubing will work with the assistance of gravity to
drain the bile. Therefore, the tubing and drainage bag should be below the t-tube insertion site
(which is at or below the waist) to help drain bile.

3. Which position is best for a patient with a t-tube?

 A. Supine
 B. Semi-Fowler's
 C. Right lateral recumbent
 D. Left lateral recumbent
The answer is B. To help facilitate drainage (remember in order for the t-tube to work it needs
the assistance of gravity), positioning the patient at about 30-45 degrees (the Semi-Fowler's
position) will be the best.
4. A patient is post-op day 4 from a t-tube placement. Which finding below requires you
to notify the physician?

 A. Drainage from the t-tube is yellowish green.


 B. Drainage from the t-tube within the past 24 hours is approximately 925 cc.
 C. Blood tinged drainage from the t-tube has decreased.
 D. Patient reports a decrease in nausea.
The answer is B. A drainage amount of 500 cc or more within a 24 hour period is abnormal and
the physician should be notified. On post-op day 4 the drainage should be decreasing (NOT
increasing). It is normal for the drainage to be yellowish green. Also blood tinged drainage will
decrease in the t-tube at this time (fresh post-op like day 1-2 it may be blood tinged but this will
decrease over time). The patient reporting a decrease in nausea is a positive sign.

5. The physician orders a patient’s t-tube to be clamped 1 hour before and 1 hour after
meals. You clamp the t-tube as prescribed. While the tube is clamped which finding
requires immediate nursing intervention?

 A. The t-tube is not draining.


 B. The t-tube tubing is below the patient’s waist.

 C. The patient reports nausea and abdominal pain.


 D. The patient’s stool is brown and formed.
The answer is C. A nurse should ONLY clamp a t-tube with a physician’s order. Most physicians
will prescribe to clamp the t-tube 1 hour before and 1 hour after meals. WHY? So, bile will flow
down into the small intestine (instead out of the body) during times when food is in the small
intestine to help with the digestion of fats. This is to help the small intestine adjust to the flow of
bile in preparation for the removal of the t-tube (remember normally it received bile when the
gallbladder contracted but now it will flow from the liver to the small intestine continuously).
Option C is an abnormal finding. The patient should not report nausea or abdominal pain when
the tube is blocked. This could indicate a serious problem. Option A is correct because the t-
tube should not be draining because it’s clamped. Option B is correct because the t-tube tubing
should be below or at the patient’s waist level. Option D is correct because this shows the body
is digesting fats and bilirubin is exiting the body through the stool (remember bilirubin is found in
the bile and gives stool its brown color…it would be light colored if the bilirubin was not present).
You would NOT want to see steatorrhea (fat/greasy liquid stools) because this shows the bile
isn’t being delivered to help digest the fats.
6. You’re assessing a patient’s t-tube and note that it is not draining bile. The patient is
reporting nausea. The nurse will first?

 A. Notify the physician


 B. Assess if the tubing from the t-tube is kinked or clamped.
 C. Flush the tubing.
 D. Administer an antiemetic medication per physician order.

1. The gallbladder is found on the __________ side of the body and is located under the
____________. It stores __________.

 A. right; pancreas; bilirubin


 B. left; liver; bile
 C. right; thymus' bilirubin
 D. right; liver; bile
The answer is D. The gallbladder is found in the RIGHT side of the body and is located under
the LIVER. It stores BILE.

2. Which statements below are CORRECT regarding the role of bile? Select all that apply:

 A. Bile is created and stored in the gallbladder.

 B. Bile aids in digestion of fat soluble vitamins, such as A, D, E, and K.

 C. Bile is released from the gallbladder into the duodenum.

 D. Bile contains bilirubin.


The answer are B, C, and D. Option A is INCORRECT because bile is created in the LIVER (not
gallbladder), but bile is stored in the gallbladder.

3. You’re providing a community in-service about gastrointestinal disorders. During your


teaching about cholecystitis, you discuss how cholelithiasis can lead to this condition.
What are the risk factors for cholelithiasis that you will include in your teaching to the
participants? Select all that apply:

 A. Being male
 B. Underweight

 C. Being female

 D. Older age

 E. Native American
 F. Caucasian

 G. Pregnant

 H. Family History

 I. Obesity
The answers are C, D, E, G, H and I. Cholelithiasis is the formation of gallstones. Risk factors
include: being female, older age (over 40), Native American or Mexican American descent,
pregnant, obesity, and family history.

4. A patient is being transferred to your unit with acute cholecystitis. In report the
transferring nurse tells you that the patient has a positive Murphy’s Sign. You know that
this means:

 A. The patient stops breathing in when the examiner palpates under the ribs on the
right upper side of the abdomen at the midclavicular line.
 B. The patient stops breathing out when the examiner palpates under the ribs on the right
upper side of the abdomen at the midclavicular line.

 C. The patient verbalizes pain when the lower right quadrant is palpated.
 D. The patient reports pain when pressure is applied to the right lower quadrant but then
reports an increase in pain intensity when the pressure is released.
The answer is A. Murphy’s Sign can occur with cholecystitis. This occurs when the patient is
placed in the supine position and the examiner palpates under the ribs on the right upper side of
the abdomen. The examiner will have the patient breathe out and then take a deep breath in.
The examiner will simultaneously (while the patient is breathing in) palpate on this area under
the ribs at the midclavicular line (hence the location of the gallbladder). It is a POSITIVE
Murphy’s Sign when the patient stops breathing in during palpation due to pain.
5. Your patient is post-op day 3 from a cholecystectomy due to cholecystitis and has a T-
Tube. Which finding during your assessment of the T-Tube requires immediate nursing
intervention?

 A. The drainage from the T-Tube is yellowish/green in color.


 B. There is approximately 750 cc of drainage within the past 24 hours.
 C. The drainage bag and tubing is at the patient's waist.

 D. The patient is in the Semi-Fowler's position.


The answer is B. A T-Tube should not drain more than about 500 cc of drainage per day (within
24 hours). A T-Tube’s drainage will go from bloody tinged (fresh post-op) to yellowish/green
within 2-3 days. The drainage bag and tubing should be below the site of insertion (at or below
the patient's waist so gravity can help drainage the bile), and the patient should be in Semi-
Fowler's to Fowler's position to help with draining the bile.

6. The physician orders a patient’s T-Tube to be clamped 1 hour before and 1 hour after
meals. You clamp the T-Tube as prescribed. While the tube is clamped which finding
requires you to notify the physician?

 A. The T-Tube is not draining.


 B. The T-Tube tubing is below the patient’s waist.
 C. The patient reports nausea and abdominal pain.
 D. The patient’s stool is brown and formed.
The answer is C. A nurse should ONLY clamp a T-Tube with a physician’s order. Most
physicians will prescribe to clamp the T-tube 1 hour before and 1 hour after meals. WHY? So,
bile will flow down into the small intestine (instead out of the body) during times when food is in
the small intestine to help with the digestion of fats. This is to help the small intestine adjust to
the flow of bile in preparation for the removal of the t-tube (remember normally it received bile
when the gallbladder contracted but now it will flow from the liver to the small intestine
continuously). Option C is an abnormal finding. The patient should not report nausea or
abdominal pain when the tube is blocked. This could indicate a serious problem. Option A is
correct because the T-tube should not be draining because it’s clamped. Option B is correct
because the T-tube tubing should be below or at the patient’s waist level. Option D is correct
because this shows the body is digesting fats and bilirubin is exiting the body through the stool
(remember bilirubin is found in the bile and gives stool its brown color…it would be light colored
if the bilirubin was not present). You would NOT want to see steatorrhea (fat/greasy liquid
stools) because this shows the bile isn’t being delivered to help digest the fats.

7. Your recent admission has acute cholecystitis. The patient is awaiting a


cholecystostomy. What signs and symptoms are associated with this condition? Select
all that apply:
 A. Right lower quadrant pain with rebound tenderness
 B. Negative Murphy’s Sign

 C. Epigastric pain that radiates to the right scapula

 D. Pain and fullness that increases after a greasy or spicy meal

 E. Fever

 F. Tachycardia

 G. Nausea
The answers are C, D, E, F, and G. Option A and B are not associated with cholecystitis, but a
POSITIVE Murphy’s Sign is.

8. A patient in the emergency room has signs and symptoms associated with
cholecystitis. What testing do you anticipate the physician will order to help diagnose
cholecystitis? Select all that apply:

 A. Lower GI series

 B. Abdominal ultrasound

 C. HIDA Scan (Hepatobiliary Iminodiacetic AciD scan)


 D. Colonoscopy
The answers are B and C. These two tests can assess for cholecystitis. A lower GI series would
not assess the gallbladder but the lower portions of the GI system like the rectum and large
intestine. Option D is wrong because it would also assess the lower portions of the GI system.

9. You’re precepting a nursing student who is helping you provide T-Tube drain care.
You explain to the nursing student that the t-shaped part of the drain is located in what
part of the biliary tract?

 A. Cystic duct
 B. Common hepatic duct
 C. Common bile duct
 D. Pancreatic duct
The answer is C. The “T-shaped” part of the drain is located in the common bile duct and helps
deliver bile to the duodenum (small intestine).

10. Your patient is unable to have a cholecystectomy for the treatment of cholecystitis.
Therefore, a cholecystostomy tube is placed to help treat the condition. Which statement
about a cholecystostomy (C-Tube) is TRUE?

 A. The C-Tube is placed in the cystic duct of the gallbladder and helps drain infected bile
from the gallbladder.
 B. Gallstones regularly drain out of the C-Tube, therefore, the nurse should flush the tube

regularly to ensure patency.


 C. The C-Tube is placed through the abdominal wall and directly into the
gallbladder where it will drain infected bile from the gallbladder.
 D. The tubing and drainage bag of the C-Tube should always be level with the insertion
site to ensure the tube is draining properly.
The answer is C. This is the only correct statement about a cholecystostomy. A
cholecystostomy, also sometimes called a C-Tube, is placed when a patient can't immediately
have the gallbladder removed (cholecystectomy) due to cholecystitis. It is placed through the
abdominal wall and into the gallbladder. It will drain infected bile (NOT gallstones). The tubing
and drainage bag should be at or below waist level so it drains properly.

11. A patient, who has recovered from cholecystitis, is being discharged home. What
meal options below are best for this patient?

 A. Baked chicken with steamed carrots and rice


 B. Broccoli and cheese casserole with gravy and mashed potatoes
 C. Cheeseburger with fries
 D. Fried chicken with a baked potato
The answer is A. The patient should eat a low-fat diet and avoid greasy/fatty/gassy foods.
Option B is wrong because this contains dairy/animal fat like the cheese and gravy, and broccoli
is known to cause gas. Option C and D are greasy food options.

12. Your patient is diagnosed with acute cholecystitis. The patient is extremely
nauseous. A nasogastric tube is inserted with GI decompression. The patient reports a
pain rating of 9 on 1-10 scale and states the pain radiates to the shoulder blade. Select all
the appropriate nursing interventions for the patient:

 A. Encourage the patient to consume clear liquids.

 B. Administered IV fluids per MD order.

 C. Provide mouth care routinely.

 D. Keep the patient NPO.

 E. Administer analgesic as ordered.


 F. Maintain low intermittent suction to NG tube.

1. The liver receives blood from two sources. The _____________ is responsible for
pumping blood rich in nutrients to the liver.

 A. hepatic artery
 B. hepatic portal vein
 C. mesenteric artery
 D. hepatic iliac vein
The answer is B. The liver receives blood from two sources. The hepatic portal vein is
responsible for pumping blood rich in nutrients to the liver.

2. Which statements are INCORRECT regarding the anatomy and physiology of the liver?
Select all that apply:

 A. The liver has 3 lobes and 8 segments.


 B. The liver produces bile which is released into the small intestine to help digest fats.

 C. The liver turns urea, a by-product of protein breakdown, into ammonia.


 D. The liver plays an important role in the coagulation process.
The answers are A and C. The liver has 2 lobes (not 3), and the liver turns ammonia (NOT
urea), which is a by-product of protein breakdown, into ammonia. All the other statements are
true about liver’s anatomy and physiology.

3. You’re providing an in-service on viral hepatitis to a group of healthcare workers. You


are teaching them about the types of viral hepatitis that can turn into chronic infections.
Which types are known to cause ACUTE infections ONLY? Select all that apply:

 A. Hepatitis A
 B. Hepatitis B
 C. Hepatitis C
 D. Hepatitis D

 E. Hepatitis E
The answers are A and E. Only Hepatitis A and E cause ACUTE infections…not chronic.
Hepatitis B, C, and D can cause both acute and chronic infections.

4. Which patients below are at risk for developing complications related to a chronic
hepatitis infection, such as cirrhosis, liver cancer, and liver failure? Select all that apply:

 A. A 55-year-old male with Hepatitis A.

 B. An infant who contracted Hepatitis B at birth.

 C. A 32-year-old female with Hepatitis C who reports using IV drugs.

 D. A 50-year-old male with alcoholism and Hepatitis D.


 E. A 30-year-old who contracted Hepatitis E.
The answers are B, C, and D. Infants or young children who contract Hepatitis B are at a very
high risk of developing chronic Hepatitis B (which is why option B is correct). Option C is correct
because most cases of Hepatitis C turn into chronic cases and IV drug use increases this risk
even more. Option D is correct because Hepatitis D occurs when Hepatitis B is present and
constant usage of alcohol damages the liver. Therefore, the patient is at high risk of developing
chronic hepatitis. Hepatitis A and E tend to only cause acute infections….not chronic.

5. A patient is diagnosed with Hepatitis A. The patient asks how a person can become
infected with this condition. You know the most common route of transmission is?
 A. Blood
 B. Percutaneous

 C. Mucosal
 D. Fecal-oral
The answer is D. Hepatitis A is most commonly transmitted via the fecal-oral route.

6. Which of the following is NOT a common source of transmission for Hepatitis A?


Select all that apply:

 A. Water
 B. Food

 C. Semen

 D. Blood
The answers are C and D. The most common source for transmission of Hepatitis A is water
and food.

7. A 36-year-old patient’s lab work show anti-HAV and IgG present in the blood. As the
nurse you would interpret this blood work as?

 A. The patient has an active infection of Hepatitis A.


 B. The patient has recovered from a previous Hepatitis A infection and is now
immune to it.

 C. The patient is in the preicetric phase of viral Hepatitis.


 D. The patient is in the icteric phase of viral Hepatitis.
The answer is B. When a patient has anti-HAV (antibodies of the Hepatitis A virus) and IgG, this
means the patient HAD a past infection of Hepatitis A but it is now gone, and the patient is
immune to Hepatitis A now. If the patient had anti-HAV and IgM, this means the patient has an
active infection of Hepatitis A.

8. TRUE or FALSE: A patient with Hepatitis A is contagious about 2 weeks before signs
and symptoms appear and 1-3 weeks after the symptoms appear.
 True

 False
The answer is TRUE.

9. A 25-year-old patient was exposed to the Hepatitis A virus at a local restaurant one
week ago. What education is important to provide to this patient?

 A. Inform the patient to notify the physician when signs and symptoms of viral Hepatitis

start to appear.
 B. Reassure the patient the chance of acquiring the virus is very low.
 C. Inform the patient it is very important to obtain the Hepatitis A vaccine immediately to
prevent infection.
 D. Inform the patient to promptly go to the local health department to receive
immune globulin.
The answer is D. Since the patient was exposed to Hepatitis A, the patient would need to take
preventive measures to prevent infection because infection is possible. The patient should not
wait until signs and symptoms appear because the patient can be contagious 2 weeks BEFORE
signs and symptoms appear. The vaccine would not prevent Hepatitis A from this exposure, but
from possible future exposures because it takes the vaccine 30 days to start working. The best
answer is option D. The patient would need to receive immune globulin to provide temporary
immunity within 2 weeks of exposure.

10. Select all the ways a person can become infected with Hepatitis B:

 A. Contaminated food/water

 B. During the birth process

 C. IV drug use

 D. Undercooked pork or wild game

 E. Hemodialysis

 F. Sexual intercourse
The answers are B, C, E, and F. Hepatitis B is spread via blood and body fluids. It could be
transmitted via the birthing process, IV drug use, hemodialysis, or sexual intercourse etc.
11. A patient has completed the Hepatitis B vaccine series. What blood result below
would demonstrate the vaccine series was successful at providing immunity to Hepatitis
B?

 A. Positive IgG
 B. Positive HBsAg
 C. Positive IgM
 D. Positive anti-HBs
The answer is D. A positive anti-HBs (Hepatitis B surface antibody) indicates either a past
infection of Hepatitis B that is now cleared and the patient is immune, OR that the vaccine has
been successful at providing immunity. A positive HBsAg (Hepatitis B surface antigen) indicates
an active infection.

12. A patient has lab work drawn and it shows a positive HBsAg. What education will you
provide to the patient?

 A. Avoid sexual intercourse or intimacy such as kissing until blood work is


negative.
 B. The patient is now recovered from a previous Hepatitis B infection and is now immune.
 C. The patient is not a candidate from antiviral or interferon medications.

 D. The patient is less likely to develop a chronic infection.


The answer is A. A positive HBsAg (hepatitis B surface antigen) indicates an active Hepatitis B
infection. Therefore, the patient should avoid sexual intercourse and other forms of intimacy
until their HBsAg is negative.

13. A patient with Hepatitis A asks you about the treatment options for this condition.
Your response is?

 A. Antiviral medications
 B. Interferon
 C. Supportive care

 D. Hepatitis A vaccine
The answer is C. There is no current treatment for Hepatitis A but supportive care and rest.
Treatments for the other types of Hepatitis such as B, C, and D include antiviral or interferon
(mainly the chronic cases) along with rest.

14. A patient was exposed to Hepatitis B recently. Postexposure precautions include


vaccination and administration of HBIg (Hepatitis B Immune globulin). HBIg needs to be
given as soon as possible, preferably ___________ after exposure to be effective.

 A. 2 weeks
 B. 24 hours
 C. 1 month
 D. 7 days
The answer is B. HBIg should be given 24 hours after exposure to maximum effectiveness of
temporary immunity against Hepatitis B. It would be given within 12 hours after birth to an infant
born to a mother who has Hepatitis B.

15. You’re providing education to a patient with an active Hepatitis B infection. What will
you include in their discharge instructions? Select all that apply:

 A. “Take acetaminophen as needed for pain.”


 B. “Eat large meals that are spread out through the day.”

 C. “Follow a diet low in fat and high in carbs.”


 D. “Do not share toothbrushes, razors, utensils, drinking cups, or any other type of

personal hygiene product.”


 E. “Perform aerobic exercises daily to maintain strength.”
The answers are C and D. The patient should NOT take acetaminophen (Tylenol) due to its
effective on the liver. The patient should eat small (NOT large), but frequent meals…this may
help with the nausea. The patient should rest (not perform aerobic exercises daily) because this
will help with liver regeneration.

16. What is the MOST common transmission route of Hepatitis C?

 A. Blood transfusion

 B. Sharps injury
 C. Long-term dialysis
 D. IV drug use
The answer is D. IV drug use is the MOST common transmission route of Hepatitis C.

17. A patient is diagnosed with Hepatitis D. What statement is true about this type of viral
Hepatitis? Select all that apply:

 A. The patient will also have the Hepatitis B virus.


 B. Hepatitis D is most common in Southern and Eastern Europe, Mediterranean,

and Middle East.


 C. Prevention of Hepatitis D includes handwashing and the Hepatitis D vaccine.
 D. Hepatitis D is most commonly transmitted via the fecal-oral route.
The answers are A and B. These are true statements about Hepatitis D. Prevention for Hepatitis
D includes handwashing and the Hepatitis B vaccine (since it occurs only with the Hepatitis B
virus). It is transmitted via blood.

18. Select all the signs and symptoms associated with Hepatitis?

 A. Arthralgia
 B. Bilirubin 1 mg/dL
 C. Ammonia 15 mcg/dL

 D. Dark urine
 E. Vision changes

 F. Yellowing of the sclera

 G. Fever

 H. Loss of appetite
The answers are A, D, F, G, and H. The bilirubin and ammonia levels are normal in these
options, but they would be abnormal in Hepatitis. A normal bilirubin is 1 or less, and a normal
ammonia is 15-45 mcg/dL.
19. A patient with Hepatitis has a bilirubin of 6 mg/dL. What findings would correlate with
this lab result? Select all that apply:

 A. None because this bilirubin level is normal

 B. Yellowing of the skin and sclera

 C. Clay-colored stools
 D. Bluish discoloration on the flanks of the abdomen

 E. Dark urine

 F. Mental status changes


The answers are B, C, and E. This is associated with a high bilirubin level. A normal bilirubin
level is 1 or less.

20. A patient with Hepatitis is extremely confused. The patient is diagnosed with Hepatic
Encephalopathy. What lab result would correlate with this mental status change?

 A. Ammonia 100 mcg/dL


 B. Bilirubin 7 mg/dL
 C. ALT 56 U/L
 D. AST 10 U/L
The answer is A. When ammonia levels become high (normal 15-45 mcg/dL) it affects brain
function. Therefore, the nurse would see mental status changes in a patient with this ammonia
level.

21. The physician writes an order for the administration of Lactulose. What lab result
indicates this medication was successful?

 A. Bilirubin <1 mg/dL


 B. ALT 8 U/L

 C. Ammonia 16 mcg/dL
 D. AST 10 U/L
The answer is C. Lactulose is ordered to decrease a high ammonia level. It will cause excretion
of ammonia via the stool. A normal ammonia level would indicate the medication was successful
(normal ammonia level 15-45 mcg/dL).

22. How is Hepatitis E transmitted?

 A. Fecal-oral
 B. Percutaneous
 C. Mucosal
 D. Body fluids
The answer is A.

23. Which patient below is at MOST risk for developing a complication related to a
Hepatitis E infection?

 A. A 45-year-old male with diabetes.


 B. A 26-year-old female in the 3rd trimester of pregnancy.

 C. A 12-year-old female with a ventricle septal defect.


 D. A 63-year-old male with cardiovascular disease.
The answer is B. Patients who are in the 3rd trimester of pregnancy are at a HIGH risk of
developing a complication related to a Hepatitis E infection.

24. What is the BEST preventive measure to take to help prevent ALL types of viral
Hepatitis?

 A. Vaccination
 B. Proper disposal of needles
 C. Hand hygiene

 D. Blood and organ donation screening


The answer is C. Hand hygiene can help prevent all types of viral hepatitis. However, not all
types of viral Hepatitis have a vaccine available or are spread through needle sticks or
blood/organs donations. Remember Hepatitis A and E are spread only via fecal-oral routes.
25. Select all the types of viral Hepatitis that have preventive vaccines available in the
United States?

 A. Hepatitis A

 B. Hepatitis B
 C. Hepatitis C
 D. Hepatitis D
 E. Hepatitis E
The answers are A and B. Currently there is only a vaccine for Hepatitis A and B in the U.S.

26. A patient is prescribed Peginterferon alfa-2a. The nurse will prepare to administer this
medication what route?

 A. Oral
 B. Intramuscular
 C. Subcutaneous

 D. Intravenous
The answer is C. This medication is administered subq.

27. A patient with viral Hepatitis states their flu-like symptoms have subsided. However,
they now have yellowing of the skin and sclera along with dark urine. Based on this
finding, this is what phase of Hepatitis?

 A. Icteric
 B. Posticteric
 C. Preicteric

 D. Convalescent
The answer is A. The Preicteric (prodromal) Phase: flulike symptoms...joint pain, fatigue,
nausea vomiting, abdominal pain change in taste, liver enzymes and bilirubin
increasing….Icteric Phase: decrease in the flu-like symptoms but will have jaundice and dark
urine (buildup of bilirubin) yellowing of skin and white part of the eyeball, clay-colored stool
(bilirubin not going to stool to give it's normal brown color) enlarged liver and pain in this
area….Posticteric (convalescent) Phase: jaundice and dark urine start to subside and stool
returns to normal brown color, liver enzymes and bilirubin decrease to normal
28. During the posticteric phase of Hepatitis the nurse would expect to find? Select all
that apply:

 A. Increased ALT and AST levels along with an increased bilirubin level

 B. Decreased liver enzymes and bilirubin level


 C. Flu-like symptoms

 D. Resolved jaundice and dark urine


The answers are B and D. Posticteric (convalescent) Phase: jaundice and dark urine start to
subside and stool returns to normal brown color, liver enzymes and bilirubin decrease to normal

1. Which condition is NOT a known cause of cirrhosis?

 A. Obesity
 B. Alcohol consumption
 C. Blockage of the bile duct
 D. Hepatitis C
 E. All are known causes of cirrhosis
The answer is E. All of these conditions can cause cirrhosis.

2. The liver receives it blood supply from two sources. One of these sources is called the
_________________, which is a vessel network that delivers blood _____________ in
nutrients but ________ in oxygen.

 A. hepatic artery, low, high


 B. hepatic portal vein, high, low
 C. hepatic lobule, high, low
 D. hepatic vein, low, high
The answer is B. Majority of the blood flow to the liver comes from the hepatic portal vein. This
vessel network delivers blood HIGH in nutrients (lipids, proteins, carbs etc.) from organs that aid
in the digestion of food, but the blood is POOR in oxygen. The organs connected to the hepatic
portal vein are: small/large intestine, pancreas, spleen, stomach. Rich oxygenated blood comes
from the hepatic artery to the liver.

3. A patient with late-stage cirrhosis develops portal hypertension. Which of the


following options below are complications that can develop from this condition? Select
all that apply:

 A. Increase albumin levels

 B. Ascites

 C. Splenomegaly
 D. Fluid volume deficient

 E. Esophageal varices
The answer are B, C, and E. Portal Hypertension is where the portal vein becomes narrow due
to scar tissue in the liver, which is restricting the flow of blood to the liver. Therefore, pressure
becomes increased in the portal vein and affects the organs connected via the vein to the liver.
The patient may experience ascites, enlarged spleen "splenomegaly", and esophageal varices
etc.

4. Your patient with cirrhosis has severe splenomegaly. As the nurse you will make it
priority to monitor the patient for signs and symptoms of? Select all that apply:

 A. Thrombocytopenia
 B. Vision changes

 C. Increased PT/INR

 D. Leukopenia
The answers are A, C, and D. A patient with an enlarged spleen (splenomegaly) due to cirrhosis
can experience thrombocytopenia (low platelet count), increased PT/INR (means it takes the
patient a long time to stop bleeding), and leukopenia (low white blood cells). The spleen stores
platelets and WBCs. An enlarged spleen can develop due to portal hypertension, which causes
the platelets and WBCs to become stuck inside the spleen due to the increased pressure in the
hepatic vein (hence lowering the count and the body’s access to these important cells for
survival).

5. A patient is admitted with hepatic encephalopathy secondary to cirrhosis. Which meal


option selection below should be avoided with this patient?

 A. Beef tips and broccoli rabe


 B. Pasta noodles and bread
 C. Cucumber sandwich with a side of grapes
 D. Fresh salad with chopped water chestnuts
The answer is A. Patients who are experiencing hepatic encephalopathy are having issues with
toxin build up in the body, specifically ammonia. Remember that ammonia is the byproduct of
protein breakdown, and normally the liver can take the ammonia from the protein breakdown
and turn it into urea (but if the cirrhosis is severe enough this can’t happen). Therefore, the
patient should consume foods LOW in protein until the encephalopathy subsides. Option A is
very high in protein while the others are low in protein. Remember meats, legumes, eggs,
broccoli rabe, certain grains etc. are high in protein.

6. During your morning assessment of a patient with cirrhosis, you note the patient is
disoriented to person and place. In addition while assessing the upper extremities, the
patient's hands demonstrate a flapping motion. What lab result would explain these
abnormal assessment findings?

 A. Decreased magnesium level


 B. Increased calcium level
 C. Increased ammonia level
 D. Increased creatinine level
The answer is C. Based on the assessment findings and the fact the patient has cirrhosis, the
patient is experiencing hepatic encephalopathy. This is due to the buildup of toxins in the blood,
specifically ammonia. The flapping motion of the hands is called "asterixis". Therefore, an
increased ammonia level would confirm these abnormal assessment findings.

7. You are receiving shift report on a patient with cirrhosis. The nurse tells you the
patient’s bilirubin levels are very high. Based on this, what assessment findings may you
expect to find during your head-to-toe assessment? Select all that apply:

 A. Frothy light-colored urine


 B. Dark brown urine

 C. Yellowing of the sclera


 D. Dark brown stool

 E. Jaundice of the skin


 F. Bluish mucous membranes
The answers are B, C, and E. High bilirubin levels are because the hepatocytes are no longer
able to properly conjugate the bilirubin because they are damaged. This causes bilirubin to leak
into the blood and urine (rather than entering the bile and being excreted in the stool).
Therefore, the bilirubin stays in the blood and will enter the urine. This will cause the patient to
experience yellowing of the skin, sclera of the eyes, and mucous membranes (“jaundice”) and
have dark brown urine. The stools would be CLAY-COLORED not dark brown (remember
bilirubin normally gives stool it brown color but it will be absent).

8. A 45 year old male has cirrhosis. The patient reports concern about the development
of enlarged breast tissue. You explain to the patient that this is happening because?

 A. The liver cells are removing too much estrogen from the body which causes the
testicles to produce excessive amounts of estrogen, and this leads to gynecomastia.
 B. The liver is producing too much estrogen due to the damage to the liver cells, which
causes the level to increase in the body, and this leads to gynecomastia.
 C. The liver cells are failing to recycle estrogen into testosterone, which leads to
gynecomastia.
 D. The liver cells are failing to remove the hormone estrogen properly from the
body, which causes the level to increase in the body, and this leads to

gynecomastia.
The answer is D.

9. You're providing an in-service to new nurse graduates about esophageal varices in


patients with cirrhosis. You ask the graduates to list activities that should be avoided by
a patient with this condition. Which activities listed are correct: Select all that apply

 A. Excessive coughing
 B. Sleeping on the back
 C. Drinking juice
 D. Alcohol consumption

 E. Straining during a bowel movement

 F. Vomiting
The answers are A, D, E, and F. Esophageal varices are dilated vessels that are connected
from the throat to the stomach. They can become enlarged due to portal hypertension in
cirrhosis and can rupture (this is a medical emergency). The patient should avoid activities that
could rupture these vessels, such as excessive cough, vomiting, drinking alcohol, and
constipation (straining increases thoracic pressure.)

10. While providing mouth care to a patient with late-stage cirrhosis, you note a pungent,
sweet, musty smell to the breath. This is known as:

 A. Metallic Hepatico
 B. Fetor Hepaticus
 C. Hepaticoacidosis
 D. Asterixis
The answer is B.

11. The physician orders Lactulose 30 mL by mouth per day for a patient with cirrhosis.
What findings below demonstrates the medication is working effectively? Select all that
apply:

 A. Decrease albumin levels

 B. Decrease in Fetor Hepaticus


 C. Patient is stuporous.

 D. Decreased ammonia blood level


 E. Presence of asterixis
The answer is B and D. A patient with cirrhosis may experience a complication called hepatic
encephalopathy. This will cause the patient to become confused (they may enter into a coma),
have pungent, musty smelling breath (fetor hepaticus), asterixis (involuntary flapping of the
hands) etc. This is due to the buildup of ammonia in the blood, which affects the brain.
Lactulose can be prescribed to help decrease the ammonia levels. Therefore, if the medication
is working properly to decrease the level of ammonia the patient would have improving mental
status (NOT stuporous), decreased ammonia blood level, decreasing or absence of asterixis,
and decreased ammonia blood level.

12. ________ reside in the liver and help remove bacteria, debris, and old red blood cells.

 A. Hepatocytes
 B. Langerhan cells

 C. Enterocytes
 D. Kupffer cells
The answer is D. Kupffer cells perform this function and are one of the two types of cells found
in the liver lobules (the functional units of the liver). These cells play a role in helping the
hepatocytes turn parts of the old red blood cells into bilirubin.

13. Which of the following is NOT a role of the liver?

 A. Removing hormones from the body


 B. Producing bile

 C. Absorbing water
 D. Producing albumin

1. Inside the pancreas are special cells that secrete digestive enzymes and hormones.
The cells that secrete digestive enzymes are known as ______________ cells.

 A. Islet of Langerhans
 B. Protease
 C. Acinar
 D. Amylase
The answer is C. Acinar cells secrete digestive enzymes such as amylase, protease, and lipase.
2. From the pancreas and gallbladder, the common bile duct and pancreatic duct open
into the ____________ where digestive enzymes and bile flow into the duodenum via the
major duodenal papilla which is surrounded by a muscular valve that controls the
release of digestive enzymes known as the ______________.

 A. ampulla of vater, sphincter of Oddi


 B. papilla of vater, sphincter of Oddi
 C. minor duodenal papilla, ampulla of vater

 D. jejunum, sphincter of pylori


The answer is A. Digestive enzymes from the pancreas duct and bile from the common bile duct
flow through the ampulla of vater (this is where the pancreatic duct and common bile duct form
together) into the duodenum via the major duodenal papilla (also called the papilla of vater)
which is surrounded by the sphincter of Oddi. This sphincter is a muscular valve that controls
the release of digestive enzymes/bile and prevents reflux of stomach contents into the pancreas
and bile duct.

3. Select-ALL-that-apply: In the pancreas, the acinar cells release:

 A. Amylase
 B. Somatostatin

 C. Lipase

 D. Protease
The answers are A, C, and D. Acinar cells secrete digestive enzymes into the pancreatic ducts.
These enzymes are: Amylase: breaks down carbs to glucose, Protease: breaks down proteins
to amino acids, Lipase: breaks down fats

4. You're caring for a 45 year old patient who is admitted with suspected acute
pancreatitis. The patient reports having extreme mid-epigastric pain that radiates to the
back. The patient states the pain started last night after eating fast food. As the nurse,
you know the two most common causes of acute pancreatitis are:

 A. High cholesterol and alcohol abuse


 B. History of diabetes and smoking

 C. Pancreatic cancer and obesity


 D. Gallstones and alcohol abuse
The answer is D. Main causes of acute pancreatitis are gallstones and alcohol consumption.
Heavy, long-term alcohol abuse is the main cause of CHRONIC pancreatitis.

5. Which patient below is at MOST risk for CHRONIC pancreatitis?

 A. A 25 year old female with a family history of gallstones.


 B. A 35 year old male who reports social drinking of alcohol.
 C. A 15 year old female with cystic fibrosis.
 D. A 66 year old female with stomach cancer.
The answer is C. Patients in options A and B are at slight risk for ACUTE pancreatitis not
chronic. Remember the main causes of ACUTE pancreatitis are gallstones and alcohol
consumption. In option C, the patient with cystic fibrosis is at MAJOR risk for CHRONIC
pancreatitis because they are lacking the protein CFTR which plays a role in the movement of
chloride ions to help balance salt and water in the epithelial cells that line the ducts of the
pancreas. There is a decreased production of bicarbonate secretion by the epithelial cells.
Therefore, this leads to thick mucus in the pancreatic ducts that can lead to blockage of the
pancreatic ducts which can cause the digestive enzymes to activate and damage the pancreas.
Overtime, the pancreas will experience fibrosis of the pancreas’ tissue and will no longer
produce digestive enzyme to help with food digestion.

6. Your patient with acute pancreatitis is scheduled for a test that will use a scope to
assess the pancreas, bile ducts, and gallbladder. The patient asks you, "What is the
name of the test I’m going for later today?" You tell the patient it is called:

 A. MRCP
 B. ERCP
 C. CT scan of the abdomen
 D. EGD
The answer is B. ERCP (Endoscopic Retrograde Cholangio-Pancreatography) is used to
diagnosis and sometimes treat the causes of pancreatitis. It will assess the pancreas, bile ducts,
and gallbladder. In addition, the doctor may be able to remove gallstones, dilate the blocked
ducts with a stent or balloon, drain presenting cysts etc.

7. A patient is admitted to the ER with the following signs and symptoms: very painful
mid-epigastric pain felt in the back, elevated glucose, fever, and vomiting. During the
head-to-toe assessment, you notice bluish discoloration around the belly button. As the
nurse, you know this is called?
 A. Grey-Turner's Sign

 B. McBurney's Sign
 C. Homan's Sign
 D. Cullen's Sign
The answer is D. This is known as Cullen's Sign. It represents retroperitoneal bleeding from the
leakage of digestive enzymes from the inflamed pancreas into the surrounding tissues which is
causing bleeding and it is leaking down to umbilicus tissue. Remember the C in Cullen for
"circle" and the belly button forms a circle. The patient can also have Grey-Turner's Sign which
is a bluish discoloration at the flanks (side of the abdomen). Remember this by TURNER ("turn
her" over on her side) which is where the bluish discoloration will be.

8. While assisting a patient with chronic pancreatitis to the bathroom, you note the
patient's stool to be oily/greasy in appearance. In your documentation you note this as:

 A. Steatorrhea
 B. Melena
 C. Currant
 D. Hematochezia
The answer is A. Steatorrhea is an oily/greasy appearance of the stool which can occur in
chronic pancreatitis. This occurs due to the inability of the pancreas to produce digestive
enzymes which help break down fats. Fats are not being broken down; therefore, it is being
excreted into the stool. Melena is used to describe tarry/black stool, hematochezia is used to
describe red stools, and currant are jelly type stools.

9. A patient with acute pancreatitis is reporting excessive thirst, excessive voiding, and
blurred vision. As the nurse, it is priority you?

 A. Reassure the patient this is normal with pancreatitis


 B. Check the patient's blood glucose
 C. Assist the patient with drinking a simple sugar drink like orange juice
 D. Provide a dark and calm environment
The answer is B. Patients with acute pancreatitis are at risk for hyperglycemia (the signs and
symptoms the patient are reporting are classic symptoms of hyperglycemia). Remember the
endocrine function of the pancreas (which is to release insulin/glucagon etc. is insufficient) so
the nurse must monitor the patient's blood glucose levels even if the patient is not diabetic.
10. A patient who received treatment for pancreatitis is being discharged home. You're
providing diet teaching to the patient. Which statement by the patient requires immediate
re-education about the diet restrictions?

 A. "It will be hard but I will eat a diet low in fat and avoid greasy foods."
 B. "It is very important I limit my alcohol intake to no more than 2-3 glasses of wine
a week."
 C. "I will concentrate on eating complex carbohydrates rather than refined

carbohydrates."
 D. "I will purchase foods that are high in protein."
The answer is B. A patient with pancreatitis should AVOID any amount of alcohol because of its
effects on the pancreas. Remember alcohol is a cause of both acute and chronic pancreatitis.
All the other options are correct.

11. The physician orders a patient with pancreatitis to take a pancreatic enzyme. What
assessment finding demonstrates the pancreatic enzymes are working properly?

 A. Abdominal girth is decreased


 B. Skin turgor is less than 2 seconds
 C. Blood glucose is 250
 D. Stools appear formed and solid
The answer is D. Pancreatic enzymes help the body break down carbs, proteins, and fats
because the body is not sufficiently producing digestive enzymes anymore. Hence, the stool will
not appear as oily or greasy (decrease in steatorrhea) but appear solid and formed.

12. During a home health visit, you are assessing how a patient takes the prescribed
pancreatic enzyme. The patient is unable to swallow the capsule whole, so they open the
capsule and mix the beads inside the capsule with food/drink. Which food or drink is safe
for the patient to mix the beads with?

 A. Pudding
 B. Ice cream
 C. Milk
 D. Applesauce
The answer is D. The patient should mix the medications with acidic foods like applesauce. It is
very important the patient does NOT use alkaline foods for mixing (like dairy products, pudding
etc.) because they can damaged the enzyme.

1. You're educating a group of nursing students about the pathophysiology of Celiac


Disease. You ask the group to identify the specific protein that plays a role in the immune
reaction experienced in Celiac Disease. Which answer is correct?

 A. Zein
 B. Globulins
 C. Gliadins
 D. Glutamate
The answer is C. There are several proteins in wheat. One of them is called GLUTEN. Gluten
itself is constructed with a group of proteins called GLIADIN and glutenin. Gliadin is the problem
with Celiac Disease because it triggers an immune response.

2. A patient is suspected to be suffering from Celiac Disease. The physician orders an


endoscopy. If the patient has Celiac Disease, what finding will be discovered with the
endoscopy?

 A. Over exaggerated intestinal villi


 B. Ulcerations in the small intestine, specifically the Jejunum
 C. Flat intestinal villi
 D. Cobble-stone appearance throughout the small intestine
The answer is C. Normally, the intestinal villi look like little finger-like projections. The villi aid in
the absorption of the nutrients by increasing the surface area for absorption. However, when
Celiac Disease occurs the villi are DAMAGED (attacked by the immune system) and they will
appear FLAT. This will decrease the surface area for absorption and lead to malnourishment
issues in the patient.

3. Which of the following is not an anti-body blood test ordered by a physician to


diagnose Celiac Disease?

 A. Antinuclear antibody (ANA)


 B. Tissue Transglutaminase Antibodies (tTG-IgA)
 C. IgA Endomysial antibody (EMA)

 D. IgA serum
The answer is A. Option A is a type of test used to assess for Lupus NOT Celiac Disease.
Options B, C, D are antibodies screened in patients with suspected Celiac Disease.

4. A patient, who was recently diagnosed with Celiac Disease, has blister type bumps on
the elbow and knees. The patient reports it is extremely itchy. As the nurse, you know
this is as known as:

 A. Seborrheic Dermatitis
 B. Psoriasis
 C. Dyshidrotic Eczema
 D. Dermatitis Herpetiformis
The answer is D. Patients will Celiac Disease may exhibit skin rashes. Dermatitis Herpetiformis
is a VERY itchy blister type rash that can be present on the elbows, knees, buttocks, or hairline.
All the other options are not typically present in Celiac Disease.

5. During an outpatient clinic visit, a female patient reports feeling abdominal


bloating/pain, and diarrhea when eating foods that contain wheat or rye. The patient
states her mother was diagnosed with Celiac Disease 5 years ago. What other symptoms
will you assess the patient for that can be present in Celiac Disease? SELECT-ALL-
THAT-APPLY:

 A. Unexplained Weight loss

 B. Jelly-like stools

 C. Mouth ulcers

 D. Menstrual irregularities
 E. Pain at McBurney's Point
 F. Ribbon-like stools
 G. Inability to tolerate dairy products

 H. Enamel changes
The answers are A, C, D, G, and H. These options are seen in Celiac Disease. Option B is seen
in Intussusception. Option E is seen in appendicitis. Option F is seen in Hirschsprung's Disease.

6. Your patient was admitted 3 days ago for treatment of severe malnourishment
secondary to Celiac Disease. The patient is doing well and will be discharged tomorrow.
When you arrive to the patient's room, the patient's friends and family are visiting and
have brought dinner for the patient. Which food item below should the patient avoid
consuming?

 A. Pork barbeque sandwich

 B. Steak and steamed broccoli


 C. Braised chicken with carrots
 D. Vegetables and rice
The answer is A. The pork is okay to eat but the bread of the sandwich is not. Sandwich bread
(bun) typically is made up of wheat which contains gluten. All the other options are okay to eat.

7. Gliadin is a wheat prolamin, which is a plant storage protein that is high in the amino
acids particularly ____________ and ___________.

 A. lysine and proline


 B. glutamine and cysteine
 C. gliadin and gluten
 D. proline and glutamine
The answer is D. Gliadin is a wheat prolamin, which is a plant storage protein that is high in the
amino acids particularly PROLINE and GLUTAMINE.

8. You're helping a mother, whose child was recently diagnosed with Celiac Disease,
read food labels. Which items below, if listed as the ingredients, should the mother avoid
feeding her child?

 A. Millet
 B. Wheat

 C. Malt

 D. Corn
 E. Buckwheat

 F. Rye
The answers are B, C, F. These items should be avoided in Celiac Disease. Options A, D, E
(don't let the "wheat" part of buckwheat confuse you...they are not related).

1. A patient reports frequent heartburn twice a week for the past 4 months. What other
symptoms reported by the patient may indicate the patient has GERD? SELECT-ALL-
THAT-APPLY:

 A. Bitter taste in mouth

 B. Dry cough
 C. Melena

 D. Difficulty swallowing
 E. Smooth, red tongue
 F. Murphy's Sign
The answers are A, B, D. These are signs and symptoms seen with GERD. Melena is seen with
gastrointestinal bleeding as in peptic ulcer disease. Smooth, red tongue is seen with vitamin
B12 deficiency, and Murphy's Signs is seen with cholecystitis.

2. Your patient, who is presenting with signs and symptoms of GERD, is scheduled to
have a test that assesses the function of the esophagus' ability to squeeze food down
into the stomach and the closer of the lower esophageal sphincter. The patient asks you,
"What is the name of the test I'm having later today?" You tell the patient the name of the
test is:
 A. Lower Esophageal Gastrointestinal Series
 B. Transesophageal echocardiogram
 C. Esophageal manometry
 D. Esophageal pH monitoring
The answer is C. An esophageal manometry assesses the function of the esophagus' ability to
squeeze the food down and how the lower esophageal sphincter closes.

3. After dinner time, during hourly rounding, a patient awakes to report they feel like
"food is coming up" in the back of their throat and that there is a bitter taste in their
mouth. What nursing intervention will you perform next?

 A. Perform deep suctioning


 B. Assist the patient into the Semi-Fowler's position

 C. Keep the patient NPO


 D. Instruct the patient to avoid milk products
The answer is B. The patient is experiencing regurgitation. The clues in this scenario are the
patient signs and symptoms along with the time of day (after dinner time...the patient just ate a
meal and is sleeping..we can assume they are lying down). If a patient has reflux disease, the
lower esophageal sphincter is weak and after a meal when a person lies down to sleep the food
can regurgitate into the throat which will cause the patient to feel like "food in coming up" in the
back of the throat and bitter taste in the mouth. Placing the patient in semi-fowler's position will
help alleviate this.

4. During a home health visit, you are helping a patient develop a list of foods they
should avoid due to GERD. Which items in the patient's pantry should be avoided?
SELECT-ALL-THAT-APPLY:

 A. Hot and Spicy Pork Rinds

 B. Peppermint Patties
 C. Green Beans

 D. Tomato Soup

 E. Chocolate Fondue
 F. Almonds
 G. Oranges
The answers are A, B, D, E, G. Patients with GERD should avoid foods that relax the lower
esophageal sphincter such as greasy/fatty foods (Hot and Spicy Pork Rinds), peppermint
(peppermint patties), acidic or citrus foods/juice (tomato soup and oranges), chocolate
(chocolate fondue), along with coffee and soft drinks.

5. After providing education to a patient with GERD. You ask the patient to list 4 things
they can do to prevent or alleviate signs and symptoms of GERD. Which statement is
INCORRECT?

 A. "It is best to try to consume small meals throughout the day than eat 3 large

ones."
 B. "I'm disappointed that I will have to limit my intake of peppermint and spearmint
because I love eating those types of hard candies."
 C. "It is important I avoid eating right before bedtime."
 D. "I will try to lie down after eating a meal to help decrease pressure on the lower
esophageal sphincter."
The answer is D. This statement is incorrect. The patient should have said I will AVOID lying
down after eating a meal to help decrease pressure on the lower esophageal sphincter. It is
important a patient does not immediately lie down after eating but wait for about 1 hour.

6. You're collecting a patient's medication history that has GERD. Which medication
below is NOT typically used to treat GERD?

 A. Colesevelam "Welchol"
 B. Omeprazole "Prilosec"
 C. Metoclopramide "Reglan"
 D. Ranitidine HCL "Zantac"
The answer is A. Options B is a proton-pump Inhibitors (PPIs) and it decreases stomach acid
and helps the esophagus heal. Option C is a type of prokinetic drug and prevents delayed
gastric emptying by improving pressure in lower esophageal sphincter and it improves
peristalsis of the GI tract. Option D is a histamine receptor blocker and it blocks histamine.
When histamine is released it causes the parietal cells to release HCL but this response will be
blocked so gastric acid secretion will be decreased. Option A is a drug used in gallbladder
disease.
7. A patient is taking Bethanechol "Urecholine" for treatment of GERD. This is known as
what type of drug?

 A. Proton-pump inhibitor
 B. Histamine receptor blocker
 C. Prokinetic
 D. Mucosal Healing Agent
The answer is C. This drug is known as a prokinetic drug. It prevents delayed gastric emptying
by improving pressure in lower esophageal sphincter and improves peristalsis of the GI tract.

8. Which of the following does NOT play a role in the development of GERD?

 A. Pregnancy
 B. Hiatal hernia
 C. Usage of antihistamines or calcium channel blockers

 D. All the above play a role in GERD


The answer is D. All the options above play a role in the development of GERD. These options
can weaken the lower esophageal sphincter and cause it to not close properly.

1. In the stomach lining, the parietal cells release _________ and the chief cells release
__________ which both play a role in peptic ulcer disease.

 A. pepsin, hydrochloric acid


 B. pepsinogen, pepsin
 C. pepsinogen, gastric acid
 D. hydrochloric acid, and pepsinogen
The answer is D. In the stomach lining, the parietal cells release HYDROCHLORIC ACID and
the chief cells release PEPSINOGEN which both plays a role in peptic ulcer disease.
Pepsinogen then mixes with the hydrochloric acid and turns into pepsin.
2. A patient has developed a duodenal ulcer. As the nurse, you know that which of the
following plays a role in peptic ulcer formation. Select ALL that apply:

 A. Spicy foods

 B. Helicobacter pylori

 C. NSAIDs
 D. Milk

 E. Zollinger-Ellison Syndrome
The answers are B, C, and E. Helicobacter pylori and NSAIDS are the most common causes for
peptic ulcer formation. Zollinger-Ellison Syndrome can cause peptic ulcers but it is not as
common as H. pylori or NSAIDS. Foods and stress are no longer thought to cause ulcers.
Certain foods and stress can irritate ulcers or prolong healing but there is no evidence to
suggest they cause them.

3. You're educating a group of patients at an outpatient clinic about peptic ulcer


formation. Which statement is correct about how peptic ulcers form?

 A. "An increase in gastric acid is the sole cause of peptic ulcer formation."
 B. "Peptic ulcers can form when acid penetrates unprotected stomach mucosa.
This causes histamine to be released which signals to the parietal cells to release
more hydrochloric acid which erodes the stomach lining further."
 C. "Peptic ulcers form when acid penetrates unprotected stomach mucosa. This causes
pepsin to be released which signals to the parietal cells to release more pepsinogen
which erodes the stomach lining further."
 D. "The release of prostaglandins cause the stomach lining to breakdown which allows
ulcers to form."
The answer is B. Ulcers form when acid penetrates unprotected stomach mucosa. This causes
histamine to be released which signals to the parietal cells to release more hydrochloric acid
which erodes the stomach lining further...hence why option C is wrong. Option A is wrong
because although peptic ulcers can from with increase gastric acid, this is not the sole cause of
peptic ulcer formation. A breakdown in the defense mechanisms along with gastric acid leads to
peptic ulcer formation. For example, h. pylori and regular NSAID usage leads to the breakdown
of the stomach lining which allows stomach acid to penetrate and erode the lining. Option D is
wrong because prostaglandins actually protect the stomach lining by causing the stomach cells
to release mucous rich in bicarb, controls acid amounts via the parietal cells, and regulates
perfusion to the stomach.
4. Your patient is diagnosed with peptic ulcer disease due to h.pylori. This bacterium has
a unique shape which allows it to penetrate the stomach mucosa. You know this
bacterium is:

 A. Rod shaped
 B. Spherical shaped
 C. Spiral shaped
 D. Filamentous shaped
The answer is C. Helicobacter pylori (h. pylori) are spiral shaped which all them to penetrate
down into the stomach lining to reside.

5. Helicobacter pylori can live in the stomach's acidic conditions because it secretes
___________ which neutralizes the acid.

 A. ammonia

 B. urease
 C. carbon dioxide
 D. bicarbonate
The answer is B. H. pylori can live in the acidic conditions of the stomach because it secretes
urease which produces ammonia to neutralize the acid.

6. The physician orders a patient with a duodenal ulcer to take a UREA breath test. Which
lab value will the test measure to determine if h. pylori is present?

 A. Ammonia
 B. Urea
 C. Hydrochloric acid
 D. Carbon dioxide
The answer is D. If h. pylori are present, the bacteria will release urease which produces
ammonia and carbon dioxide. For the test, the patient will ingest a urea tablet and breath
samples will be analyzed for carbon dioxide levels.

7. A patient arrives to the clinic for evaluation of epigastric pain. The patient describes
the pain to be relieved by food intake. In addition, the patient reports awaking in the
middle of the night with a gnawing pain in the stomach. Based on the patient's
description this appears to be what type of peptic ulcer?

 A. Duodenal
 B. Gastric
 C. Esophageal
 D. Refractory
The answer is A. The patient signs and symptoms describe a duodenal ulcer. Gastric ulcer tend
to not cause pain in the middle of the night and epigastric pain in worst with food.

8. A patient with chronic peptic ulcer disease underwent a gastric resection 1 month ago
and is reporting nausea, bloating, and diarrhea 30 minutes after eating. What condition is
this patient most likely experiencing?

 A. Gastroparesis

 B. Fascia dehiscence
 C. Dumping Syndrome
 D. Somogyi effect
The answer is C. After a gastric resection the stomach is not able to regulate the movement of
food due to the removal of sections of the stomach (usually the pyloric valve and duodenum).
Therefore, the food enters into the small intestine too fast before the stomach can finish
digesting it. The partially digested food will act hypertonically and cause water from the blood to
enter jejunum. This will cause a fluid shift leading to bowel swelling, diarrhea, and nausea etc.

9. Thinking back to the patient in question 8, select ALL the correct statements on how to
educate this patient about decreasing their symptoms:

 A. "It is best to eat 3 large meals a day rather than small frequent meals."

 B. "After eating a meal lie down for 30 minutes."

 C. "Eat a diet high in protein, fiber, and low in carbs."


 D. "Be sure to drink at least 16 oz. of milk with meals."
The answers are B and C. The patient in question 8 is exhibiting signs and symptoms of
dumping syndrome. The patient should eat small but frequent meals (NOT 3 large meals a day),
lie down for 30 minutes after meals, avoid sugary drinks and foods, and follow a high protein,
high fiber, and low-carb diet, and avoid consuming drinks while eating but afterwards.

10. A patient is recovering from discomfort from a peptic ulcer. The doctor has ordered
to advance the patient’s diet to solid foods. The patient's lunch tray arrives. Which food
should the patient avoid eating?

 A. Orange
 B. Milk
 C. White rice
 D. Banana
The answer is A. When an ulcer is actively causing signs and symptoms, the patient should
avoid acidic foods like tomatoes or citric fruits/juices, chocolate, alcohol, fried foods and
caffeine. These foods can irritate the ulcer site. Instead the patient should consume alkalotic or
bland foods like milk, white rice or bananas.

11. Which statement is INCORRECT about Histamine-receptor blockers?

 A. "H2 blockers block histamine which causes the chief cells to decrease the
secretion of hydrochloric acid."
 B. "Ranitidine and Famotidine are two types of histamine-receptor blocker

medications."
 C. "Antacids and H2 blockers should not be given together."
 D. All the statements are CORRECT.
The answer is A. This statement is false. H2 blockers block histamine which causes the
PARTIETAL (not chief) cells to decrease the secretion of hydrochloric acid.

12. You are providing discharge teaching to a patient taking Sucralfate (Carafate). Which
statement by the patient demonstrates they understand how to take this medication?

 A. "I will take this medication at the same time I take Ranitidine."
 B. "I will always take this medication on an empty stomach."

 C. "It is best to take this medication with antacids."


 D. "I will take this medication once a week."
The answer is B. This statement is the only correct statement about how to take Carafate. It
should always be taken on an empty stomach without food so it can coat the site of ulceration.
This medication should NOT be taken with H2 blockers (Ranitidine) or antacids because these
drugs affect the absorption of Carafate.

13. Select all the medications a physician may order to treat a H. Pylori infection that is
causing a peptic ulcer?

 A. Proton-Pump Inhibitors
 B. Antacids
 C. Anticholinergics
 D. 5-Aminosalicylates

 E. Antibiotics

 F. H2 Blockers

 G. Bismuth Subsalicylates
The answers are: A, E, F, and G. All these medications can be used to treat an h. pylori
infection that is causing a peptic ulcer.

14. A physician prescribes a Proton-Pump Inhibitor to a patient with a gastric ulcer.


Which medication is considered a PPI?

 A. Pantoprazole
 B. Famotidine
 C. Magnesium Hydroxide
 D. Metronidazole
The answer is A. Pantoprazole is the only PPI listed. Remember PPIs tend to end with the
letters "prazole".

15. A patient with a peptic ulcer is suddenly vomiting dark coffee ground emesis. On
assessment of the abdomen you find bloating and an epigastric mass in the abdomen.
Which complication may this patient be experiencing?
 A. Obstruction of pylorus
 B. Upper gastrointestinal bleeding
 C. Perforation
 D. Peritonitis
The answer is B. This patient is most likely experiencing an upper GI bleeding. Signs and
symptoms of a possible GI bleeding with a peptic ulcer include: vomiting coffee ground emesis
along with bloating, and abdominal mass.

1. Most patients with diverticulosis are most likely to have diverticula located in the?

 A. Transverse colon
 B. Sigmoid Colon
 C. Rectum
 D. Ascending Colon
The answer is B: sigmoid colon. Diverticulosis can occur throughout the GI tract, however, it
tends to be most common in the sigmoid colon.

2. You're providing nursing education to a group of nursing students about diverticulosis


versus diverticulitis. Which statement by the nursing students demonstrate they
understand the education provided?

 A. "Most patients with diverticulitis are asymptomatic."


 B. "Diverticulosis tends to occur in young women with a family history of diverticulosis."
 C. "Diverticulitis, if not treated, can lead to abscess formation and peritonitis."

 D. "Patients with diverticulosis should at all times avoid eating seeds and nuts."
The answer is C. This statement is the only true statement. Option A is wrong because patients
with diverticulOSIS (not diverticulitis) are usually asymptomatic. Patients with diverticulitis will
have symptoms of cramping like abdominal pain which is mainly felt in the left lower quadrant,
blood in stool, constipation, fever. Option B is wrong because diverticulosis tends to occur in
middle to older-aged adults not young. Option D is wrong because research now shows that
consuming seeds or nuts does not cause diverticulitis as it was once thought.

3. A patient with a history of diverticulosis is admitted with abdominal pain. The


physician suspects diverticulitis. What other findings would correlate with diverticulitis?
SELECT-ALL-THAT-APPLY:

 A. Abdominal pain that is mainly present in the upper right quadrant

 B. Unrelenting cramping type pain


 C. Pain found at McBurney's Point

 D. Blood in stool

 E. Fever

 F. Reports of constipation

 G. Abdominal bloating
 H. Positive Cullen's Sign
The answers are : B, D, E, F, and G. These are typical signs and symptoms found with
diverticulitis. Option C is found in appendicitis and Option H is found in acute pancreatitis.
Option A is wrong because abdominal pain is typically found in the left lower quadrant not upper
right.

4. A patient asks what type of testing is performed to assess for diverticulosis. As the
nurse, you know that which test below is used to assess for diverticulosis?

 A. Colonoscopy
 B. Fleets enema
 C. Bronchoscopy
 D. Cystoscopy
The answer is A. This is the only option that can assess for diverticulosis.
5. A patient is experiencing an acute episode of diverticulitis. The patient is having
abdominal pain, temperature 102.6 'F, and elevated WBCs. As the nurse, you know it is
important to:

 A. Encourage intake of high-fiber foods

 B. Monitor the patient for peritonitis


 C. Apply a heating pad to the patient's abdomen to help alleviate pain
 D. Encourage intake of full liquids
The answer is B. The patient experiencing an episode of acute diverticulitis is at risk for abscess
due to the presenting infection in the affected pouch/pouches which can lead to peritonitis if the
abscess ruptures. Therefore, the nurse should monitor the patient for signs and symptoms of
peritonitis which include: increased HR, RR, abdominal tenderness/bloating, and high fever.

6. In regards to question 5, this patient signs and symptoms are starting to subside.
Which of the following food items would be best for the patient to consume?

 A. Oatmeal and bran


 B. Orange juice and eggs
 C. Chicken broth and Jello
 D. Salad with chicken
The answer is C. During the recovery phase of diverticulitis, once the symptoms start subsiding,
the patient should start out with clear liquids like broth, jello, ice etc. and then low-fiber foods
until healed. After the patient has fully recovered, they should consume a high-fiber diet and
stay hydrated.

7. You're providing discharge teaching to a patient who was hospitalized with


diverticulitis. Which statement by the patient requires you to re-educate the patient?

 A. "It is important I consume a diet high in fiber and keep hydrated to keep my stool soft."
 B. "The physician prescribed me to take Psyllium every day which will help prevent
constipation."
 C. "I will be sure to always cook and skin my fruits and vegetables rather than

eating them fresh."


 D. "I will notify my physician if I develop abdominal pain and fever."
The answer is C. The patient should consume fresh fruits and vegetables because they contain
the most fiber. Fruits and vegetables that have been skinned or cooked have low amounts of
fiber in them. The patient needs to follow a high-fiber diet..not low-fiber. A low fiber diet is only
followed when the patient has experienced an episode of diverticulitis and is not fully healed
from the inflammation.

8. True or False: Most patients with chronic diverticulitis require surgery at some point,
such as a bowel resection. If the healthy bowel cannot be reconnected right away, a
permanent colostomy will be created until it can be reconnected.

 True

 False
The answer is False. Most patients with chronic diverticulitis require surgery at some point, such
as a bowel resection. If the healthy bowel cannot be reconnected right away, a TEMPORARY
(NOT permanent) colostomy will be created until it can be reconnected.

1. True or False: A patient with Crohn's Disease can experience inflammation in the large
intestine that affects mainly the mucosa (inner layer) of the bowel.

 True
 False
The answer is FALSE: A patient with Crohn's Disease can experience inflammation throughout
the GI Tract (mainly in the terminal ileum and beginning of the colon) from the mouth to anus
(not just the large intestine) and it affects the WHOLE bowel lining (not just the mucosa layer).

2. A patient with Crohn's Disease is MOST likely to have the disease is what part of the GI
tract?

 A. Rectum
 B. Duodenum of the small intestine
 C. Terminal Ileum
 D. Descending colon
The answer is C. Crohn's disease is MOST likely to affect the terminal ileum. However, it may
affect any area of the GI tract.

3. You're providing teaching to a patient who has been newly diagnosed with Crohn's
Disease. Which statement by the patient's spouse requires re-education?

 A. "Crohn's Disease can be scattered throughout the GI tract in patches with some areas
appearing healthy while others are diseased."

 B. "There is no cure for Crohn's Disease."


 C. "Strictures are a common complication with Crohn's Disease."
 D. "Crohn's Disease can cause the haustra of the large intestine to lose its form."
The answer is D. All the statements are true except option D. ULCERATIVE COLITIS can cause
the haustra of the large intestine to lose its form. This is not common with Crohn's Disease.

4. A physician is explaining to a patient that the patient has a type of Crohn's Disease
that is found in both the ileum and colon. As the nurse, you know this type of Crohn's
Disease is called?

 A. Gastroduodenal Crohn's Disease


 B. Granulomatous Colitis
 C. Ileitis
 D. Ileocolitis
The answer is D. This patient has ileocolitis which affects parts of the colon and ileum.
Gastroduodenal Crohns affects the stomach and duodenum which is the first part of the small
intestine. Granulomatous colitis affects only the colon. Ileitis affects the ileum.

5. Select ALL of the following that are complications associated with Crohn's Disease:

 A. Cobble-stone appearance of GI lining


 B. Lead-pipe sign
 C. Toxic megacolon

 D. Fistula
 E. Abscess

 F. Anal Fissure
The answers are A, D, E, and F. These are all complications found with Crohn's Disease. Lead-
pipe sign and toxic megacolon are complications associated with ulcerative colitis.

6. Your patient with Crohn's Disease is admitted with an opening that has formed
between the bowel and bladder. As the nurse, you know this is what type of complication
associated with this disease?

 A. Enterovesical Fistula
 B. Rectovaginal Stricture
 C. Enteroenteric Fistula

 D. Perianal Fissure
The answer is A. These scenario describes a fistula which is an abnormal passage that forms
deep in the wall of the intestine to form an opening between intestine to intestine, intestine to
organ, or intestine to skin’s surface. This specific patient is experiencing an enterovesical fistula
which is an abnormal passage between the bowel and bladder.

7. A patient experiencing a flare-up with Crohn's Disease is ordered complete bowel rest
by the physician. You are administering TPN (total parental nutrition) per physician
order. When developing the patient's nursing plan of care, which nursing diagnosis is
MOST important to include in the care plan?

 A. Risk for allergy response


 B. Risk for unstable blood glucose level
 C. Risk for imbalance nutrition: more than body requirements
 D. Risk for imbalanced nutrition: less than body requirements
The answer is D. Patients with Crohn's Disease are at risk for undernourishment due to how the
disease process effects the body (small intestine is inflamed which is the area of the gut that
absorbs most of the nutrients from food). With severe cases of Crohn's Disease, the physician
may order complete bowel rest (where the patient will be nothing by mouth (NPO) and nothing
will enter the GI system....so the patient is at even more risk for imbalance nutrition. Physicians
may prescribe an IV solution (total parental nutrition) which will be given through a central line
that contains nutrients (so it enters the bloodstream...bypassing the gut). However, there is still
a risk for imbalanced nutrition (less than body requirements) for the patient. The nurse must
monitor the patient's nutrition status very closely such as daily weights, hydration status,
electrolytes, skin, etc.
8. A patient is receiving treatment for Crohn's Disease. Which food found on the patient's
food tray should the patient avoid?

 A. Fresh Salad
 B. White rice

 C. Baked chicken
 D. Cooked skinless apples
The answer is A. Patients who are experiencing flare-ups of Crohn's Disease should avoid high
fiber foods, foods that are hard to digest, spicy foods, dairy products etc. Therefore, the patient
should avoid a fresh salad. This contain vegetables which are high in fiber and hard to digest.
The gut needs to rest. It is best for the patient to consume low fiber and high protein foods.
White rice and fruits/vegetables that are cooked/skinless are low in fiber. Baked chicken is a
good source of protein for the patient.

9. A physician has prescribed a patient with a severe case of Crohn's Disease to take a
drug that works by suppressing the immune system. This medication achieves this by
blocking a protein that plays a role the inflammatory process. Which drug does this
describe?

 A. Azathioprine
 B. Sulfasalazine

 C. Infliximab
 D. Prednisone
The answer is C. Infliximab (Remicade) is a TNF-blocker (biologic drug) which blocks tumor
necrosis factor which plays a role in the inflammatory response system. Azathioprine is an
immunosupressor which suppresses the immune system but does NOT block TNF.
Sulfasalazine is an 5-Aminosalicylate which is an anti-inflammatory medication. Prednisone is a
corticosteroid which decreases inflammation.

10. A patient with Crohn's Disease is taking corticosteroids. The patient is complaining of
extreme thirst, polyuria, and blurred vision. What is your next nursing action?

 A. Check the patient's blood glucose


 B. Give the patient a food containing sugar (ex: orange juice)
 C. Administer oxygen via nasal cannula
 D. D. Assess bowel sounds
The answer is A. A side effect of corticosteroids is hyperglycemia. Extreme thirst, polyuria, and
blurred vision are classic signs and symptoms of hyperglycemia. Therefore, the nurse should
check the patient's blood glucose to confirm the hyperglycemia.

1. The nurse is doing an assessment on a client during the first postoperative day after
abdominal surgery. Which of the following manifestations does the nurse report immediately?

 Leg swelling and calf pain


 Decreased bowel sounds
 Mild abdominal distention
 Inability to void immediately after urinary catheter is removed

2. When preparing a client for insertion of a nasogastric tube, it is essential for the nurse to
include which of the following aspects of the procedure?

 Measure the tube from the tip of the nose to the earlobe to the xiphoid process
 Instruct the client to avoid swallowing when the tube is felt in the back of the throat

 Assist the client to assume a left-side-lying or recumbent position


 Tilt the client’s head back when the tube is being inserted

3. The nurses on a unit are planning for stoma care for clients who have a stoma for fecal
diversion. Which stomal diversion poses the highest risk for skin breakdown?

 Sigmoid colostomy
 Transverse colostomy
 Ileostomy
 Ileal conduit
4. In planning the post procedure care for a client who has a barium enema, the nurse should
include which of the following? Select all that apply:

 Administer a laxative
 Encourage fluids
 Observe and record the amount of rectal drainage
 Maintain bed rest for 12 hours
 Monitor the client for a rise in body temperature and abdominal pain
 Position the client on the right side

5. A client is diagnosed with gastroesophageal reflux disease (GERD). The nurse’s instruction
to the client regarding diet should be to?

 Focus on three average size meals a day


 Avoid all raw fruit and vegetables
 Increase intake of milk products
 Decrease intake of fatty foods

6. An 82 year-old client complains of chronic constipation. To improve bowel function, the nurse
should FIRST suggest?

 Increasing fiber intake to 20-30 grams daily


 Monitoring a balance between activity and rest
 Avoidance of binding foods such as cheese and chocolate
 Daily use of laxatives

7. The nurse is assessing a comatose client receiving gastric tube feedings. Which of the
following assessments requires an IMMEDIATE response from the nurse?
 Decrease in bowel sounds
 Urine output of 250 cc in past eight hours
 Aspiration of a residual of 100cc of formula
 Decreased breath sounds in right lower lobe

8. The nurse is caring for a client with cholecystitis. Which clinical manifestation would the nurse
expect the client to exhibit?

 Dysphagia
 Fever
 Bradycardia
 Hiccups

9. The nurse is caring for a client with peptic ulcer disease. Which of the following observations
should the nurse report immediately? Select all that apply:

 Diarrhoea
 Headache
 Thirst
 Hypotension
 Restlessness
 Tachycardia

10. The nurse is assessing a client’s gastrointestinal tract. Which of the following subjective
assessments should be included?

 Haematuria
 Diarrhoea
 Generalized red abdominal rash
 Rebound tenderness
11. A client with acute viral hepatitis has a serum bilirubin of 3.6 mg/dL on admission. A nurse
should expect the client to report:

 A high fever for several days.


 Tar-colored bowel movements.
 Dark orange urine.
 Red, irritated eyes.

12. An 82 year-old client complains of chronic constipation. To improve bowel function, the
nurse should FIRST suggest?
 Increasing fiber intake to 20-30 grams daily
 Monitoring a balance between activity and rest
 Avoidance of binding foods such as cheese and chocolate
 Daily use of laxatives

13. The nurse is caring for a client who is receiving total parenteral nutrition (hyperalimentation
and lipids). What is the PRIORITY nursing action on every eight hour shift?

 Change the tubing under sterile conditions


 Adjust the infusion rate to provide for total volume
 Monitor blood pressure, temperature and weight
 Check urine glucose, acetone and specific gravity

14. A client is complaining of epigastric pain after eating. The physician suspects peptic ulcer
disease (PUD) and orders a urea breath test for H. pylori. Which would the nurse include in the
teaching plan for test preparation?

 Clear liquids the day before the test


 High-fat meal two hours before the test
 Asking the client to bring a sputum sample with her for analysis
 Nothing by mouth the night before the test
15. The nurse is doing an assessment on a client during the first postoperative day after
abdominal surgery. Which of the following manifestations does the nurse report immediately?

 Decreased bowel sounds


 Mild abdominal distention
 Inability to void immediately after urinary catheter is removed
 Leg swelling and calf pain

16. Which of the following entries on a client’s progress notes is the MOST complete?

 Client’s urinary output adequate


 Client seems anxious about low salt diet
 Demerol 75mg administered for severe abdominal pain
 100 cc of dark green drainage from Nasogastric tube

17. The nurse is planning the care of a client who has had an abdominal–perineal resection for
cancer of the colon. Which interventions should the nurse implement? Select all that apply.

 Position the client semi-recumbent.


 Irrigate the JP drains every shift.
 Provide meticulous skin care to stoma.
 Maintain the indwelling catheter.
 Assess the flank incision.

18. The nurse is caring for a client with gastroenteritis. Which of the following nursing measures
should receive priority in the client’s plan of care?
 Assist the client to wash hands and face before meals
 Maintain a clean environment free from odours
 Encourage fluids and monitor intake and output
 Provide foods the client likes and allow plenty of time for meals

19. The healthcare provider performs a paracentesis on a client with ascites and 3 liters of fluid
are removed. Which assessment parameter is most critical for the nurse to monitor following the
procedure?

 Gag reflex.
 Pedal pulses.
 Vital signs.
 Breath sounds.

20. The nurse in the emergency room observes a physician examining a client with possible
appendicitis. The physician presses downward on the right lower quadrant of the abdomen and
asks the client to instruct him when he feels pain (application of pressure or pressure release).
What is the physician assessing for?

 Ascites
 Turner’s sign
 Rovsing’s sign
 Rebound tenderness

21. The nurse is explaining the proper use of syrup of ipecac to a group of parents. For which of
the following accidental poisonings is the treatment appropriate?

 Chewable vitamins
 Oven cleaner
 Kerosene
 Drain cleaner

22. The maximum safe height at which the container of fluid can be held when administering an
enema is?

 66cm (26 inches)


 37cm (15 inches)
 30cm (12 inches)
 45cm (18 inches)

23. Which sign/symptom should the nurse expect to find in a client diagnosed with ulcerative
colitis?
 Hard, rigid abdomen.
 Twenty bloody stools a day.
 Oral temperature of 102˚F.
 Urinary stress incontinence.

1. The advice the nurse would give to a client that will most likely result in a reduction of dental
caries is

a. brush and floss regularly.


b. see the dentist once a year.
c. eliminate carbonated beverages.
d. avoid fluoride rinses.
2. The nurse explains to a client asking about a "pulpectomy" that this procedure is also
commonly called a

a. dental filling.
b. scaling.
c. root canal.
d. tooth extraction.

3. The nurse encourages the client to obtain routine dental care because plaque formation can
lead to

a. periodontal disease.
b. oral cancer.
c. herpes simplex infection.
d. stomatitis.

4. The nurse should be aware that the dental problem most likely to cause a person with
periodontitis to seek treatment is

a. decay.
b. pain.
c. loose teeth.
d. headaches.

5. A client has undergone tooth extraction. The clients's statement indicating that the client did
not fully understand discharge instructions is

a. "I can take analgesics for pain."


b. "I plan to eat soft foods for several days."
c. " I can drink hot tea or coffee if I don't feel like eating."
d. " I may use a normal-saline mouth rinse."

6. The nursing diagnosis is Acute Pain related to altered oral mucous membrane and
ulcerations for a client with Vincent's angina. To wash the mouth, the client should be instructed
to use

a. a commercial mouthwash.
b. Dakin's solution.
c. saline mouth rinses.
d. half-strength peroxide.

7. When developing the care plan for a client with leukoplakia, the nurse would consider that the
primary problem leukoplakia poses to the client is

a. cancer.
b. discomfort.
c. infection.
d. purulent secretions.
8. A client is diagnosed as having carcinoma of the oral cavity. The finding that indicates a high
risk for this disease is

a. having had rheumatic fever as a child.


b. ingesting icy cold liquids.
c. working as a construction worker for 6 years.
d. having a history of alcohol abuse.

9. The nurse caring for a client who has recently undergone oral surgery has made a nursing
diagnosis of Imbalanced Nutrition: Less than Body Requirements related to oral pain and
difficulty eating. The nursing intervention that would best assist the client to achieve the goal of
maintaining weight is

a. administering analgesics before meals.


b. teaching the client to avoid putting food directly on the suture site.
c. increasing the time interval between oral care and mealtime.
d. suctioning secretions from the mouth.

10. A client who had extensive oral surgery 5 days earlier has the nursing diagnosis of
Imbalanced Nutrition: Less than Body Requirements related to altered oral mucosa and surgical
procedure. The most appropriate caution by the nurse when the client resumes oral feedings is

a. "The capacity of your mouth will be smaller."


b. "It will be painful to eat for some time."
c. "You may have difficulty feeling the food in your mouth."
d. "Often clients lose their sense of taste following surgery."

11. The nurse caring for a client receiving diuretics who develops parotitis would

a. ask the physician to discontinue the diuretics.


b. discontinue the use of dental floss.
c. encourage the client to suck sugarless candy.
d. restrict oral fluids.

12. The immediate priority in postoperative nursing care for a client after glossectomy is to

a. maintain a patent airway.


b. provide analgesia.
c. monitor for hemorrhage.
d. assist with body image issues.

13. The nurse would assess the client with an early mechanical obstruction of the esophagus for

a. aspiration.
b. dysphagia.
c. coughing.
d. vomiting.

14. The nurse should anticipate that a client with mechanical obstruction of the esophagus
would initially have difficulty swallowing

a. carbonated beverages.
b. bread.
c. mashed potatoes.
d. saliva.

15. Reviewing a client's chart notes, the nurse finds that the client has odynophagia. The nurse
questions the client about experience with

a. dulled taste.
b. knife-like pain.
c. intermittent difficulty with swallowing.
d. throbbing sensations in the throat.

16. Recently a client has been diagnosed with achalasia and is bothered greatly by the
substernal pain. The nurse should encourage the client to

a. begin a reducing diet.


b. eat foods with a dry consistency.
c. take aspirin before going to sleep.
d. sleep with the head of the bed elevated.
17. The nurse explains to the client that bougienage is a procedure that will

a. increase esophageal peristalsis.


b. create a mechanical barrier to reflux.
c. dilate the lower esophagus.
d. alter the gastric pH.

18. The nurse caring for a client who is to undergo the insertion of a PEG tube can best explain
how a PEG tube differs from a gastrostomy tube by describing

a. diameters of the tubes.


b. methods of insertion.
c. locations of the tubes.
d. procedures used for feedings.

19. The nurse planning a teaching program for a client about postoperative care after a
thoracotomy approach to an esophagomyotomy would include information about

a. insertion of a Blakemore tube.


b. prepartion for closed-chest drainage.
c. drainage of a T tube.
d. application of a Hemovac.

20. The statement made by a postoperative client after esophagomyotomy that indicates that
the client has a misunderstanding about the discharge plan is
a. "I will use a board under my mattress."
b. "I'm going to sleep on several pillows."
c. "Any fever should be reported immediately."
d. "It is OK to fall asleep in a chair."

21. The nurse should be mindful that the factor in a client's history most likely to result in
esophageal reflux is

a. very-high-fiber diet.
b. heavy consumption of coffee daily.
c. long-term sedentary lifestyle.
d. eating foods high in sodium.

22. During the assessment of a client admitted for evaluation of gastroesophageal reflux
disease (GERD), the client begins to complain of severe pain that radiates to the jaw. The client
is sitting upright in bed, her color is ruddy, and vital signs are within normal limits, but she asks
for the nitroglycerin (NTG) tablets in her purse. The nurse realizes that the clinical
manifestations demonstrated by the client are

a. classic manifestations of a myocardial infarction, and the physician should be paged


immediately.
b. specifically associated with GERD and not myocardial infarction, but the NTG should be
allowed if the client wants to use it.
c. greatly influenced by fear related to the location of the pain, and the use of NTG should be
discouraged.
d. indications that a thorough pain assessment should be done to determine the etiology
of the pain, and the NTG should be given at once.

23. Metoclopramide (Reglan) is prescribed for a client with GERD. In explaining the action of
this drug, the nurse's statement that most accurately describes the action of metoclopramide is

a. "It decreases the time food and fluids are in the stomach."
b. "It acts as an antacid to reduce gastric acidity."
c. "It helps to promote movement in the esophagus."
d. "It has a local anesthetic effect on the lower esophagus and stomach."

24. The nursing instruction that would be included in a client's teaching plan to prevent or delay
the development of hiatal hernia is

a. avoid drinking carbonated beverages rapidly.


b. sit in an upright position in a straight-backed chair.
c. avoid heavy lifting and stooping.
d. consume a high-carbohydrate, low-fat diet.
25. For a client with diverticula of the esophagus, the nurse would tell the client to avoid

a. deep-breathing exercises after meals.


b. sleeping with the head of the bed elevated.
c. ingestion of carbonated drinks.
d. vigorous exercise after eating.

26. A client has undergone radiation therapy to reduce the size of an esophageal tumor. The
nurse should be especially vigilant in assessing for

a. prolonged epistaxis.
b. sudden onset of diarrhea.
c. esophageal stenosis.
d. projectile vomiting.

27. The nurse assesses a knowledge deficit in a client who is scheduled for the placement of an
Angelchik prosthesis when the client says
a. "I am so glad that I won't have to have surgery to get this thing applied."
b. "My doctor said this procedure may not work."
c. "I am going to have a poker chip-like thing tied to my esophagus."
d. "This gadget is going to help keep my stomach contents where they belong."

28. The nurse instructs a group of clients who smoke that the leading type of malignant oral
cancer is

a. basal cell carcinoma.


b. neurofibroma.
c. hemangioma.
d. squamous cell carcinoma.

29. During a health interview, the nurse informs a client with GERD that of all the drugs the
client is presently taking, the drug that will aggravate the clinical manifestations of GERD is

a. theophylline.
b. Vioxx.
c. Lanoxin.
d. Lasix.

30. A client with a rolling hiatal hernia complains of a feeling of fullness after eating and difficulty
breathing. When the client says, "I think I should lie down for awhile," the nurse should remind
the client
a. to lie on the left side for at least 15 minutes.
b. that lying down may increase the distress.
c. to drink milk or eat a small snack before lying down.
d. that arching the back while lying down will reduce the discomfort.
ANS: b

31. A lesion of the mouth that often indicates an early squamous cell carcinoma is erythroplakia,
which can be identified by its

a. velvety appearance.
b. roughened, leathery appearance.
c. rigid delineated border that bleeds easily.
d. concave, ulcer-like lesion.

1. The instructor removes the tube the student has set out in preparation for passage of along
tube for intestinal decompression. The tube that is inappropriate for intestinal
decompression is the

a. Cantor tube.
b. Harris tube.
c. Miller-Abbott tube.
d. Salem sump tube.

2. Before inserting a gastrointestinal (GI) tube, the nurse assists the client to the

a. High-Fowler's position.
b. Side-lying position.
c. supine position.
d. lithotomy position.

3. The nurse inserting a GI tube should instruct the client to

a. cough vigorously.
b. swallow.
c. perform the Valsalva maneuver.
d. tilt the chin upward.

4. The nurse has just finished inserting a nasogastric (NG) tube in an 80-year-old client
who has been having difficulty swallowing. The best measure to test for placement of
the tube is

a. inspecting the posterior pharynx for correct positioning of the tube.


b. asking the client to speak.
c. placing the end of the tube in a glass of water.
d. aspirating gastric contents.
.

5. The nurse planning to irrigate a nasogastric (NG) tube prepares to use

a. half-strength peroxide.
b. sterile water.
c. normal saline.
d. tap water.
ANS: c

6. A 48-year-old client with acute gastritis is admitted to the emergency department for
GI bleeding. The nurse would anticipate that the client's history will include
a. anorexia.
b. alcohol abuse.
c. a high-carbohydrate, high-fat diet.
d. a recent course of oral penicillin.

7. The client with chronic gastritis is experiencing abdominal pain. The client asks,
"Which of my medications will relieve my belly pain now?" The nurse responds that
the medication that will provide the client with the fastest relief is

a. Prilosec.
b. Flagyl.
c. Gaviscon.
d. Biaxin.

8. The nurse is caring for a client with chronic gastritis of the atrophic type. Mindful of
the diagnosis, the nurse would anticipate the finding of

a. cardiac disease.
b. emphysema.
c. intestinal parasites.
d. pernicious anemia.

9. The nurse is explaining defensive and aggressive factors to a client with peptic ulcer
disease (PUD). The factor the nurse describes as an aggressive component in the
development of ulceration is

a. blood flow to the mucosa.


b. biliary acid volume.
c. mucus barrier.
d. regulation of secretion.

10. The effect on the GI mucosa that the nurse should anticipate if cortisone is not given
with food is
a. increased transit time of GI contents.
b. dramatic increase in gastric pH.
c. increase in amount of GI secretions.
d. susceptibility of the mucosa to injury.

11. A beverage that the nurse should caution a client with peptic ulcer to avoid because
of its stimulatory effect on gastric secretions is

a. apple juice.
b. milk.
c. water.
d. lemonade.

12. The nurse should anticipate that the person with a head injury is most likely to
develop

a. Curling's ulcer.
b. Cushing's ulcer.
c. chronic gastritis.
d. pyloric stenosis.

13. In the history of a client with a duodenal ulcer, the nurse would anticipate the finding
that
a. Food causes pain.
b. the client complains of spitting blood.
c. pain may awaken the client in the middle of the night.
d. certain types of food increase distress.

14. The nurse administers alternating doses of two antacids into the NG tube of a client
with a duodenal ulcer. The finding that best indicates that this drug regimen has been
successful is

a. absent NG tube drainage.


b. increase in gastric pH.
c. mild diarrhea.
d. decreased abdominal rigidity.

15. The nurse administers an anticholinergic drug to a client with a healed gastric ulcer.
Because it is a side effect, the event that would cause the nurse to withhold the next
scheduled dose of this medication is

a. diarrhea.
b. bradycardia.
c. excessive salivation.
d. urinary retention.
16. Ranitidine (Zantac) is prescribed for a client with a gastric ulcer. The statement that
best indicates to the nurse that the client understands the action of this drug is

a. "This drug reduces stomach juices."


b. "I will take this medication at bedtime to relieve pain."
c. "Ranitidine will activate protective mucous barriers."
d. "Stomach acids are neutralized by this medication."

17. The nurse caring for a client with a duodenal ulcer finds the client diaphoretic,
with knees drawn up to a hard, tender abdomen. The nurse should initially

a. assess for bowel sounds.


b. give as-needed (prn) pain medication.
c. move the client to a high-Fowler's position.
d. call the physician.

18. The nurse is monitoring a client who has had gastric surgery for manifestations of
dumping syndrome. The nurse should be particularly watchful for the early
manifestation of

a. hypertension.
b. vertigo.
c. pain.
d. tenesmus.

19. The nurse clarifies that a high-protein, high-fat, low-carbohydrate, dry diet is
prescribed for a client after gastric surgery because this diet

a. is quickly digested.
b. Does not cause diarrhea.
c. is slow to leave the stomach.
d. does not dilate the stomach.

20. An older male client with chronic gastritis asks the nurse, "Do you think I'll get
gastric cancer?" The most accurate response by the nurse is
a. "Chronic gastritis is not associated with the development of gastric cancer in men over 40."

b. "Women with chronic gastritis are more likely to get gastric cancer than men who have
chronic gastritis."
c. "Individual predictions are not possible, but chronic gastritis is associated with the
development of gastric cancer."
d. "As long as the chronic gastritis is diagnosed early, there is little risk of gastric cancer."

21. Immediate nursing actions to achieve critical goals in the postoperative period for a client
who has just returned from a Billroth II procedure should include

a. maintaining the client in the semi-Fowler's position.


b. ensuring frequent coughing and deep breathing.
c. irrigating the client's NG tube every hour.
d. encouraging oral intake.

22. The nurse clarifies that a tube feeing is also called

a. lavage.
b. gavage.
c. bougienage.
d. foulage.

23. The nurse practitioner is having difficulty passing a Harris tube through a client's pylorus.
The appropriate nursing intervention at this time is to
a. advance the tube slowly 2 inches.

b. securely tape the tube to the client's nose.


c. assist the client to lie on the right side.
d. give the client fluid to sip through a straw.

24. The nurse explains that the surgical procedure in which the parietal cells are
denervated is

a. an adrenalectomy.
b. a proximal vagotomy.
c. a Billroth II.
d. a pyloroplasty.

25. Before tube insertion, the nurse performs the NEX measurement, which is the

a. distance from the tip of the nose to the ear lobe and to the xiphoid.
b. length of a tube from the hub to the tip converted to centimeters.
c. distance from the ear lobe to the umbilicus.
d. width of the lumen of the tube multiplied by the length

1. The end-products of carbohydrate and protein digestion are absorbed by the

a. bloodstream.
b. blood vessels in the villi.
c. lymphatic system.
d. lymph vessels in the villi.

2. A 56-year-old patient with diverticulitis has been admitted to the medical unit. The nurse will
most likely document which assessment in the charting?
a. Chest pain
b. Pain in the upper right quadrant
c. Pain in the left lower quadrant
d. Progressive weight loss

3. An ileostomy was performed on a patient for the treatment of debilitating ulcerative colitis
disease. A problem the nurse should watch for in patients after this surgery is

a. fluid imbalance.
b. sexual activity restriction.
c. skin excoriation.
d. the collecting appliance being bulky and large.

4. A patient with cancer of the esophagus has been receiving radiation therapy. The nurse
realizes that this patient should be assessed for
a. aspiration from fistula formation.
b. hemorrhage.
c. incompetence of the suture line.
d. dumping syndrome.

5. A patient has been admitted for diagnostic procedures including an


esophagogastroduodenoscopy. The nurse explains to this patient that during this procedure, the
physician will

a. use a long, rigid, fiberoptic scope.


b. order NPO status 1 hour before the procedure.
c. view the esophagus, stomach, and upper small intestine.
d. visualize the intestine but cannot remove polyps.

6. The nurse determines that a patient has a knowledge deficit regarding her diagnosis of
achalasia. The nurse begins patient teaching by explaining that achalasia is
a. a white patch on the mouth or tongue mucosa.

b. caused by frequent ulceration on the lip.


c. caused by excess exposure to sun and wind.
d. caused by the inability of a muscle to relax.

7. Following a gastrectomy, the nurse would anticipate that the patient would need to be
assessed for

a. renal failure.
b. vitamin B12 deficiency.
c. obesity complications.
d. continuing his routine diet.
8. A 63-year-old patient is admitted with acute diverticulitis. The most appropriate nursing
intervention to lessen this patient's signs and symptoms of increased flatus and chronic
constipation alternating with diarrhea, anorexia, and nausea would be to

a. encourage a diet high in fiber content.


b. reduce oral intake to rest the bowel.
c. encourage fluids to prevent dehydration.
d. administer laxatives to prevent secondary constipation.

9. The patient, age 32, has ulcerative colitis, and his condition is deteriorating. An ileostomy is
scheduled. After the procedure, this patient may be at risk for

a. Activity intolerance.
b. Sexual dysfunction.
c. Disturbed body image.
d. Ineffective thermoregulation.

10. Dumping syndrome after a Billroth II procedure occurs when high-carbohydrate foods are
ingested over a period of less than 20 minutes. A nursing measure that will prevent or minimize
dumping syndrome is to administer the feeding

a. by bolus to prevent continuous intestinal distention.


b. with large amounts of fluids to maintain hydration.
c. high in carbohydrates and protein.
d. in six small daily meals high in protein and fat.

11. The emergency room staff is caring for a patient with an acute inflammatory intestinal
disorder who is being observed to rule out appendicitis. Which intervention is contraindicated?

a. Give nothing by mouth until seen by a health care provider


b. Measure intake and output
c. Apply heat to the abdomen
d. Encourage bed rest

12. After barium swallow contrast studies are performed, the nurse should ensure that the
patient

a. drinks adequate amounts of water.


b. remains in bed for 6 hours.
c. eats nothing until the gag reflex returns.
d. expels all barium rectally.

13. A patient, age 36, is admitted with diarrhea and dehydration. The physician has ordered
several diagnostic studies of the patient's stools. When obtaining a stool specimen to be
examined for ova and parasites, the nurse should

a. use an oil retention enema to facilitate collection.


b. refrigerate the specimen immediately.
c. instruct the patient to obtain three different stool specimens on subsequent days.
d. check the specimen for the presence of occult blood.

14. The patient, age 32, is admitted with possible appendicitis after being evaluated by the
physician. It is appropriate for the nurse to administer

a. fluid and electrolyte replacement.


b. heat to decrease discomfort.
c. a cleansing enema.
d. an oral laxative.

15. A patient, age 84, has a history of a large left inguinal hernia. He is complaining of nausea,
vomiting, abdominal distention, and inguinal pain. A serious complication of a hernia in which
the blood supply to the tissue becomes occluded is called a(n)

a. strangulated hernia.
b. hiatal hernia.
c. incarcerated hernia.
d. sliding hernia.

16. A patient had a ruptured diverticulum in his descending colon. He has undergone a
transverse loop colostomy. He asks the nurse the purpose of this procedure, and the nurse tells
him that it is
a. a temporary colostomy to allow healing of the bowel by diverting feces.
b. to allow for more thorough irrigations.
c. to provide two stomas for fecal elimination.
d. to prevent chronic constipation.

17. The patient complains that he will never adjust to his colostomy. In this situation, it would be
best for the nurse to

a. encourage him to express his concern.


b. suggest that he discuss his concerns with his physician.
c. counsel him that everything will be all right.
d. explain that his concerns will be dealt with when he is taught how to care for his colostomy.

18. The most important nursing intervention to assure the patency of a nasogastric tube (NG) is
to:

a. Clamp nasogastric tube 30 minutes twice a day.


b. Monitor NG for patency and irrigate with sterile normal saline PRN as ordered.
c. Cleanse nares at least once each shift; lubricate with a petrolatum ointment.
d. Administer mouth care every 24 hours.

19. A progressive wavelike movement that occurs involuntarily in hollow tubes of the body,
especially in the alimentary canal, to propel fluids, gas, and digestive substances forward is
called
a. perisclerium.
b. periprostatic.
c. pleurolysis.
d. peristalsis.

20. A NANDA-accepted nursing diagnosis that could be written for a patient with an
abdominoperineal resection and a permanent colostomy would include

a. Disturbed body image.


b. Ineffective thermoregulation.
c. Ineffective protection.
d. Autonomic dysreflexia.

21. Bowel sound assessment is especially important for a postoperative patient who has had
abdominal surgery as it can determine the:

a. need for a cathartic.


b. return of peristalsis.
c. presence of singultus.
d. presence of eructation.

22. A NANDA-accepted nursing diagnosis that could be written for the patient who is
hemorrhaging and in hypovolemic shock from a bleeding peptic ulcer would include

a. Ineffective tissue perfusion (gastrointestinal).


b. Unilateral neglect.
c. Constipation.
d. Disuse syndrome (gastrointestinal).

23. The goals of diet management in a patient with inflammatory bowel disease are: (Select all
that apply.)

a. Restrict fluids to decrease peristalsis.


b. Prevent weight loss.
c. Correct and prevent malnutrition.
d. Provide adequate nutrition.
e. Replace fluid and electrolyte losses.
f. Limit high-caloric foods.

a. a,b,c
b. b,c,d,e
c. a,c,f
d. b,c,d,f
24. What nursing interventions would be appropriate for inflammatory bowel disease diagnoses
of Imbalanced Nutrition: less than body requirements related to bowel hypermotility and
decreased absorption?

a. Provide three specific balanced meals a day.


b. Restrict fluid to 1000 mL per day.
c. Provide at least six small frequent meals per day.
d. Allow alcohol and caffeine products.

25. Which nursing intervention would be the highest priority in evaluating a patient with
peritonitis from a ruptured appendix?

a. Assessment of severity, location, and duration of pain


b. Assessment of vital signs
c. Preventing wound infection
d. Promoting balanced nutrition

26. Sulfasalazine is the recommended medication for treatment of Crohn's disease. Patient
teaching should include:

a. taking medication 2 hours before meals.


b. limiting fluid intake.
c. ensuring adequate hydration to prevent crystallization in kidneys.
d. increased effectiveness of oral contraceptives.

27. The most lethal complication of a peptic ulcer is

a. bleeding.
b. perforation.
c. severe pain.
d. gastric outlet obstruction.

28. Symptoms of GERD (gastroesophageal reflux disease) can be modified or eliminated by


which nursing interventions?

a. Eat three large meals daily.


b. Follow a high-protein, high-fat diet.
c. Remain upright for 1 to 2 hours post meals.
d. Eat a snack 1 hour before bedtime.

29. The purpose of antibiotic therapy in treating stomach disorders is that it

a. eradicates H. pylori.
b. inhibits gastric acid secretion.
c. protects the gastric mucosa.
d. neutralizes or reduces the acidity of stomach contents.

30. Peptic ulcers are often common in the aging population. Which medications should be taken
with caution to help prevent this problem?
a. Antibiotics
b. Antacids
c. NSAIDs
d. Laxatives

31. You administered the medication GoLYTELY to a 78-year-old woman in preparation for
diagnostic tests. In planning for her care, which would be most appropriate?

a. Side rails up to prevent falling


b. Door closed for privacy
c. Available bedside commode for possible weakness
d. Monitor vital signs

32. A patient is admitted with a diagnosis of Crohn's disease. What nursing interventions would
be appropriate when caring for this patient? (Select all that apply.)
a. Daily weight
b. Monitor I & O every shift
c. Fluid restriction
d. Accessibility to bedside commode

33. A patient was recently diagnosed with colorectal cancer. His wife asks the nurse, "What
prevents colon cancer?" The nurse's answer should include which factors? (Select all that
apply.)

a. A diet high in fiber


b. Familial predisposition of a cancer-causing gene
c. Regular checkups
d. A diet high in animal fats

34. Constipation is a problem for many older adults. The medical management to prevent
constipation includes (Select all that apply):

a. Decreasing physical activity


b. Decreasing fluid intake
c. Nutritional diet high in fiber
d. Increasing fluid intake
e. Increasing daily activity
f. Decreasing strict daily routines

35. Flexible sigmoidoscopy should be performed every years.


ANS:
5
five

36. Colonoscopy should be performed every years.


ANS:
10
ten

-M.J. calls to tell the nurse that her 85-year-old mother has been nauseated all day and has
vomited twice. Before the nurse hangs up and calls the health care provider, she should instruct
M.J. to:

a. administer antispasmodic drugs and observe skin turgor


b. give her mother sips of water and elevate the head of her bed to prevent aspiration
c. offer her mother a high-protein liquid supplement to drink to maintain her nutritional needs
d. offer her mother large quantities of Gatorade to drink because older adults are at rick for
sodium depletion

-The nurse explains to the patient with Vincent's infection that treatment will include:

a. smallpox vaccinations
b. viscous lidocaine rinses
c. amphotericin B suspension
d. topical application of antibiotics

-The nurse teaching young adults about behaviors that put them at risk for oral cancer includes:

a. discouraging use of chewing gum


b. avoiding use of perfumed lip gloss
c. avoiding use of smokeless tobacco
d. discouraging drinking of carbonated beverages

-The nurse explains to the patient with GERD that this disorder:
a. results in acid erosion of the esophagus from frequent vomiting
b. will require surgical wrapping or repair of the pyloric sphincter to control the symptoms
c. is the protrusion of a portion of the stomach into the esophagus through an opening in the
diaphragm
d. often involves relaxation of the lower esophageal sphincter, allowing stomach
contents to back up into the esophagus
-A patient who has undergone an esophagectomy for esophageal cancer develops increasing
pain, fever, and dyspnea when a full liquid diet is started postoperatively. The nurse recognizes
that these symptoms are more indicative of:

a. an intolerance to the feedings


b. extension of the tumor into the aorta
c. leakage of fluid or foods into the mediastinum
d. esophageal perforation with fistula formation into the lung
.
-The pernicious anemia that may accompany gastritis is due to:

a. chronic autoimmune destruction of cobalamin stores in the body


b. progressive gastric atrophy from chronic breakage in the mucosal barrier and blood loss
c. a lack of intrinsic factor normally produced by acid-secreting cells of the gastric
mucosa
d. hyperchlorhydria resulting from an increase in acid-secreting parietal cells and degradation of
RBCs.

-The nurse is teaching the patient and family that peptic ulcers are:

a. caused by a stressful lifestyle and other acid-producing factors such as H. pylori.


b. inherited within families and reinforced by bacterial spread of Staphylococcus aureus in
childhood
c. promoted by factors that tend to cause over secretion of acid, such as excess dietary fats,
smoking, and H. pylori.
d. promoted by a combination of factors that may result in erosion of the gastric mucosa,
including certain drugs and alcohol
-An optimal teaching plan for an outpatient with stomach cancer receiving radiation therapy
should include information about:

a. cancer support groups, alopecia, and stomatitis


b. avitaminosis, ostomy care, and community resources
c. prosthetic devices, skin conductance, and grief counseling
d. wound and skin care, nutrition, drugs, and community resources

-The teaching plan for the patient being discharged after an acute episode of upper GI bleeding
includes information concerning the importance of (select all that apply):

a. only taking aspirin with milk or bread products


b. avoiding taking aspirin and drugs containing aspirin.
c. only taking drugs prescribed by the health care provider.
d. taking all drugs 1 hours before mealtime to prevent further bleeding
e. reading all OTC drug labels to avoid those containing stearic acid and calcium

-Several patients are seen at an urgent care center with symptoms of nausea, committing, and
diarrhea that began 2 hours ago while attending a large family reunion potluck dinner. You
question the patients specifically about foods they ingested containing:

a. beef
b. meat and milk
c. poultry and eggs
d. home-preserved vegetables

1. A client complains of right lower abdominal pain and nausea and appendicitis is suspected.
Which of the following lab reports would assist in confirming the diagnosis?

a. 5,000/mm3 WBC (white blood cells); 50% neutrophils


b. 10,000/mm3 WBC; 75% neutrophils
c. 15,000/mm3 WBC; 85% neutrophils
d. 2,000/mm3 WBC; 40% neutrophils

2. A client is admitted with the diagnosis Rule out appendicitis. Which nursing action is MOST
important?

a. assessing the client's pain threshold


b. avoiding administering analgesics
c. encouraging the family to visit
d. monitoring laboratory reports

3. Clients with diverticulitis will frequently complain of abdominal pain located in which area of
the abdomen?

a. generalized
b. left lower side
c. right lower side
d. umbilicus area

4. A client is admitted with diverticulitis of 1 day's duration. Which of these nursing interventions
should be included in the client's plan of care?

a. Offer oral fluids frequently.


b. Ambulate ad lib.
c. Keep the client in a private room.
d. Maintain NPO.

5. A client who has diverticulitis should be instructed to follow which of these prescribed diets
when the acute episode is resolved?

a. bland, low residue


b. clear liquids
c. high fiber
d. high protein

6. Which of these statements about Crohn's disease is TRUE?


a. It affects only the mucosal lining of the bowel.
b. It is treated by colostomy.
c. It can occur anywhere in the small and large intestines.
d. It can produce 15 to 20 liquid stools containing blood daily.

7. A client who has ulcerative colitis has an increased chance of developing which of the
following?

a. viral pneumonia
b. colorectal cancer
c. pernicious anemia
d. intestinal obstruction

8. An ulcerative colitis client is having a severe flare-up. What most likely will be the treatment
the physician prescribes next for this client?
a. decompression with NG tube
b. total parenteral nutrition for bowel rest
c. a course of antibiotics
d. clear liquid diet

9. Clients with IBS may be prescribed what type of medication prior to meals to prevent
diarrhea?
a. bulk forming
b. antispasmodic
c. antidiarrheal
d. antihistamine

10. Paralytic ileus can develop as the result of:

a. adhesions that obstruct the bowel's lumen


b. interrupted blood flow to a portion of the bowel, which then becomes necrotic
c. interrupted nerve transmission to the bowel with a portion becoming paralyzed
d. a tumor obstructing the bowel's lumen

11. A client complains of colicky abdominal pain, nausea, constipation, and bloating. During the
nursing assessment, the client vomits fecal material. Which of these conditions should be
suspected?

a. abdominal hernia
b. duodenal ulcer
c. intestinal obstruction
d. pancreatitis

12. Nursing interventions for clients with intestinal obstruction include all of the following. Which
action would be a priority?

a. administering IV fluid and electrolyte replacements as ordered


b. checking weight daily
c. giving oral care frequently
d. monitoring intake and output every shift

13. A client is lifting a box and experiences a sudden pain in the lower right side and upper
thigh, with a "bulge" in the groin area. The nurse notes a tight band around the bulge. The client
has probably developed which type of hernia?

a. abdominal, reducible
b. femoral, strangulated
c. inguinal, incarcerated
d. umbilical, strangulated
14. Replacing the hernia into the abdomen and reinforcing the weakened muscle wall with wire,
mesh or fascia is called:

a. herniorrhaphy
b. hernioplasty
c. hernia reduction
d. herniorectoplasty

15. A client develops peritonitis as a result of a ruptured appendix. The nurse should carefully
monitor the client for evidence of which of these complications?

a. hypothermia
b. thrombosis
c. hepatitis
d. paralytic ileus

16. A client who has surgical treatment for peritonitis would be encouraged to do all of the
following postoperatively. Which is the MOST important for this client?

a. active exercises of the ankles and knees


b. pulmonary hygiene
c. splinting the abdominal incision with a pillow
d. turning every 2 hours

17. A postoperative client has been receiving narcotic analgesics every 4 hours for incisional
pain for 2 days and has now developed severe constipation. An enema has been prescribed by
the health care provider. Client education about preventing constipation should emphasize that
enemas and laxatives are not recommended for long-term use, because they can:

a. cause fluid and electrolyte imbalances


b. interrupt normal bowel function
c. remove normal bowel flora and cause infections
d. traumatize the intestinal lining

18. An older adult client tells the nurse about experiencing chronic constipation. The nurse
should suggest that the client increase intake of which of these foods?

a. baked potatoes and rice


b. milk and cheese
c. fish and red meat
d. apples and broccoli

19. Which one of these statements are TRUE about colon cancer?

a. All colorectal cancers arise from polyps.


b. Prognosis is not very good even if caught early.
c. Stools are usually guiaic negative.
d. There is no mucus in the stool.

20. A client with external hemorrhoids has called the primary physician's office and has asked
the nurse how much fiber and what volume of fluids should be ingested daily to help with the
discomfort of the hemorrhoids. What should the nurse tell the client?

a. 5-10 g fiber, 1000 ml


b. 10-20 g fiber, 1500 ml
c. 20-30 g fiber, 2500 ml
d. 30-40 g fiber, 4000 ml

1. Which of the following are characteristics of ulcerative colitis? (Select all that apply.)

a. affects only the colon


b. affects the entire thickness of the bowel
c. has potential for hemorrhage in acute phase
d. surgery usually does not cure or limit the progress of the disease
e. has a genetic link
f. client may have 15 to 20 liquid stools/day

2. The nurse is teaching a client strategies to manage ulcerative colitis. Which of the following
would be included in the client's teaching plan? (Select all that apply.)

a. Eat three large meals a day


b. Encourage client to keep food diary and avoid foods that increase symptoms
c. Exercise regularly
d. Practice stress relieving measures
e. Enjoy caffeinated beverages in large quantities
The nurse is monitoring a client admitted to the hospital with a diagnosis of appendicitis who is
scheduled for surgery in 2 hours. The client begins to complain of increased abdominal pain and
begins to vomit. On assessment, the nurse notes that the abdomen is distended and bowel
sounds are diminished. Which is the most appropriate nursing intervention?

A. Notify the health care provider (HCP).


B. Administer the prescribed pain medication.
C. Call and ask the operating room team to perform surgery as soon as possible.
D. Reposition the client and apply a heating pad on the warm setting to the client's abdomen.

A client has just had a hemorrhoidectomy. Which nursing interventions are appropriate for this
client? Select all that apply.

A. Administer stool softeners as prescribed.


B. Instruct the client to limit fluid intake to avoid urinary retention.
C. Encourage a high-fiber diet to promote bowel movements without straining.
D. Apply cold packs to the anal-rectal area over the dressing until the packing is
removed.
E. Help the client to a Fowler's position to place pressure on the rectal area and decrease
bleeding.

The nurse is planning to teach a client with gastroesophageal reflux disease (GERD) about
substances to avoid. Which items should the nurse include on this list? Select all that apply.

A. Coffee
B. Chocolate
C. Peppermint
D. Nonfat milk
E. Fried chicken
F. Scrambled eggs

A client has undergone esophagogastroduodenoscopy. The nurse should place highest priority
on which item as part of the client's care plan?

1. Monitoring the temperature


2. Monitoring complaints of heartburn
3. Giving warm gargles for a sore throat
4. Assessing for the return of the gag reflex

The nurse is providing dietary teaching for a client with a diagnosis of chronic gastritis. The
nurse instructs the client to include which foods rich in vitamin B12 in the diet? Select all that
apply.

A. Nuts
B. Corn
C. Liver
D. Apples
E. Lentils
F. Bananas

The nurse is monitoring a client with a diagnosis of peptic ulcer. Which assessment finding
would most likely indicate perforation of the ulcer?

A. Bradycardia
B. Numbness in the legs
C. Nausea and vomiting
D. A rigid, boardlike abdomen

The nurse is caring for a client following a gastrojejunostomy (Billroth II procedure). Which
postoperative prescription should the nurse question and verify?

A. Leg exercises
B. Early ambulation
C. Irrigating the nasogastric tube
D. Coughing and deep-breathing exercises

The nurse is providing discharge instructions to a client following gastrectomy and should
instruct the client to take which measure to assist in preventing dumping syndrome?

A. Ambulate following a meal.


B. Eat high-carbohydrate foods.
C. Limit the fluids taken with meals.
D. Sit in a high Fowler's position during meals.

The nurse is providing discharge teaching for a client with newly diagnosed Crohn's disease
about dietary measures to implement during exacerbation episodes. Which statement made by
the client indicates a need for further instruction?

A. "I should increase the fiber in my diet."


B. "I will need to avoid caffeinated beverages."
C. "I'm going to learn some stress reduction techniques."
D. "I can have exacerbations and remissions with Crohn's disease."
The nurse is doing an admission assessment on a client with a history of duodenal ulcer. To
determine whether the problem is currently active, the nurse should assess the client for which
sign(s)/symptom(s) of duodenal ulcer?

A. Weight loss
B. Nausea and vomiting
C. Pain relieved by food intake
D. Pain radiating down the right arm

A client with hiatal hernia chronically experiences heartburn following meals. The nurse should
plan to teach the client to avoid which action because it is contraindicated with a hiatal hernia?

A. Lying recumbent following meals


B. Consuming small, frequent, bland meals
C. Taking H2-receptor antagonist medication
D. Raising the head of the bed on 6-inch (15 cm) blocks

The nurse is providing care for a client with a recent transverse colostomy. Which observation
requires immediate notification of the health care provider?

A. Stoma is beefy red and shiny


B. Purple discoloration of the stoma
C. Skin excoriation around the stoma
D. Semi-formed stool noted in the ostomy pouch

A client had a new colostomy created 2 days earlier and is beginning to pass malodorous flatus
from the stoma. What is the correct interpretation by the nurse?

A. This is a normal, expected event.


B. The client is experiencing early signs of ischemic bowel.
C. The client should not have the nasogastric tube removed.
D. This indicates inadequate preoperative bowel preparation.

.
A client has just had surgery to create an ileostomy. The nurse assesses the client in the
immediate postoperative period for which most frequent complication of this type of surgery?

A. Folate deficiency
B. Malabsorption of fat
C. Intestinal obstruction
D. Fluid and electrolyte imbalance

The nurse provides instructions to a client about measures to treat inflammatory bowel
syndrome (IBS). Which statement by the client indicates a need for further teaching?

A. "I need to limit my intake of dietary fiber."


B. "I need to drink plenty, at least 8 to 10 cups daily."
C. "I need to eat regular meals and chew my food well."
D. "I will take the prescribed medications because they will regulate my bowel patterns."

The nurse is monitoring a client for the early signs and symptoms of dumping syndrome. Which
findings indicate this occurrence?

A. Sweating and pallor


B. Bradycardia and indigestion
C. Double vision and chest pain
D. Abdominal cramping and pain

The nurse inspects the color of the drainage from a nasogastric tube on a postoperative client
approximately 24 hours after gastric surgery. Which finding indicates the need to notify the
health care provider (HCP)?

A. Dark red drainage


B. Dark brown drainage
C. Green-tinged drainage
D. Light yellowish-brown drainage

The nurse is caring for a client admitted to the hospital with a suspected diagnosis of acute
appendicitis. Which laboratory result should the nurse expect to note if the client does have
appendicitis?

A. Leukopenia with a shift to the left


B. Leukocytosis with a shift to the left
C. Leukopenia with a shift to the right
D. Leukocytosis with a shift to the right

After undergoing Billroth I gastric surgery, the client experiences fatigue and complains of
numbness and tingling in the feet and difficulties with balance. On the basis of these symptoms,
the nurse suspects which postoperative complication?

A. Stroke
B. Pernicious anemia
C. Bacterial meningitis
D. Peripheral arterial disease

A client experiencing chronic dumping syndrome makes the following comments to the nurse.
Which one indicates the need for further teaching?

A. "I eat at least 3 large meals each day."


B. "I eat while lying in a semirecumbent position."
C. "I have eliminated taking liquids with my meals."
D. "I eat a high-protein, low- to moderate-carbohydrate diet."

The nurse obtains an admission history for a client with suspected peptic ulcer disease (PUD).
Which client factor documented by the nurse would increase the risk for PUD?

A. Recently retired from a job


B. Significant other has a gastric ulcer
C. Occasionally drinks 1 cup of coffee in the morning
D. Takes nonsteroidal antiinflammatory drugs (NSAIDs) for osteoarthritis

A client who has undergone gastric surgery has a nasogastric (NG) tube connected to low
intermittent suction that is not draining properly. Which action should the nurse take initially?

A. Call the surgeon to report the problem.


B. Reposition the NG tube to the proper location.
C. Check the suction device to make sure it is working.
D. Irrigate the NG tube with saline to remove the obstruction.

The nurse is caring for a client with ulcerative colitis. Which finding does the nurse determine is
consistent with this diagnosis?
A. Hypercalcemia
B. Hypernatremia
C. Frothy, fatty stools
D. Decreased hemoglobin

A client with acute ulcerative colitis requests a snack. Which is the most appropriate snack for
this client?

A. Carrots and ranch dip


B. Whole-grain cereal and milk
C. A cup of popcorn and a cola drink
D. Applesauce and a graham cracker

The nurse is teaching an older client about measures to prevent constipation. Which statement
by the client indicates a need for further teaching?

A. "I walk 1 to 2 miles every day."


B. "I need to decrease fiber in my diet."
C. "I have a bowel movement every other day."
D. "I drink 6 to 8 glasses of water every day."

The nurse is caring for a client after abdominal surgery and creation of a colostomy. The nurse
is assessing the client for a prolapsed stoma and should expect to note which observation if this
is present?

A. A sunken and hidden stoma


B. A narrow and flattened stoma
C. A stoma that is dusky or bluish
D. A stoma that is elongated with a swollen appearance

The nurse is providing instructions to a client with a colostomy about measures to reduce the
odor from the colostomy. Which client statement indicates that the educational session was
effective?

A. "I should be sure to eat at least 1 cucumber every day."


B. "Beet greens, parsley, or yogurt will help to control the colostomy odor."
C. "I will need to increase my egg intake and try to eat ½ to 1 egg per day."
D. "Green vegetables such as spinach and broccoli will prevent odor, and I should eat these
foods every day."

The nurse has provided dietary instructions to a client with a diagnosis of peptic ulcer disease.
Which client statement indicates that education was effective?

A. "Baked foods such as chicken or fish are all right to eat."


B. "Citrus fruits and raw vegetables need to be included in my daily diet."
C. "I can drink beer as long as I consume only a moderate amount each day."
D. "I can drink coffee or tea as long as I limit the amount to 2 cups daily."
The nurse has provided home care instructions to a client who had a subtotal gastrectomy. The
nurse instructs the client on the signs and symptoms associated with dumping syndrome. Which
client statement indicates that teaching was effective?

A. "It will cause diaphoresis and diarrhea."


B. "I have to monitor for hiccups and diarrhea."
C. "It will be associated with constipation and fever."
D. "I have to monitor for fatigue and abdominal pain."

The nurse is providing instructions to a client regarding measures to minimize the risk of
dumping syndrome. The nurse should make which suggestion to the client?

A. Maintain a high-carbohydrate diet.


B. Increase fluid intake, particularly at mealtime.
C. Maintain a low Fowler's position while eating.
D. Ambulate for at least 30 minutes following each meal.

A client with peptic ulcer disease states that stress frequently causes exacerbation of the
disease. The nurse determines that which item mentioned by the client is most likely to be
responsible for the exacerbation?

A. Sleeping 8 to 10 hours a night


B. Ability to work at home periodically
C. Eating 5 or 6 small meals per day
D. Frequent need to work overtime on short notice

The nurse is giving dietary instructions to a client who has a new colostomy. The nurse should
encourage the client to eat foods representing which diet for the first 4 to 6 weeks
postoperatively?

A. Low fiber
B. Low calorie
C. High protein
D. High carbohydrate

The nurse has implemented a bowel maintenance program for an unconscious client. The nurse
would evaluate the plan as best meeting the needs of the client if which method was successful
in stimulating a bowel movement?

A. Fleet enema
B. Fecal disimpaction
C. Glycerin suppository
D. Soap solution enema (SSE)

The nurse is preparing to teach a client with a new colostomy about how to perform a colostomy
irrigation. Which information should the nurse include in the teaching plan?

A. Use 500 to 1000 mL of warm tap water.


B. Suspend the irrigant 36 inches above the stoma.
C. Insert the irrigation cone ½ inch into the stoma.
D. If cramping occurs, open the irrigation clamp farther.

A client in a long-term care facility is being prepared to be discharged to home in 2 days. The
client has been eating a regular diet for a week; however, he is still receiving intermittent enteral
tube feedings and will need to receive these feedings at home. The client states concern that he
will not be able to continue the tube feedings at home. Which nursing response is most
appropriate at this time?

A. "Do you want to stay here in this facility for a few more days?"
B. "Have you discussed your feelings with your health care provider?"
C. "You need to talk to your health care provider about these concerns."
D. "Tell me more about your concerns with your diet after going home."

The nurse should anticipate that the health care provider (HCP) will prescribe which treatment
for a client with pernicious anemia?

A. Oral iron tablets


B. Blood transfusions
C. Gastric tube feedings
D. Vitamin B12 injections

A client arrives at the hospital emergency department complaining of acute right lower quadrant
abdominal pain, and appendicitis is suspected. Laboratory tests are performed, and the nurse
notes that the client's white blood cell (WBC) count is elevated. On the basis of these findings,
the nurse should question which health care provider (HCP) prescription documented in the
client's medical record?

A. Apply a cold pack to the abdomen.


B. Administer 30 mL of milk of magnesia (MOM).
C. Maintain nothing by mouth (nil per os [NPO]) status.
D. Initiate an intravenous (IV) line for the administration of IV fluids.

The nurse is providing dietary instructions to a client with a diagnosis of irritable bowel
syndrome. The nurse determines that education was effective if the client states the need to
avoid which food?

A. Rice
B. Corn
C. Broiled chicken
D. Cream of wheat

Diphenoxylate hydrochloride with atropine sulfate is prescribed for a client with ulcerative colitis.
The nurse should monitor the client for which therapeutic effect of this medication?

A. Decreased diarrhea
B. Decreased cramping
C. Improved intestinal tone
D. Elimination of peristalsis

Sulfasalazine is prescribed for a client with a diagnosis of ulcerative colitis, and the nurse
instructs the client about the medication. Which statement made by the client indicates a need
for further teaching?

A. "The medication will cause constipation."


B. "I need to take the medication with meals."
C. "I may have increased sensitivity to sunlight."
D. "This medication should be taken as prescribed."

A client who has been advanced to a solid diet after undergoing a subtotal gastrectomy. What is
the appropriate nursing intervention in preventing dumping syndrome?

A. Remove fluids from the meal tray.


B. Give the client 2 large meals per day.
C. Ask the client to sit up for 1 hour after eating.
D. Provide concentrated, high-carbohydrate foods.

The nurse is caring for a client with gastroesophageal reflux disease (GERD) and provides
client education on measures to decrease GERD. Which statement made by the client indicates
a need for further teaching?

A. "I plan to eat 4 to 6 small meals a day."


B. "I should sleep in the right side-lying position."
C."I plan to have a snack 1 hour before going to bed."
D. "I will stop having a glass of wine each evening with dinner."

The registered nurse is precepting a new nurse who is caring for a client with pernicious anemia
as a result of gastrectomy. Which statement made by the new nurse indicates understanding of
this diagnosis?

A. "It's due to insufficient production of vitamin B12 in the colon."


B. "Increased production of intrinsic factor in the stomach leads to this type of anemia."
C. "Overproduction of vitamin B12 in the large intestine can result in pernicious anemia."
D. "Decreased production of intrinsic factor by the stomach affects absorption of vitamin
B12 in the small intestine."

A client with appendicitis is scheduled for an appendectomy. The nurse providing preoperative
teaching for the client describes the location of the appendix by stating that it is attached to
which part of the gastrointestinal (GI) system?

A. Ileum
B. Cecum
C. Rectum
D. Jejunum

The nurse is caring for a group of clients on the surgical nursing unit. The nurse anticipates that
the client who underwent which procedure is most likely to have some long-term residual
difficulty with absorption of nutrients?

A. Colectomy
B. Appendectomy
C. Ascending colostomy
D. Small bowel resection

The nurse is caring for a client with spinal cord injury (SCI) who is participating in a bowel
retraining program. What should the nurse anticipate to promote during the bowel retraining
program?

A. Sufficiently low water content in the stool


B. Low intestinal roughage that promotes easier digestion
C. Constriction of the anal sphincter based on voluntary control
D. Stimulation of the parasympathetic reflex center at the S1 to S4 level in the spinal cord

The nurse is evaluating the plan of care for a client with peptic ulcer disease (PUD) who is
experiencing acute pain. The nurse determines that the expected outcomes have been met if
the nursing assessment reveals which result?

A. The client reports some pain before meals.


B. The client frequently is awakened at 2 a.m. with heartburn.
C. The client has eliminated any irritating foods from the diet.
D. The client's pain is minimal with histamine H2-receptor antagonists.

A client is readmitted to the hospital with dehydration after surgery for creation of an ileostomy.
The nurse assesses that the client has lost 3 lb of weight, has poor skin turgor, and has
concentrated urine. The nurse interprets the client's clinical picture as correlating most closely
with recent intake of which medication, which is contraindicated for the ileostomy client?

A. Folate
B. Biscodyl
C. Ferrous sulfate
D. Cyanocobalamin

A home care nurse is visiting a client with a diagnosis of pernicious anemia that developed as a
result of gastric surgery. The nurse instructs the client that in this disorder because the stomach
lining produces a decreased amount of a substance known as the intrinsic factor, the client will
need which medication?

A. Vitamin B12 injections


B. Vitamin B6 injections
C. An antibiotic
D. An antacid

A client arrives at the hospital emergency department complaining of acute right lower quadrant
abdominal pain. Appendicitis is suspected, and appropriate laboratory tests are performed. The
emergency department nurse reviews the test results and notes that the client's white blood cell
(WBC) count is elevated. The nurse also reviews the prescriptions from the health care provider
(HCP). The nurse should contact the HCP to question which prescription if noted in the client's
record?

A. Maintain a semi Fowler's position.


B. Maintain on NPO (nothing by mouth) status.
C.Apply a heating pad to the lower abdomen for comfort.
D. Initiate an intravenous (IV) line with the administration of IV fluids.

The nurse is caring for a client prescribed enteral feeding via a newly inserted nasogastric (NG)
tube. Before initiating the enteral feeding, the nurse should perform which action first?

A. Warm the feeding to 103°F (39.4°C).


B. Confirm NG placement by x-ray study.
C. Make sure the continuous enteral feeding tubing is primed.
D. Position the head of the client's bed to 30 degrees or greater.

The nurse is caring for a client who had a subtotal gastrectomy. The nurse should assess the
client for which signs and symptoms of dumping syndrome?

A. Diarrhea, chills, and hiccups


B. Weakness, diaphoresis, and diarrhea
C. Fever, constipation, and rectal bleeding
D. Abdominal pain, elevated temperature, and weakness

The nurse is caring for a client who has just returned from the operating room after the creation
of a colostomy. The nurse is assessing the drainage in the pouch attached to the site where the
colostomy was formed and notes serosanguineous drainage. Which nursing action is
appropriate based on this assessment?

A. Apply ice to the stoma site.


B. Apply pressure to the stoma site.
C. Notify the health care provider (HCP).
D. Document the amount and characteristics of the drainage.

A client has a large, deep duodenal ulcer diagnosed by endoscopy. Which sign or symptom
indicative of a complication should the nurse look for during the client's postprocedure
assessment?
A. Bradycardia
B. Nausea and vomiting
C. Numbness in the legs
D. A rigid, boardlike abdomen

The nurse is assisting a client with Crohn's disease to ambulate to the bathroom. After the client
has a bowel movement, the nurse should assess the stool for which characteristic that is
expected with this disease?

A. Blood in the stool


B. Chalky gray stool
C. Loose, watery stool
D. Dry, hard, constipated stool

The nurse is assessing a client with a duodenal ulcer. The nurse interprets that which sign or
symptom is most consistent with the typical presentation of duodenal ulcer?

A. Weight loss
B. Nausea and vomiting
C. Pain that is relieved by food intake
D. Pain that radiates down the right arm
C. Pain that is relieved by food intake

The nurse teaches a preoperative client about the use of a nasogastric (NG) tube for the
planned surgery. Which statement indicates to the nurse that the client understands when the
tube can be removed in the postoperative period?

A. "When I can tolerate food without vomiting."


B. "When my gastrointestinal system is healed enough."
C. "When my health care provider says the tube can come out."
D. "When my bowels begin to function again, and I begin to pass gas."

A client with gastritis asks the nurse at a screening clinic about analgesics that will not cause
epigastric distress. The nurse should tell the client that which medication is unlikely to cause
epigastric distress?

A. Ibuprofen
B. Indomethacin
C. Acetaminophen
D. Naproxen sodium

The nurse is caring for a client who is receiving bolus feedings via a nasogastric tube. As the
nurse is finishing the feeding, the client asks for the bed to be positioned flat for sleep. The
nurse understands that which is the appropriate position for this client at this time?

A. Head of bed flat, with the client supine for 60 minutes


B. Head of bed flat, with the client in the supine position for at least 30 minutes
C. Head of bed elevated 30 to 45 degrees, with the client in the right lateral position for 60
minutes
D. Head of bed in a semi Fowler's position, with the client in the left lateral position for 60
minutes

The nurse is preparing to administer an intermittent enteral feeding though a nasogastric (NG)
tube. Which priority assessment should the nurse perform?

A. Observe for digestion of formula.


B. Assess fluid and electrolyte status.
C. Evaluate absorption of the last feeding.
D. Evaluate percussion tone of the stomach.
C. Evaluate absorption of the last feeding.

The nurse has inserted a nasogastric (NG) tube to the level of the oropharynx and has
repositioned the client's head in a flexed-forward position. The client has been asked to begin
swallowing. The nurse starts to slowly advance the NG tube with each swallow. The client
begins to cough, gag, and choke. Which actions should the nurse take that will result in proper
tube insertion and promote client relaxation? Select all that apply.

A. Pull the tube back slightly.


B. Instruct the client to breathe slowly.
C. Assist the client to take sips of water.
D. Continue to slowly advance the tube to the desired distance.
E. Check the back of the pharynx using a tongue blade and flashlight.

The client with a small bowel obstruction asks the nurse to explain the purpose of the
nasogastric tube attached to continuous gastric suction. The nurse determines that teaching has
been effective if the client makes which statement?

A. "It will help to provide me with nourishment."


B. "It will help to relieve the congestion from excess mucus."
C. "It is used to remove gastric contents for laboratory testing."
D. "It will help to remove gas and fluids from my stomach and intestine."

A client is scheduled for an upper gastrointestinal (GI) endoscopy. Which assessment is


essential to include in the plan of care following the procedure?

A. Assessing pulses
B. Monitoring urine output
C. Monitoring for rectal bleeding
D. Assessing for the presence of the gag reflex

.
A client with a gastric ulcer is prescribed both magnesium hydroxide and cimetidine twice daily.
How should the nurse schedule the medications for administration?

A. Drink 8 ounces of water between taking each medication.


B. Administer the cimetidine and magnesium hydroxide at the same time twice daily.
C. Administer each dose of cimetidine 1 hour prior to the administration of magnesium
hydroxide.
D. Collaborate with the health care provider (HCP), as the client should not be receiving both
medications.

The nurse is caring for a client admitted to the hospital with suspected acute appendicitis. Which
laboratory result should the nurse expect to note if the client does have appendicitis?

A. White blood cell (WBC) count of 4000 mm3 (4 × 109/L)


B. WBC count of 8000 mm3 (8 × 109/L)
C. WBC count of 18,000 mm3 (18 × 109/L)
D. WBC count of 26,000 mm3 (26 × 109/L)

The nurse is teaching the postgastrectomy client about measures to prevent dumping
syndrome. Which statement by the client indicates a need for further teaching?

A. "I need to lie down after eating."


B. "I need to drink liquids with meals."
C. "I need to avoid concentrated sweets."
D. "I need to eat small meals 6 times daily."

The nurse is caring for a client with pernicious anemia. Which prescription by the health care
provider (HCP) should the nurse anticipate?

A. Iron
B. Folic acid
C. Vitamin B6
D. Vitamin B12

.
A client presents to the emergency department with upper gastrointestinal (GI) bleeding and is
in moderate distress. In planning care, which nursing action should be the priority for this client?

A. Assessment of vital signs


B. Complete abdominal examination
C. Thorough investigation of precipitating events
D. Insertion of a nasogastric tube and Hematest of emesis

The nurse has given postprocedure instructions to a client who has undergone a colonoscopy.
Which statement by the client indicates the need for further teaching?

A. "It is normal to feel gassy or bloated after the procedure."


B. "The abdominal muscles may be tender from the procedure."
C. "It is all right to drive once I've been home for an hour or so."
D. "Intake should be light at first and then progress to regular intake."

The nurse is reviewing the medication record of a client with acute gastritis. Which medication, if
noted on the client's record, should the nurse question?

A. Digoxin
B. Furosemide
C. Indomethacin
D. Propranolol hydrochloride

The nurse is caring for a client postoperatively after creation of a colostomy. What is an
appropriate potential client problem?

A. Fear
B. Sexual dysfunction
C. Disturbed body image
D. Imbalanced nutrition: more than body requirements

The nurse is caring for a hospitalized client with a diagnosis of ulcerative colitis. Which finding, if
noted on assessment of the client, should the nurse report to the health care provider (HCP)?

A. Hypotension
B. Bloody diarrhea
C. Rebound tenderness
D. A hemoglobin level of 12 mg/dL (120 mmol/L)

The medication history of a client with peptic ulcer disease reveals intermittent use of several
medications. The nurse would teach the client that which of these medications are not a part of
the treatment plan because of its irritating effects on the lining of the gastrointestinal tract?

A. Nizatidine
B. Sucralfate
C. Ibuprofen
D. Omeprazole

The nurse should instruct a client with an ileostomy to include which action as part of essential
care of the stoma?

A. Massage the area below the stoma.


B. Take in high-fiber foods such as nuts.
C. Limit fluid intake to prevent diarrhea.
D. Cleanse the peristomal skin meticulously.

A client with Crohn's disease is experiencing acute pain, and the nurse provides information
about measures to alleviate the pain. Which statement by the client indicates the need for
further teaching?

A. "I know I can massage my abdomen."


B. "I will continue using antispasmodic medication."
C. "One of the best things I can do is use relaxation techniques."
D. "The best position for me is to lie supine with my legs straight."

A client with ulcerative colitis has a prescription to begin a salicylate compound medication to
reduce inflammation. What instruction should the nurse give the client regarding when to take
this medication?

A. On arising
B. After meals
C. On an empty stomach
D. 30 minutes before meals

A client is admitted to the hospital with a diagnosis of acute diverticulitis. What should the nurse
expect to be prescribed for this client?

A. NPO (nothing by mouth) status


B. Ambulation at least 4 times daily
C. Cholinergic medications to reduce pain
D. Coughing and deep breathing every 2 hours

The nurse is performing an admission assessment on a client who has been admitted to the
hospital with a diagnosis of suspected gastric ulcer. The nurse is asking the client questions
about pain. Which client statement supports the diagnosis of gastric ulcer?

A. "The pain doesn't usually come right after I eat."


"B. The pain gets so bad that it wakes me up at night."
C. "The pain that I get is located on the right side of my chest."
D. "My pain comes shortly after I eat, maybe a half-hour or so later."

A client receiving a cleansing enema complains of pain and cramping. The nurse should take
which corrective action?

A. Discontinue the enema.


B. Reassure the client, and continue the flow.
C. Raise the enema bag so that the solution can be completed quickly.
D. Clamp the tubing for 30 seconds, and restart the flow at a slower rate.

A client with a history of gastrointestinal upset has been diagnosed with acute diverticulitis. The
nurse should anticipate a prescription from the health care provider for which type of diet for this
client?

A. A low-fat diet
B. A low-fiber diet
C. A high-protein diet
D. A high-carbohydrate diet
The nurse is providing discharge instructions for a client following a Roux-en-Y gastric bypass
surgery 3 days ago. What will the nurse include in the instructions? Select all that apply.

A. Do not drink fluids with meals.


B. Avoid foods high in carbohydrates.
C. Take an extended-release multivitamin daily.
D. Maintain a clear liquid diet for about 6 weeks.
E. Eat 6 small meals a day that are high in protein.

The nurse cares for a client following a Roux-en-Y gastric bypass surgery. Which nursing
intervention is appropriate?

A. Encourage the client to ambulate.


B. Position the client on the left side.
C. Frequently irrigate the nasogastric tube (NG) with 30 mL saline.
D. Discourage the use of the patient-controlled analgesia (PCA) machine.

The nurse is providing instructions to a client diagnosed with irritable bowel syndrome (IBS) who
is experiencing abdominal distention, flatulence, and diarrhea. What interventions should the
nurse include in the instructions? Select all that apply.

A. Eat yogurt.
B. Take loperamide to treat diarrhea.
C. Use stress management techniques.
D. Avoid foods such as cabbage and broccoli.
E. Decrease fiber intake to less than 15 g/day.

The nurse is caring for a client experiencing an exacerbation of Crohn's disease. Which
intervention should the nurse anticipate the health care provider prescribing?

A. Enteral feedings
B. Fluid restrictions
C. Oral corticosteroids
D. Activity restrictions
C. Oral corticosteroids

The nurse caring for a client diagnosed with inflammatory bowel disease (IBD) recognizes that
which classifications of medications may be prescribed to treat the disease and induce
remission? Select all that apply.

A. Antidiarrheal
B. Antimicrobial
C. Corticosteroid
D. Aminosalicylate
E. Biological therapy
F. Immunosuppressant
During a home care visit, an adult client complains of chronic constipation. What should the
nurse tell the client to do?

A. Increase potassium in the diet.


B. Include rice and bananas in the diet.
C. Increase fluid and dietary fiber intake.
D. Increase the intake of sugar-free products.

The nurse is monitoring a client admitted to the hospital with a diagnosis of appendicitis who is
scheduled for surgery in 2 hours. The client begins to complain of increased abdominal pain and
begins to vomit. On assessment, the nurse notes that the abdomen is distended and bowel
sounds are diminished. Which is the most appropriate nursing intervention?

1. Notify the health care provider (HCP).


2. Administer the prescribed pain medication.
3. Call and ask the operating room team to perform the surgery as soon as possible.
4. Reposition the client and apply a heating pad on the warm setting to the client's abdomen.

A client has been admitted to the hospital with a diagnosis of acute pancreatitis and the nurse is
assessing the client's pain. What type of pain is consistent with this diagnosis?

1. Burning and aching, located in the left lower quadrant and radiating to the hip
2. Severe and unrelenting, located in the epigastric area and radiating to the back
3. Burning and aching, located in the epigastric area and radiating to the umbilicus
4. Severe and unrelenting, located in the left lower quadrant and radiating to the groin

The nurse is assessing a client who is experiencing an acute episode of cholecystitis. Where
should the nurse anticipate the location of the pain?

1. Right lower quadrant, radiating to the back


2. Right lower quadrant, radiating to the umbilicus
3. Right upper quadrant, radiating to the left scapula and shoulder
4. Right upper quadrant, radiating to the right scapula and shoulder

A client is admitted to the hospital with viral hepatitis, complaining of "no appetite" and
"losing my taste for food." What instruction should the nurse give the client to provide
adequate nutrition?

1. Select foods high in fat.


2. Increase intake of fluids, including juices.
3. Eat a good supper when anorexia is not as severe.
4. Eat less often, preferably only three large meals daily.
A client has developed hepatitis A after eating contaminated oysters. The nurse assesses the
client for which expected assessment finding?

1. Malaise
2. Dark stools
3. Weight gain
4. Left upper quadrant discomfort
1. Malaise

A client has just had a hemorrhoidectomy. Which nursing interventions are appropriate for this
client? Select all that apply.

1. Administer stool softeners as prescribed.


2. Instruct the client to limit fluid intake to avoid urinary retention.
3. Instruct the client to avoid activities that will initiate vasovagal responses.
4. Encourage a high-fiber diet to promote bowel movements without straining.
5. Apply cold packs to the anal-rectal area over the dressing until the packing is
removed.
6. Help the client to a Fowler's position to place pressure on the rectal area and decrease
bleeding.

The nurse is planning to teach a client with gastroesophageal reflux disease about substancesto
avoid. Which items should the nurse include on this list? Select all that apply.

1. Coffee
2. Chocolate
3. Peppermint
4. Nonfat milk
5. Fried chicken
6. Scrambled eggs

A client has undergone esophagogastroduodenoscopy. The nurse should place highest priority
on which item as part of the client's care plan?

1. Monitoring the temperature


2. Monitoring complaints of heartburn
3. Giving warm gargles for a sore throat
4. Assessing for the return of the gag reflex

The nurse has taught the client about an upcoming endoscopic retrograde
cholangiopancreatography procedure. The nurse determines that the client needs
furtherinformation if the client makes which statement?

1. "I know I must sign the consent form."


2. "I hope the throat spray keeps me from gagging."
3. "I'm glad I don't have to lie still for this procedure."
4. "I'm glad some IV medication will be given to relax me."

The health care provider has determined that a client with hepatitis has contracted the infection
from contaminated food. The nurse understands that this client is most likely experiencing what
type of hepatitis?

1. Hepatitis A
2. Hepatitis B
3. Hepatitis C
4. Hepatitis D

The nurse is caring for a client with a diagnosis of chronic gastritis. The nurse monitors the
client, knowing that this client is at risk for which vitamin deficiency?

1. Vitamin A
2. Vitamin B12
3. Vitamin C
4. Vitamin E

The nurse is assessing a client 24 hours following a cholecystectomy. The nurse notes that the
T-tube has drained 750 mL of green-brown drainage since the surgery. Which nursing
intervention is most appropriate?

1. Clamp the T-tube.


2. Irrigate the T-tube.
3. Document the findings.
4. Notify the health care provider.

The nurse is monitoring a client with a diagnosis of peptic ulcer. Which assessment finding
would most likely indicate perforation of the ulcer?

1. Bradycardia
2. Numbness in the legs
3. Nausea and vomiting
4. A rigid, boardlike abdomen

Tachycardia may occur as hypovolemic shock develops. Numbness in the legs is not an
associated finding. The nurse is caring for a client following a Billroth II procedure. Which
postoperative
prescription should the nurse question and verify?

1. Leg exercises
2. Early ambulation
3. Irrigating the nasogastric tube
4. Coughing and deep-breathing exercises

The nurse is providing discharge instructions to a client following gastrectomy and should
instruct the client to take which measure to assist in preventing dumping syndrome?
1. Ambulate following a meal.
2. Eat high-carbohydrate foods.
3. Limit the fluids taken with meals.
4. Sit in a high Fowler's position during meals.

The nurse is reviewing the prescription for a client admitted to the hospital with a diagnosis of
acute pancreatitis. Which interventions would the nurse expect to be prescribed for the client?
Select all that apply.

1. Administer antacids as prescribed.


2. Encourage coughing and deep breathing.
3. Administer anticholinergics as prescribed.
4. Give small, frequent high-calorie feedings.
5. Maintain the client in a supine and flat position.
6. Give meperidine (Demerol) as prescribed for pain.

Test-Taking Strategy: Focus on the subject, care for the client with
The nurse is reviewing the record of a client with Crohn's disease. Which stool characteristic
should the nurse expect to note documented in the client's record?

1. Diarrhea
2. Chronic constipation
3. Constipation alternating with diarrhea
4. Stool constantly oozing from the rectum

.
The nurse is reviewing the record of a client with a diagnosis of cirrhosis and notes that there is
documentation of the presence of asterixis. How should the nurse assess for its presence?

1. Dorsiflex the client's foot.


2. Measure the abdominal girth.
3. Ask the client to extend the arms.
4. Instruct the client to lean forward.

The nurse is reviewing the laboratory results for a client with cirrhosis and notes that the
ammonia level is elevated. Which diet does the nurse anticipate to be prescribed for this
client?

1. Low-protein diet
2. High-protein diet
3. Moderate-fat diet
4. High-carbohydrate diet
The nurse is doing an admission assessment on a client with a history of duodenal ulcer. To
determine whether the problem is currently active, the nurse should assess the client for which
symptom(s) of duodenal ulcer?

1. Weight loss
2. Nausea and vomiting
3. Pain relieved by food intake
4. Pain radiating down the right arm

.
A client with hiatal hernia chronically experiences heartburn following meals. The nurse
should plan to teach the client to avoid which action because it is contraindicated with a hiatal
hernia?

1. Lying recumbent following meals


2. Consuming small, frequent, bland meals
3. Raising the head of the bed on 6-inch blocks
4. Taking H2-receptor antagonist medication

The nurse is assessing for stoma prolapse in a client with a colostomy. What should the nurse
observe if stoma prolapse occurs?

1. Protruding stoma
2. Sunken and hidden stoma
3. Narrowed and flattened stoma
4. Dark- and bluish-colored stoma

A client had a new colostomy created 2 days earlier and is beginning to pass malodorous
flatus from the stoma. What is the correct interpretation by the nurse?

1. This is a normal, expected event.


2. The client is experiencing early signs of ischemic bowel.
3. The client should not have the nasogastric tube removed.
4. This indicates inadequate preoperative bowel preparation.

A client has just had surgery to create an ileostomy. The nurse assesses the client in the
immediate postoperative period for which most frequent complication of this type of surgery?

1. Folate deficiency
2. Malabsorption of fat
3. Intestinal obstruction
4. Fluid and electrolyte imbalance
The nurse is doing preoperative teaching with a client who is about to undergo creation of a
Kock pouch. The nurse interprets that the client has the best understanding of the nature of the
surgery if the client makes which statement?
1. "I will be able to pass stool by the rectum eventually."
2. "The drainage from this type of ostomy will be formed."
3. "I will need to drain the pouch regularly with a catheter."
4. "I will need to wear a drainage bag for the rest of my life."

The nurse is monitoring a client for the early signs and symptoms of dumping syndrome. Which
findings indicate this occurrence?

1. Sweating and pallor


2. Bradycardia and indigestion
3. Double vision and chest pain
4. Abdominal cramping and pain

The health care provider schedules a patient for a colonoscopy. In preparing him for this
procedure, you explain that:

a. he will be sedated to minimize discomfort


b. the procedure takes only about six minutes
c. he may have a liquid breakfast before the procedure
d. he can go back to work as soon as the procedure is over

The nurse is caring for several patients with gastrointestinal problems. Which patient is most
likely to need a guaiac (hemooccult) test?

a. patient reports dark amber-colored urine


b. patient reports black discoloration of stool
c. Patient vomits small amounts of yellow emesis
d. patient complains of right upper quadrant pain

Which healthy lifestyle choice can decrease the risk of pancreatic cancer?

a. Eat a well-balanced diet


b. Obtain 30 minutes of exercise at least three times a week
c. Refrain from drinking any alcohol
d. refrain from smoking cigarettes
When patients undergo diagnostic tests of the gastrointestinal system, elderly patients in
particular must be closely watched for:

a. dehydration and electrolyte imbalance


b. nausea and vomiting
c. diarrhea
d. constipation from contrast media
Hyperactive bowel sounds in one quadrant and absent bowel sounds in other quadrants plus
nausea and vomiting may indicate:

a. pancreatitis
b. cholecystitis
c. gastrointestinal bleeding
d. intestinal obstruction

Your patient is to have a liver biopsy. What teaching points should be included in the
instructions (SATA)

a. Nothing is allowed by mouth for 4-8 hours prior to the procedure


b. local or general anesthesia will be used
c. patient will be positioned on the right side for the procedure
d. The procedure takes about an hour
e. It will be necessary to lie on the right side for 1-2 hours postprocedure
d. Heavy lifting or strenuous activities are to be avoided for 1-2 weeks

The nurse hears in report that a patient is suspected of having ascites. WHich action is the
nurse most likely to initiate for this specific condition?

a. Elevate the head of the bed 30-45 degrees


b. Assess for pain every 30-60 minutes
c. Perform serial measurements of abdominal girth
d. Slightly elevate legs and buttocks to help expel flatus

The patient is supposed to be NPO for 12 hours prior to diagnostic testing. The nursing
assistant reports that the patient just drank a soda and ate a sandwich. What should the nurse
do first?

a. Explain the purpose of NPO to the patient


b. Make an incident report that includes all the relevant facts
c. Cancel the test and reschedule it for the next day
d. Notify the charge nurse and diagnostic technician

A nurse hears in report that a patient has stomatitis. Which intervention is the nurse most likely
to initiate for this patient?

a. check for tube gastric residual before feeding


b. assess for a typical 24 hour eating pattern
c. offer frequent mouth care and artificial saliva
d. perform a physical assessment with a skinfold measurement

The nurse is caring for a patient with a gastrostomy tube. Which nursing intervention is
appropriate?

a. flush the tube after each feeding


b. instruct the patient to take some practice swallows
c. give only thickened liquids
d. handle the tube using sterile technique

Prior to administering an enteral feeding through a feeding tube, the nurse checks for residual.
What is the best rationale for performing this nursing action?

a. checking for residual is likely to be in the procedural manual


b. the appearance of residual should be documented
c. excessive volume can cause regurgitation and aspiration
d. The nutritionist bases formula recommendations on residual content

Following a stroke, a patient has dysphagia and a nursing diagnosis of altered swallowing
ability. What would be the best outcome statement?

a. patient will receive pureed foods and thickened liquids


b. patient will increase swallowing muscle strength
c. patient will control breathing while swallowing
d. patient will not aspirate when swallowing

A patient is second day postoperative for GI surgery and has a nasogastric tube attached to
suction. There is little drainage in the suction container after four hours. What should the nurse
do first?

a. Check to see that the NG tube is still in the correct place by checking the mark near
the nose
b. try to aspirate some stomach contents
c. irrigate the tube with the ordered amount of solution
d. Instill 2 mL of air into the pigtail of the NG tube with a dry syringe

A patient is suspected of having esophageal cancer. Which diagnostic test is the health care
provider most likely to order to make the definitive diagnosis?

a. Esophageal manometry with 24 hour pH monitoring


b. Gastric analysis and serum tests for H. phylori
c. Upper GI series and check BMI
d. Esophagogastroduodenoscopy with biopsy

Proton pump inhibitors such as rabeprazole (AcipHex) work by:

a. Increasing HCl secretion in the stomach


b. neutralizing stomach acid
c. both neutralizing and suppressing stomach acid
d. Suppressing the secretion of stomach acid

A nursing implication for esomeprazole (Nexium) is to:

a. administer the drug only with food


b. not to administer the drug with digoxin (lanoxin)
c. administer the drug along with an antacid
d. Do not administer the drug along with orange or grapefruit juice

When caring for a patient with nasogastric suction, you would test the drainage for blood if you
found:

a. copious greenish drainage in the container


b. material resembling coffee grounds in the tubing
c. complaints of continued nausea by the patient
d. a rising BP and pulse rate.

-The nurse is to obtain a stool specimen from a client who reported that he is taking iron
supplements. The nurse would expect the stool to be which color?

a) Black
b) Green
c) Dark brown
d) Red

-The nurse is providing care to a client who has had a percutaneous liver biopsy. The nurse
would monitor the client for which of the following?

a) Return of the gag reflex


b) Passage of stool
c) Intake and output
d) Signs and symptoms of bleeding

-A client has recently obtained a set of dentures. Which of the following will the nurse do during
a complete physical examination?

a) Have the client rinse with warm salt water before assessing the oral cavity.
b) Remove the plates to visualize the oral cavity.
c) Remove the plates and rinse under hot water to remove bacteria.
d) Brush the dentures before examining the oral cavity.
Remove the plates to visualize the oral cavity.

-Which of the following is the major carbohydrate that tissue cells use as fuel?

a) Glucose
b) Chyme
c) Proteins
d) Fats

.
-When assisting with preparing a client scheduled for a barium swallow, which of the following
would be appropriate to include?

a) Avoid the intake of red meat before the procedure.


b) Avoid smoking for at least 12 to 24 hours before the procedure.
c) Take vitamin K before the procedure.
d) Take three cleansing enemas before the procedure.
Avoid smoking for at least 12 to 24 hours before the procedure.

-Which of the following should the nurse complete prior to assessing the abdomen of a 35-year-
old man?

a) Ask the client to empty his bladder.


b) Dim the lights for privacy.
c) Prepare for a prostate examination.
d) Assist the client to a Fowler's position.

-A nurse is teaching a client with malabsorption syndrome about the disorder and its treatment.
The client asks which part of the GI tract absorbs food. The nurse tells the client that products of
digestion are absorbed mainly in the:

a) large intestine.
b) small intestine.
c) rectum.
d) stomach.

-A client is diagnosed with megaloblastic anemia caused by vitamin B12 deficiency. The
physician begins the client on cyanocobalamin (Betalin-12), 100 mcg I.M. daily. Which
substance influences vitamin B12 absorption?

a) Histamine
b) Hydrochloric acid
c) Intrinsic factor
d) Liver enzyme
-From the following profiles of clients, which client would be most likely to undergo the
diagnostic test of cholecystography?

a) Mark, suspected of having stones in the gallbladder


b) Steven, suspected of having a tumor in the colon
c) Sandra, suspected of having lesions in the liver
d) Andrew, suspected of having esophageal abnormalities

-When examining the abdomen of a client with complaints of nausea and vomiting, which of the
following would the nurse do first?

a) Auscultation
b) Percussion
c) Palpation
d) Inspection

-The nurse is answering questions regarding fecal matter for a client who is scheduled for a
colon resection. The client is asking questions regarding the composition of the fecal matter and
when it becomes a formed mass. The nurse is most correct to state at which location?

a) Duodenum
b) Ileum
c) Cecum
d) Sigmoid colon

-The cecum is located at the beginning of the large intestine.


Upon review of a client's chart, the nurse notes the client has been receiving antiemetics every
6 to 8 hours. What in this client's history may necessitate such frequency?

a) Adrenal gland removal 3 days ago


b) Multiple leg fractures
c) Pituitary tumor
d) Treatment for cancer

-A nurse is admitting a severely malnourished and nonverbal client to the hospital. Identify
which of the following factors contribute to the nutritional function of the digestive system. Select
all that apply.

a) Increased secretion of gastrin


b) Peristaltic contractions in the stomach
c) Absence of intrinsic factor in the gastric mucosa
d) Intact dentition of the upper and lower teeth
e) Increased saliva production
-A client receives a local anesthetic to suppress the gag reflex for a diagnostic procedure of the
upper GI tract. Which of the following nursing interventions is advised for this patient?

a) The client should not be given any food and fluids until the gag reflex returns.
b) The client should be monitored for cramping or abdominal distention.
c) The client should be monitored for any breathing-related disorder or discomforts.
d) The client's fluid output should be measured for at least 24 hours after the procedure.
The client should not be given any food and fluids until the gag reflex returns.

-While palpating a client's right upper quadrant (RUQ), the nurse would expect to find which
structure?

a) Spleen
b) Appendix
c) Sigmoid colon
d) Liver

-A client is to have an upper GI procedure with barium ingestion and abdominal


ultrasonography. While scheduling these diagnostic tests, the nurse must consider which
factor?

a) The client may eat a light meal before either test.


b) Both tests need to be done before breakfast.
c) The upper GI should be scheduled before the ultrasonography.
d) The ultrasonography should be scheduled before the GI procedure.

-After teaching a group of students about the various organs of the upper gastrointestinal tract
and possible disorders, the instructor determines that the teaching was successful when the
students identify which of the following structures as possibly being affected?

a) Liver
b) Stomach
c) Large intestine
d) Ileum

-A client with a recent history of rectal bleeding is being prepared for a colonoscopy. Initially,
how should the nurse position the client for this test?

a) Bent over with hands touching the floor


b) Lying on the left side with knees bent
c) Lying on the right side with legs straight
d) Prone with the torso elevated

-The nurse is teaching a patient scheduled for a colonoscopy. Which of the following should be
included as part of the preparation for the procedure?

a) Consume at least 3 quarts of water 30 minutes before the test.


b) Do not void for at least 30 minutes before the test.
c) Spray or gargle with a local anesthetic.
d) Follow the dietary and fluid restrictions and bowel preparation procedures.
-The patient is not advised to consume 3 quarts of water nor to void before the test. These
interventions may be essential for tests that involve ultrasonographic procedures.
Which of the following is an age-related change of the GI system?

a) Increased mucus secretion


b) Increased motility
c) Weakened gag reflex
d) Hypertrophy of the small intestine

-While completing an abdominal assessment, the nurse will use which landmark as the upper
boundary for auscultating bowel sounds?

a) Umbilicus
b) Xiphoid process
c) Symphysis pubis
d) T12 to L3 vertebrae

-The nurse has been directed to position a patient for an examination of the abdomen. What
position should the nurse place the patient in for the examination?

a) Reverse Trendelenburg position to facilitate the natural propulsion of intestinal contents


b) Supine position with the knees flexed to relax the abdominal muscles
c) Prone position with pillows positioned to alleviate pressure on the abdomen
d) Semi-Fowler's position with the left leg bent to minimize pressure on the abdomen

.
-The nurse is instructing the client who was newly diagnosed with peptic ulcers. Which of the
following diagnostic studies would the nurse anticipate reviewing with the client?

a) Serum antibodies for H. pylori


b) A complete blood count including differential
c) Gastric analysis
d) A sigmoidoscopy

-Which of the following is considered the gold standard for the diagnosis of liver disease?

a) Cholecystography
b) Paracentesis
c) Ultrasonography
d) Biopsy
-A client undergoing a diagnostic examination for gastrointestinal disorder was given
polyethylene glycol/electrolyte solution as a part of the test preparation. Which of the following
measures should the nurse take once the solution is administered?

a) Instruct the client to have low-residue meals.


b) Permit the client to drink only clear liquids.
c) Provide saline gargles to the client.
d) Allow the client to ingest fat-free meal.

-Instructing the client to have low-residue meals is a pretest procedure for barium enema. A
client is offered saline gargles after esophagogastroduodenoscopy. Which of the following is an
age-related change of the gastrointestinal system?

a) Weakened gag reflex


b) Increased mucus secretion
c) Hypertrophy of the small intestine
d) Increased motility

-The nurse is performing an abdominal assessment. The nurse should perform the assessment
in which of the following orders?

a) Inspection, palpation, percussion, auscultation


b) Auscultation, inspection, percussion, palpitation
c) Inspection, auscultation, percussion, palpation
d) Auscultation, percussion, inspection, palpation

-A nurse is providing postprocedure instructions for a client who had an


esophagogastroduodenoscopy. The nurse should perform which action?

a) Tell the client to call back in the morning so she can give him instructions over the phone.
b) Review the instructions with the person accompanying the client home.
c) Give instructions to the client immediately before discharge.
d) Tell the client there aren't specific instructions for after the procedure.

-When preparing a client for magnetic resonance imaging (MRI) of the abdomen, which
statement would indicate the need to notify the physician?

a) "I brought earphones to shut out the loud noise."


b) "I left all my jewelry and my watch at home."
c) "I really don't like to be in small, enclosed spaces."
d) "I haven't had anything to eat or drink since midnight last night."

-
A client is scheduled for magnetic resonance imaging (MRI). During the client teaching, the
nurse will discuss which of the following?

a) "You must be NPO for the day before the examination."


b) "The examination will take only 15 minutes."
c) "You must remove all jewelry but can wear your wedding ring."
d) "Do you experience any claustrophobia?"
-The nurse is teaching the client about the upcoming endoscopic retrograde
cholangiopancreatography (ERCP). Although the nurse instructs on several pertinent points of
care, which is emphasized?

a) The client will change positions frequently throughout the procedure.


b) The client will have moderate sedation.
c) The client will fast prior to the procedure.
d) The client will receive antibiotics before and after the procedure.

.
-Which of the following terms describes a gastric secretion that combines with vitamin B12 so
that it can be absorbed?

a) Amylase
b) Trypsin
c) Pepsin
d) Intrinsic factor

-Cystic fibrosis, a genetic disorder characterized by pulmonary and pancreatic dysfunction,


usually appears in young children but can also affect adults. If the pancreas was functioning
correctly, where would the bile and pancreatic enzymes enter the GI system?

a) Jejunum
b) Duodenum
c) Cecum
d) Ileum

-Which of the following is true statement regarding older patients, considering the age-related
effects on their GI system?

a) They have no awareness of the filling reflex.


b) They tend to have increased muscle tone and mass.
c) They tend to have higher physiologic reserves to compensate for fluid loss.
d) They tend usually to have less control of the rectal sphincter.

-Which of the following would be most important to ensure that a client does not retain any
barium after a barium swallow?

a) Observing the color of urine.


b) Placing any stool passed in a specific preservative.
c) Monitoring the stool passage and its color.
d) Monitoring the volume of urine.

-A client frequently reports constipation. The nurse asks the client about his bowel habits. Which
of the following would be the most likely contributing factor related to constipation?

a) Resisting the urge to defecate several times a day


b) A vegan, organic lifestyle
c) A fiber-rich diet
d) Having a formed bowel movement only every other day
-Over time the stool becomes firm and the client may experience constipation.
Gastrin has which of the following effects on gastrointestinal (GI) motility?

a) Increased motility of the stomach


b) Contraction of the ileocecal sphincter
c) Relaxation of the colon
d) Relaxation of gastroesophageal sphincter

-Which of the following is an age-related change in the esophagus?

a) Increased motility
b) Weakened gag reflex
c) Increased muscle tone
d) Increased emptying

-One or two bowel sounds in 2 minutes would be documented as which of the following?

a) Hyperactive
b) Hypoactive
c) Normal
d) Absent

-A home care nurse is caring for a client with complaints of epigastric discomfort who is
scheduled for a barium swallow. Which statement by the client indicates an understanding of
the test?

a) "I'll take a laxative to clear my bowels before the test."


b) "I'll drink full liquids the day before the test."
c) "There is no need for special preparation before the test."
d) "I'll avoid eating or drinking anything 6 to 8 hours before the test."
"
- Using gastric analysis, the nurse would expect that a patient diagnosed with peptic ulcer would
secrete which of the following?

a) Small amount of acid


b) Little or no acid
c) No acid under basal condition or after stimulation
d) An excess amount of acid

-The nurse is caring for a client recovering from an esophagogastroduodenoscopy (EGD).


Which of the following client symptoms would require further nursing assessment?

a) Thirst
b) Abdominal distention
c) Drowsiness
d) Sore throat
-The nurse is a preparing a patient for a barium enema. The nurse should place the patient on
which of the following prior to the procedure?

a) Soft diet 1 day prior


b) Clear liquids day before
c) High-fiber diet 1 to 2 days prior
d) Nothing by mouth (NPO) 2 days prior

-The nurse is assisting the physician with a gastric acid stimulation test for a patient. What
medication should the nurse prepare to administer subcutaneously to stimulate gastric
secretions?

a) Mucomyst
b) Pentagastrin
c) Atropine
d) Robinul (glycopyrrolate)

-What part of the GI tract begins the digestion of food?

a) Mouth
b) Stomach
c) Esophagus
d) Duodenum

-A client is scheduled for several diagnostic tests to evaluate her gastrointestinal function. After
teaching the client about these tests, the nurse determines that the client has understood the
teaching when she identifies which test as not requiring the use of a contrast medium?

a) Upper GI series
b) Colonoscopy
c) Computer tomography
d) Small bowel series

-Which of the following is an enzyme secreted by the gastric mucosa?

a) Pepsin
b) Bile
c) Ptyalin
d) Trypsin

-During assessment of a patient complaining of dyspepsia, the nurse is aware that abdominal
pain associated with indigestion is usually which of the following?

a) Less severe after an intake of fatty foods


b) In the left lower quadrant
c) Described as cramping or burning
d) Relieved by the intake of coarse vegetables, which stimulate peristalsis
-A nurse is preparing a client with Crohn's disease for a barium enema. What should the nurse
do the day before the test?

a) Serve the client his usual diet.


b) Encourage plenty of fluids.
c) Order a high-fiber diet.
d) Serve dairy products.

-Because dairy products leave a residue, they aren't allowed the evening before the test. Clear
liquids only are allowed the evening before the test.
Following ingestion of carrots or beets, the nurse would expect which alteration in stool color?

a) Black
b) Milky white
c) Red
d) Yellow

-A nurse assesses the abdomen of a newly admitted client. Which finding would necessitate
further investigation?

a) Flat appearance below the umbilicus


b) Striae of lateral abdomen
c) Rounded contour
d) Asymmetrical upper quadrants

-A patient asks the nurse why the physician ordered the blood test carcinoembryonic antigen
(CEA). The nurse answers:

a) "It indicates if a cancer is present."


b) "It tells the physician what type of cancer is present."
c) "It determines functionality of the liver."
d) "It detects a protein normally found in the blood."

-A nurse is assessing a client who underwent esophagogastroduodenoscopy (EGD) for


postoperative complications. Which sign or symptom is a complication of this procedure?

a) Sore throat
b) Drooling
c) Bloody secretions
d) Absent gag reflex

-A client with extreme weakness, pallor, weak peripheral pulses, and disorientation is admitted
to the emergency department. His wife reports that he has been "spitting up blood." A Mallory-
Weiss tear is suspected, and the nurse begins taking a client history from the client's wife. The
question by the nurse that demonstrates her understanding of Mallory-Weiss tearing is:

a) "Has your partner recently fallen or injured his chest?"


b) "What spices and condiments does your spouse use on food?"
c) "Is your partner being treated for tuberculosis?"
d) "Has your partner had recent forceful vomiting?"
-The nurse is assessing a client who is stating gastrointestinal upset and a feeling of bloating.
Which type of meal would the nurse anticipate causing these types of symptoms?

a) Hamburger and French fries


b) Salmon with cheddar mashed potatoes
c) Steamed rice with pork and broccoli
d) Grilled chicken on a spinach salad

-A patient is being prepared for esophageal manometry. The nurse should inform the patient to
withhold what medication for 48 hours prior to the procedure?

a) Metoprolol (Lopressor)
b) Amiodarone (Cordarone)
c) Aspirin
d) Calan (Verapamil)

-The nurse is scheduling gastrointestinal (GI) diagnostic testing for a client. Which GI test
should be scheduled first?

a) Barium swallow
b) Barium enema
c) Small bowel series
d) Radiography of the gallbladder

-The nurse is investigating a patient's complaint of pain in the duodenal area. Where should the
nurse perform the assessment?

a) Left lower quadrant


b) Periumbilical area, followed by the right lower quadrant
c) Hypogastrium in the right or left lower quadrant
d) Epigastric area and consider possible radiation of pain to the right subscapular region

-During a colonoscopy with moderate sedation, the patient groans with obvious discomfort and
begins bleeding from the rectum. The patient is diaphoretic and has an increase in abdominal
girth from distention. What complication of this procedure is the nurse aware may be occurring?

a) Infection
b) Rectal fissure
c) Bowel perforation
d) Colonic polyp

-A nursing instructor tells the class that review of oral hygiene is an important component during
assessment of the gastrointestinal system. One of the students questions this statement. Which
of the following explanations from the nurse educator is most appropriate?

a) "Injury to oral mucosa or tooth decay can lead to difficulty in chewing food."
b) "Bad breath will encourage ingestion of fatty foods to mask odor."
c) "Mouth sores are caused by bacteria that can thin the villi of the small intestine."
d) "Decaying teeth secrete toxins that decrease the absorption of nutrients."
-The nurse is assessing a 50-year-old, dark-skinned African American man and has noted that
he appears jaundice. Most likely, the nurse made this observation by assessing which part of
his body?

a) Nail beds
b) Skin
c) Top of the hands and feet
d) Mucous membranes

-A patient tells the nurse that his stool was colored yellow. The nurse assesses for which of the
following?

a) Recent foods ingested


b) Pilonidal cyst
c) Occult blood
d) Ingestion of bismuth

-Which of the following is an enzyme secreted by the gastric mucosa?

a) Pepsin
b) Trypsin
c) Bile
d) Ptyalin

-When assisting with preparing a client scheduled for a barium swallow, which of the following
would be appropriate to include?

a) Take three cleansing enemas before the procedure.


b) Take vitamin K before the procedure.
c) Avoid the intake of red meat before the procedure.
d) Avoid smoking for at least a day before the procedure.

-Which of the following is the primary function of the small intestine?

a) Secretion
b) Peristalsis
c) Absorption
d) Digestion
-A 24-year-old athlete is admitted to the trauma unit following a motor-vehicle collision. The
client is comatose and has developed ascites as a result of the accident. You are explaining the
client's condition to his parents. In your education, what do you indicate is the primary function
of the small intestine?

a) Digest fats
b) Digest proteins
c) Absorb water
d) Absorb nutrients

-A home care nurse is caring for a client with complaints of epigastric discomfort who is
scheduled for a barium swallow. Which statement by the client indicates an understanding of
the test?

a) "I'll avoid eating or drinking anything 6 to 8 hours before the test."


b) "I'll drink full liquids the day before the test."
c) "I'll take a laxative to clear my bowels before the test."
d) "There is no need for special preparation before the test."

-A nurse is providing postprocedure instructions for a client who had an


esophagogastroduodenoscopy. The nurse should perform which action?

a) Tell the client there aren't specific instructions for after the procedure.
b) Tell the client to call back in the morning so she can give him instructions over the phone.
c) Give instructions to the client immediately before discharge.
d) Review the instructions with the person accompanying the client home.

-The nurse is assessing a client following laparoscopy. The client states that his stomach looks
bloated and asks if this is normal. How will the nurse respond?

a) "I am not sure about this. Let me get another nurse."


b) "Yes, your abdomen may appear larger as a result of the injection of carbon dioxide
for visualization."
c) "Do you need to use the restroom? You may have to have a bowel movement."
d) "No, this should not occur. I will call the physician right away."

.
-A client is to have an upper GI procedure with barium ingestion and abdominal
ultrasonography. While scheduling these diagnostic tests, the nurse must consider which
factor?

a) The ultrasonography should be scheduled before the GI procedure.


b) The upper GI should be scheduled before the ultrasonography.
c) The client may eat a light meal before either test.
d) Both tests need to be done before breakfast.

-Which of the following would be most important to ensure that a client does not retain any
barium after a barium swallow?

a) Observing the color of urine.


b) Placing any stool passed in a specific preservative.
c) Monitoring the stool passage and its color.
d) Monitoring the volume of urine.

-A nursing instructor tells the class that review of oral hygiene is an important component during
assessment of the gastrointestinal system. One of the students questions this statement. Which
of the following explanations from the nurse educator is most appropriate?

a) "Bad breath will encourage ingestion of fatty foods to mask odor."


b) "Injury to oral mucosa or tooth decay can lead to difficulty in chewing food."
c) "Decaying teeth secrete toxins that decrease the absorption of nutrients."
d) "Mouth sores are caused by bacteria that can thin the villi of the small intestine."

-One or two bowel sounds in 2 minutes would be documented as which of the following?

a) Hypoactive
b) Hyperactive
c) Normal
d) Absent
.
-A nurse assesses the abdomen of a newly admitted client. Which finding would necessitate -
further investigation?

a) Flat appearance below the umbilicus


b) Asymmetrical upper quadrants
c) Striae of lateral abdomen
d) Rounded contour

-The hydrogen breath test was developed to evaluate which type of absorption?

a) Protein
b) Vitamin B12
c) Carbohydrate
d) Fat

.
-Gastrin has which of the following effects on gastrointestinal (GI) motility?

a) Relaxation of the colon


b) Contraction of the ileocecal sphincter
c) Increased motility of the stomach
d) Relaxation of gastroesophageal sphincter

-Which of the following should be included as part of the preprocedure teaching for a patient
scheduled for a proctosigmoidoscopy involving the lower GI structures?

a) Consume at least three quarts of water 30 minutes before the test


b) Follow the dietary and fluid restrictions and bowel preparation procedures
c) Spray or gargle with a local anesthetic
d) Do not void for at least 30 minutes before the test
-The patient is not advised to consume three quarts of water and is not advised to void before
the test. These interventions may be essential for tests that involve ultrasonographic
procedures. Which of the following sequence should be used to assess the abdomen?

a) Palpation, inspection, percussion, auscultation


b) Auscultation, inspection, percussion, palpation
c) Inspection, auscultation, percussion, palpation
d) Percussion, auscultation, palpation, inspection
Inspection, auscultation, percussion, palpation

-Upon review of a client's chart, the nurse notes the client has been receiving antiemetics every
6 to 8 hours. What in this client's history may necessitate such frequency?

a) Pituitary tumor
b) Adrenal gland removal 3 days ago
c) Treatment for cancer
d) Multiple leg fractures

-One or two bowel sounds in 2 minutes would be documented as which of the following?

a) Hyperactive
b) Absent
c) Normal
d) Hypoactive

-The nurse asks a client to point to where she feels pain. The client asks why this is important.
The nurse's best response would be which of the following?

a) "This determines the pain medication to be ordered."


b) "The area may determine the severity of the pain."
c) "If the doctor massages over the exact painful area, the pain will disappear."
d) "Often the area of pain is referred from another area."

-A client is diagnosed with megaloblastic anemia caused by vitamin B12 deficiency. The
physician begins the client on cyanocobalamin (Betalin-12), 100 mcg I.M. daily. Which
substance influences vitamin B12 absorption?

a) Intrinsic factor
b) Histamine
c) Liver enzyme
d) Hydrochloric acid

-Which of the following digestive enzymes aids in the digesting of starch?

a) Bile
b) Trypsin
c) Amylase
d) Lipase
-A few hours after eating hot and spicy chicken wings, a client presents with lower chest pain.
He wonders if he is having a heart attack. How should the nurse proceed first?

a) Further investigate the initial complaint.


b) Explain that fatty foods can mimic chest pain.
c) Administer an over-the-counter antacid tablet.
d) Call for an immediate electrocardiogram.

-A home care nurse is caring for a client with complaints of epigastric discomfort who is
scheduled for a barium swallow. Which statement by the client indicates an understanding of
the test?

a) "I'll take a laxative to clear my bowels before the test."


b) "I'll drink full liquids the day before the test."
c) "I'll avoid eating or drinking anything 6 to 8 hours before the test."
d) "There is no need for special preparation before the test."

-A client undergoing a diagnostic examination for gastrointestinal disorder was given


polyethylene glycol/electrolyte solution as a part of the test preparation. Which of the following
measures should the nurse take once the solution is administered?

a) Allow the client to ingest fat-free meal.


b) Instruct the client to have low-residue meals.
c) Provide saline gargles to the client.
d) Permit the client to drink only clear liquids.
Permit the client to drink only clear liquids.

-A client with extreme weakness, pallor, weak peripheral pulses, and disorientation is admitted
to the emergency department. His wife reports that he has been "spitting up blood." A Mallory-
Weiss tear is suspected, and the nurse begins taking a client history from the client's wife. The
question by the nurse that demonstrates her understanding of Mallory-Weiss tearing is:

a) "Is your partner being treated for tuberculosis?"


b) "Has your partner recently fallen or injured his chest?"
c) "Has your partner had recent forceful vomiting?"
d) "What spices and condiments does your spouse use on food?"

.
-A nurse is teaching a client with malabsorption syndrome about the disorder and its treatment.
The client asks which part of the GI tract absorbs food. The nurse tells the client that products of
digestion are absorbed mainly in the:

a) large intestine.
b) small intestine.
c) rectum.
d) stomach.

-Digestive enzymes contribute to the breakdown of foods for the body's use. Stomach
contractions mix the food and gastric secretions and, eventually, the chyme moves to the small
intestine. Which nutrient actually delays feeling hungry?
a) Fats
b) Minerals
c) Carbohydrates
d) Proteins

-Cystic fibrosis, a genetic disorder characterized by pulmonary and pancreatic dysfunction,


usually appears in young children but can also affect adults. If the pancreas was functioning
correctly, where would the bile and pancreatic enzymes enter the GI system?

a) Cecum
b) Jejunum
c) Duodenum
d) Ileum
Duodenum

-The nurse is to obtain a stool specimen from a client who reported that he is taking iron
supplements. The nurse would expect the stool to be which color?

a) Red
b) Black
c) Dark brown
d) Green

-The nurse is preparing to examine the abdomen of a client complaining of a change in his
bowel pattern. The nurse would place the client in which position?

a) Lithotomy
b) Knee-chest
c) Supine with knees flexed
d) Left Sim's lateral

-A client recently started a new medication to treat a suspected ulcer. She asks the nurse how
this medicine is helping. Which of the following would be the best response by the nurse
regarding anti-ulcerative medications?

a) "This medication helps to digest food more slowly, allowing acid to attach to the food
particles."
b) "This medication reduces the acid secretion in your stomach."
c) "The medication is allowing the acid to be eliminated more quickly in the stool."
d) "This medication acts to reduce the volume of acid and foods that can enter the small
intestine."

-A nurse assesses the abdomen of a newly admitted client. Which finding would necessitate
further investigation?

a) Flat appearance below the umbilicus


b) Rounded contour
c) Asymmetrical upper quadrants
d) Striae of lateral abdomen
-A client is scheduled for several diagnostic tests to evaluate her gastrointestinal function. After
teaching the client about these tests, the nurse determines that the client has understood the
teaching when she identifies which test as not requiring the use of a contrast medium?

a) Small bowel series


b) Colonoscopy
c) Upper GI series
d) Computer tomography

-Which of the following should the nurse advise a client due to undergo a diagnostic test for a
disorder of the GI system, to ensure that the client experiences none or minimal discomfort?

a) The client should not expel gas and test fluids from the bowel when he or she
experiences the urge during the procedure
b) The client should take a sedative before the procedure to avoid the possibility of experiencing
any discomfort
c) The client should lie down in a supine position for at least three hours before the test to
reduce any discomfort during the test
d) The client should inform the test personnel if he or she experiences pressure or cramping
during the instillation of test fluids

-A client has recently obtained a set of dentures. Which of the following will the nurse do during
a complete physical examination?

a) Remove the plates and rinse under hot water to remove bacteria.
b) Brush the dentures before examining the oral cavity.
c) Remove the plates to visualize the oral cavity.
d) Have the client rinse with warm salt water before assessing the oral cavity.

-A client is scheduled for bowel resection with anastomosis involving the large intestine.
Because of the surgical site, the nurse formulates the nursing diagnosis of Risk for infection. To
complete the nursing diagnosis statement, the nurse should add which "related-to" phrase?

a) Related to the presence of bacteria at the surgical site


b) Related to malnutrition secondary to bowel resection with anastomosis
c) Related to major surgery required by bowel resection
d) Related to the presence of a nasogastric (NG) tube postoperatively

-A client with diabetes begins to have digestive problems and is told by the physician that they
are a complication of the diabetes. Which of the following explanations from the nurse is most
accurate?

a) The pancreas secretes digestive enzymes.


b) Elevated glucose levels cause bacteria overgrowth in the large intestine.
c) The nerve fibers of the intestinal lining are experiencing neuropathy.
d) Insulin has an adverse effect of constipation.

-A client who had a colonoscopy with removal of a polyp is being prepared for discharge. Which
of the following would the nurse include in the discharge instructions?

a) "You might experience some nausea and vomiting for a day or so. This is normal."
b) "Call your physician if there is even slight bleeding with your first bowel movement."
c) "You might feel some cramping and gas but these usually go away in about a day."
d) "Be sure to eat high fiber foods when you get home to help you move your bowels."

-A client presents to the emergency department, reporting that he has been vomiting every 30 to
40 minutes for the past 8 hours. Frequent vomiting puts this client at risk for which imbalances?

a) Metabolic alkalosis and hyperkalemia


b) Metabolic acidosis and hypokalemia
c) Metabolic alkalosis and hypokalemia
d) Metabolic acidosis and hyperkalemia

-If a client has abdominal surgery and a portion of the small intestine is removed, the client is at
risk for which of the following?

a) Cirrhosis
b) Gastric ulcers
c) Constipation
d) Malabsorption syndrome

-When describing the role of the pancreas to a client with a pancreatic dysfunction, the nurse
would identify which substance as being acted on by pancreatic lipase?

a) Protein
b) Glucose
c) Triglycerides
d) Starch

-Which patient teaching component is important for the nurse to communicate regarding pain
management prior to or during diagnostic testing for a disorder of the GI system?

a) The patient should inform the test personnel if he or she experiences pressure or
cramping during the instillation of test fluids.
b) The patient should not expel gas and test fluids from the bowel when he or she experiences
the urge during the procedure.
c) The patient should take a sedative before the procedure to avoid the possibility of
experiencing any discomfort.
d) The patient should lie down in a supine position for at least 3 hours before the test to reduce
any discomfort during the test.

-An examiner is performing the physical assessment of the rectum, perianal region, and anus.
While this examination can be uncomfortable for many clients, health care providers must
approach it in a prepared, confident manner. Which of the following considerations will help this
examination flow smoothly and efficiently for both provider and client? Select all that apply.

a) Position the client on the right side with the knees up to the chest.
b) Dim the lights to decrease the client's embarrassment.
c) Cleanse gloved fingers with water to allow for easy insertion.
d) Ask the client to produce a bowel movement after the procedure.
e) Ask the client to bear down for visual inspection.

.
-The nurse assesses a client who is reporting mild abdominal cramping. How will the nurse
proceed with assessment of this client?

a) Auscultation, inspection, percussion, palpation


b) Inspection, auscultation, percussion, palpation
c) Palpation, inspection, percussion, auscultation
d) Inspection, percussion, auscultation, palpation

-The nurse is caring for a man who has experienced a spinal cord injury. Throughout his
recovery, the client expects to gain control of his bowels. The nurse's best response to this
client would be which of the following?

a) "Over time, the nerve fibers will regrow new tracts, and you can have bowel movements
again."
b) "It is not going to happen. Your nerve cells are too damaged."
c) "Wearing an undergarment will become more comfortable over time."
d) "Having a bowel movement is a spinal reflex requiring intact nerve fibers. Yours are
not intact."

-A high school senior is diagnosed with anorexia nervosa and is hospitalized for severe
malnutrition. The treatment team is planning to use behavior modification. What rationale should
a nurse identify as the reasoning behind this therapy choice?

A. This therapy will increase the client's motivation to gain weight.


B. This therapy will reward the client for perfectionist achievements.
C. This therapy will provide the client with control over behavioral choices.
D. This therapy will protect the client from parental overindulgence.

-Which information related to a client's home environment should a nurse associate with the
development of this disorder?

A. The home environment maintains loose personal boundaries.


B. The home environment places an overemphasis on food.
C. The home environment is overprotective and demands perfection.
D. The home environment condones corporal punishment.

-A client's altered body image is evidenced by claims of "feeling fat" even though the client is
emaciated. Which is the appropriate outcome criterion for this client's problem?

A. The client will consume adequate calories to sustain normal weight.


B. The client will cease strenuous exercise programs.
C. The client will perceive an ideal body weight and shape as normal.
D. The client will not express a preoccupation with food.
-A nurse observes dental deterioration when assessing a client diagnosed with bulimia nervosa.
What explains this assessment finding?

A. The emesis produced during purging is acidic and corrodes the tooth enamel.
B. Purging causes the depletion of dietary calcium.
C. Food is rapidly ingested without proper mastication.
D. Poor dental and oral hygiene leads to dental caries.

-Why are behavior modification programs the treatment of choice for clients diagnosed with
eating disorders?

A. These programs help clients correct distorted body image.


B. These programs address underlying client anger.
C. These programs help clients manage uncontrollable behaviors.
D. These programs allow clients to maintain control.

-A potential Olympic figure skater collapses during practice and is hospitalized for severe
malnutrition. Anorexia nervosa is diagnosed. Which client statement best reflects the underlying
etiology of this disorder?

A. "Skaters need to be thin to improve their daily performance."


B. "All the skaters on the team are following an approved 1,200-calorie diet."
C. "When I lose skating competitions, I also lose my appetite."
D. "I am angry at my mother. I can only get her approval when I win competitions."

-The family of a client diagnosed with anorexia nervosa becomes defensive when the treatment
team calls for a family meeting. Which is the appropriate nursing reply?

A. "Tell me why this family meeting is causing you to be defensive. All clients are required to
participate in two family sessions."
B. "Eating disorders have been correlated to certain familial patterns; without addressing
these, your child's condition will not improve."
C. "Family dynamics are not linked to eating disorders. The meeting is to provide your child with
family support."
D. "Clients diagnosed with anorexia nervosa are part of the family system, and any alteration in
family processes needs to be addressed."

-A client diagnosed with bulimia nervosa has been attending a mental health clinic for several
months. Which factor should a nurse identify as an appropriate indicator of a positive client
behavioral change?

A. The client gains 2 pounds in 1 week.


B. The client focuses conversations on nutritious food.
C. The client demonstrates healthy coping mechanisms that decrease anxiety.
D. The client verbalizes an understanding of the etiology of the disorder.

-A morbidly obese client is prescribed an anorexiant medication. The nurse should prepare to
teach the client about which medication?

A. Diazepam (Valium)
B. Dexfenfluramine (Redux)
C. Sibutramine (Meridia)
D. Pemoline (Cylert)

-A nursing instructor is teaching students about the differences between the symptoms of
anorexia nervosa and the symptoms of bulimia nervosa. Which student statement indicates that
learning has occurred?

A. "Clients diagnosed with anorexia nervosa experience extreme nutritional deficits,


whereas clients diagnosed with bulimia nervosa do not."
B. "Clients diagnosed with bulimia nervosa experience amenorrhea, whereas clients diagnosed
with anorexia nervosa do not."
C. "Clients diagnosed with bulimia nervosa experience hypotension, edema, and lanugo,
whereas clients diagnosed with anorexia nervosa do not."
D. "Clients diagnosed with anorexia nervosa have eroded tooth enamel, whereas clients
diagnosed with bulimia nervosa do not."

-A client diagnosed with bulimia nervosa is to receive fluoxetine (Prozac) by oral solution. The
medication is supplied in a 100 mL bottle. The label reads 20 mg/5 mL. The doctor orders 60
mg q day. Which dose of this medication should the nurse dispense?

A. 25 mL
B. 20 mL
C. 15 mL
D. 10 mL

-A client who is 5 foot 6 inches tall and weighs 98 pounds is admitted with a medical diagnosis
of anorexia nervosa. Which nursing diagnosis would take priority at this time?

A. Ineffective coping R/T food obsession


B. Altered nutrition: less than body requirements R/T inadequate food intake
C. Risk for injury R/T suicidal tendencies
D. Altered body image R/T perceived obesity

-A nurse is seeing a client in an outpatient clinic for treatment of anorexia nervosa. Which is the
most appropriate, correctly written short-term outcome for this client?

A. The client will use stress-reducing techniques to avoid purging.


B. The client will discuss chaos in personal life and be able to verbalize a link to purging.
C. The client will gain 2 pounds prior to the next weekly appointment.
D. The client will remain free of signs and symptoms of malnutrition and dehydration.

-When a community health nurse arrives at the home of a client diagnosed with bulimia
nervosa, the nurse finds the client on the floor unconscious. The client has a history of using
laxatives for purging. To what would the nurse attribute this client's symptoms?

A. Increased creatinine and blood urea nitrogen (BUN) levels


B. Abnormal electroencephalogram (EEG)
C. Metabolic acidosis
D. Metabolic alkalosis
-A group of nurses are discussing how food is used in their families and the effects this might
have on their ability to work with clients diagnosed with eating disorders. Which of these nurses
will probably be most effective with these clients?

A. The nurse who understands the importance of three balanced meals a day
B. The nurse who permits children to have dessert only after finishing the food on their plate
C. The nurse who refuses to engage in power struggles related to food consumption
D. The nurse who grew up poor and frequently did not have enough food to eat

-A nurse working with a client diagnosed with bulimia nervosa asks the client to recall a time in
life when food could be consumed without purging. Which is the purpose of this nursing
intervention?

A. To gain additional information about the progression of the disease process


B. To emphasize that the client is capable of consuming food without purging
C. To incorporate specific foods into the meal plan to reflect pleasant memories
D. To assist the client to become more compliant with the treatment plan

-A client diagnosed with anorexia nervosa stopped eating 5 months ago and lost 25% of total
body weight. Which subjective client response would the nurse assess to support this medical
diagnosis?

A. "I do not use any laxatives or diuretics to lose weight."


B. "I am losing lots of hair. It's coming out in handfuls."
C. "I know that I am thin, but I refuse to be fat!"
D. "I don't know why people are worried. I need to lose this weight."

-A nursing diagnosis of ineffective coping R/T feelings of loneliness AEB bingeing then purging
when alone, is assigned to a client diagnosed with bulimia nervosa. Which is an appropriate
outcome related to this nursing diagnosis?

A. The client will identify two alternative methods of dealing with isolation by day 3.
B. The client will appropriately express angry feelings about lack of control by week 2.
C. The client will verbalize two positive self attributes by day 3.
D. The client will list five ways that the body reacts to bingeing and purging.

-A nurse responsible for conducting group therapy on an eating disorder unit schedules the
sessions immediately after meals. Which is the rationale for scheduling group therapy at this
time?

A. To shift the clients' focus from food to psychotherapy


B. To prevent the use of maladaptive defense mechanisms
C. To promote the processing of anxiety associated with eating
D. To focus on weight control mechanisms and food preparation

-Which nursing intervention is appropriate when caring for clients diagnosed with either anorexia
nervosa or bulimia nervosa?

A. Provide privacy during meals.


B. Remain with the client for at least 1 hour after the meal.
C. Encourage the client to keep a journal to document types of food consumed.
D. Restrict client privileges when provided food is not completely consumed.

-A nurse should identify topiramate (Topamax) as the drug of choice for which of the following
conditions? (Select all that apply.)

A. Binge eating with obesity


B. Bingeing and purging with a diagnosis of bulimia nervosa
C. Weight loss with a diagnosis of anorexia nervosa
D. Amenorrhea with a diagnosis of anorexia nervosa
E. Emaciation with a diagnosis of bulimia nervosa

1. Which of the following complications is thought to be the most common cause of


appendicitis?

1. A fecalith
2. Bowel kinking
3. Internal bowel occlusion
4. Abdominal bowel swelling

2. Which of the following terms best describes the pain associated with appendicitis?

1. Aching
2. Fleeting
3. Intermittent
4. Steady

3. Which of the following nursing interventions should be implemented to manage a client with
appendicitis?

1. Assessing for pain


2. Encouraging oral intake of clear fluids
3. Providing discharge teaching
4. Assessing for symptoms of peritonitis

The client with appendicitis will have pain that should be controlled with analgesia. The nurse
should discourage oral intake in preparation for surgery. Discharge teaching is important;
however, in the acute phase, management should focus on minimizing preoperative
complications and recognizing when such may be occurring.
Which of the following definitions best describes gastritis?

1. Erosion of the gastric mucosa


2. Inflammation of a diverticulum
3. Inflammation of the gastric mucosa
4. Reflux of stomach acid into the esophagus

5. Which of the following substances is most likely to cause gastritis?

1. Milk
2. Bicarbonate of soda, or baking soda
3. Enteric coated aspirin
4. Nonsteriodal anti-imflammatory drugs

6. Which of the following definitions best describes diverticulosis?

1. An inflamed outpouching of the intestine


2. A noninflamed outpouching of the intestine
3. The partial impairment of the forward flow of intestinal contents
4. An abnormal protrusion of an organ through the structure that usually holds it.

7. Which of the following types of diets is implicated in the development of diverticulosis?

1. Low-fiber diet
2. High-fiber diet
3. High-protein diet
4. Low-carbohydrate diet

8. Which of the following mechanisms can facilitate the development of diverticulosis into
diverticulitis?

1. Treating constipation with chronic laxative use, leading to dependence on laxatives


2. Chronic constipation causing an obstruction, reducing forward flow of intestinal contents
3. Herniation of the intestinal mucosa, rupturing the wall of the intestine
4. Undigested food blocking the diverticulum, predisposing the area to bacterial
invasion.

9. Which of the following symptoms indicated diverticulosis?

1. No symptoms exist
2. Change in bowel habits
3. Anorexia with low-grade fever
4. Episodic, dull, or steady midabdominal pain

10. Which of the following tests should be administered to a client suspected of having
diverticulosis?

1. Abdominal ultrasound
2. Barium enema
3. Barium swallow
4. Gastroscopy
11. Medical management of the client with diverticulitis should include which of the following
treatments?

1. Reduced fluid intake


2. Increased fiber in diet
3. Administration of antibiotics
4. Exercises to increase intra-abdominal pressure

12. Crohn's disease can be described as a chronic relapsing disease. Which of the following
areas in the GI system may be involved with this disease?

1. The entire length of the large colon


2. Only the sigmoid area
3. The entire large colon through the layers of mucosa and submucosa
4. The small intestine and colon; affecting the entire thickness of the bowel

13. Which area of the alimentary canal is the most common location for Crohn's disease?

1. Ascending colon
2. Descending colon
3. Sigmoid colon
4. Terminal ileum

.
14. Which of the following factors is believed to be linked to Crohn's disease?

1. Constipation
2. Diet
3. Hereditary
4. Lack of exercise

15. Which of the following factors is believed to cause ulcerative colitis?

1. Acidic diet
2. Altered immunity
3. Chronic constipation
4. Emotional stress

16. Fistulas are most common with which of the following bowel disorders?

1. Crohn's disease
2. Diverticulitis
3. Diverticulosis
4. Ulcerative colitis

17. Which of the following areas is the most common site of fistulas in client's with Crohn's
disease?

1. Anorectal
2. Ileum
3. Rectovaginal
4. Transverse colon

18. Which of the following associated disorders may a client with ulcerative colitis exhibit?

1. Gallstones
2. Hydronephrosis
3. Nephrolithiasis
4. Toxic megacolon

19. Which of the following associated disorders may the client with Crohn's disease exhibit?

1. Ankylosing spondylitis
2. Colon cancer
3. Malabsorption
4. Lactase deficiency

20. Which of the following symptoms may be exhibited by a client with Crohn's disease?

1. Bloody diarrhea
2. Narrow stools
3. N/V
4. Steatorrhea

21. Which of the following symptoms is associated with ulcerative colitis?

1. Dumping syndrome
2. Rectal bleeding
3. Soft stools
4. Fistulas

22. If a client had irritable bowel syndrome, which of the following diagnostic tests would
determine if the diagnosis is Crohn's disease or ulcerative colitis?

1. Abdominal computed tomography (CT) scan


2. Abdominal x-ray
3. Barium swallow
4. Colonoscopy with biopsy

23. Which of the following interventions should be included in the medical management of
Crohn's disease?

1. Increasing oral intake of fiber


2. Administering laxatives
3. Using long-term steroid therapy
4. Increasing physical activity

24. In a client with Crohn's disease, which of the following symptoms should not be a direct
result of antibiotic therapy?

1. Decrease in bleeding
2. Decrease in temperature
3. Decrease in body weight
4. Decrease in the number of stools

25. Surgical management of ulcerative colitis may be performed to treat which of the following
complications?

1. Gastritis
2. Bowel herniation
3. Bowel outpouching
4. Bowel perforation

.
26. Which of the following medications is most effective for treating the pain associated with
irritable bowel disease?

1. Acetaminophen
2. Opiates
3. Steroids
4. Stool softeners

27. During the first few days of recovery from ostomy surgery for ulcerative colitis, which of the
following aspects should be the first priority of client care?

1. Body image
2. Ostomy care
3. Sexual concerns
4. Skin care

28. Colon cancer is most closely associated with which of the following conditions?

1. Appendicitis
2. Hemorrhoids
3. Hiatal hernia
4. Ulcerative colitis

29. Which of the following diets is most commonly associated with colon cancer?

1. Low-fiber, high fat


2. Low-fat, high-fiber
3. Low-protein, high-carbohydrate
4. Low carbohydrate, high protein

30. Which of the following diagnostic tests should be performed annually over age 50 to screen
for colon cancer?

1. Abdominal CT scan
2. Abdominal x-ray
3. Colonoscopy
4. Fecal occult blood test
31. Radiation therapy is used to treat colon cancer before surgery for which of the following
reasons?

1. Reducing the size of the tumor


2. Eliminating the malignant cells
3. Curing the cancer
4. Helping the bowel heal after surgery

32. Which of the following symptoms is a client with colon cancer most likely to exhibit?

1. A change in appetite
2. A change in bowel habits
3. An increase in body weight
4. An increase in body temperature

33. A client has just had surgery for colon cancer. Which of the following disorders might the
client develop?

1. Peritonitis
2. Diverticulosis
3. Partial bowel obstruction
4. Complete bowel obstruction

34. A client with gastric cancer may exhibit which of the following symptoms?

1. Abdominal cramping
2. Constant hunger
3. Feeling of fullness
4. Weight gain

35. Which of the following diagnostic tests may be performed to determine if a client has gastric
cancer?

1. Barium enema
2. Colonoscopy
3. Gastroscopy
4. Serum chemistry levels

36. A client with gastric cancer can expect to have surgery for resection. Which of the following
should be the nursing management priority for the preoperative client with gastric cancer?

1. Discharge planning
2. Correction of nutritional deficits
3. Prevention of DVT
4. Instruction regarding radiation treatment

37. Care for the postoperative client after gastric resection should focus on which of the
following problems?

1. Body image
2. Nutritional needs
3. Skin care
4. Spiritual needs

38. Which of the following complications of gastric resection should the nurse teach the client to
watch for?

1. Constipation
2. Dumping syndrome
3. Gastric spasm
4. Intestinal spasms

39. A client with rectal cancer may exhibit which of the following symptoms?

1. Abdominal fullness
2. Gastric fullness
3. Rectal bleeding
4. Right upper quadrant pain

40. A client with which of the following conditions may be likely to develop rectal cancer?

1. Adenomatous polyps
2. Diverticulitis
3. Hemorrhoids
4. Peptic ulcer disease

41. Which of the following treatments is used for rectal cancer but not for colon cancer?

1. Chemotherapy
2. Colonoscopy
3. Radiation
4. Surgical resection

42. Which of the following conditions is most likely to directly cause peritonitis?

1. Cholelithiasis
2. Gastritis
3. Perforated ulcer
4. Incarcerated hernia

43. Which of the following symptoms would a client in the early stages of peritonitis exhibit?

1. Abdominal distention
2. Abdominal pain and rigidity
3. Hyperactive bowel sounds
4. Right upper quadrant pain

44. Which of the following laboratory results would be expected in a client with peritonitis?

1. Partial thromboplastin time above 100 seconds


2. Hemoglobin level below 10 mg/dL
3. Potassium level above 5.5 mEq/L
4. White blood cell count above 15,000

45. Which of the following therapies is not included in the medical management of a client with
peritonitis?

1. Broad-spectrum antibiotics
2. Electrolyte replacement
3. I.V. fluids
4. Regular diet

46. Which of the following aspects is the priority focus of nursing management for a client with
peritonitis?

1. Fluid and electrolyte balance


2. Gastric irrigation
3. Pain management
4. Psychosocial issues

47. A client with irritable bowel syndrome is being prepared for discharge. Which of the following
meal plans should the nurse give the client?

1. Low fiber, low-fat


2. High fiber, low-fat
3. Low fiber, high-fat
4. High-fiber, high-fat

48. A client presents to the emergency room, reporting that he has been vomiting every 30 to 40
minutes for the past 8 hours. Frequent vomiting puts him at risk for which of the following?

1. Metabolic acidosis with hyperkalemia


2. Metabolic acidosis with hypokalemia
3. Metabolic alkalosis with hyperkalemia
4. Metabolic alkalosis with hypokalemia

49. Five days after undergoing surgery, a client develops a small-bowel obstruction. A Miller-
Abbott tube is inserted for bowel decompression. Which nursing diagnosis takes priority?

1. Imbalanced nutrition: Less than body requirements


2. Acute pain
3. Deficient fluid volume
4. Excess fluid volume
3. Deficient fluid volume

50. When teaching an elderly client how to prevent constipation, which of the following
instructions should the nurse include?

1. "Drink 6 glasses of fluid each day."


2. "Avoid grain products and nuts."
3. "Add at least 4 grams of bran to your cereal each morning."
4. "Be sure to get regular exercise."
51. In a client with diarrhea, which outcome indicates that fluid resuscitation is successful?

1. The client passes formed stools at regular intervals


2. The client reports a decrease in stool frequency and liquidity
3. The client exhibits firm skin turgor
4. The client no longer experiences perianal burning.

52. When teaching a community group about measures to prevent colon cancer, which
instruction should the nurse include?

1. "Limit fat intake to 20% to 25% of your total daily calories."


2. "Include 15 to 20 grams of fiber into your daily diet."
3. "Get an annual rectal examination after age 35."
4. "Undergo sigmoidoscopy annually after age 50."

53. A 30-year old client experiences weight loss, abdominal distention, crampy abdominal pain,
and intermittent diarrhea after birth of her 2nd child. Diagnostic tests reveal gluten-induced
enteropathy. Which foods must she eliminate from her diet permanently?

1. Milk and dairy products


2. Protein-containing foods
3. Cereal grains (except rice and corn)
4. Carbohydrates

54. After a right hemicolectomy for treatment of colon cancer, a 57-year old client is reluctant to
turn while on bed rest. Which action by the nurse would be appropriate?

1. Asking a coworker to help turn the client


2. Explaining to the client why turning is important.
3. Allowing the client to turn when he's ready to do so
4. Telling the client that the physician's order states he must turn every 2 hours

55. A client has a percutaneous endoscopic gastrostomy tube inserted for tube feedings. Before
starting a continuous feeding, the nurse should place the client in which position?

1. Semi-Fowlers
2. Supine
3. Reverse Trendelenburg
4. High Fowler's

56. An enema is prescribed for a client with suspected appendicitis. Which of the following
actions should the nurse take?

1. Prepare 750 ml of irrigating solution warmed to 100*F


2. Question the physician about the order
3. Provide privacy and explain the procedure to the client
4. Assist the client to left lateral Sim's position

57. The client being seen in a physician's office has just been scheduled for a barium swallow
the next day. The nurse writes down which of the following instructions for the client to follow
before the test?
1. Fast for 8 hours before the test
2. Eat a regular supper and breakfast
3. Continue to take all oral medications as scheduled.
4. Monitor own bowel movement pattern for constipation

58. The nurse is monitoring a client for the early signs of dumping syndrome. Which symptom
indicates this occurrence?

1. Abdominal cramping and pain


2. Bradycardia and indigestion
3. Sweating and pallor
4. Double vision and chest pain

59. The nurse is preparing a discharge teaching plan for the client who had an umbilical hernia
repair. Which of the following would the nurse include in the plan?

1. Restricting pain medication


2. Maintaining bedrest
3. Avoiding coughing
4. Irrigating the drain

60. The nurse is caring for a hospitalized client with a diagnosis of ulcerative colitis. Which
finding, if noted on assessment of the client, would the nurse report to the physician?

1. Bloody diarrhea
2. Hypotension
3. A hemoglobin of 12 mg/dL
4. Rebound tenderness

61. The nurse is reviewing the record of a client with Crohn's disease. Which of the following
stool characteristics would the nurse expect to note documented on the client's record?

1. Chronic constipation
2. Diarrhea
3. Constipation alternating with diarrhea
4. Stool constantly oozing from the rectum

62. The nurse is performing a colostomy irrigation on a client. During the irrigation, a client
begins to complain of abdominal cramps. Which of the following is the most appropriate nursing
action?

1. Notify the physician


2. Increase the height of the irrigation
3. Stop the irrigation temporarily.
4. Medicate with dilaudid and resume the irrigation

63. The nurse is teaching the client how to perform a colostomy irrigation. To enhance the
effectiveness of the irrigation and fecal returns, what measure should the nurse instruct the
client to do?
1. Increase fluid intake
2. Reduce the amount of irrigation solution
3. Perform the irrigation in the evening
4. Place heat on the abdomen
1. Increase fluid intake
.
64. The nurse is reviewing the physician's orders written for a client admitted with acute
pancreatitis. Which physician order would the nurse question if noted on the client's chart?

1. NPO status
2. Insert a nasogastric tube
3. An anticholinergic medication
4. Morphine for pain

65. The nurse is doing an admission assessment on a client with a history of duodenal ulcer. To
determine whether the problem is currently active, the nurse would assess the client for which of
the following most frequent symptom(s) of duodenal ulcer?

1. Pain that is relieved by food intake


2. Pain that radiated down the right arm
3. N/V
4. Weight loss

66. The nurse instructs the ileostomy client to do which of the following as a part of essential
care of the stoma?

1. Cleanse the peristomal skin meticulously


2. Take in high-fiber foods such as nuts
3. Massage the area below the stoma
4. Limit fluid intake to prevent diarrhea.

67. The client who has undergone creation of a colostomy has a nursing diagnosis of Disturbed
body image. The nurse would evaluate that the client is making the most significant progress
toward identified goals if the client:

1. Watches the nurse empty the colostomy bag


2. Looks at the ostomy site
3. Reads the ostomy product literature
4. Practices cutting the ostomy appliance

68. The nurse is assessing for stoma prolapse in a client with a colostomy. The nurse would
observe which of the following if stoma prolapse occurred?

1. Sunken and hidden stoma


2. Dark- and bluish-colored stoma
3. Narrowed and flattened stoma
4. Protruding stoma

69. The client with a new colostomy is concerned about the odor from the stool in the ostomy
drainage bag. The nurse teaches the client to include which of the following foods in the diet to
reduce odor?
1. Yogurt
2. Broccoli
3. Cucumbers
4. Eggs

70. The nurse has given instructions to the client with an ileostomy about foods to eat to thicken
the stool. The nurse determines that the client needs further instructions if the client stated to
eat which of the following foods to make the stools less watery?

1. Pasta
2. Boiled rice
3. Bran
4. Low-fat cheese

71. The client has just had surgery to create an ileostomy. The nurse assesses the client in the
immediate post-op period for which of the following most frequent complications of this type of
surgery?

1. Intestinal obstruction
2. Fluid and electrolyte imbalance
3. Malabsorption of fat
4. Folate deficiency

72. The nurse is doing pre-op teaching with the client who is about to undergo creation of a
Kock pouch. The nurse interprets that the client has the best understanding of the nature of the
surgery if the client makes which of the following statements?

1. "I will need to drain the pouch regularly with a catheter."


2. "I will need to wear a drainage bag for the rest of my life."
3. "The drainage from this type of ostomy will be formed."
4. "I will be able to pass stool from my rectum eventually."

73. The client with a colostomy has an order for irrigation of the colostomy. The nurse used
which solution for irrigation?

1. Distilled water
2. Tap water
3. Sterile water
4. Lactated Ringer's

74. A nurse is monitoring a client admitted to the hospital with a diagnosis of appendicitis. The
client is scheduled for surgery in 2 hours. The client begins to complain of increased abdominal
pain and begins to vomit. On assessment the nurse notes that the abdomen is distended and
the bowel sounds are diminished. Which of the following is the most appropriate nursing
intervention?

1. Administer dilaudid
2. Notify the physician
3. Call and ask the operating room team to perform the surgery as soon as possible
4. Reposition the client and apply a heating pad on a warm setting to the client's abdomen.
75. The client has been admitted with a diagnosis of acute pancreatitis. The nurse would assess
this client for pain that is:

1. Severe and unrelenting, located in the epigastric area and radiating to the back.
2. Severe and unrelenting, located in the left lower quadrant and radiating to the groin.
3. Burning and aching, located in the epigastric area and radiating to the umbilicus.
4. Burning and aching, located in the left lower quadrant and radiating to the hip.

76. The client with Crohn's disease has a nursing diagnosis of acute pain. The nurse would
teach the client to avoid which of the following in managing this problem?

1. Lying supine with the legs straight


2. Massaging the abdomen
3. Using antispasmodic medication
4. Using relaxation techniques

77. A client with ulcerative colitis has an order to begin salicylate medication to reduce
inflammation. The nurse instructs the client to take the medication:

1. 30 minutes before meals


2. On an empty stomach
3. After meals
4. On arising

78. During the assessment of a client's mouth, the nurse notes the absence of saliva. The client
is also complaining of pain near the area of the ear. The client has been NPO for several days
because of the insertion of a NG tube. Based on these findings, the nurse suspects that the
client is developing which of the following mouth conditions?

1. Stomatitis
2. Oral candidiasis
3. Parotitis
4. Gingivitis

79. The nurse evaluates the client's stoma during the initial post-op period. Which of the
following observations should be reported immediately to the physician?

1. The stoma is slightly edematous


2. The stoma is dark red to purple
3. The stoma oozes a small amount of blood
4. The stoma does not expel stool

80. When planning care for a client with ulcerative colitis who is experiencing symptoms, which
client care activities can the nurse appropriately delegate to a unlicensed assistant? Select all
that apply.

1. Assessing the client's bowel sounds


2. Providing skin care following bowel movements
3. Evaluating the client's response to antidiarrheal medications
4. Maintaining intake and output records
5. Obtaining the client's weight.
81. Which goal of the client's care should take priority during the first days of hospitalization for
an exacerbation of ulcerative colitis?

1. Promoting self-care and independence


2. Managing diarrhea
3. Maintaining adequate nutrition
4. Promoting rest and comfort

82. A client's ulcerative colitis symptoms have been present for longer than 1 week. The nurse
recognizes that the client should be assessed carefully for signs of which of the following
complications?

1. Heart failure
2. DVT
3. Hypokalemia
4. Hypocalcemia

83. A client who has ulcerative colitis has persistent diarrhea. He is thin and has lost 12 pounds
since the exacerbation of his ulcerative colitis. The nurse should anticipate that the physician
will order which of the following treatment approaches to help the client meet his nutritional
needs?

1. Initiate continuous enteral feedings


2. Encourage a high protein, high-calorie diet
3. Implement total parenteral nutrition
4. Provide six small meals a day.

84. Digoxin preparations and absorbents should not be given simultaneously. As a nurse, you
are aware that if these agents are given simultaneously, which of the following will occur?

1. Increased absorption of digoxin


2. Decreased absorption of digoxin
3. Increased absorption of the absorbent
4. Decreased absorption of the absorbent

85. When used with hyperacidic disorders of the stomach, antacids are given to elevate the
gastric pH to:

1) 2.0
2) 4.0
3) 6.0
4) >8.0

86. One of your patients is receiving digitalis orally and is also to receive an antacid at the same
time. Your most appropriate action, based on the pharmacokinetics of antacids, is to:

1. Delay the digitalis for 1 to 2 hours until the antacid is absorbed


2. Give the antacid at least 2 to 4 hours before administering the digitalis
3. Administer both medications as ordered and document in nurse's notes
4. Contact the physician regarding the drug interaction and request a change in the time
of dosing of the drugs.

87. The nurse would teach patients that antacids are effective in treatment of hyperacidity
because they:

1. Neutralize gastric acid


2. Decrease stomach motility
3. Decrease gastric pH
4. Decrease duodenal pH

Antacids work by neutralizing gastric acid, which would cause an increase in pH. They do not
affect gastric motility.
88. The nurse would monitor for which of the following adverse reactions to aluminum-
containing antacids such as aluminum hydroxide (Amphojel)?

1. Diarrhea
2. Constipation
3. GI upset
4. Fluid retention

89. The nurse would question an order for which type of antacid in patients with chronic renal
failure?

1. Aluminum-containing antacids
2. Calcium-containing antacids
3. Magnesium-containing antacids
4. All of the above.

90. The nurse would monitor a patient using sodium bicarbonate to treat gastric hyperacidity for
signs and symptoms of:

1. Metabolic alkalosis
2. Metabolic acidosis
3. Hyperkalemia
4. Hypercalcemia

91. Which of the following nursing diagnoses is appropriate for a patient receiving famotidine
(Pepcid)?

1. Increased risk for infection due to immunosuppression


2. Potential risk for bleeding related to thrombocytopenia.
3. Alteration in urinary elimination related to retention
4. Alteration in tissue perfusion related to hypertension

92. Histamine2-receptor antagonists:

1. Compete with histamine for binding sites on the parietal cells


2. Irreversibly bind to H+/K+ATPase
3. Cause a decrease in stomach pH
4. Decrease signs and symptoms of allergies related to histamine release
93. Proton pump inhibitors:

1. Gastric ulcer formation


2. GERD
3. Achlorhydria
4. Diverticulosis

94. A patient unable to tolerate oral medications may be prescribed which of the following
proton pump inhibitors to be administered intravenously?

1. lansoprazole (Prevacid)
2. omeprazole (Prilosec)
3. pantoprazole (Protonix)
4. esomeprazole (Nexium)
3. pantoprazole (Protonix)

95. When administering sucralfate (Carafate) to a patient with a nasogastric tube, it is important
to:

1. Crush the tablet into a fine powder before mixing with water
2. Administer with a bolus tube feeding
3. Allow the tablet to dissolve in water before administering
4. Administer with an antacid for maximum benefit

96. Sucralfate (Carafate) achieves a therapeutic effect by:

1. Neutralizing gastric acid


2. Enhancing gastric absorption
3. Forming a protective barrier around gastric mucosa
4. Inhibiting gastric acid secretion
3. Forming a protective barrier around gastric mucosa

97. To avoid fecal impaction, psyllium (Metamucil) should be administered with at least how
many ounces of fluid?

1) 4
2) 6
3) 8
4) 10

98. Bismuth subsalicylate (Pepto-Bismol), as an absorbent, has which of the following


mechanisms of action?

1. Decreased GI motility
2. Decreased gastric secretions
3. Increased fluid absorption
4. Binding to diarrhea-causing bacteria for excretion

99. Side effects of loperamide (Imodium) include all of the following except?

1. Diarrhea
2. epigastric pain
3. Dry mouth
4. Anorexia

100. The mechanism of action of diphenoxylate (Lotomil) is:

1. An increase in intestinal excretion of water


2. An increase in intestinal motility
3. A decrease in peristalsis in the intestinal wall
4. A decrease in the reabsorption of water in the bowel

101. Over the past year, a woman has cooked gourmet meals for her family but eats only tiny
servings. She wears layered, loose clothing and now has amenorrhea. Her current weight is 95
pounds, a loss of 35 pounds. Which medical diagnosis is most likely?

1. Binge eating disorder


2. Anorexia nervosa
3. Bulimia nervosa
4. Pica

102. Family dynamics are thought to be a major influence in the development of anorexia
nervosa. Which statement regarding a client's home environment should a nurse associate with
the development of anorexia nervosa?

1. The home environment maintains loose personal boundaries.


2. The home environment places an overemphasis on food.
3. The home environment is overprotective and demands perfection.
4. The home environment condones corporal punishment.

103. A client's altered body image is evidenced by claims of feeling fat, even though the client is
emaciated. Which is the appropriate outcome criterion for this clients problem?

1. The client will consume adequate calories to sustain normal weight.


2. The client will cease strenuous exercise programs.
3. The client will perceive personal ideal body weight and shape as normal.
4. The client will not express a preoccupation with food.

104. A nurse is counseling a client diagnosed with bulimia nervosa about the symptom of tooth
enamel deterioration. Which explanation for this complication of bulimia nervosa, should the
nurse provide?

1. The emesis produced during purging is acidic and corrodes the tooth enamel.
2. Purging causes the depletion of dietary calcium.
3. Food is rapidly ingested without proper mastication.
4. Poor dental and oral hygiene leads to dental caries.

105. A nurse is teaching a client diagnosed with an eating disorder about behavior-modification
programs. Why is this intervention the treatment of choice?

1. It helps the client correct a distorted body image.


2. It addresses the underlying client anger.
3. It manages the client's uncontrollable behaviors.
4. It allows clients to maintain control.

106. A potential Olympic figure skater collapses during practice and is hospitalized for severe
malnutrition. Anorexia nervosa is diagnosed. Which client statement best reflects insight related
to this disorder?

1. Skaters need to be thin to improve their daily performance.


2. All the skaters on the team are following an approved 1200-calorie diet.
3. The exercise of skating reduces my appetite but improves my energy level.
4. I am angry at my mother. I can only get her approval when I win competitions.

107. The family of a client diagnosed with anorexia nervosa becomes defensive when the
treatment team calls for a family meeting. Which is the appropriate nursing response?

1. Tell me why this family meeting is causing you to be defensive. All clients are required to
participate in two family sessions.
2. Eating disorders have been correlated to certain familial patterns; without addressing
these, your child's condition will not improve.
3. Family dynamics are not linked to eating disorders. The meeting is to provide your child with
family support.
4. Clients diagnosed with anorexia nervosa are part of the family system, and any alteration in
family processes needs to be addressed.

108. A client diagnosed with bulimia nervosa has been attending a mental health clinic for
several months. Which factor should a nurse identify as an appropriate indicator of a positive
client behavioral change?

1. The client gained two pounds in one week.


2. The client focused conversations on nutritious food.
3. The client demonstrated healthy coping mechanisms that decreased anxiety.
4. The client verbalized an understanding of the etiology of the disorder.

109. A morbidly obese client is prescribed an anorexiant medication. The nurse should expect
to teach the client about which medication?

1. Phentermine (Mirapront)
2. Dexfenfluramine (Redux)
3. Sibutramine (Meridia)
4. Pemoline (Cylert)

110. A nurse is attempting to differentiate between the symptoms of anorexia nervosa and the
symptoms of bulimia. Which statement delineates the difference between these two disorders?

1. Clients diagnosed with anorexia nervosa experience extreme nutritional deficits,


whereas clients diagnosed with bulimia nervosa do not.
2. Clients diagnosed with bulimia nervosa experience amenorrhea, whereas clients diagnosed
with anorexia nervosa do not.
3. Clients diagnosed with bulimia nervosa experience hypotension, edema, and lanugo,
whereas clients diagnosed with anorexia nervosa do not.
4. Clients diagnosed with anorexia nervosa have eroded tooth enamel, whereas clients
diagnosed with bulimia nervosa do not.

111. A client diagnosed with a history of anorexia nervosa comes to an outpatient clinic after
being medically cleared. The client states, "My parents watch me like a hawk and never let me
out of their sight." Which nursing diagnosis would take priority at this time?

1. Altered nutrition less than body requirements


2. Altered social interaction
3. Impaired verbal communication
4. Altered family processes

112. A nurse should identify topiramate (Topamax) as the drug of choice for which of the
following conditions? (SATA)

1. Binge eating with a diagnosis of obesity


2. Bingeing and purging with a diagnosis of bulimia nervosa
3. Weight loss with a diagnosis of anorexia nervosa
4. Amenorrhea with a diagnosis of anorexia nervosa
5. Emaciation with a diagnosis of bulimia nervosa

113. A nursing instructor is teaching about the DSM-5 criteria for the diagnosis of binge-eating
disorder. Which of the following student statements indicates that further instruction is needed?
(SATA)

1. In this disorder, binge eating occurs exclusively during the course of bulimia nervosa.
2. In this disorder, binge eating occurs, on average, at least once a week for three months.
3. In this disorder, binge eating occurs, on average, at least two days a week for six
months.
4. In this disorder, distress regarding binge eating is present.
5. In this disorder, distress regarding binge eating is absent.

114. Which of the following would contribute to a clients excessive weight gain? (SATA)

1. A hypothalamus lesion
2. Hyperthyroidism
3. Diabetes mellitus
4. Cushings disease
5. Low levels of serotonin

115. Disturbed body image is the nursing diagnosis for a patient diagnosed with an eating
disorder. Which outcome indicator is most appropriate to monitor?

a. Weight, muscle, and fat are congruent with height, frame, age, and sex.
b. Calorie intake is within the required parameters of the treatment plan.
c. Weight reaches the established normal range for the patient.
d. Patient expresses satisfaction with body appearance.

116. A patient who is referred to the eating disorders clinic has lost 35 pounds in the past 3
months. To assess the patient's oral intake, the nurse should ask:
a. Do you often feel fat?
b. Who plans the family meals?
c. What do you eat in a typical day?
d. What do you think about your present weight?

117. A patient diagnosed with anorexia nervosa virtually stopped eating 5 months ago and has
lost 25% of body weight. A nurse asks, "Describe what you think about your present weight and
how you look." Which response by the patient is most consistent with the diagnosis?

a. I am fat and ugly.


b. What I think about myself is my business.
c. I am grossly underweight, but that's what I want.
d. I am a few pounds overweight, but I can live with it.

118. A patient was diagnosed with anorexia nervosa. The history shows the patient virtually
stopped eating 5 months ago and has lost 25% of body weight. The patient's current serum
potassium is 2.7 mg/dl. Which nursing diagnosis applies?

a. Adult failure to thrive, related to abuse of laxatives as evidenced by electrolyte imbalances


and weight loss
b. Disturbed energy field, related to physical exertion in excess of energy produced through
caloric intake as evidenced by weight loss and hyperkalemia
c. Ineffective health maintenance, related to self-induced vomiting as evidenced by swollen
parotid glands and hyperkalemia
d. Imbalanced nutrition: less than body requirements, related to malnutrition as
evidenced by loss of 25% of body weight and hypokalemia

119. Outpatient treatment is planned for a patient diagnosed with anorexia nervosa. Select the
most important outcome related to the nursing diagnosis: Imbalanced nutrition: less than body
requirements. Within 1 week, the patient will:

a. weigh self accurately using balanced scales.


b. limit exercise to less than 2 hours daily.
c. select clothing that fits properly.
d. gain 1 to 2 pounds.

120. Which nursing intervention has priority as a patient diagnosed with anorexia nervosa
begins to gain weight?

a. Assess for depression and anxiety.


b. Observe for adverse effects of re-feeding.
c. Communicate empathy for the patients feelings.
d. Help the patient balance energy expenditure and caloric intake.

121. A patient diagnosed with anorexia nervosa is resistant to weight gain. What is the rationale
for establishing a contract with the patient to participate in measures designed to produce a
specified weekly weight gain?

a. Because severe anxiety concerning eating is expected, objective and subjective data must be
routinely collected.
b. Patient involvement in decision-making increases a sense of control and promotes
compliance with the treatment.
c. A team approach to planning the diet ensures that physical and emotional needs of the
patient are met.
d. Because of increased risk for physical problems with re-feeding, obtaining patient permission
is required.

122. The nursing care plan for a patient diagnosed with anorexia nervosa includes the
intervention Monitor for complications of re-feeding. Which body system should a nurse closely
monitor for dysfunction?

a. Renal
b. Endocrine
c. Central nervous
d. Cardiovascular

123. A psychiatric clinical nurse specialist uses cognitive therapy techniques with a patient
diagnosed with anorexia nervosa. Which statement by the staff nurse supports this type of
therapy?

a. What are your feelings about not eating the food that you prepare?
b. You seem to feel much better about yourself when you eat something.
c. It must be difficult to talk about private matters to someone you just met.
d. Being thin does not seem to solve your problems. You are thin now but still unhappy.

124. An appropriate intervention for a patient diagnosed with bulimia nervosa who binges and
purges is to teach the patient to:

a. eat a small meal after purging.


b. avoid skipping meals or restricting food.
c. concentrate oral intake after 4 PM daily.
d. understand the value of reading journal entries aloud to others.

125. What behavior by a nurse caring for a patient diagnosed with an eating disorder indicates
the nurse needs supervision?

a. The nurses comments are nonjudgmental.


b. The nurse uses an authoritarian manner when interacting with the patient.
c. The nurse teaches the patient to recognize signs of increasing anxiety and ways to intervene.
d. The nurse refers the patient to a self-help group for individuals with eating disorders.

126. A nursing diagnosis for a patient diagnosed with bulimia nervosa is: Ineffective coping,
related to feelings of loneliness as evidenced by overeating to comfort self, followed by self-
induced vomiting. The best outcome related to this diagnosis is, Within 2 weeks the patient will:

a. appropriately express angry feelings.


b. verbalize two positive things about self.
c. verbalize the importance of eating a balanced diet.
d. identify two alternative methods of coping with loneliness.
127. Which nursing intervention has the highest priority for a patient diagnosed with bulimia
nervosa?

a. Assist the patient to identify triggers to binge eating.


b. Provide corrective consequences for weight loss.
c. Explore patient needs for health teaching.
d. Assess for signs of impulsive eating.

128. One bed is available on the inpatient eating disorders unit. Which patient should be
admitted? The patient whose weight dropped from:

a. 150 to 100 pounds over a 4-month period. Vital signs: temperature, 35.9 C; pulse, 38
beats/min; blood pressure, 60/40 mm Hg
b. 120 to 90 pounds over a 3-month period. Vital signs: temperature, 36 C; pulse, 50 beats/min;
blood pressure, 70/50 mm Hg
c. 110 to 70 pounds over a 4-month period. Vital signs: temperature, 36.5 C; pulse, 60
beats/min; blood pressure, 80/66 mm Hg
d. 90 to 78 pounds over a 5-month period. Vital signs: temperature, 36.7 C; pulse, 62 beats/min;
blood pressure, 74/48 mm Hg

129.While providing health teaching for a patient diagnosed with bulimia nervosa, a nurse
should emphasize information about:

a. self-monitoring of daily food and fluid intake.


b. establishing the desired daily weight gain.
c. recognizing symptoms of hypokalemia.
d. self-esteem maintenance.

130. As a patient admitted to the eating disorders unit undresses, a nurse observes that the
patients body is covered by fine, downy hair. The patient weighs 70 pounds and is 5 feet, 4
inches tall. Which condition should be documented?

a. Amenorrhea
b. Alopecia
c. Lanugo
d. Stupor

131. A patient being admitted to the eating disorders unit has a yellow cast to the skin and fine,
downy hair covering the body. The patient weighs 70 pounds; height is 5 feet, 4 inches. The
patient is quiet and says only, "I won't eat until I look thin." What is the priority initial nursing
diagnosis?

a. Anxiety, related to fear of weight gain


b. Disturbed body image, related to weight loss
c. Ineffective coping, related to lack of conflict resolution skills
d. Imbalanced nutrition: less than body requirements, related to self-starvation

132. A nurse conducting group therapy on the eating disorders unit schedules the sessions
immediately after meals for the primary purpose of:
a. maintaining patient's concentration and attention.
b. shifting the patient's focus from food to psychotherapy.
c. focusing on weight control mechanisms and food preparation.
d. processing the heightened anxiety associated with eating.

133. Physical assessment of a patient diagnosed with bulimia nervosa often reveals:

a. prominent parotid glands.


b. peripheral edema.
c. thin, brittle hair.
d. amenorrhea.

134. Which personality characteristic is a nurse most likely to assess in a patient diagnosed with
anorexia nervosa?

a. Carefree flexibility
b. Rigidity, perfectionism
c. Open displays of emotion
d. High spirits and optimism

135. Which assessment finding for a patient diagnosed with an eating disorder meets a criterion
for hospitalization?

a. Urine output: 40 ml/hr


b. Pulse rate: 58 beats/min
c. Serum potassium: 3.4 mEq/L
d. Systolic blood pressure: 62 mm Hg

136. Which statement is a nurse most likely to hear from a patient diagnosed with anorexia
nervosa?

a. I would be happy if I could lose 20 more pounds.


b. My parents don't pay much attention to me.
c. I'm thin for my height.
d. I have nice eyes.

137. Which nursing diagnosis is more applicable for a patient diagnosed with anorexia nervosa
who restricts intake and is 20% below normal weight than for a 130-pound patient diagnosed
with bulimia nervosa who purges?

a. Powerlessness
b. Ineffective coping
c. Disturbed body image
d. Imbalanced nutrition: less than body requirements

138. An outpatient diagnosed with anorexia nervosa has begun re-feeding. Between the first
and second appointments, the patient gained 8 pounds. The nurse should:

a. assess lung sounds and extremities.


b. suggest the use of an aerobic exercise program.
c. positively reinforce the patient for the weight gain.
d. establish a higher goal for weight gain the next week.

139. When a nurse finds a patient diagnosed with anorexia nervosa vigorously exercising before
gaining the agreed-upon weekly weight, the nurse should state:

a. You and I will have to sit down and discuss this problem.
b. It bothers me to see you exercising. You'll lose more weight.
c. Lets discuss the relationship between exercise and weight loss and how that affects your
body.
d. According to our agreement, no exercising is permitted until you have gained a
specific amount of weight.

140. A patient diagnosed with anorexia nervosa has a body mass index (BMI) of 14.8 kg/m2.
Which assessment finding is most likely to accompany this value?

a. Cachexia
b. Leukocytosis
c. Hyperthermia
d. Hypertension

141. The nurse is monitoring a client admitted to the hospital with a diagnosis of appendicitis
who is scheduled for surgery in 2 hours. The client begins to complain of increased abdominal
pain and begins to vomit. On assessment, the nurse notes that the abdomen is distended and
bowel sounds are diminished. Which is the most appropriate nursing intervention?

1. Notify the health care provider (HCP).


2. Administer the prescribed pain medication.
3. Call and ask the operating room team to perform surgery as soon as possible.
4. Reposition the client and apply a heating pad on the warm setting to the client's abdomen.

142. A client admitted to the hospital with a suspected diagnosis of acute pancreatitis is being
assessed by the nurse. Which assessment findings would be consistent with acute pancreatitis?
(SATA)

1. Diarrhea
2. Black, tarry stools
3. Hyperactive bowel sounds
4. Gray-blue color at the flank
5. Abdominal guarding and tenderness
6. Left upper quadrant pain with radiation to the back

143. The nurse is assessing a client who is experiencing an acute episode of cholecystitis.
Which of these clinical manifestations support this diagnosis? (SATA)

1. Fever
2. Positive Cullen's sign
3. Complaints of indigestion
4. Palpable mass in the left upper quadrant
5. Pain in the upper right quadrant after a fatty meal
6. Vague lower right quadrant abdominal discomfort

144. A client is diagnosed with viral hepatitis, complaining of "no appetite" & "losing my taste for
food." What instruction should the nurse give the client to provide adequate nutrition?

1. Select foods high in fat.


2. Increase intake of fluids, including juices.
3. Eat a good supper when anorexia is not as severe.
4. Eat less often, preferably only 3 large meals daily.

145. A client has developed hepatitis A after eating contaminated oysters. The nurse assesses
the client for which expected assessment finding?

1. Malaise
2. Dark stools
3. Weight gain
4. Left upper quadrant discomfort

146. A client has just had a hemorrhoidectomy. Which nursing interventions are appropriate for
this client? (SATA)

1. Administer stool softeners as prescribed.


2. Instruct the client to limit fluid intake to avoid urinary retention.
3. Encourage a high-fiber diet to promote bowel movements without straining.
4. Apply cold packs to the anal-rectal area over the dressing until the packing is
removed.
5. Help the client to a Fowler's position to place pressure on the rectal area and decrease
bleeding.

147. The nurse is planning to teach a client with gastroesophageal reflux disease (GERD) about
substances to avoid. Which items should the nurse include on this list? (SATA)

1. Coffee
2. Chocolate
3. Peppermint
4. Nonfat milk
5. Fried chicken
6. Scrambled eggs

148. A client has undergone esophagogastroduodenoscopy (EGD). The nurse should place
highest priority on which item as part of the client's care plan?

1. Monitoring the temperature


2. Monitoring complaints of heartburn
3. Giving warm gargles for a sore throat
4. Assessing for the return of the gag reflex

149. The nurse has taught the client about an upcoming endoscopic retrograde
cholangiopancreatography (ERCP) procedure. The nurse determines that the client needs
further information if the client makes which statement?
1. "I know I must sign the consent form."
2. "I hope the throat spray keeps me from gagging."
3. "I'm glad I don't have to lie still for this procedure."
4. "I'm glad some intravenous medication will be given to relax me."

150. The health care provider has determined that a client has contracted hepatitis-A based on
flu-like symptoms & jaundice. Which statement made by the client supports this medical
diagnosis?

1. "I have had unprotected sex with multiple partners."


2. "I ate shellfish about 2 weeks ago at a local restaurant."
3. "I was an intravenous drug abuser in the past & shared needles."
4. "I had a blood transfusion 30 years ago after major abdominal surgery."

151. The nurse is providing dietary teaching for a client with a diagnosis of chronic gastritis. The
nurse instructs the client to include which foods rich in vitamin B12 in the diet? (SATA)

1. Nuts
2. Corn
3. Liver
4. Apples
5. Lentils
6. Bananas

152. The nurse is assessing a client 24 hours following a cholecystectomy. The nurse notes that
the T-tube has drained 750 mL of green-brown drainage since the surgery. Which nursing
intervention is most appropriate?

1. Clamp the T-tube.


2. Irrigate the T-tube.
3. Document the findings.
4. Notify the health care provider.

153. The nurse is monitoring a client with a diagnosis of peptic ulcer. Which assessment finding
would most likely indicate perforation of the ulcer?

1. Bradycardia
2. Numbness in the legs
3. Nausea and vomiting
4. A rigid, board-like abdomen

154.The nurse is caring for a client following a gastrojejunostomy (Billroth II procedure). Which
postoperative prescription should the nurse question and verify?

1. Leg exercises
2. Early ambulation
3. Irrigating the nasogastric tube
4. Coughing & deep-breathing exercises
155. The nurse is providing discharge instructions to a client following gastrectomy & should
instruct the client to take which measure to assist in preventing dumping syndrome?

1. Ambulate following a meal.


2. Eat high-carbohydrate foods.
3. Limit the fluids taken with meals.
4. Sit in a high Fowler's position during meals.

156. The nurse is reviewing the prescription for a client admitted to the hospital with a diagnosis
of acute pancreatitis. Which interventions would the nurse expect to be prescribed for the client?
(SATA)

1. Maintain NPO (nothing by mouth) status.


2. Encourage coughing and deep breathing.
3. Give small, frequent high-calorie feedings.
4. Maintain the client in a supine and flat position.
5. Give hydromorphone IV as prescribed for pain.
6. Maintain intravenous fluids at 10 mL/hour to keep the vein open.

157. The nurse is providing discharge teaching for a client with newly diagnosed Crohn's
disease about dietary measures to implement during exacerbation episodes. Which statement
made by the client indicates a need for further instruction?

1. "I should increase the fiber in my diet."


2. "I will need to avoid caffeinated beverages."
3. "I'm going to learn some stress reduction techniques."
4. "I can have exacerbations and remissions with Crohn's disease."

158. The nurse is reviewing the record of a client with a diagnosis of cirrhosis and notes that
there is documentation of the presence of asterixis. How should the nurse assess for its
presence?

1. Dorsiflex the client's foot.


2. Measure the abdominal girth.
3. Ask the client to extend the arms.
4. Instruct the client to lean forward.

159. The nurse is reviewing the laboratory results for a client with cirrhosis and notes that the
ammonia level is 85 mcg/dL (51 mcmol/L). Which dietary selection does the nurse suggest to
the client?

1. Roast pork
2. Cheese omelet
3. Pasta with sauce
4. Tuna fish sandwich

160. The nurse is doing an admission assessment on a client with a history of duodenal ulcer.
To determine whether the problem is currently active, the nurse should assess the client for
which sign(s)/symptom(s) of duodenal ulcer?
1. Weight loss
2. Nausea and vomiting
3. Pain relieved by food intake
4. Pain radiating down the right arm

161. A client with hiatal hernia chronically experiences heartburn following meals. The nurse
should plan to teach the client to avoid which action because it is contraindicated with a hiatal
hernia?

1. Lying recumbent following meals


2. Consuming small, frequent, bland meals
3. Taking H2-receptor antagonist medication
4. Raising the head of the bed on 6-inch (15 cm) blocks

162. The nurse is providing care for a client with a recent transverse colostomy. Which
observation requires immediate notification of the health care provider?

1. Stoma is beefy red and shiny


2. Purple discoloration of the stoma
3. Skin excoriation around the stoma
4. Semi-formed stool noted in the ostomy pouch

163. A client had a new colostomy created 2 days earlier and is beginning to pass malodorous
flatus from the stoma. What is the correct interpretation by the nurse?

1. This is a normal, expected event.


2. The client is experiencing early signs of ischemic bowel.
3. The client should not have the nasogastric tube removed.
4. This indicates inadequate preoperative bowel preparation.

164. A client has just had surgery to create an ileostomy. The nurse assesses the client in the
immediate postoperative period for which most frequent complication of this type of surgery?

1. Folate deficiency
2. Malabsorption of fat
3. Intestinal obstruction
4. Fluid and electrolyte imbalance

165. The nurse provides instructions to a client about measures to treat inflammatory bowel
syndrome (IBS). Which statement by the client indicates a need for further teaching?

1. "I need to limit my intake of dietary fiber."


2. "I need to drink plenty, at least 8 to 10 cups daily."
3. "I need to eat regular meals and chew my food well."
4. "I will take the prescribed medications because they will regulate my bowel patterns."

166. The nurse is monitoring a client for the early signs and symptoms of dumping syndrome.
Which findings indicate this occurrence?

1. Sweating and pallor


2. Bradycardia and indigestion
3. Double vision and chest pain
4. Abdominal cramping and pain

167. A client presents to the emergency department with upper gastrointestinal bleeding and is
in moderate distress. In planning care, what is the priority nursing action for this client?

1. Assessment of vital signs


2. Completion of abdominal examination
3. Insertion of the prescribed nasogastric tube
4. Thorough investigation of precipitating events

168. The nurse is caring for a client with acute pancreatitis and is monitoring the client for
paralytic ileus. Which piece of assessment data should alert the nurse to this occurrence?

1. Inability to pass flatus


2. Loss of anal sphincter control
3. Severe, constant pain with rapid onset
4. Firm, nontender mass palpable at the lower right costal margin

169. The nurse inspects the color of the drainage from a nasogastric tube on a postoperative
client approximately 24 hours after gastric surgery. Which finding indicates the need to notify the
health care provider (HCP)?

1. Dark red drainage


2. Dark brown drainage
3. Green-tinged drainage
4. Light yellowish-brown drainage

170. A sexually active young adult client has developed viral hepatitis. Which client statement
indicates the need for further teaching?

1. "I should avoid drinking alcohol."


2. "I can go back to work right away."
3. "My partner should get the vaccine."
4. "A condom should be used for sexual intercourse."

171. The nurse is caring for a client admitted to the hospital with a suspected diagnosis of acute
appendicitis. Which laboratory result should the nurse expect to note if the client does have
appendicitis?

1. Leukopenia with a shift to the left


2. Leukocytosis with a shift to the left
3. Leukopenia with a shift to the right
4. Leukocytosis with a shift to the right

172. After performing an initial abdominal assessment on a client with nausea and vomiting, the
nurse should expect to note which finding?

1. Waves of loud gurgles auscultated in all 4 quadrants


2. Low-pitched swishing auscultated in 1 or 2 quadrants
3. Relatively high-pitched clicks or gurgles auscultated in all 4 quadrants
4. Very high-pitched, loud rushes auscultated especially in 1 or 2 quadrants

173. After undergoing Billroth I gastric surgery, the client experiences fatigue and complains of
numbness and tingling in the feet and difficulties with balance. On the basis of these symptoms,
the nurse suspects which postoperative complication?

1. Stroke
2. Pernicious anemia
3. Bacterial meningitis
4. Peripheral arterial disease

174. A client experiencing chronic dumping syndrome makes the following comments to the
nurse. Which one indicates the need for further teaching?

1. "I eat at least 3 large meals each day."


2. "I eat while lying in a semirecumbent position."
3. "I have eliminated taking liquids with my meals."
4. "I eat a high-protein, low- to moderate-carbohydrate diet."

175. The nurse obtains an admission history for a client with suspected peptic ulcer disease
(PUD). Which client factor documented by the nurse would increase the risk for PUD?

1. Recently retired from a job


2. Significant other has a gastric ulcer
3. Occasionally drinks 1 cup of coffee in the morning
4. Takes nonsteroidal antiinflammatory drugs (NSAIDs) for osteoarthritis

176. A client who has undergone gastric surgery has a nasogastric (NG) tube connected to low
intermittent suction that is not draining properly. Which action should the nurse take initially?

1. Call the surgeon to report the problem.


2. Reposition the NG tube to the proper location.
3. Check the suction device to make sure it is working.
4. Irrigate the NG tube with saline to remove the obstruction.

177. The nurse is caring for a client with ulcerative colitis. Which finding does the nurse
determine is consistent with this diagnosis?

1. Hypercalcemia
2. Hypernatremia
3. Frothy, fatty stools
4. Decreased hemoglobin

178. A client with acute ulcerative colitis requests a snack. Which is the most appropriate snack
for this client?

1. Carrots and ranch dip


2. Whole-grain cereal and milk
3. A cup of popcorn and a cola drink
4. Applesauce and a graham cracker
179. The nurse is teaching the client with viral hepatitis about the stages of the disease. The
nurse should explain to the client that the second stage of this disease is characterized by which
specific assessment findings? (SATA)

1. Jaundice
2. Flulike symptoms
3. Clay-colored stools
4. Elevated bilirubin levels
5. Dark or tea-colored urine

180. The nurse is teaching an older client about measures to prevent constipation. Which
statement by the client indicates a need for further teaching?

1. "I walk 1 to 2 miles every day."


2. "I need to decrease fiber in my diet."
3. "I have a bowel movement every other day."
4. "I drink 6 to 8 glasses of water every day."

181. The nurse is giving instructions to a client with cholecystitis about food to exclude from the
diet. Which food item identified by the client indicates that the educational session was
successful?

1. Fresh fruit
2. Brown gravy
3. Fresh vegetables
4. Poultry without skin

182. The nurse is performing an assessment on a client with acute pancreatitis who was
admitted to the hospital. Which assessment question would most specifically elicit information
regarding the pain that is associated with acute pancreatitis?

1. "Does the pain in your stomach radiate to your back?"


2. "Does the pain in your lower abdomen radiate to your hip?"
3. "Does the pain in your lower abdomen radiate to your groin?"
4. "Does the pain in your stomach radiate to your lower middle abdomen?"

183. The nurse is caring for a client after abdominal surgery and creation of a colostomy. The
nurse is assessing the client for a prolapsed stoma and should expect to note which observation
if this is present?

1. A sunken and hidden stoma


2. A narrow and flattened stoma
3. A stoma that is dusky or bluish
4. A stoma that is elongated with a swollen appearance

184. The nurse is providing instructions to a client with a colostomy about measures to reduce
the odor from the colostomy. Which client statement indicates that the educational session was
effective?

1. "I should be sure to eat at least 1 cucumber every day."


2. "Beet greens, parsley, or yogurt will help to control the colostomy odor."
3. "I will need to increase my egg intake and try to eat ½ to 1 egg per day."
4. "Green vegetables such as spinach and broccoli will prevent odor, and I should eat these
foods every day."

185. The nurse has provided dietary instructions to a client with a diagnosis of peptic ulcer
disease. Which client statement indicates that education was effective?

1. "Baked foods such as chicken or fish are all right to eat."


2. "Citrus fruits and raw vegetables need to be included in my daily diet."
3. "I can drink beer as long as I consume only a moderate amount each day."
4. "I can drink coffee or tea as long as I limit the amount to 2 cups daily."

186. The nurse has provided home care instructions to a client who had a subtotal gastrectomy.
The nurse instructs the client on the signs and symptoms associated with dumping syndrome.
Which client statement indicates that teaching was effective?

1. "It will cause diaphoresis and diarrhea."


2. "I have to monitor for hiccups and diarrhea."
3. "It will be associated with constipation and fever."
4. "I have to monitor for fatigue and abdominal pain."

187. The nurse is providing instructions to a client regarding measures to minimize the risk of
dumping syndrome. The nurse should make which suggestion to the client?

1. Maintain a high-carbohydrate diet.


2. Increase fluid intake, particularly at mealtime.
3. Maintain a low Fowler's position while eating.
4. Ambulate for at least 30 minutes following each meal.

188. A client with peptic ulcer disease states that stress frequently causes exacerbation of the
disease. The nurse determines that which item mentioned by the client is most likely to be
responsible for the exacerbation?

1. Sleeping 8 to 10 hours a night


2. Ability to work at home periodically
3. Eating 5 or 6 small meals per day
4. Frequent need to work overtime on short notice

189. The nurse is giving dietary instructions to a client who has a new colostomy. The nurse
should encourage the client to eat foods representing which diet for the first 4 to 6 weeks
postoperatively?

1. Low fiber
2. Low calorie
3. High protein
4. High carbohydrate

190. A client with viral hepatitis is discussing with the nurse the need to avoid alcohol and
states, "I'm not sure I can avoid alcohol." What is the most appropriate nursing response?
1. "I don't believe that."
2. "Everything will be all right."
3. "I'm not sure that I understand. Would you please explain?"
4. "I think you should talk more with the health care provider about this."

191. A client with viral hepatitis is having difficulty coping with the disorder. Which question by
the nurse is the most appropriate in identifying the client's coping problem?

1. "Do you have a fever?"


2. "Are you losing weight?"
3. "Have you enjoyed having visitors?"
4. "Do you rest sometime during the day?"

192. A client with viral hepatitis states, "I am so yellow." What is the most appropriate nursing
action?

1. Assist the client in expressing feelings.


2. Restrict visitors until the jaundice subsides.
3. Perform most of the activities of daily living for the client.
4. Provide information to the client only when he or she requests it.

193. A client with viral hepatitis has no appetite, and food makes the client nauseated. Which
nursing intervention is appropriate?

1. Encourage foods that are high in protein.


2. Monitor for fluid and electrolyte imbalance.
3. Explain that high-fat diets usually are better tolerated.
4. Explain that most daily calories need to be consumed in the evening hours.

194. The nurse has implemented a bowel maintenance program for an unconscious client. The
nurse would evaluate the plan as best meeting the needs of the client if which method was
successful in stimulating a bowel movement?

1. Fleet enema
2. Fecal disimpaction
3. Glycerin suppository
4. Soap solution enema (SSE)

195. The nurse checks the gastric residual of an unconscious client receiving nasogastric tube
feedings continuously at 50 mL/hr. The nurse notes that the residual is 250 mL at 0800 and 300
mL at 0900. The nurse determines that the client is experiencing which complication?

1. Air in the stomach


2. Too slow an infusion rate
3. Delayed gastric emptying
4. Early signs of peptic ulcer

196. The nurse is developing a teaching plan for a client with viral hepatitis. The nurse should
plan to include which information in the teaching session?
1. The diet should be low in calories.
2. Meals should be large to conserve energy.
3. Activity should be limited to prevent fatigue.
4. Alcohol intake should be limited to 2 ounces per day.

197. The nurse is preparing to teach a client with a new colostomy about how to perform a
colostomy irrigation. Which information should the nurse include in the teaching plan?

1. Use 500 to 1000 mL of warm tap water.


2. Suspend the irrigant 36 inches above the stoma.
3. Insert the irrigation cone ½ inch into the stoma.
4. If cramping occurs, open the irrigation clamp farther.

198. Family dynamics are thought to be a major influence in the development of anorexia
nervosa. Which information related to a client's home environment should a nurse associate
with the development of this disorder?

A. The home environment maintains loose personal boundaries.


B. The home environment places an overemphasis on food.
C. The home environment is overprotective and demands perfection.
D. The home environment condones corporal punishment.

199. A client's altered body image is evidenced by claims of "feeling fat" even though the client
is emaciated. Which is the appropriate outcome criterion for this client's problem?

A. The client will consume adequate calories to sustain normal weight.


B. The client will cease strenuous exercise programs.
C. The client will perceive an ideal body weight and shape as normal.
D. The client will not express a preoccupation with food.

200. A nurse observes dental deterioration when assessing a client diagnosed with bulimia
nervosa. What explains this assessment finding?

A. The emesis produced during purging is acidic and corrodes the tooth enamel.
B. Purging causes the depletion of dietary calcium.
C. Food is rapidly ingested without proper mastication.
D. Poor dental and oral hygiene leads to dental caries.

201. Why are behavior modification programs the treatment of choice for clients diagnosed with
eating disorders?

A. These programs help clients correct distorted body image.


B. These programs address underlying client anger.
C. These programs help clients manage uncontrollable behaviors.
D. These programs allow clients to maintain control.

202. A potential Olympic figure skater collapses during practice and is hospitalized for severe
malnutrition. Anorexia nervosa is diagnosed. Which client statement best reflects the underlying
etiology of this disorder?

A. "Skaters need to be thin to improve their daily performance."


B. "All the skaters on the team are following an approved 1,200-calorie diet."
C. "When I lose skating competitions, I also lose my appetite."
D. "I am angry at my mother. I can only get her approval when I win competitions."

203. The family of a client diagnosed with anorexia nervosa becomes defensive when the
treatment team calls for a family meeting. Which is the appropriate nursing reply?

A. "Tell me why this family meeting is causing you to be defensive. All clients are required to
participate in two family sessions."
B. "Eating disorders have been correlated to certain familial patterns; without addressing
these, your child's condition will not improve."
C. "Family dynamics are not linked to eating disorders. The meeting is to provide your child with
family support."
D. "Clients diagnosed with anorexia nervosa are part of the family system, and any alteration in
family processes needs to be addressed."

204. A client diagnosed with bulimia nervosa has been attending a mental health clinic for
several months. Which factor should a nurse identify as an appropriate indicator of a positive
client behavioral change?

A. The client gains 2 pounds in 1 week.


B. The client focuses conversations on nutritious food.
C. The client demonstrates healthy coping mechanisms that decrease anxiety.
D. The client verbalizes an understanding of the etiology of the disorder.

205. A nursing instructor is teaching students about the differences between the symptoms of
anorexia nervosa and the symptoms of bulimia nervosa. Which student statement indicates that
learning has occurred?

A. "Clients diagnosed with anorexia nervosa experience extreme nutritional deficits,


whereas clients diagnosed with bulimia nervosa do not."
B. "Clients diagnosed with bulimia nervosa experience amenorrhea, whereas clients diagnosed
with anorexia nervosa do not."
C. "Clients diagnosed with bulimia nervosa experience hypotension, edema, and lanugo,
whereas clients diagnosed with anorexia nervosa do not."
D. "Clients diagnosed with anorexia nervosa have eroded tooth enamel, whereas clients
diagnosed with bulimia nervosa do not."

206. A client who is 5 foot 6 inches tall and weighs 98 pounds is admitted with a medical
diagnosis of anorexia nervosa. Which nursing diagnosis would take priority at this time?

A. Ineffective coping R/T food obsession


B. Altered nutrition: less than body requirements R/T inadequate food intake
C. Risk for injury R/T suicidal tendencies
D. Altered body image R/T perceived obesity

207. A nurse is seeing a client in an outpatient clinic for treatment of anorexia nervosa. Which is
the most appropriate, correctly written short-term outcome for this client?

A. The client will use stress-reducing techniques to avoid purging.


B. The client will discuss chaos in personal life and be able to verbalize a link to purging.
C. The client will gain 2 pounds prior to the next weekly appointment.
D. The client will remain free of signs and symptoms of malnutrition and dehydration.

208. When a community health nurse arrives at the home of a client diagnosed with bulimia
nervosa, the nurse finds the client on the floor unconscious. The client has a history of using
laxatives for purging. To what would the nurse attribute this client's symptoms?

A. Increased creatinine and blood urea nitrogen (BUN) levels


B. Abnormal electroencephalogram (EEG)
C. Metabolic acidosis
D. Metabolic alkalosis

209. A group of nurses are discussing how food is used in their families and the effects this
might have on their ability to work with clients diagnosed with eating disorders. Which of these
nurses will probably be most effective with these clients?

A. The nurse who understands the importance of three balanced meals a day
B. The nurse who permits children to have dessert only after finishing the food on their plate
C. The nurse who refuses to engage in power struggles related to food consumption
D. The nurse who grew up poor and frequently did not have enough food to eat

210. A client diagnosed with anorexia nervosa stopped eating 5 months ago and lost 25% of
total body weight. Which subjective client response would the nurse assess to support this
medical diagnosis?

A. "I do not use any laxatives or diuretics to lose weight."


B. "I am losing lots of hair. It's coming out in handfuls."
C. "I know that I am thin, but I refuse to be fat!"
D. "I don't know why people are worried. I need to lose this weight."

210. A nursing diagnosis of ineffective coping R/T feelings of loneliness AEB bingeing then
purging when alone, is assigned to a client diagnosed with bulimia nervosa. Which is an
appropriate outcome related to this nursing diagnosis?

A. The client will identify two alternative methods of dealing with isolation by day 3.
B. The client will appropriately express angry feelings about lack of control by week 2.
C. The client will verbalize two positive self attributes by day 3.
D. The client will list five ways that the body reacts to bingeing and purging.

211.A nurse responsible for conducting group therapy on an eating disorder unit schedules the
sessions immediately after meals. Which is the rationale for scheduling group therapy at this
time?

A. To shift the clients' focus from food to psychotherapy


B. To prevent the use of maladaptive defense mechanisms
C. To promote the processing of anxiety associated with eating
D. To focus on weight control mechanisms and food preparation

212. Which nursing intervention is appropriate when caring for clients diagnosed with either
anorexia nervosa or bulimia nervosa?
A. Provide privacy during meals.
B. Remain with the client for at least 1 hour after the meal.
C. Encourage the client to keep a journal to document types of food consumed.
D. Restrict client privileges when provided food is not completely consumed.

213. How do cultural stereotypes contribute to the development of eating disorders?

A Eating disorders result from biological and genetic factors.


B There is a strong emphasis on low body weight justifying high self-esteem.
C Cultural stereotypes increase an individual's insight regarding his or her own personal weight
issues.
D The stereotypes identify the population at risk for developing eating disorders.

214. According to the family systems theory, family behavior characteristics associated with
anorexia include (select all that apply):

A Isolation between family members.


B Successful conflict resolutio
C Unclear boundaries between family members.
D Family members' preoccupation with food and eating.
E Individual autonomy.
C Unclear boundaries between family members.

215. The most common coexisting mental health issue associated with anorexia and bulimia is:

A Anxiety.
B Agoraphobia.
C Depression.
D Panic attacks.

216. A nurse is teaching a group of adolescents about the risk factors and complications of
anorexia nervosa. Which of the following complications should the nurse stress as the most
serious?

A Increased risk of mortality


B Depression
C Ineffective family relationships
D Ineffective coping skills

217. Which of the following objective data would the nurse expect to find in the client with
anorexia nervosa?

A A score of 13 on the Mini-Mental State Exam


B Feeling isolated and lonely
C Osteoporosis
D Preoccupation with food

218. The nursing diagnosis for a client with bulimia is Fluid Volume Deficit. Nursing interventions
specific to the fluid volume deficit include:

A Weighing the client after each meal.


B Monitoring the client for at least 1 hour after meals.
C Monitoring body temperature every 4 hours.
D Ensuring daily consumption of 1000 to 2000 mL of liquid.

219. Educational guidelines for family members of clients with eating disorders include:

A Expecting a full recovery within 6 months of starting treatment.


B Scheduling family activities that include food.
C Expressing love and affection both verbally and physically.
D Recognizing the client's need to have his or her behaviors controlled by family members.

220. The nurse observes a client admitted with anorexia nervosa doing repeated, vigorous sit-
ups. The most appropriate action from the nurse is to:

A Tell the client she cannot do exercises.


B Allow the client to complete the exercise routine.
C Take away the client's visitor privileges.
D Interrupt the behavior and offer to walk with the client.

221. The nurse is assessing a client with severe anorexia nervosa. Which of the following
physical findings should be immediately reported to the physician?

A Pulse rate of 102


B Blood pressure of 80/40mm Hg
C Amenorrhea
D Urine output of 50cc/hour
The nurse is presenting information at the community health fair about normal
defecation patterns across the lifespan. Which of the following factors would
not be part of the discussion?

Diet
Fluid intake and output
Medications
Gender

The elderly population is known to use laxatives with regularity. In advising an older
adult practicing this habit, the nurse would identify all of the following except: (Select all
that apply.)

*Consistent use of laxatives inhibits natural defecation reflexes, and is thought to


cause rather that cure constipation.
*Habitual use of laxatives eventually requires larger or stronger doses because
the effect is progressively reduced.
*Laxatives may interfere with fluid and electrolyte balance.
*Laxatives increase the absorption of certain vitamins.

The nurse encounters a 75-year-old in the emergency department, with complaints of


nausea, diarrhea, and anorexia. He has been evaluated, and it is determined that he
can be treated at home. In discussing the guidelines of managing diarrhea, the nurse
knows the client understands his care measures when he says:

"I will drink two glasses of water a day to prevent dehydration."


"I will drink tea when I get home."
"I will increase foods with fiber, like oatmeal."
"I will eat fried chicken for supper."

A client suffering with ulcerative colitis has discussed the need for a temporary
colostomy to rest the colon and help the healing process. The colostomy will be located
in the descending colon. The type of stool that the client can expect from this stoma is:

Liquid that cannot be regulated


Malodorous and mushy drainage
Increasingly solid
Liquid fecal drainage

After having a transverse colostomy constructed for colon cancer, discharge planning
for home care would include teaching about the ostomy appliance. Information
appropriate for this intervention would include:
*Instructing the client to report redness, swelling, fever, or pain at the site to the
physician for evaluation of infection
*Nothing can be done about the concerns of odor with the appliance.
*Ordering appliances through the client's health care provider
*The appliance will not be needed when traveling.

appliances are almost always worn throughout the day and when traveling.
At the local wellness fair, the nurse is asked to share information on having healthy
bowel life. Included in this area is the topic of having a healthy defecation. The nurse
should include which of the following information as appropriate action to follow?

Eliminating high-fiber foods in your diet


Defecating only once a day. Ignore any other urges.
Establishing a regular exercise regimen
Drinking four glasses of water a day

An 80-year-old client is in the emergency department. The client complains of diarrhea


and vomiting for the past two days. In assessing the client, it is noted that his skin is dry
and can be tented, he has lost eight pounds, and is itchy. Which NANDA diagnosis
would be most appropriate to use with this client in making his plan of care?

Risk for Deficient fluid volume related to prolonged diarrhea and vomiting
Risk for fluid volume excess related to prolonged diarrhea and vomiting
Risk for normal fluid volume related to prolonged diarrhea and vomiting
Risk for hidden fluid related to prolonged diarrhea and vomiting

When using a rectal tube in helping a client expel flatulence, the most appropriate
intervention to be followed by the nurse is

Have client in the supine position


Insert rectal tube, no lubrication needed
Leave tube in for one hour
Encourage the client to assume various positions in bed once the tube is
inserted.

A client has had a stroke, and can no longer move her bowels on her own accord. A
bladder-training program is to be established for her. Before beginning this program, the
client and her family members must understand what is involved with this care. Which of
the following would be the most appropriate directions or information to share?

Maintain the daily routine for six weeks


Only allow client to defecate once a day
Administer a cathartic suppository 30 minutes before the client's defection time
to stimulate peristalsis
Fluid intake, decreased fiber in diet, intake of hot drinks, and increased exercise all
influence one's ability to perform the action of defection on a regular basis.

The nurse has completed the administration of a cleaning enema for a client being
prepared for intestinal surgery. Complete documentation by the nurse of this event
includes all but which of the following assessments? (Select all that apply.)

Type of solution
Length of time solution retained
Relief of flatus and abdominal distention
Amount of return

Clients should be taught that repeatedly ignoring the sensation of needing to defecate
could result in which of the following?

Constipation
Diarrhea
Incontinence
Hemorrhoids

Which statement provides evidence that an older adult who is prone to constipation is in
need of further teaching?

"I need to drink one and a half to two quarts of liquids each day."
"I need to take a laxative such as Milk of Magnesia if I don't have a BM every day."
"If my bowel pattern changes on its own, I should call you."
"Eating my meals at regular times is likely to result in regular bowel movements."
"I need to take a laxative such as Milk of Magnesia if I don't have a BM every day."

A client is scheduled for a colonoscopy. The nurse will provide information to the client
about which type of enema?

Oil retention
Return flow
High, large volume
Low, small volume

The nurse is most likely to report which finding to the primary care provider for a client
who has an established colostomy?

The stoma extends 1/2 in. above the abdomen.


The skin under the appliance looks red briefly after removing the appliance.
The stoma color is a deep red-purple.
An ascending colostomy delivers liquid feces.
Which goal is the most appropriate for clients with diarrhea related to ingestion of an
antibiotic for an upper respiratory infection?

The client will wear a medical alert bracelet for antibiotic allergy.
The client will return to his or her previous fecal elimination pattern.
The client verbalizes the need to take an antidiarrheal medication prn.
The client will increase intake of insoluble fiber such as grains, rice, and cereals.

A client with a new stoma who has not had a bowel movement since surgery last week
reports feeling nauseous. What is the appropriate nursing action?

Prepare to irrigate the colostomy.


After assessing the stoma and surrounding skin, notify the surgeon.
Assess bowel sounds and administer antiemetic.
Administer a bulk-forming laxative, and encourage increased fluids and exercise.

The nurse assesses a client's abdomen several days after abdominal surgery. It is firm,
distended, and painful to palpate. The client reports feeling "bloated." The nurse
consults with the surgeon, who orders an enema. The nurse prepares to give what kind
of enema?

Soapsuds
Retention
Return flow
Oil retention

Which of the following is most likely to validate that a client is experiencing intestinal
bleeding?

Large quantities of fat mixed with pale yellow liquid stool


Brown, formed stools
Semisoft tar-colored stools
Narrow, pencil-shaped stool

Which nursing diagnoses is/are most applicable to a client with fecal incontinence?
Select all that apply.

Bowel Incontinence
Risk for Deficient Fluid Volume
Disturbed Body Image
Social Isolation
Risk for Impaired Skin Integrity
A student nurse (SN) is assigned to care for a client with a sigmoidostomy. The student
will assess which ostomy site?

Option 1
Option 2 Ascending Colon
Option 3 Transverse Colon
Option 4 Descending Colon
Option 5 Sigmoid Colon

The nurse is working with a client whose main diet is rice, eggs, and lean meat. It is
most important for the nurse to teach the client which of the following things about a diet
high in these foods? [Hint]

This diet needs more fluid intake to move it through GI tract.


No additional fiber or bulk is needed, as this diet is rich in it.
Eating this diet may result in the passage of watery stools.
This diet may produce a significant amount of bloating and flatus.

The nurse is administering a medication containing iron to a client. The nurse does
some teaching about the iron medication. Which of the following statements will the
nurse most need to make when instructing the client about taking iron? [Hint]

"Diarrhea may result from taking iron supplements."


"Iron may cause your stools to appear black in color."
"The urine may become brown when taking iron."
"You will need additional vitamin C when taking iron."

You are going to have a client collect a stool sample for a guaiac test, on three
consecutive stools. In order to avoid false positive results, you will instruct your client
about the foods to eat and foods to avoid for a day or two prior to the test. Which of the
following meals selected by the client would indicate they understood your instructions?
(Select ALL that apply)

sirloin steak, a garden salad, and bread


fried chicken, mashed potatoes, and cake
fresh fruit salad bowl and cookies
a hamburger and a bowl of chili
sirloin steak, a garden salad, and bread
Your assigned client, who has a history of heart disease, has a tendency to strain when
having a bowel movement. You instruct the client not to strain for which of the following
reasons? [Hint]

It increases intrathoracic pressure.


It can suddenly increase the blood pressure.
Straining reduces the amount of available oxygen.
The medial nerve is stimulated during straining.

You answer the call light of a hospitalized client who states they are having a lot of
flatulence and asks for a carbonated drink with a straw. Which of the following things
would be best for you to do to help this client reduce the amount of flatulence? [Hint]

Get the client a cola drink with a straw.


Offer the client some chewing gum.
Give the client a straw but offer a noncarbonated drink
Give the client a noncarbonated drink without a straw

You are instructing a client in the clinic about diphenoxylate (Lomotil), which the
physician has just ordered for diarrhea. Which of the following instructions would be
most important to give a client who is on this medication? [Hint]

Take the medication with food.


Increase the dose until results are satisfactory.
Do not give to children under the age of 12.
Do not take before driving or using running machinery.

How high above the rectum should the nurse giving a high-cleansing enema hold the
enema solution? [Hint]

5 to 7 inches
8 to 9 inches
10 to 12 inches
14 to 16 inches

The colon's three main functions include which one of the following in addition to that of
fecal elimination? [Hint]

to eliminate excess fluid


to excrete excess electrolytes with the fecal matter
excretion of a substance that sloughs off dead cells
to excrete mucus and protect the intestine from bacteria
The colon's main functions are the absorption of water and electrolytes, the mucal
protection of the intestinal wall, and fecal elimination.
Which of the following factors would be most likely to decrease the movement of chyme
in the intestine? [Hint]

depression
anxiety
exercise
anger

When checking the stool of a client, you notice there is a whitish discoloration and some
white specks in the stool. Which of the following medications do you suspect the client
is taking? [Hint]

Non-enteric coated aspirin


Antacids
New-generation cephalosporins
Antimalarials

The nurse has assessed that a patients stool has changed from brown to dark black
and sticky. The nurse suspects:

a.blockage of the bile duct.


b.blockage of the pancreatic duct.
c.recent excessive intake of milk products.
d.presence of occult blood.

The nurse has documented that a patient has had two episodes of steatorrhea, which
means that the character of the stool is:

a.hard and clay colored


b.frothy and foul smelling.
c.very liquid and streaked with blood.
d.soft and filled with mucus.

The nurse should plan interventions to combat constipation in a patient:

a.being treated for diabetes mellitus.


b.who has a routine order for Metamucil.
c.who just completed barium studies of the bowel.
d.with orders to ambulate with assistance.
A patient who is undergoing barium radiograph studies is more prone to constipation
than are the other patients.A patient calls the nurse at the health clinic and reports that
since his trip to Mexico, he has been experiencing diarrhea. The nurse suggests he try
the antidiarrheal drug:
a.docusate sodium (Colace).
b.loperamide (Imodium).
c.polycarbophil (FiberCon).
d.senna (Senokot).

An elderly resident in a long-term care facility has experienced constant diarrhea for 3
days and is now exhibiting signs and symptoms of dehydration. The nurse initiates an
intervention to offer small amounts of ________ frequently.
ANS: C

The patient may be developing electrolyte imbalance because of the diarrhea, so the
best fluid source to offer is Gatorade, which has sodium and potassium. This should be
offered 1 to 2 ounces at a time.
A nurse has performed abdominal assessments on four patients. After reviewing the
findings, the nurse is least concerned about problems with bowel elimination for the
patient with abdomen _____ bowel sounds in all four quadrants.
a.
nondistended, firm, with hypoactive
b.
nondistended, soft, with active
c.
distended, firm, with hypoactive
d.
distended, soft, with hyperactive

Normal abdominal assessment data are an abdomen that is soft and nondistended and
that has active bowel sounds in all four quadrants.
A nurse is monitoring bowel elimination of a patient who has a history of constipation.
The nurse implements measures to assist with bowel elimination if the patient has not
had a bowel movement within how many days?
a.5
b.3
c.2
d.1

If bowel evacuation has not occurred within 3 days, measures should be taken to assist
the patient.
A nurse is preparing a cleansing enema for an adult patient who is constipated and has
not responded to laxative use. Before giving the enema, the nurse should:
a.cool the solution to 70 F.
b.warm the solution in the microwave.
c.keep the solution at room temperature.
d.warm the solution to 105 F.

A patient scheduled for bowel surgery has an order to receive enemas until clear. The
nurse is aware that no more than three enemas should be given because:
a.
repeated enemas may cause more flatus.
b.
the patient may develop an irritated rectum.
c.
repeated enemas may cause electrolyte imbalance.
d.
the patient may develop severe diarrhea.

. A nurse is digitally removing a fecal impaction from a patient. The nurse should stop
the procedure immediately and take corrective action if the patients:

a.blood pressure increases from 110/84 to 118/88 mm Hg.


b.pulse rate decreases from 78 to 52 beats/min.
c.respiratory rate increases from 16 to 24 breaths/min.
d.temperature increases from 98.8 F to 99.0 F.

Stimulation of the sphincter may cause a vagal response as evidenced by bradycardia.


A nurse is reinforcing teaching with a patient who will begin a bowel training program.
An intervention this program does not include is:

a.regularly scheduled time for toileting.


b.fluid intake of at least 2500 mL daily.
c.use of a suppository.
d.use of an enema.

The nurse is caring for an anxious patient who is scheduled for surgery for colostomy
placement. While the nurse is talking to the patient, the patient states, I am so scared.
The nurses most supportive response would be:
a.Surgeries like yours are very safe.
b.What about your colostomy scares you?
c.Why are you scared?
d.Sounds like someone has been telling you horror stories.

.
The patient asks the nurse how an ileostomy differs from a colostomy. The most
informative response by the nurse would be that a(n):

a.ileostomy is performed to remove stool from the colon, whereas a colostomy is the
removal of lower portions of bowel, diverting intestinal contents.
b.ileostomy has effluent that is more formed, whereas a colostomy has effluent that is
liquid.
c.colostomy is an opening into the colon, whereas an ileostomy is an opening at
the ileum.
d.ileostomy requires irrigating, whereas a colostomy requires catheterizing.

. The nurse points out that age-related changes in the intestinal tract are relatively
insignificant. The changes include: (Select all that apply.)

a.atrophy of the villi in the small intestine.


b.increased incidence of hemorrhoids.
c.decreased absorption of fats and vitamin B12.
d.creation of excessive flatus.
e.decreased motility in the large intestine.

A client asks the RN why it is more difficult to use a bedpan for defecating than sitting
on the toilet. Which of the following is the best response?

1. The sitting position decreases the contractions of the muscles of the pelvic floor.
2. The sitting position increases the downward pressure on the rectum, making it
easier to pass stool.
3. The sitting position increases the pressure within the abdomen.
4. The sitting position inhibits the urge to urinate, allowing one to defecate.

A client asks the nurse why expelled flatus is foul-smelling. What should the nurse
respond?

1. The actions of microorganisms within the GI tract are responsible for the odor.
2. The clients emotions are causing the gas formation.
3. The sensory nerves in the rectum are being stimulated.
4. The client has swallowed too much air while eating.

.
Which client would benefit from interventions to decrease the risk of developing
constipation?

1. An adult who is on bed rest


2. An infant who is breast-fed
3. A school-age child at recess
4. A toddler who is now walking
The nurse is taking care of a client who states that he ignores the urge to defecate
when he is at work. Which of the following responses by the nurse would explain why
this practice should be changed?

1. If you continue to ignore the urge to defecate, the urge is ultimately lost.
2. It is best to suppress the urge than suffer embarrassment at work.
3. This is a common practice, and it will strengthen the reflex later.
4. You will get the urge later; dont worry.

What activity will the nurse perform when assessing a clients fecal elimination status?

1. Obtain a nursing history


2. Interpret results of diagnostic tests
3. Perform a physical examination
4. Goal setting with the client

The nurse would instruct a client with frequent bouts of diarrhea to:

1. Change the daily routine.


2. Decrease fluid consumption.
3. Increase fiber in the diet.
4. Note the precipitating event.

Which client behavior would indicate that teaching to prevent constipation was
effective?

1. The client continues to ask for his pain medication.


2. The client decreases his fluid consumption.
3. The client refuses to eat the bran flakes on his tray.
4. The client walks around the unit several times a day

.
What nursing diagnosis would the nurse select as appropriate to address bowel
evacuation for a client who is on bed rest?

1. Bowel Incontinence
2. Constipation
3. Diarrhea
4. Disturbed Body Image

What goal would be applicable for a client who is experiencing diarrhea?


1. Client will defecate regularly.
2. Client will increase the amount of sugar in the diet.
3. Client will limit fluid intake.
4. Client will regain normal stool consistency.
Which assessment technique will the nurse use first when examining a client with a
fecal elimination problem?

1. Auscultation
2. Inspection
3. Palpation
4. Percussion

Which assessment data would indicate compromised gastrointestinal function?

1. Bowel sounds active in all four quadrants


2. Clay color stool
3. Increased appetite
4. Semisolid and moist stool

What should the nurse instruct a client prior to administering a cleansing enema?

1. Hold the solution for a short time.


2. Lie in the left lateral position.
3. Lie in the right lateral position.
4. Take fast breaths through the nose.

A client is prescribed a saline enema. Since this solution is hypertonic, the nurse would
expect the enema to cause which action?

1. Exerts osmotic pressure and draws fluid from the interstitial space into the
colon
2. Exerts a lower osmotic pressure than the surrounding interstitial fluid
3. Exerts the same osmotic pressure as the interstitial fluid surrounding the colon
4. Stimulates peristalsis by increasing the volume in the colon and irritating the colon

A client experiencing hard, dry feces is scheduled for an enema. The nurse realizes that
the type of solution that would be the best for the client would be:
Standard Text: Select all that apply.

1. Hypertonic.
2. Hypotonic.
3. Soapsuds.
4. Oil retention.
5. Isotonic.
A client with DKA is being treated in the ED. What would the nurse suspect?
a. Comatose state
b. Decreased Urine Output
c. Increased respiration and an increase in pH.
d. Elevated blood glucose level and low plasma bicarbonate level.

Which electrolyte replacement should the nurse anticipate being ordered by the health-
care provider in the client diagnosed with DKA who has just been admitted to the ICU?
a. Glucose
b. Potassium
c. Calcium
d. Sodium Potassium

The nurse is monitoring a client newly diagnosed with diabetes mellitus for signs of
complications. Which sign, if exhibited in the client, would indicate hyperglycemia?
a. polyuria
b. diaphoresis
c. hypertension
d. increased pulse rate

The nurse performs a physical assessment on a client with type 2 diabetes mellitus.
Findings include a fasting blood glucose level of 120 mg/dl, temp of 101 degrees, pulse
of 88 beats/min, respirations of 22 breaths/minute, and blood pressure of 100/72 mm
Hg. Which finding would be of MOST concern to the nurse?
a. pulse
b. respiration
c. temperature
d. blood pressure

What disease/disorder can lead to anemia?


a. heart disease
b. peripheral vascular disease
c. renal disease
d. lung cancer

What are some modifiable risk factors to diabetes select all that apply?
a. obesity
b. sedentary life style
c. poor nutrition
d. hypotension
contribute to the development of high blood sugar and increase your chances of
becoming insulin resistant due to the high amount of sugar that your body is used to.
A nurse is caring for a client admitted to the emergency department with diabetic
ketoacidosis (DKA). In the acute phase, the priority nursing action is to prepare to:
A. Correct the acidosis
B. Administer 5% dextrose intravenously
C. Administer regular insulin intravenously
D. Apply a monitor for an electrocardiogram

The nurse is caring for a patient with liver cirrhosis who developed ascites and requires
paracentesis. Relief of which symptom indicated that the paracentesis was effective?
A. Pruritus
B. Dyspnea
C. Jaundice
D. Peripheral Neuropathy

"The nurse caring for a 54-year-old patient hospitalized with diabetes mellitus would
look for which of the following laboratory test results to obtain information on the
patient's past glucose control?
a. prealbumin level
b. urine ketone level
c. fasting glucose level
d. glycosylated hemoglobin level

"The nurse is teaching a community class to people with Type 2 diabetes mellitus.
Which explanation would explain the development of Type 2 diabetes?
a. The islet cells in the pancreas stop producing insulin.
b. The client eats too many foods that are high in sugar.
c. The pituitary gland does not produce vasopression.
d. The cells become resistant to the circulating insulin.

The nurse is planning to teach a client with GERD about substances to avoid. Which
items should the nurse include on this list? Select all that apply.
a. coffee
b. chocolate
c. peppermint
d. nonfat milk
e. fried chicken
f. scrambled eggs
The nurse is reviewing the record of a client with Crohn's disease. Which stool
characteristic should the nurse expect to note documented in the client's record?
a. Diarrhea
b. Chronic constipation
c. Constipation alternating with diarrhea
d. Stool constantly oozing from the rectum

.
A nurse is treating a client for dehydration. The nurse expects to see which of the
following symptoms?
a) Rapid weak pulse, tachypnea, hypotension
b) Dry mucous membranes, increased hematocrit, decreased urine specific gravity
c) Rapid strong pulse, extreme thirst, decreased urine output
d) Dry skin, emaciation, hypertension
e) Dry mouth, bradypnea, hypotension

Which patient in a hospital is most likely to develop a urinary tract infection?


a) 55 y/o woman treated for a fractured wrist
b) 20 y/o postpartum woman who had a vaginal delivery
c) 55 y/o postpartum woman who had a cesarean section
d) 20 y/o woman who is sexually active, has multiple partners, and uses condoms with a
spermicidal lubricant
e) 55 y/o man with BPH

A 27 week gestation patient is admitted to the ED with complaints of severe nausea and
vomiting for the last 48 hours. Which of the following would be appropriate nursing
interventions for this patient? Select all that apply
a. Encourage immediate use of clear liquid diet to replace fluid deficit
b. Monitor VS, skin turgor and intake/output
c. Administer antiemetic medications, as ordered
d. Place patient in prone position
e. Monitor amount, color and specific gravity of urine

Which of the following patients is at highest risk for GERD?


a. full term (40wk gestation) obese mother admitted to L&D
b. 95 year old male admitted for malnutrition
c. A 5 year old in surgery for a tibial fracture
d. A 55 year old female patient with A-FIB

A pregnant woman complains of heartburn. As her nurse you inform her of the following
preventative measures that can be used to prevent GERD: Select all that apply.
a. Eat smaller more frequent meals
b. Eat meals right before bedtime
c. Elevate the head of the bed
d. Avoid smoking and alcohol
e. Wear tight clothing
A woman at 9 weeks gestation complains of frequent urination occurring 9-12 times a
day. The nurse informs the patient:

a. This finding is abnormal since you should urinate 4-6 times a day
b. You should lower your fluid intake to 1500 ml a day to decrease urination
c. It is normal to urinate more frequently in the first trimester since the enlarging
uterus is pressing on the badder
d. This symptom should not occur until the second trimester when the baby is larger.
We will have to double check your due date using an ultrasound

As a nurse, you are teaching a 17 year old who is 26 weeks pregnant about the
importance of proper nutrition for her and her baby. The patient states she doesn't have
any money to buy healthy food. Without proper nutrition, the baby is at risk for: Select all
that apply

a. Low birth weight


b. Poor dentition
c. Learning disabilities
d. Blindness

A woman at 32 weeks gestation complains of an inability to control her bladder and


asks you why she has urinary incontinence. What would be best response?
a. "I would recommend buying some depends until you give birth"
b. "Pregnancy is a risk factor for urinary incontinence because of the weight of
the expanding uterus on the bladder."
c. "Sometimes prenatal vitamins will cause urinary incontinence"
d. "This is an abnormal condition. We should contact your physician"

A patient comes into the doctor's office complaining of nausea and vomiting and is
showing signs of dehydration. The Nurse is worried about an electrolyte imbalance. The
doctor orders the patient's serum electrolyte levels checked. the nurse notes that the
Potassium levels are low. The nurse knows this is a problem because the role of
potassium in the body is what? Select all that apply.
a. Transmitting nerve and other electrical impulses
b. Regulating cardiac impulse transmission and muscle contraction
c. Regulating ECF balance and vascular volume
d. Maintaining cardiac pacemaker (automaticity)
e. Skeletal and smooth muscle function

A pregnant patient who recently discovered that she is pregnant has come to the
doctor's office to determine how far along she was in the pregnancy. The fetus has been
estimated to be in its 4th week. The nurse knows that the first trimester increases the
mother's risk for FVD. In teaching the patient about maintaining appropriate fluid volume
what should the nurse include? Select all that apply.
a. Avoid caffeinated drinks
b. Drink diet drinks regularly
c. Consume 11-12 cups of fluids a day
d. Exercise outside during the hottest part of the day but only for 30 min at a time.
e. Avoid alcohol

Patient admitted with a chief complaint of vomiting and nausea, patient is 12 weeks
gestation with 1 gravida. Patient states "I have thrown up 4 times in the last 8 hours and
I have felt nauseous for the last few days." What nursing diagnosis best describes this
patient's condition?
a. fluid volume excess
b. decreased cardiac output
c. fluid volume deficit
d. activity intolerance

Patient is admitted to the ED with a UTI and is 22 weeks gestation. The patient
complains of abdominal pain 7/10 and has not urinated in the last 24 hr. What are the
possible nursing diagnoses for this patient? Select all that apply.
a. Activity intolerance
b. Excess fluid volume
c. Acute pain
d. decreased cardiac output
e. impaired urinary elimination

A postoperative client has been placed on a clear liquid diet. The nurse should provide
the client with which item(s) that are allowed to be consumed on this diet? Select all that
apply.
a. Broth
b. Coffee
c. Gelatin
d. Pudding
e. Vegetable juice
f. Pureed vegetables

A client who has a history of Crohn's disease is admitted to the hospital with fever,
diarrhea, cramping, abdominal pain, and weight loss. The nurse should monitor the
client for:
a. Hyperalbuminemia
b. Thrombocytopenia
c. Hypokalemia
d. Hypercalcemia

A client with Type 1 diabetes mellitus has diabetic ketoacidosis. Which of the following
findings has the greatest effect on fluid loss?
a. Hypotension
b. Decreased serum potassium level
c. Rapid, deep respirations
d. Warm, dry skin

The nurse is assessing a client with a 4-day history of diarrhea. The nurse assesses the
client for which of the following? Select all that apply.
a. Bradycardia
b. Hyperglycemia
c. Hypertension
d. Lethargy
e. Skin tenting
f. Tachycardia

A client arrives at the emergency department with complaints of low abdominal pain and
hematuria. The client is afebrile. The nurse next assesses the client to determine a
history of which condition?
a. Pyelonephritis
b. Glomerulonephritis
c. Trauma to the bladder or abdomen
d. Renal cancer in the client's family

The nurse is teaching a new prenatal client about her iron deficiency anemia during
pregnancy. Which statement indicates that the client needs further instruction about her
anemia?
a. "I will need to take iron supplements now."
b. "I may have anemia because my family is of Asian descent."
c. "I am considered anemic if my hemoglobin is below 11g/dL (110g/L)."
d. "The anemia increases the workload on my heart."

The nurse is providing instructions regarding treatment of hemorrhoids to a client who is


in the second trimester of pregnancy. Which statement by the client indicates a need for
further instruction?
a. "I should avoid straining during bowel movement."
b. "I can gently replace the hemorrhoids into the rectum."
c. "I can apply ice packs to the hemorrhoids to reduce the swelling."
d. "I should apply heat packs to the hemorrhoids to help them shrink."

The clinic nurse is providing instructions to a pregnant client regarding measures that
assist in alleviating heartburn. Which statement by the client indicates an understanding
of the instructions?
a. "I should avoid between-meal snacks."
b. "I should lie down for an hour after eating."
c. "I should use spices for cooking rather than using salt."
d. "I should avoid eating foods that produce gas and fatty foods."
A woman who is two months pregnant complains of nausea after taking her prenatal
vitamin. The nurse should instruct the patient to do which of the following to alleviate
this side effect? Select all that apply.
a. Crush the vitamin and mix it with orange juice
b. Take the vitamin at night
c. Take the vitamin in the morning
d. Take the vitamin with food
e. Try the generic brand

A client with very dry mouth, skin and mucous membranes is diagnosed of having
dehydration. Which intervention should the nurse perform when caring for a client
diagnosed with fluid volume deficit?
a. Assessing urinary intake and output
b. Obtaining the client's weight weekly at different times of the day
c. Monitoring arterial blood gas (ABG) results
d. Maintaining I.V. therapy at the keep-vein-open rate

What laboratory test is a common measure of renal function?


a. CBC
b. BUN/CREATININE
c. Glucose
d. ALT- alanine amino transferase

The red blood cell (RBC) production in a client with chronic renal failure (CRF) has
decreased. The nurse should monitor this client for:
a. nausea and vomiting
b. dyspnea and cyanosis
c. fatigue and weakness
d. thrush and circumoral pallor

Which client is at the highest risk of being admitted to the Emergency Department with
severe nausea and vomiting? Select all that apply.
a. A 47-year-old with a 3-hour history of chest pressure
b. A 61-year-old reporting sudden onset of vertigo
c. A 72-year-old with an asthma exacerbation
d. A 23-year-old who sustained a head injury in a fall
e. A 19-year-old who is 6 weeks pregnant

The client also reports nausea and vomiting and appears pale, diaphoretic, and
anxious. The physician tentatively diagnosed renal calculi and orders flat-plate
abdominal X-rays. Renal calculi can form anywhere in the urinary tract. What is their
most common formation site?
a. Kidney
b. Ureter
c. Bladder
d. Urethra
Neonates of mothers with diabetes are at risk for which complication following birth?
a. Atelectasis
b. Microcephaly
c. Pneumothorax
d. Macrosomia

A nurse is assessing a client with a urinary tract infection who takes an antihypertensive
drug. The nurse reviews the client's urinalysis results pH 6.8, RBC 3 per high power
field, color-yellow, specific gravity 1.030 . The nurse should:
a. Encourage the client to increase fluid intake.
b. Withhold the next dose of antihypertensive medication.
c. Restrict the client's sodium intake
d. Encourage the client to eat at least half of a banana per day

A pregnant patient is admitted with excessive thirst, increased urination, & has a
medical diagnosis of diabetes insipidus. The nurse chooses which of the following
nursing diagnoses as most appropriate?
a. Risk for Imbalanced Fluid Volume
b. Excess Fluid Volume
c. Imbalanced Nutrition
d. Ineffective Tissue Perfusion

A client is 33 weeks pregnant and has had diabetes since she was 21. When checking
her fasting blood sugar level, which values indicate the client's disease was controlled.
a. 45 mg/dl
b. 85 mg/dl
c. 120 mg/dl
d. 136 mg/dl

Which of the following statements best describes hyperemesis gravidarum?


a. Severe anemia leading to electrolyte, metabolic, and nutritional imbalances in the
absence of other medical problems.
b. Severe nausea and vomiting leading to electrolyte, metabolic, and nutritional
imbalances in the absence of other medical problems.
c. Loss of appetite and continuous vomiting that commonly results in dehydration and
ultimately decreasing maternal nutrients
d. Severe nausea and diarrhea that can cause gastrointestinal irritation and possibly
internal bleeding

A female client 16 weeks gestational comes into the clinic where a nurse is working.
The physician examines the client and determines that she has developed cystitis.
Which of the following are the most commonly assessed findings in cystitis that this
client would present with? Select all that apply
a. urgency dysuria
b. Dehydration
c. Nocturia
d. suprapubic pain
e. flank pain

The nurse is inserting an indwelling catheter into a female client. Which interventions
should be implemented? Rank in the order of performance.
a. Explain the procedure to the client.
b. Set up the sterile field.
c. Inflate the catheter bulb.
d. Place absorbent pads under the client.
e. Clean the perineum from clean to dirty with Betadine.
In order of performance: A, D, B, C, E.

The nurse is caring for a pregnant client diagnosed with acute pyelonephritis. Which
scientific rationale supports the client being hospitalized for this condition?
a. The client must be treated aggressively to prevent maternal/fetal
complications.
b. The nurse can force the client to drink fluids and avoid nausea and vomiting.
c. The client will be dehydrated and there won't be sufficient blood flow to the baby.
d. Pregnant clients historically are afraid to take the antibiotics as ordered.

An anemic woman, 34 weeks pregnant, is taking iron supplements. Which of the


following foods should the patient take with iron?
a. Carrots
b. Fortified milk
c. Orange juice
d. Yogurt

A pregnant patient in her first trimester comes to the clinic concerned because she has
urinary frequency and white vaginal discharge. The nurse's appropriate response to the
woman is:
a. Are you practicing a polygamous relationship?
b. This is an abnormal finding and could be associated with cervical cancer.
c. You don't have to worry, both are normal discomforts of pregnancy.
d. You might have a urinary tract infection and must see a doctor for antibiotic
treatment.

A postpartum client is diagnosed with cystitis. The nurse should plan for which priority
nursing action in the care of the client?
a. Providing sitz baths
b. Encouraging fluid intake
c. Placing ice on the perineum
d. Monitoring hemoglobin and hematocrit levels
The nurse is caring for a postpartum patient. The nurse observes which of the following
signs that make you concerned for endometritis? Select all that apply
a. Constipation
b. Cramping
c. Hemorrhoids
d. Purulent, foul-smelling lochia
e. Uterine tenderness

A patient is having trouble adjusting to breastfeeding and disagrees with her husband
on the importance of breastfeeding compared to formula. Which member of the
healthcare team should the nurse consult?
a. Charge nurse
b. Lactation consultant
c. Physician
d. social worker

After instruction of primigravid client at 8 weeks gestation about measure to overcome


early morning nausea and vomiting, which of the following client statements indicates
the need for additional teaching?
a. "I'll eat dry crackers or toast before arising in the morning".
b. "I'll drink adequate fluid separate from my meals or snacks".
c. "I 'll eat two large meals daily with frequent protein snacks"
d. "I'll snack on a small amount of carbohydrates throughout the day."

A multigravid client thought to be at 14 weeks gestation report that she is experiencing


such severe morning sickness that" she has not been able to keep anything down for a
week."The nurse should assess for signs and symptoms of which of the following?
a. Hypercalcemia.
b. Hyperbilirubinemia.
c. Hypokalemia.
d. Hyperglycemia.
If the patient was instructed to avoid foods that may have a laxative effect, the nurse
would advise the patient to avoid which of the following foods?
A) Chinese
B) Alcohol
C) Eggs
D) Pasta

All the foods listed as such alcohol have a constipating effect


Which of the following is a true statement about the effects of medication on bowel
illumination?
A) Diarrhea commonly occurs with amoxicillin clavulanate use
B) Anticoagulants cause a white discoloration of the stool
C) Narcotic analgesics increased Gastrointestinal mobility
D) Iron salts in pair digestion and cause a green store

Anticoagulants may result in the stool having a pink to red to black appearance,
whereas iron salts also cause a black stool. Narcotic analgesics decrease gastric
mobility.
Mr. Jay has a fecal impaction. The nurse correctly administers an oil-retention Enema
by doing which of the following?
A) Administering a large volume solution 500 to 1000 ml
B) Mixing milk and molasses and equal parts for an enema
C) Instructing the patient to retain the enema for at least 30 seconds
D) Administering the enema while the patient is sitting on a toilet

The usual amount of solution administered with a retention Enema is 150 to 200 mL for
an adult. The milk and molasses mixture is a carminative enema That helps to expel
flats, As does the Harrison flush procedure
As the nurse prepares to assist Mrs. P with her newly created Ileostomy, She is aware
of which of the following?
A) An appliance will not be required on the continual basis
B) The size of the stoma stabilizes within two weeks
C) Irrigation is necessary for regulation
D) Fecal drainage will be liquid

And appliance is usually required on a continual basis because the fecal drainage is
liquid. Stomas size usually stabilizes within 4 to 6 weeks, and Ileostomy Irrigation is not
necessary because fecal matter is liquid
Which class of laxative acts by causing the stool to absorb water and swell?
A)Bulk-forming
B)Emollient
C)Lubricant
D)Stimulant
Emollients Lubricate the stool; Lubricants soften the stool, making it easier to pass: and
Stimulants promotes peristalsis by irritating the intestinal mucosa or stimulating nerve
ending in the intestinal wall
Mr. T is nervous about a colonoscopy scheduled for tomorrow. The nurse describes the
test by explaining that it allows which of the following?
A) Visual examination of the esophagus and stomach
B) Visual examination of the large intestine
C) Radiographic examination of the large intestine
D) Fluoroscopic examination of the small intestine

An esophagogastroduodenoscopy Allows visual examination of the esophagus and


stomach. The radiographic examination of the large intestine refers to a barium enema,
and a fluoroscopic Examination of the small intestines refers to an upper gastrointestinal
series
A bowel training program includes which of the following?
A) Using a diet that is low in bulk
B) Decreasing fluid intake to 1000 mL
C) Administering an enema once a day to stimulate peristalsis
D) Allowing ample time for Evacuation

For a bowel training program to be effective, the patient must have ample time for
evacuation usually 20 to 30 minutes. Fluid intake is increased to 2500 to 3000 mL, food
high in bulk is recommended as part of the program: and a daily enema is not
administered in bowel training program. A cathartic Suppository maybe use 30 minutes
before the patients usually defecation time to stimulate peristalsis
Your patient complains of an excessive flatulence. When reviewing your patients dietary
intake, which foods, if eaten regularly, would you identify as possibly responsible?
A) Meet
B) Cauliflower
C) Potatoes
D) Ice cream

Cauliflower is a gas producing food that relates in flatulence


A barium Enema should be done before an upper gastrointestinal series because of
which of the following?
A) Retained barium may cloud the colon
B)barium Can cause lower Gastrointestinal bleeding
C) The physicians orders are in that sequence
D)barium Is absorbed readily in the lower intestine
The barium And I'm should always perceive the upper gastrointestinal series because
retained barium from the latter may take several days to pass through the
gastrointestinal tract and may cloud anatomic detail on the barium enema studies
Nurses should recommend avoiding the habitual use of laxatives. Which of the following
is the rationale for this?
A) They will cause a fecal impaction
B) They will cause chronic constipation
C) They change the pH of the Gastrointestinal track
D) They inhibit the intestinal enzymes

Habitual use of laxatives is the most common cause of chronic constipation


When explaining the action of a hyper tonic solution enema, the nurse incorporates
which of the following as the basis for action?
A) bowel mucosa irritations
B) Diffusion of water out of colon
C) Osmosis of water into colon
D) Softening of fecal contents

Hypertonic solutions draw water into the colon a By osmosis that's stimulating the
defecation reflex. Orrills solutions soften fecal contents, and soup solutions distend the
intestines and irritate the bowel mucosa
Which of the following are included in the nursing plan of care to prevent adverse
effects when caring for patients with a nasogastric tube in place for gastric
decompression's? Select all that apply.
A) Irrigation with Saline
B) Measure the length of exposed tube
C) Measure the pH of the aspirated tube contents
D) Administer frequent oral hygiene
A,B,C,D

After checking placement, NGT should be Irrigated with 30 to 60 mL of normal saline to


maintain patency. The frequency is determined by facility policy, medical order, and
nursing judgment. The length of the exposed tubes should be measured after insertion
and routinely thereafter, as part of the assessment to verify placement and ensure the
tube has not dislodged. Measuring the pH of aspirated two contents is one way to
validate to placement in the intestinal tract. The other methods is to visually Assess
aspirate to confirm gastric contents. Patients with NGTs Often experience discomfort
related to irritation to nasal and throat mucosa And drying of the mucous membranes.
Frequent oral hygiene should be administered as well as applying lubricant to the lips
During removal of a fecal impaction, which of the following could occur Because of
vaginal stimulation?
A) Bradycardia
B)Atelectasid
C) Tachycardia
D) Cardiac tamponade
Removing a fecal impaction manually may result in stimulation of the vaginal nerve and
resulting bradycardia
Which of the following would be a common nursing diagnosis for patient with an
ileostomy?
A) Disturbed body image
B) Constipation
C) Delayed growth and development
D) Excess fluid volume

Constipation does not occur with the Ileostomy because the drainage is liquid. Growth
and development are not affected by the formation of an Ileostomy. Excess fluid Byam
is unlikely to occur because the drainage is liquid and probably continual
Which of the following is an appropriate nursing action to promote regular bowel habits?
A) Encourage the patient to avoid moving his bowels until a certain time of day
B) Encourage the patient to avoid excess fluid intake and too much fiber
C) Avoid strenuous exercise to limit stress on the abdominal muscles and impair
peristalsis
D) Assisting the patient to a normal position as possible to defecate
The nurse knows that most nutrients are absorbed in which portion of the digestive
tract?

a. Stomach

b. Duodenum

c. Ileum

d. Cecum

The nurse would expect the least formed stool to be present in which portion of the
digestive tract?

a. Ascending

b. Descending

c. Transverse

d. Sigmoid

Which of the following is not a function of the large intestine?

a. Absorbing nutrients

b. Absorbing water

c. Secreting bicarbonate

d. Eliminating waste

The nurse is caring for a patient who is confined to the bed. The nurse asks the patient
if he needs to have a bowel movement 30 minutes after eating a meal because

a. The digested food needs to make room for recently ingested food.

b. Mastication triggers the digestive system to begin peristalsis.

c. The smell of bowel elimination in the room would deter the patient from eating.

d. More ancillary staff members are available after meal times.


A nurse is assisting a patient in making dietary choices that promote healthy bowel
elimination. Which menu option should the nurse recommend?

a. Grape and walnut chicken salad sandwich on whole wheat bread

b. Broccoli and cheese soup with potato bread

c. Dinner salad topped with hard-boiled eggs, cheese, and fat-free dressing

d. Turkey and mashed potatoes with brown gravy

A patient informs the nurse that she was using laxatives three times daily to lose weight.
After stopping use of the laxative, the patient had difficulty with constipation and
wonders if she needs to take laxatives again. The nurse educates the patient that

a. Long-term laxative use causes the bowel to become less responsive to stimuli,
and constipation may occur.

b. Laxatives can cause trauma to the intestinal lining and scarring may result, leading to
decreased peristalsis.

c. Natural laxatives such as mineral oil are safer than chemical laxatives for relieving
constipation.

d. Laxatives cause the body to become malnourished, so when the patient begins
eating again, the body absorbs all of the food, and no waste products are produced.

A patient with a hip fracture is having difficulty defecating into a bed pan while lying in
bed. Which action by the nurse would assist the patient in having a successful bowel
movement?

a. Administering laxatives to the patient

b. Raising the head of the bed

c. Preparing to administer a barium enema

d. Withholding narcotic pain medication

Laxatives would not give the patient control over bowel movements. A barium enema is
a diagnostic test, not an intervention to promote defecation. Pain relief measures should
be given; however, preventative action should be taken to prevent constipation.
Which patient is most at risk for increased peristalsis?

a. A 5-year-old child who ignores the urge to defecate owing to embarrassment


b. A 21-year-old patient with three final examinations on the same day

c. A 40-year-old woman with major depressive disorder

d. An 80-year-old man in an assisted-living environment

A patient expresses concerns over having black stool. The fecal occult test is negative.
Which response by the nurse is most appropriate?

a. This is probably a false negative; we should rerun the test.

b. Do you take iron supplements?

c. You should schedule a colonoscopy as soon as possible.

d. Sometimes severe stress can alter stool color.

Which physiological change can cause a paralytic ileus?

a. Chronic cathartic abuse

b. Surgery for Crohns disease and anesthesia

c. Suppression of hydrochloric acid from medication

d. Fecal impaction

Fecal impactions occur in which portion of the colon?

a. Ascending

b. Descending

c. Transverse

d. Rectum

The nurse provides knows that a bowel elimination schedule would be most beneficial in
the plan of care for which patient?

a. A 40-year-old patient with an ileostomy

b. A 25-year-old patient with Crohns disease

c. A 30-year-old patient with C. difficile


d. A 70-year-old patient with stool incontinence

Which nursing intervention is most effective in promoting normal defecation for a patient
who has muscle weakness in the legs that prevents ambulation?

a. Elevate the head of the bed 45 degrees 60 minutes after breakfast.

b. Use a mobility device to place the patient on a bedside commode.

c. Give the patient a pillow to brace against the abdomen while bearing down.

d. Administer a soap suds enema every 2 hours.

The nurse is devising a plan of care for a patient with the nursing diagnosis of
Constipation related to opioid use. Which of the following outcomes would the nurse
evaluate as successful for the patient to establish normal defecation?

a. The patient reports eliminating a soft, formed stool.

b. The patient has quit taking opioid pain medication.

c. The patients lower left quadrant is tender to the touch.

d. The nurse hears bowel sounds present in all four quadrants.

15. The nurse is emptying an ileostomy pouch for a patient. Which assessment finding
would the nurse report immediately?

a. Liquid consistency of stool

b. Presence of blood in the stool

c. Noxious odor from the stool

d. Continuous output from the stoma

The nurse would anticipate which diagnostic examination for a patient with black tarry
stools?

a. Ultrasound

b. Barium enema

c. Upper endoscopy
d. Flexible sigmoidoscopy

The nurse has attempted to administer a tap water enema for a patient with fecal
impaction with no success. What is the next priority nursing action?

a. Preparing the patient for a second tap water enema

b. Donning gloves for digital removal of the stool

c. Positioning the patient on the left side

d. Inserting a rectal tube

The nurse should question which order?

a. A normal saline enema to be repeated every 4 hours until stool is produced

b. A hypertonic solution enema with a patient with fluid volume excess

c. A Kayexalate enema for a patient with hypokalemia

d. An oil retention enema for a patient using mineral oil laxatives

The nurse is preparing to perform a fecal occult blood test. The nurse plans to properly
perform the examination by

a. Applying liberal amounts of stool to the guaiac paper.

b. Testing the quality control section before collecting the specimen section.

c. Reporting any abnormal findings to the provider.

d. Applying sterile disposable gloves.

A nurse is preparing a patient for a magnetic resonance imaging scan. Which nursing
action is most important?

a. Ensuring that the patient does not eat or drink 2 hours before the examination

b. Removing all of the patients metallic jewelry

c. Administering a colon cleansing product 12 hours before the examination

d. Obtaining an order for a pain medication before the test is performed


After a patient returns from a barium swallow, the nurses priority is to

a. Encourage the patient to increase fluids to flush out the barium.

b. Monitor stools closely for bright red blood or mucus, which indicates trauma from the
procedure.

c. Inform the patient that his bowel movements are radioactive, and that he should be
sure to flush the toilet three times.

d. Thicken all patient drinks to prevent aspiration.

While a cleansing enema is administered to an 80-year-old patient, the patient


expresses the urge to defecate. What is the next priority nursing action?

a. Positioning the patient in the dorsal recumbent position with a bed pan

b. Assisting the patient to the bedside commode

c. Stopping the enema cleansing and rolling the patient into right-lying Sims position

d. Inserting a rectal plug to contain the enema solution

A nurse is educating a patient on how to irrigate an ostomy bag. Which statement by the
patient indicates the need for further instruction?

a. I can use a fleet enema to save money because it contains the same irrigation
solution.

b. Sitting on the toilet lets the irrigation sleeve eliminate into the bowl.

c. I should never attempt to reach into my stoma to remove fecal material.

d. Using warm tap water will reduce cramping and discomfort during the procedure.

A patient is diagnosed with a bowel obstruction. The nurse chooses which type of tube
for gastric decompression?

a. Salem sump

b. Dobhoff

c. Sengstaken-Blakemore

d. Small bore
A patient had an ileostomy surgically placed 2 days ago. Which diet would the nurse
recommend to the patient to ease the transition of the new ostomy?

a. Eggs over easy, whole wheat toast, and orange juice with pulp

b. Chicken fried rice with stir fried vegetables and iced tea

c. Turkey meatloaf with white rice and apple juice

d. Fish sticks with macaroni and cheese and soda

A nurse is pouching an ostomy on a patient with an ileostomy. Which action by the


nurse is most appropriate?

a. Changing the skin barrier portion of the ostomy pouch daily

b. Selecting a pouch that is able to hold excess output to reduce the frequency of
pouch emptying

c. Thoroughly scrubbing the skin around the stoma to remove excess stool and
adhesive

d. Measuring the correct size for the barrier device while leaving a 1/8-inch space
around the stoma

The nurse knows that the ideal time to change an ostomy pouch is

a. Before eating a meal, when the patient is comfortable.

b. When the patient feels that he needs to have a bowel movement.

c. When ordered in the patients chart.

d. After the patient has ambulated the length of the hallway.

The nurse administers a cathartic to a patient. The nurse determines that the cathartic
has had a therapeutic effect when the patient

a. Has a decreased level of anxiety.

b. Experiences pain relief.

c. Has a bowel movement.

d. Passes flatulence.
An older adults perineal skin appears to be dry and thin with mild excoriation. When
providing hygiene after a bowel movement, the nurse should

a. Thoroughly scrub the skin with a wash cloth and hypoallergenic soap.

b. Apply a skin protective lotion after perineal care.

c. Tape an occlusive moisture barrier pad to the patients skin.

d. Massage the skin with deep kneading pressure.

Which nursing action best reduces risk of excoriation to the mucosal lining of the nose
from a nasogastric tube?

a. Lubricating the nares with water-soluble lubricant

b. Applying a small ice bag to the nose for 5 minutes every 4 hours

c. Instilling Xylocaine into the nares once a shift

d. Changing the tape holding the tube in place once a shift

A nurse is providing discharge teaching for a patient who is going home with a guaiac
test. Which statement by the patient indicates the need for further education?

a. If I get a positive result, I have gastrointestinal bleeding.

b. I should not eat red meat before my examination.

c. I should schedule to perform the examination when I am not menstruating.

d. I will need to perform this test three times if I have a positive result.

A nurse is caring for an older adult patient with fecal incontinence due to cathartic use.
The nurse is most concerned about which complication that has the greatest risk for
severe injury?

a. Rectal skin breakdown

b. Contamination of existing wounds

c. Falls from attempts to reach the bathroom

d. Cross-contamination into the upper GI tract


The nurse is caring for a patient with Clostridium difficile. Which of the following nursing
actions will have the greatest impact in preventing the spread of bacteria?

a. Monthly in-services about contact precautions

b. Placing all contaminated items in biohazard bags

c. Mandatory cultures on all patients

d. Proper hand hygiene techniques

A nurse is performing an assessment on a patient who has not had a bowel movement
in 3 days. The nurse would expect which other assessment finding?

a. Hypoactive bowel sounds

b. Jaundice in sclera

c. Decreased skin turgor

d. Soft tender abdomen

A nurse is caring for a patient who has had diarrhea for the past week. Which additional
assessment finding would the nurse expect?

a. Increased energy levels

b. Distended abdomen

c. Decreased serum bicarbonate

d. Increased blood pressure

The nurse is caring for a patient who had a colostomy placed yesterday. The nurse
should report which assessment finding immediately?

a. Stoma is protruding from the abdomen.

b. Stoma is moist.

c. Stool is discharging from the stoma.

d. Stoma is purple.
A patient has constipation and hypernatremia. The nurse prepares to administer which
type of enema?

a. Oil retention

b. Carminative

c. Saline

d. Tap water

A guaiac test has been ordered. The nurse knows that this is a test for

a. Bright red blood.

b. Dark black blood.

c. Blood that contains mucus.

d. Blood that cannot be seen.

The nurse should place the patient in which position when preparing to administer an
enema?

a. Left Sims position

b. Fowlers

c. Supine

d. Semi-Fowlers

The nurse is assessing a patient 2 hours after a colonoscopy. Based on the procedure
done, what focused assessment will the nurse include?

a. Bowel sounds

b. Presence of flatulence

c. Bowel movements

d. Nausea
Which of the following would the nurse expect as a normal change in the bowel
elimination as a person ages?

1. Absorptive processes are increased in the intestinal mucosa.

2. Esophageal emptying time is increased.

3. Changes in nerve innervation and sensation cause diarrhea.

4. Mastication processes are less efficient.

Duller nerve sensations may place the older adult at increased risk for fecal
incontinence.
An 8-month-old infant is hospitalized with severe diarrhea. The nurse knows that the
major problem associated with severe diarrhea is:

1. Pain in the abdominal area

2. Electrolyte and fluid loss

3. Presence of excessive flatus

4. Irritation of the perineal and rectal area

A 50-year-old male client is having a screening colonoscopy. The nurse instructs the
client that:

1. No special preparation is required

2. Light sedation is normally used

3. No metallic objects are allowed

4. Swallowing of an opaque liquid is required

A client is to have a stool test for occult blood. The nurse is instructing the nursing
assistant in the correct procedure for the test. The nursing assistant is correctly
informed that:

1. Sterile technique is used for collection

2. Stool should be collected over a 3-day period

3. The specimen should be kept warm

4. A 1-inch sample of formed stool is needed


A client who recently underwent surgery and now has a colostomy is correctly instructed
by the nurse that for the next few weeks the clients diet will include foods such as:

1. Vegetables

2. Fresh fruit

3. Whole grain breads

4. Poached eggs and rice

The client has been admitted to an acute care unit with a diagnosis of biliary disease.
The nurse suspects that the feces will appear:

1. Bloody

2. Pus filled

3. Black and tarry

4. White or clay colored

The client asks the nurse to recommend bulk-forming foods that may be included in the
diet. Which of the following should be recommended by the nurse?

1. Whole grains

2. Fruit juice

3. Rare meats

4. Milk products

The client is taking medications to promote defecation. Which of the following


instructions should be included by the nurse in the teaching plan for this client?

1. Increased laxative use often causes hyperkalemia.

2. Salt tablets should be taken to increase the solute concentration of the extracellular
fluid.

3. Emollient solutions may increase the amount of water secreted into the bowel.

4. Bulk-forming additives may turn the urine pink.


While undergoing a soapsuds enema, the client complains of abdominal cramping. The
nurse should:

1. Immediately stop the infusion

2. Lower the height of the enema container

3. Advance the enema tubing 2 to 3 inches

4. Clamp the tubing

A nurse who is caring for postoperative clients on a surgical unit knows that for 24 to 48
hours postoperatively, clients who have undergone general anesthesia may experience:

1. Colitis

2. Stomatitis

3. Paralytic ileus

4. Gastrocolic reflex

For clients with hypocalcemia, the nurse should implement measures to prevent:

1. Gastric upset

2. Malabsorption

3. Constipation

4. Fluid secretion

The client is to receive a Kayexalate enema. The nurse recognizes that this is used to:

1. Prevent further constipation

2. Remove excess potassium from the system

3. Reduce bacteria in the colon before diagnostic testing

4. Provide direct antidiarrheal medication to the intestine


The appropriate amount of fluid to prepare for an enema to be given to an average-size
school-age child is:

1. 150 to 250 mL

2. 250 to 350 mL

3. 300 to 500 mL

4. 500 to 750 mL

A client has undergone intestinal surgery and now has an incontinent ostomy. The use
of which of the following products by the client indicates that the discharge learning
goals have been achieved?

1. A powder for a yeast infection

2. Peroxide to toughen the peristomal skin

3. A commercial deodorant around the stoma

4. Alcohol to cleanse the stoma

An ostomy deodorant may be placed into the pouch, not around the stoma. Alcohol
should not be used to clean the stoma. The area may be cleaned with warm tap water.
Which of the following is an appropriate nursing intervention for a client with a
nasogastric tube in place?

1. Tape the tube up and around the ear on the side of insertion.

2. Secure the tubing to the bed by the clients head.

3. Mark the tube where it exits the nose.

4. Change the tubing daily.

The tube should be taped to the nose, not to the ear. The tubing should be secured to
the clients gown, not the bed. The tubing should not be changed daily, but it should be
irrigated daily.
The nurse instructs the client that before the fecal occult blood test (FOBT) she may
eat:

1. Whole wheat bread

2. A lean, T-bone steak


3. Veal

4. Salmon

The nurse is discussing arteriosclerosis and the effects it has on the body with an older
adult client. Although the most commonly recognized effect is on the cardiovascular
system, the nurse should include which of the following statements regarding its effect
on the gastrointestinal system to complete the discussion?

1. Circulatory problems make getting to the bathroom easily problematic.

2. The benefit you get from your food is also decreased by this condition.

3. The aging process that causes the vascular problems also causes elimination
problems.

4. The problem it creates with blood flow also affects blood flow to the bowels
and so affects elimination.

Which of the following statements made by an older adult reflects the best
understanding of the role of fiber regarding bowel patterns?

1. The more fiber I eat, the fewer problems I have with my bowels.

2. Whole grain cereal and toast for breakfast keeps my bowels moving regularly.

3. My wife makes whole grain muffins; they are really good and good for me too.

4. I use to have trouble with constipation until I started taking a fiber supplement.

Which of the following statements made by an older adult reflects the best
understanding of the role of fiber regarding good bowel health?

1. Fiber is very effective at cleaning out the bowels.

2. A high-fiber diet results in softer bowel movements.

3. Passing hard, dry stool is more uncomfortable and harder on the bowels.

4. The more fiber there is in my diet, the less risk I have of developing polyps.

The nurse is discussing food allergies with a group of mothers whose children are
allergy prone. Which of the following statements made by the nurse best describes
lactose intolerance?

1. If milk causes diarrhea, cramps, or gas, it might be an intolerance of lactose.


2. You dont have to be allergic to dairy for it to cause you problems.

3. Allergies to milk can be very dangerous, even life threatening.

4. Many children outgrow their intolerance of dairy lactose.

The nurse is discussing food allergies with a group of mothers whose children are
allergy prone. Which of the following statements made by a mother best describes
lactose intolerance?

1. My child is allergic to milk; it makes her very gassy.

2. Dairy products require a special enzyme to be digested properly.

3. Being lactose intolerant means my child cant tolerate dairy products.

4. My child gets diarrhea from dairy products because she cant digest lactose.

An adult client reports to the nurse that she has been experiencing constipation recently
and is interested in any suggestions regarding dietary changes she might make. Which
of the following suggestions provided by the nurse is most likely to minimize the clients
complaint?

1. Have you tried foods like prunes and bran?

2. You might find the new flavored bulk laxatives helpful.

3. What have you tried in the past that hasnt been helpful?

4. Increase your fluid intake; have some juice with breakfast.

A client who is 2 days postoperative reports feeling constipated to the nurse. The client
has good bowel sounds in all four quadrants and has tolerated liquids well. Her pain is
being controlled with an opioid analgesic. Which of the following interventions should
the nurse try initially?

1. Let me get you some apple juice.

2. Ambulating may get your bowels moving.

3. I'll see about getting a different pain medication.

4. Your health care provider might prescribe an enema if I call.


Which of the following statements by a client reporting constipation reflects the most
informed understanding of interventions that will aid in assuming proper bowel mobility?

1. Could it be that I need to get more exercise, even here in the hospital?

2. Is it true that drinking coffee often helps stimulate the bowels to work?

3. I guess a little high-fiber cereal might help. Can you get me some from the cafeteria?

4. May I have a cup of decaffeinated tea in addition to my breakfast juice? That


usually helps.

A client is caring for her husband who recently experienced a cerebral vascular
accident. She tells the home care nurse that she has been very anxious lately about all
the added responsibilities. She adds that she has not been sleeping well and has had
several bouts of diarrhea. Which of the following statements by the nurse focuses on
the most likely cause of the gastrointestinal problem?

1.Have you experienced increased gas and cramping in addition to the diarrhea?

2.You are under a lot of stress; that can affect your bowels and result in diarrhea.

3. I suggest you get some over-the-counter medication and keep it on hand to manage
those bouts.

4. Have you been eating a well-balanced diet since you brought your husband home?

A client is caring for her daughter, who recently suffered multiple fractures in an
automobile accident. The client tells the home care nurse that she has been really down
since all this happened. She adds that she has been constipated and not really
interested in eating. Which of the following statements by the nurse focuses on the most
likely cause of the gastrointestinal problem?

1. Actually, how long have you been constipated?

2. Are you eating fiber-rich foods like fruit and whole grains?

3. You may be depressed; emotional depression can cause constipation.

4. I suggest you get some over-the-counter mild laxative and see if that helps.
A 70-year-old client is discussing his recent difficulty with having regular bowel
movements while on a cross-country bus tour with a senior citizens group. Which of the
following assessment questions is directed toward the most likely cause of the problem?

1. Did the bus stop frequently so you could get up and walk around?

2. Did you eat enough fiber while you were on the trip?

3. Do you find using public restrooms unsettling?

4. Do you have any chronic bowel-related problems?

The nurse is caring for a 19-year-old male client with a fractured left femur whose leg
was pinned 36 hours ago and is now in traction. Which of the following stressors is
mostly likely the cause of this clients difficulty related to constipation?

1. Pain related to the fracture and its repair

2. Anxiety regarding the serious nature of the injury

3. The need to defecate in an unfamiliar, awkward position

4. Poor fluid intake after the accident and ensuing surgery

A client who was recently diagnosed with anemia and rheumatoid arthritis reports to the
nurse that she has noticed that her stool is black, and she is concerned because there
is a history of colon cancer in her family. Which of the following assessment questions is
most likely to provide information regarding this clients bowel problem?

1. What medications are you currently on?

2. When did you have your last colonoscopy?

3. Does the arthritis severely impair your mobility?

4. Would you like to have the stool tested for occult blood?

Which of the following statements made by a nurse discussing the effect of an antibiotic
on the gastrointestinal system reflects the best understanding of the possible
occurrence of diarrhea?

1. The GI tract naturally rids itself of bacterial toxins by increasing peristalsis, and that
causes diarrhea.

2. The antibiotic is responsible for killing off the GI tracts normal bacterial, and
diarrhea is the result.
3. For some, antibiotics irritate the mucous lining of the intestines, causing decreased
absorption and diarrhea.

4. When you are taking an antibiotic, your body is fighting off an infection, and
peristalsis is faster and so diarrhea occurs.

31. A client is reporting that the oral medication she was prescribed for her
hypothyroidism does not seem to be helping. The client goes on to report that she has
been experiencing tension-related headaches and constipation. She has been self-
medicating with nonsteroidal antiinflammatory drugs (NSAIDs) and bulk laxatives.
Which of the following assessment questions is most likely to provide information
regarding this clients concern regarding her thyroid problem?

1. How long have you taken Synthroid?

2. What other medications are you currently on?

3. How long have you been taking a bulk laxative?

4. Have you developed any other gastrointestinal symptoms?

32. The nurse is assessing a cognitively impaired older adult client and observes a
leaking of liquid stool from the rectum. The nurses initial intervention for this client is to:

1. Determine if the client has been eating sufficiently, especially fiber-rich foods

2. Determine how long it has been since the client had a normal-size, formed stool

3. Perform a digital examination of the rectum to determine the presence of stool

4. Call the health care provider to get a prescription for an antidiarrheal medication
The greatest risk for injury for a client who has fecal incontinence is:

1. Perineal and rectal skin breakdown

2. The contamination of existing wounds

3. Falls resulting from attempts to reach the bathroom

4. Cross-contamination into the upper gastrointestinal tract


The nurse is providing ancillary personnel with instructions regarding the proper
methods to implement when caring for a client with a Clostridium difficile infection.
Which of the following practices will have the greatest impact on containment of the
bacteria and thus prevention of cross-contamination?

1. Frequent in-services on transmission modes of C. difficile

2. Practice of proper hand hygiene by all staff

3. Appropriate handling of contaminated linen

4. Stool cultures on all suspected carriers

Which of the following clients is at greatest risk for serious complications when using the
Valsalva maneuver to expel feces?

1. 25-year-old pregnant client

2. 66-year-old male with hypertrophied prostate disease

3. 44-year-old male client with glaucoma

4. 53-year-old female with stomach cancer

The mother of an 18-month-old male client shares with the nurse that she is trying to get
her child to tell her when he needs to have a bowel movement. Which of the following
statements is the most appropriate response from the nurse?

1. I'm sure that you will be glad to have your son out of diapers.

2. I once heard of a child who was totally potty-trained by the time he was a year old.

3. Development of neuromuscular control of the bowels doesnt normally occur


until a child is between 2 and 3 year of age.

4. You will have to really be persistent about taking him to the bathroom frequently in
order to be successful.

The 35-year-old pregnant client is concerned about constipation. When weighing the
advantages and disadvantages of having a local anesthetic over a general anesthetic
for a caesarian section, the nurse shares with the client that the local will cause less risk
for constipation following surgery. The best reason that there is less constipation
following this surgery is because:

1. The client will not have to be allowed nothing by mouth (NPO) before surgery
2. The client will be able to ambulate immediately following surgery

3. The client will be able to eat following surgery

4. Local or regional anesthetic often has little or no effect on bowel activity

The client will not be able to ambulate immediately after surgery because of loss of
feeling in the lower extremities. Clients should be able to eat following nonbowel-related
surgery whether or not they have undergone a general anesthetic or a local anesthetic.
A 44-year-old male client was placed on a daily low-dose aspirin regimen by his health
care provider following a recent diagnosis of hypertension and periodic atrial fibrillation.
The client is currently hospitalized with renal stones. As the nurse is admitting the client,
he shares that he has been very tired. The nurse gathers additional data regarding his
bowel habits. The client shares that he has recently had black, tarry stools. The nurse is
most concerned that the client may have:

1. Colon cancer

2. A GI bleed from the aspirin therapy

3. Ongoing atrial fibrillation

4. Electrolyte imbalance

The nurse is counseling a 65-year-old female client on her use of mineral oil as a
laxative. One of the most important things that the nurse can share with the client is how
mineral oil can cause the decreased absorption of which of the following vitamins?

1. Vitamin C

2. Niacin

3. Vitamin D

4. Riboflavin

An active 25-year-old female client shared with the nurse that ever since she had gone
on a high-protein low-carbohydrate diet she had suffered from constipation. The client
states that the diet is working for her in terms of weight loss and would like to stay on it.
The best response from the nurse is that the client should try:

1. Consuming more low-carbohydrate fiber-rich foods like broccoli, raspberries,


blackberries, and asparagus

2. Taking a laxative when feeling constipated


3. Try a different diet with less tendency to cause constipation

4. Exercise more

The nurse knows that the client receiving enteral feedings is at risk for diarrhea. One of
the measures that the nurse can take to minimize the risk for diarrhea in this client is:

1. Making sure to chill the canned feeding before administering

2. Using strict sanitation when administering the formula

3. Not deviating from the prescribed rate of delivery for the formula

4. Not diluting or changing the strength of the prescribed formula

Upon auscultation of the clients abdomen, the nurse hears hyperactive bowel sounds
(greater than 35 per minute). The nurse knows that this can indicate which of the
following?

1. Paralytic ileus

2. Fecal impaction

3. Small intestine obstruction

4. Abdominal tumor

The health care provider has ordered a stool specimen for ova and parasites from the
43-year-old male client. The nurse knows that when collecting the specimen the stool
must be:

1. Kept on ice

2. Kept warm

3. Collected using sterile technique

4. Free from urine

The nurse is instructing the client in stomal care for an incontinent ostomy. The nurse
evaluates achievement of learning goals if the client uses:

a. Triamcinolone acetamide (Kenalog) spray for a yeast infection

b. Peroxide to toughen the periostomal skin


c. A commercial deodorant around the stoma

d. Alcohol to cleanse the stoma


ANS: a

, the nurse should have an intervention that addresses the number of grams of cellulose
that are needed for normal bowel function. How many grams should be in the daily diet?

A) 2030 g

B) 4050 g

C) 6070 g

D) >80g

A patient reports constipation. Which of the following assessment questions should the
nurse initially ask when completing the patients health history, including bowel habits?

A) Do you have a daily bowel movement?

B) How do you handle stress?

C) Do you eat fiber foods every day?

D) What medicines do you take?

The student nurse has completed a presentation to a group of senior citizens on


colorectal screening. Which statement by a participant suggests a need for further
education?

A) I will need yearly screenings for colon cancer.

B) I will have a fecal occult blood test done every 5 years.

C) I will have a flexible endoscopic exam done every 5 years.

D) My mother had colon cancer so I am at a greater risk for also developing colon
cancer.
A patient is complaining of increased flatulence. Which of the following may be a cause
of his flatulence? Select all that apply.

A) Carbonated beverages

B) Caffeinated beverages

C) Smoking

D) Drinking straws

E) Rapid ingestion of food

A nurse is providing education to an elderly patient concerning ways to prevent


constipation. Which diet choices would support that the education was successful?
Select all that apply

A) Hot tea with meals

B) A turkey sandwich with whole-grain bread

C) Prune juice with breakfast

D) Ice cream with lunch and dinner

E) Diet soda with lemon

Ice cream and diet soda do not provide any preventative measures for constipation.
The student nurse is preparing a presentation on bowel elimination. Which of the
following would be a potential cause of diarrhea that the student should include? Select
all that apply.

A) Opioids

B) Antibiotics

C) Acute stress

D) Depression

E) Increased physical activity


The nurse is assisting an elderly patient into position for a sigmoidoscopy. Which
position would the nurse place the patient in?

A) Right lateral

B) Left lateral

C) Prone

D) Semi-Fowlers

The student nurse is administering a large-volume enema to a patient. The patient


complains of abdominal cramping. What should the student nurse do first?

A) Increase the flow of the enema for approximately 30 seconds then decrease it to the
prior flow rate.

B) Stop the administration of the enema and notify the physician.

C) Stop the administration of the enema momentarily .

D) Increase the flow of the enema until all of the solution has been administered.

When caring for a patient with a new colostomy, which assessment finding would be
considered abnormal and need to be reported to the physician?

A) The stoma is pink.

B) The stoma has a small amount of bleeding.

C) The stoma is prolapsed.

D) The stoma is on the abdominal surface.

An elderly woman who is incontinent of stool following a cerebrovascular accident will


have the following nursing diagnosis

A) Bowel incontinence related to loss of sphincter control as evidenced by


inability to delay the urge to defecate

B) Diarrhea related to tube feedings as evidenced by hyperactive bowel sounds and


urgency

C) Constipation related to physiologic condition involving the deficit in neurologic


innervation as evidenced by fecal incontinence
D) Fecal retention related to loss of sphincter control and diminished spinal cord
innervation related to hemiparesis

When the nurse performs a Hemoccult test on a stool specimen, blood in the stool will
change the color on the test paper to

A) Blue

B) Brown

C) Green

D) Red

A patient has had abdominal surgery and in 72 hours develops abdominal distention
and absence of bowel sounds with pain. The nurse suspects the patient has

A) A wound infection

B) Need of greater pain relief

C) Increased activity

D) Paralytic ileus

You are educating a new colostomy patient on gas-producing foods. Which of the
following are gas-producing foods the patient may choose to avoid?

A) Lettuce

B) Rice

C) Brussels sprouts

D) Green peppers

The proliferation of Clostridium difficile causes

A) Antibiotic-associated diarrhea

B) Escherichia coli diarrhea

C) Urinary Clostridium infection

D) Anal yeast infection


An elderly patient who is wheelchair bound following a cerebrovascular accident is
being assessed by the nurse. The nurse notes the patient has seepage of stool from the
anus. The nurse knows this is indicative of

A) Constipation
B) Diarrhea
C) Fecal impaction
D) Intestinal infection

Which of the following diversions is considered a continent ostomy?

A) Colostomy
B) Ileostomy
C) Ileoconduit
D) Ileoanal

The type of stool that will be expelled into the ostomy bag by a patient who has
undergone surgery for an ileostomy will be

A) Bloody
B) Mucus filled
C) Soft semi-formed
D) Liquid consistency

A patient has completed an upper gastrointestinal x-ray, small bowel series, and lower
gastrointestinal x-ray. Following these x-rays, the nurse will need to administer

A) A low-residue diet
B) An antibiotic
C) A laxative
D) High-fiber diet

Which of the following symptoms is a known side effect of antibiotics?

A) Diarrhea
B) Constipation
C) Fecal impaction
D) Abdominal bloating

Which of the following medications causes constipation?

A) Magnesium antacids
B) Dulcolax
C) Aspirin
D) Iron supplements
The nurse needs to assess the patients elimination patterns. Which of the following
patients will most likely deny the urge to defecate?

A) Patient with anxiety and depression


B) Patient who consumes >30 g of fiber
C) Patient who has a colostomy
D) Patient 3 days post-vaginal delivery

Ignoring the urge to defecate on a continual basis leads to

A) Sudden increase in stool with mucus


B) Constipation and hard stool
C) Need to increase milk intake
D) Total loss of bowel control

A young woman has just consumed a serving of ice cream pie and develops severe
cramping and diarrhea. The school nurse suspects the woman is

A) Allergic to sugar
B) Lactose intolerant
C) Experiencing infectious diarrhea
D) Deficit in fiber

When educating an elderly patient on the prevention of constipation, the nurse should
provide which of the following educational interventions?

A) Drink three glasses of milk per day


B) Eat six servings of bread or pasta
C) Consume antacids to decrease reflux
D) Increase intake of fresh vegetables

Which of the following factors is related to developmental changes in bowel habits for
elderly patients?

A) Increase in dietary fiber can decrease peristalsis


B) Milk products cause constipation in lactose intolerance patients
C) Weakened pelvic muscles lead to constipation
D) The elderly should peel fruits before eating
In a toddler, a good indication of spinal cord maturation and ultimate bowel control is

A) Use of the flexor and extensor


B) The ability to walk
C) Parallel play
D) Recognition of peristalsis
Ans: B

The postpartum nurse is instructing a new mother that her infant will pass meconium for

A) 1 day
B) 2 days
C) 3 days
D) 4 days

When educating a breast-feeding mother on the characteristics of the stool of her


newborn, the nurse should inform her that the stool will be

A) Dark yellow
B) Bright yellow
C) Beige
D) Brown

What is meconium?

A) Semi-digested food
B) Soft brown stool
C) Secreted liquid mucus
D) Dry intestinal secretions

The student nurse studying bowel elimination learns that the following statements
accurately describe the process of peristalsis. Select all that apply.

A) The sympathetic nervous system stimulates movement.

B) The autonomic nervous system innervates the muscles of the colon.

C) Peristalsis occurs every 3 to 12 minutes.

D) Mass peristaltic sweeps occur one to four times each 24-hour period in most
people.

E) Mass peristalsis often occurs after food has been ingested.

F) One-third to one-half of ingested food waste is normally excreted in the stool within
48 hours.
Based on knowledge of the physiology of the gastrointestinal tract, what type of stools
would the nurse assess in a patient with an illness that causes the stool to pass through
the large intestine quickly?

A) hard, formed

B) black, tarry

C) soft, watery

D) dry, odorous

What term is used to describe intestinal gas?

A) feces

B) stool

C) peristalsis

D) flatus

Which of the following statements accurately describes the act of defecation?

A) Defecation refers to the emptying of the small intestine.

B) Centers in the medulla and the spinal cord govern the reflex to defecate.

C) When sympathetic stimulation occurs, the internal anal sphincter relaxes and the
colon contracts sending fecal content to the rectum.

D) Rectal distention leads to a decrease in intrarectal pressure, causing the muscles to


stretch and thereby stimulating the defecation reflex.

A nurse is assessing the stools of a breastfed baby. What is the appearance of normal
stools for this baby?

A) yellow, loose, odorless

B) brown, paste-like, some odor

C) brown, formed, strong odor

D) black, semiformed, no odor


A hospitalized toddler, previously bowel trained, has been having incontinent stools.
What would the nurse tell the parents about this behavior?

A) When he does this, scold him and he will quit.

B) I dont understand why this child is losing control.

C) This is normal when a child this age is hospitalized.

D) I will have to call the doctor and report this behavior.

A nurse caring for elderly patients in an assisted-living facility encourages patients to


eat a diet high in fiber to avoid which of the following developmental risk factors for this
group?

A) diarrhea

B) fecal incontinence

C) constipation

D) flatus

A patient is having difficulty having a bowel movement on the bedpan. What is the
physiologic reason for this problem?

A) It is painful to sit on a bedpan.

B) The position does not facilitate downward pressure.

C) The position encourages the Valsalva maneuver.

D) The cause is unknown and requires further study.

The following foods are a part of a patients daily diet: high-fiber cereals, fruits,
vegetables, 2,500 mL of fluids. What would the nurse tell the patient to change?

A) decrease high-fiber foods

B) decrease amount of fluids

C) omit fruits if eating vegetables

D) nothing; this is a good diet


A young woman comes to the Emergency Department with severe abdominal cramping
and frequent bloody stools. Food poisoning is suspected. What diagnostic test would be
used to confirm this diagnosis?

A) routine urinalysis

B) chest x-ray

C) stool sample

D) sputum sample

A patient with terminal cancer is taking high doses of a narcotic for pain. The nurse will
teach the patient or family about what common side effect of opioids?

A) inability to change positions

B) problems with communication

C) diarrhea

D) constipation

A nurse is assessing a patient the first day after colon surgery. Based on knowledge of
the effects of anesthesia and manipulation of the bowel during surgery, what focused
assessment will be included?

A) bowel sounds

B) skin turgor

C) pulse character

D) urinary output

A nurse has auscultated the abdomen in all four quadrants for 5 minutes and has not
heard any bowel sounds. How would this be documented?

A) Auscultated abdomen for bowel sounds, bowel not functioning.

B) All four abdominal quadrants auscultated. Inaudible bowel sounds.

C) Bowel sounds auscultated. Patient has no bowel sounds.

D) Patient may have bowel sounds, but they cant be heard.


A nurse is conducting an abdominal assessment. What is the rationale for palpating the
abdomen last when conducting an abdominal assessment?

A) it is the most painful assessment method

B) it is the most embarrassing assessment method

C) to allow time for the examiners hands to warm

D) it disturbs normal peristalsis and bowel motility

What are two essential techniques when collecting a stool specimen?

A) hand hygiene and wearing gloves

B) following policies and selecting containers

C) wearing goggles and an isolation gown

D) using a no-touch method and toilet paper

A Hematest for occult blood in the stool has been ordered. What is occult blood?

A) bright red visible blood

B) dark black visible blood

C) blood that contains mucus

D) blood that cannot be seen

A nurse is scheduling ordered diagnostic studies for a patient. Which of the following
tests would be performed first?

A) fecal occult blood test

B) barium study

C) endoscopic exam

D) upper gastrointestinal series

A patient has had frequent watery stools (diarrhea) for an extended period of time. The
patient also has decreased skin turgor and dark urine. Based on these data, which of
the following nursing diagnoses would be appropriate?
A) Imbalanced Nutrition: Less than Body Requirements

B) Deficient Fluid Volume

C) Impaired Tissue Integrity

D) Impaired Urinary Elimination

An infant has had diarrhea for several days. What assessments will the nurse make to
identify risks from the diarrhea?

A) heart tones

B) lung sounds

C) skin turgor

D) activity level

A patient tells the nurse that he takes laxatives every day but is still constipated. The
nurses response is based on which of the following?

A) Habitual laxative use is the most common cause of chronic constipation.

B) If laxatives are not effective, the patient should begin to use enemas.

C) A laxative that works by a different method should be used.

D) Chronic constipation is nothing to be concerned about.

A patient who has been on a medication that caused diarrhea is now off the medication.
What could the nurse suggest to promote the return of normal flora?

A) stool-softening laxatives, such as Colace

B) increasing fluid intake to 3,000 mL/day

C) drinking fluids with a high sugar content

D) eating fermented products, such as yogurt


A patient is on bedrest, and an enema has been ordered. In what position should the
nurse position the patient?

A) Fowlers

B) Sims

C) Prone

D) Sitting

A patient is having liquid fecal seepage. He has not had a bowel movement for 6 days.
Based on the data, what would the nurse assess?

A) amount of intake and output

B) color and amount of urine

C) color of the feces

D) consistency of the feces

Which of the following would be an expected outcome for a patient when the nurse is
conducting a bowel training program?

A) Have a soft, formed stool at regular intervals without a laxative.

B) Continue to use laxatives, but use one less irritating to the rectum.

C) Use oil-retention enemas on a regular basis for elimination.

D) Have a formed stool at least twice a day for 2 weeks.

A patient tells the nurse, I increased my fiber, but I am very constipated. What further
information does the nurse need to tell the patient?

A) Just give it a few more days and you should be fine.

B) Well, that shouldnt happen. Let me recommend a good laxative for you.

C) When you increase fiber in your diet, you also need to increase liquids.

D) I will tell the doctor you are having problems; maybe he can help.

A nurse is caring for a patient with a colostomy. What type of stools would she expect to
find in the colostomy bag?
A) liquid

B) watery

C) formed

D) none

A nurse is documenting the appearance of feces from a patient with a permanent


ileostomy. Which of the following would she document?

A) Ileostomy bag half filled with liquid feces.

B) Ileostomy bag half filled with hard, formed feces.

C) Colostomy bag intact without feces.

D) Colostomy bag filled with flatus and feces.

A nurse is assessing the stoma of a patient with an ostomy. What would the nurse
assess in a normal, healthy stoma?

A) pallor

B) purple-blue

C) Irritation and bleeding

D) dark red and moist

A nurse is caring for a patient who is 1 day postoperative for a temporary colostomy.
The nurse assesses no feces in the collection bag. What should the nurse do next?

A) Notify the physician immediately.

B) Ask another nurse to check her findings.

C) Nothing; this is normal.

D) Recheck the bag in 2 hours.

A nurse is providing discharge instructions for a patient with a new colostomy. Which of
the following is a recommended guideline for long-term ostomy care?

A) During the first 6 to 8 weeks after surgery, eat foods high in fiber.
B) Drink at least 2 quarts of fluids, preferably water, daily.

C) Use enteric-coated or sustained-release medications if needed.

D) Use a mild laxative if needed.

What is the correct order for an ostomy pouch change?

1. Close the end of the pouch.


2. Measure the stoma.
3. Cut the hole in the wafer.
4. Press the pouch in place over the stoma.
5. Remove the old pouch.
6. Trace the correct measurement onto the back of the wafer.
7. Assess the stoma and the skin around it.
8. Cleanse and dry the peristomal skin.

A. 5, 8, 2, 7, 3, 6, 4, 1

B. 8, 5, 6, 2, 7, 3, 4, 1

C. 8, 5, 7, 6, 2, 3, 4, 1

D. 5, 8, 7, 2, 6, 3, 4, 1

This order of tasks describes the correct way to change an ostomy pouch.
The nurse should do which of the following when placing a bedpan under an
immobilized patient?

A. Lift the patient's hips off the bed and slide the bedpan under the patient

B. After positioning the patient on the bedpan, elevate the head of the bed to a 45-
degree angle

C. Adjust the head of the bed so it is lower than the feet and use gentle but firm
pressure to push the bedpan under the patient

D. Have the patient stand beside the bed and then have him or her sit on the bedpan on
the edge of the bed
When a patient has fecal incontinence as a result of cognitive impairment, it may be
helpful to teach caregivers to do which of the following interventions?

A. Cleanse the skin with antibacterial soap and apply talcum powder to the buttocks

B. Use diapers and heavy padding on the bed

C. Initiate bowel or habit training program to promote continence

D. Help the patient to toilet once every hour

A cognitively impaired patient may have forgotten how to respond to the urge to
defecate and benefit from a structured program of bowel retraining.
A patient has not had a bowel movement for 4 days. Now she has nausea and severe
cramping throughout her abdomen. On the basis of these findings, what should the
nurse suspect?

A. An intestinal obstruction

B. Irritation of the intestinal mucosa

C. Gastroenteritis

D. A fecal impaction

Absence of bowel movement, nausea, cramping, and possibly vomiting are


characteristic of an intestinal obstruction.
The nurse is taking a health history of a newly admitted patient with a diagnosis of
possible fecal impaction. Which of the following is the priority question to ask the patient
or caregiver?

A. Have you eaten more high-fiber foods lately?

B. Are your bowel movements soft and formed?

C. Have you experienced frequent, small liquid stools recently?

D. Have you taken antibiotics recently?

Frequent or continuous oozing of liquid stools occurs when liquid fecal matter above the
impacted stool seeps around the fecal impaction.
The nurse is caring for a patient with an ileostomy. Which intervention is most
important?

A. Cleansing the stoma with hot water

B. Inserting a deodorant tablet in the stoma bag

C. Selecting or cutting a pouch with an appropriate-size stoma opening

D. Wearing sterile gloves while caring for the stoma

An elderly patient comes to the hospital with a complaint of severe weakness and
diarrhea for several days. Of the following problems, which is the most important to
assess initially?

A. Malnutrition

B. Dehydration

C. Skin breakdown

D. Incontinence

You might also like